Test Bank of Microeconomics 7th Edition Jeffrey M. Perloff

January 9, 2017 | Author: nadiaBjorlin | Category: N/A
Share Embed Donate


Short Description

Download Test Bank of Microeconomics 7th Edition Jeffrey M. Perloff...

Description

Test Bank of Microeconomics 7th Edition Jeffrey M. Perloff IF You Want To Purchase A+ Work Then Click The Link Below , Instant Download

http://www.acehomework.net/?download=testbank-of-microeconomics-7th-edition-jeffrey-mperloff

If You Face Any Problem E- Mail Us At [email protected]

IF YOU FACE ANY PROBLEM CONTACT US AT [email protected]

Microeconomics, 7e (Perloff) Chapter 1 Introduction

1.1 Microeconomics: The Allocation of Scarce Resources

1) Microeconomics studies the allocation of 1.

A) decision makers.

2.

B) scarce resources.

3.

C) models.

4.

D) unlimited resources.

2) Microeconomics is often called 1.

A) price theory.

2.

B) decision science.

3.

C) scarcity.

4.

D) resource theory.

3) Most microeconomic models assume that decision makers wish to

1.

A) make themselves as well off as possible.

2.

B) act selfishly.

3.

C) make others as well off as possible.

4.

D) None of the above.

4) Society faces trade-offs because of 1.

A) government regulations.

2.

B) profit motive.

3.

C) faceless bureaucrats.

4.

D) scarcity.

5) A market 1.

A) always involves the personal exchange of goods for money.

2.

B) allows interactions between consumers and firms.

3.

C) always takes place at a physical location.

4.

D) has no influence on prices.

6) What links the decisions of consumers and firms in a market? 1.

A) the government

2.

B) prices

3.

C) coordination officials

4.

D) microeconomics

7) The price of a good is 1.

A) always equal to the cost of producing the good.

2.

B) never affected by the number of buyers and sellers.

3.

C) usually determined in a market.

4.

D) None of the above.

8) Who or what is responsible for bringing together scarce resources to produce most of the goods and services in the U.S.? 1.

A) the U.S. government

2.

B) the United Nations

3.

C) the Federal Reserve Bank

4.

D) markets and prices

9) Which of the following is a fundamental topic addressed by microeconomics? 1.

A) whether to extend unemployment insurance

2.

B) determining how many new iPhones the Apple company should produce

3.

C) the level of inflation in the country

4.

D) the impact of interest rates on savings in the economy

10) A "twinkie tax" on fatty foods would aim to 1.

A) reduce the consumption of fatty foods.

2.

B) reduce the production of fatty foods.

3.

C) raise tax revenues for other uses.

4.

D) All of the above.

11) Which choice below illustrates the tradeoff faced by surfboard manufacturers after Clark foam shut down in 2005, eliminating 80% of the world's foam blanks used to shape surfboards? The firms decide to 1.

A) substitute foam blanks for balsa wood blanks as used in the 1940s.

2.

B) advertise their new surfboard wax.

3.

C) offer surf classes.

4.

D) B and C

12) In the Soviet Union, which boasted about giving every worker a job, some workers were given the task of digging holes and filling them again. What function of microeconomic analysis did this policy address? 1.

A) What goods/services to produce

2.

B) How to produce the goods and services

3.

C) Who gets the goods and services

4.

D) A and B

13) The objective of creating a permit trading system for SO2 (sulfur dioxide) is to 1.

A) employ environmental protection agency (EPA) workers.

2.

B) create a market in which one did not previously exist.

3.

C) make annual company tax reporting more difficult.

4.

D) increase the incidence of acid rain.

14) Under most circumstances, the application of taxes on goods will only affect who gets the goods.

15) Most modern financial centers use computers to match buyers and sellers. This absence of personal contact contradicts the definition of a market.

16) Governments do not respond to prices.

17) Explain how a market helps determine which goods and services will be produced, how to produce them, and who gets them.

1.2 Models

1) The purpose of making assumptions in an economic model is to 1.

A) force the model to yield the correct answer.

2.

B) minimize the amount of work an economist must do.

3.

C) simplify the model while keeping important details.

4.

D) express the relationship mathematically.

2) Einstein was quoted saying "Everything should be made as simple as possible, but not simpler." When it comes to economic models this means that 1.

A) models shouldn't be too complex.

2. 3. 4.

B) models shouldn't be too simple. C) models should have a level of abstraction appropriate to the topic investigated. D) all of the above

3) Economists tend to judge a model based upon 1.

A) the reality of its assumptions.

2.

B) the accuracy of its predictions.

3.

C) its simplicity.

4.

D) its complexity.

4) Which of the following is an example of a normative statement? 1.

A) A higher price for a good causes people to want to buy less of that good.

2.

B) A lower price for a good causes people to want to buy more of that good.

3.

C) To make the good available to more people, a lower price should be set.

4.

D) If you consume this good, you will be better off.

5) Every economic model should include money as a variable. This statement is 1.

A) true, because every transaction in the economy uses money.

2.

B) true, because the federal reserve is very important.

3.

C) false, because some transactions in the economy are accomplished without money.

4.

D) false, because a model can get unnecessarily complex if it includes money.

6) Which of the following is an example of a normative statement? 1.

A) Since this good is bad for you, you should not consume it.

2.

B) This good has bad health effects.

3.

C) If you consume this good, you will get sick.

4.

D) People usually get sick after consuming this good.

7) Which of the following is an example of a positive statement? 1.

A) Since this good is bad for you, you should not consume it.

2.

B) If this good is bad for you, you should not consume it.

3.

C) If you consume this good, you will get sick.

4.

D) None of the above.

8) If an important assumption is omitted from an economic model 1.

A) the model's predictions will be accurate 50% of the time.

2.

B) the model's predictions will be inaccurate.

3.

C) the model will not predict anything.

4.

D) the model will be rejected by other economists.

9) Economic models are most useful in 1.

A) predicting changes in one variable due to a change in one or more other variables.

2.

B) predicting the direction of the stock market.

3.

C) explaining the future with the past.

4.

D) generating untestable hypotheses.

10) Economic modeling requires 1.

A) mathematics.

2.

B) logic.

3.

C) calculus.

4.

D) trigonometry.

11) Economic models are most often tested 1.

A) using computer simulations.

2.

B) using data from the distant past.

3.

C) using data from the real world.

4.

D) using logic alone.

12) Which of the following is not considered an ingredient to an economic model? 1.

A) Assumptions

2.

B) Logic

3.

C) Hypotheses

4.

D) Experience

13) Economists make simplifying assumptions to 1.

A) understand extremely complex phenomenon.

2.

B) build a model that is as close to the real world as possible.

3.

C) focus on the variables that are important to an economic theory.

4.

D) A and C

14) Economists will use a model such as the law of demand 1.

A) forever, ignoring all criticism.

2.

B) until it is refuted by someone.

3.

C) until the model produces the same results with added complexity.

4.

D) until every microeconomic courses uses the model.

15) Which of the following is an example of a positive statement? 1.

A) Prices determine what goods and services are produced in a market economy.

2.

B) The government of the Soviet Union determined the three fundamental questions of microeconomics (i.e., what is produced, how it is produced, who receives the product).

3.

C) The production of goods and services should be determined by market prices.

4.

D) A and B

16) Which of the statements below is/are normative? 1.

A) Economists need to include more reality in their models.

2.

B) Economists will use a model until it is refuted by someone.

3.

C) Microeconomists study economic growth.

4.

D) B and C

17) Normative analysis offers decision makers the most valuable information when choosing among alternatives.

18) If a model fits reality but doesn't generate testable predictions, it is of little value to economists.

19) If actual experience supports two competing theories, then both theories are proven to be true.

20) Normative statements are easily debated whereas positive statements are simply rhetorical.

21) Legislators argue that a minimum wage law is instituted to help poor people. Economists can attack the minimum wage law on two fronts. First, some argue that government should not help the poor. Second, some argue that minimum wage laws actually hurt the poor because it creates unemployment. Which argument is normative and which is positive?

1.3 Uses of Microeconomic Models

1) Economic policy of the government is often based on 1.

A) microeconomic models.

2.

B) educated guessing.

3.

C) intuitive reasoning.

4.

D) hints.

2) Microeconomic models are used to 1.

A) make predictions.

2.

B) explain real-life phenomena.

3.

C) evaluate policy alternatives.

4.

D) All of the above.

3) A microeconomic model CANNOT be used to 1.

A) evaluate the impact of a price change on a firm's revenue.

2.

B) predict the impact of an increase in the minimum wage on unemployment.

3.

C) evaluate the fairness of a proposal to nationalize health insurance.

4.

D) evaluate the effect of an increase in stadium size on the price of a sport team's tickets.

4) The testing of economic models facilitates 1.

A) proving a model is false.

2.

B) proving a model is true.

3.

C) proving that one person's opinion is morally incorrect.

4.

D) proving that one society's legal structure is better than another country's.

5) Economic models are only useful in analyzing government policy. 1.

A) True, individuals are irrational and therefore economic models are useless.

2.

B) False, economic models can be used to predict individual and firm behavior.

3.

C) True, economists only model those questions for which they are hired.

4.

D) False, economic models are not even useful in analyzing government policy.

6) Economic models are only applicable to free-market countries. 1.

A) False. Economic models are applicable wherever there is scarcity.

2.

B) True. Economic models are applicable only when private citizens are free to make their own decisions.

3.

C) False. Economic models are applicable in all countries with more than one million people, regardless of political structure.

4.

D) True. Economic models are only applicable when government is small relative to the rest of the economy.

7) Governments use microeconomic models to 1.

A) predict the impact of a policy before adopting it.

2.

B) consider the tradeoff between clean air and a high level of GDP.

3.

C) determine the tradeoff between equity and efficiency.

4.

D) All of the above.

8) Individuals use microeconomic models to determine 1.

A) their optimal consumption of beer given an increase in inflation.

2.

B) their optimal amount of leisure.

3.

C) whether or not to go to college given current student loan interest rates.

4.

D) All of the above.

9) Firms do NOT use microeconomic models to 1.

A) determine what inputs will be used for production.

2.

B) determine whether they should advertise in the newspaper or on the radio.

3.

C) strategically raise profits over competitors.

4.

D) None of the above.

10) Microeconomics can be used by governments to predict the impacts of a policy and suggest solutions to problems.

11) One million automobiles have a defect that could cause the car to explode; however, only one of those cars will actually explode. Nobody knows which one car it is. When the car does explode, the victim's family will sue the automaker for $1 million and win. The defect costs $2 per car to repair. What does economics predict about the automaker's decision to repair the defect?

12) Describe in words the anatomy of an economic model.

Microeconomics, 7e (Perloff) Chapter 2 Supply and Demand

2.1 Demand

1) According to the Law of Demand, the demand curve for a good will 1.

A) shift leftward when the price of the good increases.

2.

B) shift rightward when the price of the good increases.

3.

C) slope downward.

4.

D) slope upward.

2) Assuming plasma TVs are a normal good, an increase in consumer incomes will lead to 1.

A) a rightward shift of the demand curve for plasma TVs.

2.

B) a movement upward along the demand curve for plasma TVs.

3.

C) a rightward shift of the supply curve for plasma TVs.

4.

D) no change of the demand curve for plasma TVs.

3) Consider the demand function Qd = 150 - 2P. The effects of other determinants of Qd is reflected in 1.

A) the intercept of the function.

2.

B) the slope of the function.

3.

C) neither the slope nor the intercept of the function.

4.

D) in both the slope and the intercept of the function.

4) Consider the demand functions: A) Qd = 250 - 2P B) Qd = 300 - 3P. Which of the demand functions reflects a higher level of consumer incomes? 1.

A) A

2.

B) B

3.

C) A and B reflect the same consumer incomes.

4.

D) More information is needed.

5) Holding all other factors constant, consumers demand more of a good the 1.

A) higher its price.

2.

B) lower its price.

3.

C) steeper the downward slope of the demand curve.

4.

D) steeper the upward slope of the demand curve.

6) As the price of a good increases, the change in the quantity demanded can be shown by 1.

A) shifting the demand curve leftward.

2.

B) shifting the demand curve rightward.

3.

C) moving down along the same demand curve.

4.

D) moving up along the same demand curve.

7) If the price of automobiles were to increase substantially, the demand curve for gasoline would most likely

1.

A) shift leftward.

2.

B) shift rightward.

3.

C) remain unchanged.

4.

D) become steeper.

8) If the price of automobiles were to decrease substantially, the demand curve for automobiles would most likely 1.

A) shift rightward.

2.

B) shift leftward.

3.

C) remain unchanged.

4.

D) become steeper.

9) If the price of automobiles were to decrease substantially, the demand curve for public transportation would most likely

1.

A) shift right.

2.

B) shift left.

3.

C) remain unchanged.

4.

D) remain unchanged while quantity demanded would change.

10) An increase in the demand curve for orange juice would be illustrated as a 1.

A) leftward shift of the demand curve.

2.

B) rightward shift of the demand curve.

3.

C) movement up along the demand curve.

4.

D) movement down along the demand curve.

11) The term "inverse demand curve" refers to 1.

A) a demand curve that slopes upward.

2.

B) expressing the demand curve in terms of price as a function of quantity.

3.

C) the demand for "inverses."

4.

D) the difference between quantity demanded and supplied at each price.

12) If the demand for oranges is written as Q = 100 - 5p, then the inverse demand function is 100.

A) Q = 5p - 100.

101.

B) Q = 20 - .2p.

102.

C) p = 20 - 5Q.

103.

D) p = 20 - .2Q.

13) To determine the total demand for all consumers, sum the quantity each consumer demands 1.

A) at a given price.

2.

B) at all prices and then sum this amount across all consumers.

3.

C) Both A and B will generate the same total demand.

4.

D) None of the above.

14) The above figure shows a graph of the market for pizzas in a large town. No pizzas will be demanded unless price is less than 1.

A) $0.

2.

B) $5.

3.

C) $12.

4.

D) $14.

15) The above figure shows a graph of the market for pizzas in a large town. If the price falls from $10 to $7 per pizza, the quantity of pizzas demanded will 20.

A) increase by 20.

21.

B) decrease by 30.

22.

C) increase by 30.

23.

D) decrease by 10.

16) If a yet-to-be released video game receives a positive review in a popular gaming magazine, 1.

A) the demand curve is expected to shift to the right.

2.

B) the demand curve is expected to shift to the left.

3.

C) the demand curve is not expected to change.

4.

D) for those who read the review, demand shifts to the left. For those who don't read the review, demand shifts to the right.

17) Which of the following cultural events likely increased the demand for the associated product? 1.

A) the banning of cigarette advertising on television

2. 3. 4.

B) the inclusion of Reese's Pieces in the movie E.T. C) increased environmental awareness about the impacts of sport utility vehicles (SUVs) D) concerns over "Mad Cow" disease in beef

18) The law of demand 1.

A) was passed by the 102nd U.S. Congress.

2.

B) is a natural law, much like the law of gravity.

3.

C) is considered a "law" in economics because of the overwhelming empirical evidence that supports its logic.

4.

D) is considered a "law" in economics in order to force economic models to operate fully.

19) If on Tuesday the perceived price of studying for an exam is $4 per hour but on Saturday the perceived price of studying for an exam is $10, the law of demand predicts 1.

A) more studying on Saturday and less on Tuesday.

2.

B) more studying on Tuesday and less on Saturday.

3.

C) the same amount of studying on Tuesday and Saturday.

4.

D) no studying on Tuesday or Saturday.

20) Which of the following would not change demand? 1.

A) the price of the product

2.

B) information about the product's health effects

3.

C) the income of the consumers

4.

D) the price of related products

21) If a demand curve shifts left, it implies 1.

A) as a group, consumers are willing and able to pay less for the product.

2.

B) as a group, consumers are willing and able to pay more for the product.

3.

C) government has regulated how many people can purchase the product.

4.

D) the profit motive of the firms is making the price too high.

22) If the price of gasoline were $5, many people would stop buying gasoline while others would continue to do so. This would indicate 1.

A) those who are buying gasoline value it at least $5 per gallon.

2.

B) those who are not buying gasoline value it more than $5 per gallon.

3.

C) only those who are extremely wealthy are buying gasoline.

4.

D) the price of gasoline needs to be regulated by the Federal Government.

23) Suppose the market demand curve for pizza can be expressed as QD = 100 - 2P + 3Pb, where QD is the quantity of pizza demanded, P is the price of pizza, and Pb is the price of a burrito. What is the relationship between burritos and pizza, from the point of view of consumers? 1.

A) They are independent.

2.

B) They are complements.

3.

C) They are substitutes.

4.

D) Not enough information to answer the question.

24) If the price is $5 and the quantity demanded is 100 units, then at a price of $10, the quantity demanded will be 1.

A) less than or equal to 100 units.

2.

B) greater than or equal to 100 units.

3.

C) less than or equal to 1000 units.

4.

D) equal to 100 units.

25) Assume the price of a movie is $10. Jenna demands 2 movies per week, Sam demands 3 movies per week, and Jordan demands 8 movies per week. From this information we can conclude that

1.

A) the market quantity demanded at a price of $10 is at least 13 movies per week.

2.

B) Jordan is obviously more wealthy than either Sam or Jeanna.

3.

C) Sam is irrational compared to Jenna or Jordan.

4.

D) the movie industry is unprofitable.

26) Assume Joe is only willing to pay $5 for a Ferrari sports car. 1.

A) Joe is not considered part of the demand for Ferraris.

2.

B) Joe most likely will not be sold a Ferrari.

3.

C) Joe is not considered rational.

4.

D) Joe's willingness to pay is not indicative of how much he values the Ferrari.

27) A downward sloping demand curve indicates that 1.

A) individuals all have the same valuation of the same product.

2.

B) individuals have different valuations of the same product.

3.

C) individuals have no valuations of a particular product.

4.

D) certain individuals are uninformed about certain aspects of the product.

28) The demand function for tickets to Ireland is expressed as Qd = 1600 - 0.005P + 2 PEngland + 3rain + 2Y, where P is the price for tickets to Ireland, PEngland is the price of tickets to England, rain is the inches of rain Ireland receives annually, and Y is annual household income. Assuming PEngland is $1000, rain is 30 inches per year, and income is 40,000, how much would the price of tickets to Ireland have to fall for consumers to be willing to buy one more trip per year? 1.

A) $15

2.

B) $20

3.

C) $50

4.

D) $200

29) If a good is not produced, then there is no demand for it.

30) Because people prefer name-brand pain-relieving drugs over store-brand painrelieving drugs, demand curves do not slope downward for pain-relieving drugs.

31) The quantity of a good that consumers demand depends only on the price of the good.

32) During the winter of 1997-1998, the northeastern United States experienced warmer than usual conditions. The price of home heating oil was less than it was during the previous winter, but people bought less home heating oil. This contradicts the Law of Demand.

33) Suppose an individual inverse demand curve is given as P = 2 - 1/2 qi, where qi is the quantity demanded by individual i. There are 50 individual consumers with this identical, individual inverse demand curve. Solve for the market demand curve.

34) Suppose the market demand curve for pizza can be expressed as QD = 100 - 2P + 3Pb, where QD is the quantity of pizza demanded, P is the price of a pizza, and Pb is the price of a burrito. What is the slope of this demand function, and what information does the slope provide?

35) Suppose the demand for a particular product can be expressed as Q = 100/p. Calculate the total amount spent on this good when p = 10, 20, and 50. Can you make a generalization about the mathematical form of this demand curve and consumer behavior in this market?

2.2 Supply

1) Suppose the demand curve for a good shifts rightward, causing the equilibrium price to increase. This increase in the price of the good results in 1.

A) a rightward shift of the supply curve.

2.

B) an increase in quantity supplied.

3.

C) a leftward shift of the supply curve.

4.

D) a downward movement along the supply curve.

2) An increase in the price of oil will 1.

A) shift the supply curve of oil to the left.

2.

B) shift the supply curve of oil to the right.

3.

C) leave the supply curve of oil unchanged.

4.

D) not enough information to answer the question.

3) The above figure shows a graph of the market for pizzas in a large town. No pizzas will be supplied unless the price is above 1.

A) $0.

2.

B) $5.

3.

C) $12.

4.

D) $14.

4) Supply curves 1.

A) slope upward.

2.

B) slope downward.

3.

C) are horizontal.

4.

D) can have many shapes.

5) Suppose there are 100 identical firms in the rag industry, and each firm is willing to supply 10 rags at any price. The market supply curve will be a 10.

A) vertical line where Q = 10.

11.

B) vertical line where Q = 100.

12.

C) vertical line where Q = 1000.

13.

D) horizontal line where Q = 1000.

6) The expression "increase in quantity supplied" is illustrated graphically as a 1.

A) leftward shift in the supply curve.

2.

B) rightward shift in the supply curve.

3.

C) movement up along the supply curve.

4.

D) movement down along the supply curve.

7) If the supply curve of a product changes so that sellers are now willing to sell 2 additional units at any given price, the supply curve will

1.

A) shift leftward by 2 units.

2.

B) shift rightward by 2 units.

3.

C) shift vertically up by 2 units.

4.

D) shift vertically down by 2 units.

8) The market supply curve is found by 1.

A) horizontally summing all individual supply curves.

2.

B) vertically summing all individual supply curves.

3.

C) Either A or B above since they both give the same answer.

4.

D) None of the above.

9) Technological innovation in the production of computers has led to

1.

A) a decrease in the quantity demanded for computers.

2.

B) a rightward shift of the supply curve for computers.

3.

C) a decrease in the quantity supplied of computers.

4.

D) None of the above.

10) Horizontally summing different supply curves assumes 1.

A) that individual firms cannot influence the good's price.

2.

B) that all firms operate in collusion.

3.

C) that only firms who volunteer are included in the summation.

4.

D) all firms produce the same amount of output.

11) If two firms produce the same product but have different supply curves, 1.

A) this would indicate that some other variable differs across the two firms.

2.

B) this would indicate that all variables are the same across the two firms.

3.

C) this would indicate that one or both of the firm's managers are misinformed.

4.

D) this would indicate a need for government regulation.

12) The supply curve is influenced by 1.

A) the income of consumers.

2.

B) the number of customers in the market.

3.

C) the prices of the inputs required to produce the product.

4.

D) the expectations of future profit.

13) The price of crude oil increased to $100 per barrel in early 2008. What would we expect to see happen to the supply of gasoline, which is produced using crude oil? 1.

A) The supply of gasoline will increase.

2.

B) The supply of gasoline will decrease.

3.

C) The supply of gasoline will stay the same because the government requires gasoline producers to meet statutory minimum production levels.

4.

D) The supply of gasoline will stay the same because of the profit motives of gasoline producers.

14) If ethanol production is expected to increase the price of corn, U.S. farmers will likely 1.

A) shift the supply of corn to the left.

2.

B) shift the supply of corn to the right.

3.

C) reduce the production of corn by sliding down the corn supply curve, all else equal.

4.

D) increase the production of corn by sliding up the corn supply curve, all else equal.

15) Suppose society wants to increase the number of teachers in society but doesn't want to pay teachers any more in salary and benefits. This could only occur by

1.

A) making it more expensive to become a teacher, perhaps by raising college tuition rates.

2.

B) making it less expensive to become a teacher, perhaps by offering subsidies to college tuition.

3.

C) making computers more powerful.

4.

D) increasing the income of student parents.

16) What is the total market supply, Qs, for luxury sports cars in the United States if the only three suppliers inverse supply functions are P = -10 + 5Qs1, P = 20 + 5Qs2, P = 5Qs3? 1.

A) P = 10 + 21Qs

2.

B) Qs = -2 +

3.

C) Qs = 6 +

4.

D) Qs = -2 +

17) The inverse supply curve of coffee beans equals P = -5 - 2rain + Qs, where rain equals the number of inches of rain per year. How much does quantity supplied change when rain increases from 30 inches to 40 inches per year? 40.

A) Qs decreases by 40.

41.

B) Qs increases by 40.

42.

C) Qs increases by 30.

43.

D) Qs decreases by 30.

18) The Law of Supply ensures that supply curves slope upward.

19) Suppose the following information is known about a market:

1.

Sellers will not sell at all below a price of $2.

2.

At a price of $10, any given seller will sell 10 units.

3.

There are 100 identical sellers in the market.

Assuming a linear supply curve, use this information to derive the market supply curve.

2.3 Market Equilibrium

1) Equilibrium is defined as a situation in which 1.

A) neither buyers nor sellers want to change their behavior.

2.

B) no government regulations exist.

3.

C) demand curves are perfectly horizontal.

4.

D) suppliers will supply any amount that buyers wish to buy.

2) Once an equilibrium is achieved, it can persist indefinitely because

1. 2. 3. 4.

A) shocks that shift the demand curve or the supply curve cannot occur. B) shocks to the demand curve are always exactly offset by shocks to the supply curve. C) the government never intervenes in markets at equilibrium. D) in the absence of supply/demand shocks no one applies pressure to change the price.

3) If price is initially above the equilibrium level, 1.

A) the supply curve will shift rightward.

2.

B) the supply curve will shift leftward.

3.

C) excess supply exists.

4.

D) all firms can sell as much as they want.

4) A competitive equilibrium is described by 1.

A) a price only.

2.

B) a quantity only.

3.

C) the excess supply minus the excess demand.

4.

D) a constant price and a quantity.

5) The above figure shows a graph of the market for pizzas in a large town. At a price of $14, there will be 1.

A) no pizzas supplied.

2.

B) equilibrium.

3.

C) excess supply.

4.

D) excess demand.

6) The above figure shows a graph of the market for pizzas in a large town. At a price of $5, there will be 1.

A) excess demand.

2.

B) excess supply.

3.

C) equilibrium.

4.

D) zero demand.

7) The above figure shows a graph of the market for pizzas in a large town. What are the equilibrium price and quantity? 1.

A) p = 8, Q = 60

2.

B) p = 7, Q = 40

3.

C) p = 7, Q = 70

4.

D) p = 10, Q = 40

8) The figure above shows a graph of the market for pizzas in a large town. What characterizes the equilibrium in this market? 7.

A) There is excess supply at the equilibrium price of $7.

8.

B) The government has selected the appropriate price for pizzas.

9.

C) The quantity supplied equals the quantity demanded.

10.

D) Supply equals demand.

9) The above figure shows a graph of a market for pizzas in a large town. At a price of $7, what is the amount of excess demand? 7.

A) 0; there is excess supply at $7.

8.

B) 20 units

9.

C) 30 units

10.

D) 10 units

10) The above figure shows a graph of a market for pizzas in a large town. At a price of $10, the market 1.

A) is not in equilibrium.

2.

B) has excess supply.

3.

C) does not have excess demand.

4.

D) All of the above.

11) The above figure shows three different supply-and-demand graphs. Which graph best represents the market for vacations on Mars? 1.

A) Graph A

2.

B) Graph B

3.

C) Graph C

4.

D) None of the above.

12) The above figure shows three different supply-and-demand graphs. Which graph best represents the market for workers at your nearest fast-food restaurant? 1.

A) Graph A

2.

B) Graph B

3.

C) Graph C

4.

D) None of the above.

13) The above figure shows three different supply-and-demand graphs. Which graph best represents the market for the air we are currently breathing? 1.

A) Graph A

2.

B) Graph B

3.

C) Graph C

4.

D) None of the above.

14) After tickets for a major sporting event are purchased at the official box office price, a market often develops whereby these tickets sell at prices well above the official box office price. Which of the following scenarios would NOT be able to explain this result? 1.

A) The official price was below equilibrium from the moment the tickets were available.

2.

B) Increased publicity causes the demand curve for the event to shift rightward.

3.

C) The event was not a sellout.

4.

D) Not everyone who wanted a ticket was able to buy one at the box office.

15) At equilibrium, quantity sold equals the quantity bought. This implies that 1.

A) to sell more, producers require more in payment than consumers are willing to pay.

2.

B) government regulation is necessary.

3.

C) to sell less would require a lower price but would yield greater profit.

4.

D) those who don't buy have been treated unfairly.

16) The supply and demand model assumes 1.

A) no buyer or seller can unilaterally influence the price of the product.

2.

B) each unit sold is sold at the same price.

3.

C) suppliers and demanders know the price of the product.

4.

D) All of the above.

17) A market equilibrium occurs 1.

A) only with government regulation.

2.

B) only because of the profit motive of firms.

3.

C) only because of the complacency of consumers.

4.

D) through the interaction of self-interested consumers and producers.

18) Lionfish is an aquatic invasive species in the southeastern U.S. and the Caribbean. Current removal policies focus on harvesting the lionfish for human consumption. However, a fishing license is required to fish in most southern states. Assume the supply of fishing licenses is Qs = 50 + 0.05L + 40P, where L = lionfish population and P = price of a fishing license, and the demand for fishing licenses is Qd = 1400 – 60P. What is the equilibrium price and quantity of fishing licenses when L = 1,000? 8.

A) P = $8.5, Q = 890

9.

B) P = $13, Q = 620

10.

C) P = $13.5, Q = 590

11.

D) P = $13.5, Q = 591

19) Lionfish is an aquatic invasive species in the southeastern U.S. and the Caribbean. Current removal policies focus on harvesting the lionfish for human consumption. However, a fishing license is required to fish in most southern states. Assume that the supply of fishing licenses is Qs = 50 + 0.05L + 40P, where and P = price of a fishing license, and the demand for fishing licenses is . What is the equilibrium price and quantity of fishing licenses when L = 10,000? 8.

A) P = $8.5, Q = 890

9.

B) P = $13, Q = 620

10.

C) P = $13.5, Q = 590

11.

D) P = $13.5, Q = 591

20) When a market is in disequilibrium consumers and producers change their behavior. As a result the market reaches equilibrium.

21) Suppose the market for potatoes can be expressed as follows:

Supply: QS = -20 + 10p Demand: QD = 400 - 20p

Solve for the equilibrium price and quantity.

22) Use supply-and-demand graphs to explain why parking is free at the suburban shopping mall but one typically must pay to park when shopping downtown. Answer:

23) Explain why the equilibrium price is called the market clearing price. t the equilibrium price, sellers want to sell the exact amount consumers want to buy. There is no excess demand or excess supply. The market is exactly cleared of all goods.

2.4 Shocking the Equilibrium

1) From the 1970s through the 1990s, the relative price of a college education has increased greatly. During the same time period, college enrollment has also increased. This evidence suggests that during this time period 1.

A) the demand curve for a college education has shifted leftward.

2.

B) the demand curve for a college education has shifted rightward.

3.

C) the supply curve for a college education has shifted leftward.

4.

D) the supply curve for a college education has shifted rightward.

2) The above figure shows four different markets with changes in either the supply curve or the demand curve. Which graph best illustrates the market for coffee after severe weather destroys a large portion of the coffee crop? 1.

A) Graph A

2.

B) Graph B

3.

C) Graph C

4.

D) Graph D

3) The above figure shows four different markets with changes in either the supply curve or the demand curve. Assuming coffee and tea are substitutes, which graph best illustrates the market for tea after severe weather destroys a large portion of the coffee crop?

1.

A) Graph A

2.

B) Graph B

3.

C) Graph C

4.

D) Graph D

4) The above figure shows four different markets with changes in either the supply curve or the demand curve. Which graph best illustrates the market for non-dairy coffee creamer after severe weather destroys a large portion of the coffee crop? 1.

A) Graph A

2.

B) Graph B

3.

C) Graph C

4.

D) Graph D

5) The above figure shows four different markets with changes in either the supply curve or the demand curve. Which graph best illustrates the market for computers after technological advances in making computers occur?

1.

A) Graph A

2.

B) Graph B

3.

C) Graph C

4.

D) Graph D

6) The above figure shows four different markets with changes in either the supply curve or the demand curve. Which graph best illustrates the market for computer manuals after technological advances in making computers occur? 1.

A) Graph A

2.

B) Graph B

3.

C) Graph C

4.

D) Graph D

7) The above figure shows four different markets with changes in either the supply curve or the demand curve. Which graph best illustrates the market for typewriters after technological advances in computerized word-processing software occur?

1.

A) Graph A

2.

B) Graph B

3.

C) Graph C

4.

D) Graph D

8) Suppose a market were currently at equilibrium. A rightward shift of the demand curve would cause 1.

A) an increase in price but a decrease in quantity.

2.

B) a decrease in price but an increase in quantity.

3.

C) an increase in both price and quantity.

4.

D) a decrease in both price and quantity.

9) Suppose a market were currently at equilibrium. A rightward shift of the supply curve would cause a(n) 1.

A) increase in price but a decrease in quantity.

2.

B) decrease in price but an increase in quantity.

3.

C) increase in both price and quantity.

4.

D) decrease in both price and quantity.

10) A rightward shift of the demand curve will lead to an 1.

A) increase in equilibrium price.

2.

B) excess demand at the old equilibrium price.

3.

C) increase in quantity supplied.

4.

D) All of the above.

11) A rightward shift of the supply curve will lead to a(n) 1.

A) decrease in equilibrium price.

2.

B) excess supply at the old equilibrium price.

3.

C) increase in quantity demanded.

4.

D) All of the above.

12) If the demand curve is vertical a rightward shift of the supply curve will lead to 1.

A) an increase in quantity supplied.

2.

B) an increase in quantity demanded.

3.

C) a decrease in quantity demanded.

4.

D) a decrease in price.

13) When two goods are substitutes, a shock that raises the price of one good causes the price of the other good to 1.

A) remain unchanged.

2.

B) decrease.

3.

C) increase.

4.

D) change in an unpredictable manner.

14) A drought in the Midwest will raise the price of wheat because of a 1.

A) leftward shift in the supply curve.

2.

B) rightward shift in the supply curve.

3.

C) leftward shift in the demand curve.

4.

D) rightward shift in the demand curve.

15) If pizza and tacos are substitutes, a decrease in the price of tacos would lead to a 1.

A) decrease in the demand curve for pizza.

2.

B) decrease in the quantity demanded of pizza.

3.

C) decrease in the price of pizza.

4.

D) All of the above.

16) If oranges were found to cure cancer, 1.

A) the equilibrium price of apples would likely fall.

2.

B) the equilibrium price of oranges would likely increase in the near term.

3.

C) the equilibrium quantity of oranges would likely increase.

4.

D) All of the above.

17) What can explain the fact that DVD players decreased in price during the 1990s even as more DVD players were being sold? 1.

A) The demand for DVDs was declining.

2.

B) The supply of DVD players was decreasing.

3.

C) The supply of DVD players was increasing more than the demand for DVD players.

4.

D) The demand for DVD players was increasing more than the supply of DVD players.

18) If government regulations increase the cost of producing gasoline while at the same time government regulations reduce the cost of driving a relatively inefficient sport utility vehicle (SUV), 1.

A) the supply of gasoline is expected to increase.

2.

B) the demand for gasoline is expected to decrease.

3. 4.

C) the price of gasoline is expected to increase, whereas the total consumption of gasoline can increase, decrease, or stay the same. D) the supply of gasoline is expected to decrease.

19) If the demand for high definition televisions increases and the supply of high definition televisions increases, then 1.

A) it is clear that prices will increase; the change in the quantity of televisions sold is ambiguous.

2.

B) it is clear that prices will decrease; the change in the quantity of televisions sold is ambiguous.

3.

C) it is clear that quantity sold will increase; the change in the price of televisions is ambiguous.

4.

D) it is clear that quantity sold will decrease; the change in the price of televisions is ambiguous.

20) Lionfish is an aquatic invasive species in the southeastern U.S. and the Caribbean. Current removal policies focus on harvesting the lionfish for human consumption. Assuming that lionfish is a substitute for snapper, what will likely happen to fishing efforts for lionfish if the price of snapper increases? 1.

A) Fishing efforts will increase.

2.

B) Fishing efforts will decrease.

3.

C) Fishing efforts will remain the same if supply is perfectly inelastic.

4.

D) A and C.

21) Lionfish is an aquatic invasive species in the southeastern U.S. and the Caribbean. Current removal policies focus on harvesting the lionfish for human consumption.

Assuming that lionfish is an inferior good, what will likely happen to fishing efforts if income increases? 1.

A) Fishing efforts will increase.

2.

B) Fishing efforts will decrease, especially when supply is relatively inelastic.

3.

C) Fishing efforts will decrease, especially when supply is relatively elastic.

4.

D) None of the above.

22) Tastes are starting to change for hybrid vehicles such that consumers are purchasing more hybrid vehicles and suppliers are choosing to produce more hybrid vehicles than they were five years ago. Due to the change in tastes, equilibrium price has fallen and equilibrium quantity has increased relative to the hybrid market from five years ago. What is the likely cause of this result? 1.

A) Supply increased by more than demand.

2.

B) Demand increased by more than supply.

3.

C) Demand and supply increased by the same amount.

4.

D) Only supply increased.

23) During a mild winter, the price of home heating oil is expected to be less than it would be during a normal winter. Answer: True. During a mild winter, people do not need to operate their furnace as often as in a normal winter. The demand for home heating oil lies to the left of where it would be under normal weather conditions. As a result, the price of oil falls.

24) What happens to the equilibrium price and quantity of coffee when there is a leftward shift of the supply curve for tea? Explain. Answer:

25) Suppose there is a linear downward-sloping demand curve and a linear upwardsloping supply curve for a good. The price of a substitute good increases and the price of an input to production also increases. Graph the original demand and supply curves, and the curves after the substitute good and input prices increase. How will the equilibrium price change after the substitute and input prices increase? Answer:

2.5 Equilibrium Effects of Government Interventions

1) Government actions can cause a 1.

A) shift in the supply curve.

2.

B) shift in the demand curve.

3.

C) reaction from firms in other countries.

4.

D) All of the above.

2) Municipalities that have adopted the policy of "rent control" typically set the rentals on certain apartments well below equilibrium. As a result, 1.

A) landlords have a difficult time finding tenants.

2.

B) prospective tenants have a difficult time finding available apartments.

3.

C) there is a surplus of apartments.

4.

D) All of the above.

3) When "rent controls" result in a shortage of housing, landlords 1.

A) use criteria other than price to allocate housing.

2.

B) lower the price to allocate the housing.

3.

C) attempt to attract renters.

4.

D) None of the above.

4) The above figure shows the market for apples. If a consumer group convinces the government to set a maximum price of $2 per pound, then 2.

A) 300 pounds of apples will be sold at $2.

3.

B) no apples will be supplied.

4.

C) no apples will be demanded.

5.

D) None of the above.

5) The above figure shows the market for apples. If apple farmers convince the government to set a minimum price of $4 per pound, then 4.

A) 100 pounds of apples will be sold at $4.

5.

B) no apples will be supplied.

6.

C) no apples will be demanded.

7.

D) None of the above.

6) The above figure shows the market for apples. If the government restricts output to no more than 300 pounds, then 3.

A) 300 pounds of apples will be sold at $3.

4.

B) 200 pounds of apples will be sold at $3.

5.

C) no apples will be sold.

6.

D) None of the above.

7) If a government-imposed price ceiling causes the observed price in a market to be below the equilibrium price, 1.

A) there will be excess demand.

2.

B) there will be excess supply.

3.

C) the curves will shift to make a new equilibrium at the regulated price.

4.

D) None of the above.

8) In the labor market, if the government imposes a minimum wage that is below the equilibrium wage, then 1.

A) workers who wish to work at the minimum wage will have a difficult time finding jobs.

2.

B) firms will hire fewer workers than without the minimum wage law.

3.

C) some workers may lose their jobs as a result.

4.

D) nothing will happen to the wage rate or employment.

9) Government prohibition of advertising cigarettes on television would most likely result in 1.

A) a rightward shift in the demand curve for cigarettes.

2.

B) a leftward shift in the demand curve for cigarettes.

3.

C) a rightward shift in the demand curve for television advertising time.

4.

D) no change in the market for either cigarette or television advertising.

10) The above figure shows a graph of the market for pizzas in a large town. Suppose that concern over dietary habits has led the government to impose a restriction that limits suppliers to produce no more than 40 pizzas. What will the price of pizza be as a result of this quota? 1.

A) $2

2.

B) $7

3.

C) $8

4.

D) $10

11) The above figure shows a graph of the market for pizzas in a large town. Suppose that concern over dietary habits has led the government to impose a restriction that limits suppliers to produce only 40 pizzas. As a result, for prices greater than $7, the 1.

A) supply curve is unchanged.

2.

B) supply curve is vertical.

3.

C) demand curve becomes vertical.

4.

D) demand curve becomes horizontal.

12) The above figure shows a graph of the market for pizzas in a large town. As a result of concern over the affordability of pizza, the government restricts sellers from charging a price over $7. As a result, the quantity of pizzas consumed will 1.

A) increase.

2.

B) decrease.

3.

C) remain unchanged.

4.

D) be indeterminable.

13) Agricultural price supports are 1.

A) floors.

2.

B) ceilings.

3.

C) quantity quotas.

4.

D) taxes.

14) Consumer groups tend to lobby for 1.

A) price floors.

2.

B) price ceilings.

3.

C) quantity quotas.

4.

D) taxes.

15) Producer groups tend to lobby for 1.

A) price floors.

2.

B) price ceilings.

3.

C) quantity quotas.

4.

D) taxes.

16) Quotas are most often supported by 1.

A) foreign producers.

2.

B) foreign consumers.

3.

C) domestic consumers.

4.

D) domestic producers.

17) Considering that the U.S. places a quota on imports of steel from South Korea, which of the following would NOT likely occur? 1. 2.

A) The price of steel in the United States would increase. B) The quantity of steel produced in the United States would increase or stay the same.

3.

C) The demand for steel in the United States will increase.

4.

D) The quantity demanded for steel in the United States will decrease.

18) Repealing a tariff will likely 1.

A) increase the quantity produced domestically, increase the quantity purchased domestically, and reduce the price charged domestically.

2.

B) reduce the quantity produced domestically, increase the quantity purchased domestically, and reduce the price charged domestically.

3.

C) reduce the amount produced abroad, reduce the price domestically, and reduce the quantity purchased domestically.

4.

D) increase the amount produced abroad, increase the price domestically, and reduce the quantity purchased domestically.

19) The fact that many people drive faster than the posted speed limit suggests that 1.

A) not all price floors are enforceable.

2.

B) not all price and quantity regulations are enforceable.

3.

C) individual drivers act irrationally.

4.

D) government regulation is utterly useless.

20) A price restriction that tells suppliers the minimum price they can sell their goods for is also known as 1.

A) a price ceiling.

2.

B) a quota.

3.

C) a price floor.

4.

D) deadweight loss.

21) Import quotas on steel tend to result in higher quantities of steel being sold at lower prices than would be observed in the absence of a quota. Answer: False. The opposite is true. Quotas restrict the supply curve. This reduces quantity and raises price.

22) Suppose the market for potatoes can be expressed as follows:

Supply: QS = -20 + 10p Demand: QD = 400 - 20p

If the government sets a maximum price of $10 per unit, what will be the quantity demanded and quantity supplied?

23) Usury laws place a ceiling on interest rates that lenders such as banks can charge borrowers. The interest rate is the price of a loan. Graph a binding usury law on the market for loans, and describe the effects of the law on the quantity of loans supplied and the quantity of loans demanded. Answer: See the above figure. The usury law will result in more loans being demanded and fewer loans being supplied.

24) Suppose the market for potatoes can be expressed as follows:

Supply: QS = -20 + 10p Demand: QD = 400 - 20p

Suppose the government restricts the quantity to 100 units. What will be the price of potatoes?

25) Use a supply-and-demand graph to predict what happens to sheet steel prices in the United States after quotas on Japanese and European sheet steel expire. Answer: See the above figure. The quota-restricted supply results in a higher price than the market equilibrium. When the quotas expire, the Japanese and European steel firms export into the United States based upon their own supply curves. The new equilibrium will be at a lower price.

26) Assume that the market for annual physical checkups is in equilibrium, and not everyone gets an annual physical checkup. What is the effect on price and quantity if a government regulation fixes price at the current level and requires everyone to get an annual physical checkup? Answer: See the above figure. Requiring everyone to get a physical would shift the demand curve rightward. With price remaining fixed, a shortage will result.

27) Suppose the market for a good is expressed as follows:

Inverse demand: P = 200 - 2Q Inverse supply: P = 2Q

What is the equilibrium if the government imposes a supply quota of 75 units? What is the equilibrium if the government imposes a supply quota of 25 units?

2.6 When to Use the Supply-and-Demand Model

1) It is appropriate to use the supply-and demand-model if, in a market, 1.

A) everyone is a price taker with full information about the price and quality of the good.

2.

B) firms sell identical products.

3.

C) costs of trading are low.

4.

D) All of the above.

2) Consumers and firms are known as price takers only if 1.

A) no market exists to determine the equilibrium price.

2.

B) they can set the market price.

3.

C) they cannot unilaterally affect the market price.

4.

D) excess demand exists.

3) Costs that pertain to finding a trading partner and making a trade are called 1.

A) transaction costs.

2.

B) transgression costs.

3.

C) consumption costs.

4.

D) transaction taxes.

4) It is appropriate to use the supply-and-demand model in which of the following markets? 1.

A) beer market

2.

B) car market

3.

C) wheat market

4.

D) market for breakfast cereal

5) One of the biggest benefits of the Internet auction site eBay 1.

A) is its control over prices.

2.

B) is that it increases the supply of products.

3.

C) is that it increases the demand for products.

4.

D) is that it reduces transaction costs for both sellers and buyers.

6) One reason the supply and demand model might not be appropriate to the healthcare industry? 1.

A) Consumers do not have full information.

2.

B) Providers do not know the demand for health-care services.

3.

C) Consumers do not know how to value their own health.

4.

D) The costs of finding a doctor are too low.

7) The power of the supply and demand model lies in its ability 1.

A) to generally predict how price and quantity will change with supply and demand shocks.

2.

B) to precisely predict the impact of government regulations on quantity and price.

3. 4.

C) to precisely determine the difference between price ceilings and price floors. D) to generally predict how profit motive impacts the distribution of goods and services.

8) Understanding the supply and demand model facilitates 1.

A) greed.

2.

B) charity.

3.

C) happiness.

4.

D) prediction.

9) The military is not a perfectly competitive market because 1.

A) there is only one buyer, i.e., the government, for military goods/services.

2.

B) there is imperfect information, i.e., intelligence branches.

3. 4.

C) there are heterogeneous goods/services because each military branch provides different goods/services. D) All of the above

10) The supply-and-demand model may not be appropriate in markets with large transaction costs. Answer: True. If the costs of finding a trading partner are high, no trades may occur, or trades may occur at a variety of prices.

11) If a large number of people decide to major in economics, the supply and demand model has little to say about the future wages of economists.

12) Explain why the supply-and-demand model should not be used to analyze the market for jeans.

Microeconomics, 7e (Perloff) Chapter 3 Applying the Supply-and-Demand Model

3.1 How Shapes of Supply and Demand Curves Matter

1) The change in price that results from a leftward shift of the supply curve will be greater if 1.

A) the demand curve is relatively steep than if the demand curve is relatively flat.

2.

B) the demand curve is relatively flat than if the demand curve is relatively steep.

3.

C) the demand curve is horizontal than if the demand curve is vertical.

4.

D) the demand curve is horizontal than if the demand curve is downward sloping.

2) The change in price that results from a rightward shift in demand will be greater if 1.

A) the supply curve is horizontal than if the supply curve is upward sloping.

2.

B) the supply curve is relatively steep than if the supply curve is relatively flat.

3.

C) the supply curve is upward sloping than if the supply curve is vertical.

4.

D) the supply curve is horizontal than if the supply curve is vertical.

3) If the demand curve for a good is horizontal and the price is positive, then a leftward shift of the supply curve results in

1.

A) a price of zero.

2.

B) an increase in price.

3.

C) a decrease in price.

4.

D) no change in price.

4) The above figure shows the supply and demand curves for rice in the U.S. and Japan. Assume there is no trade between the two countries. If bad weather causes the supply curves in each country to shift leftward by the same amount, then 1.

A) the price will increase in both countries.

2.

B) the price will decrease in both countries.

3.

C) the change in price cannot be determined.

4.

D) None of the above.

5) The above figure shows the supply and demand curves for rice in the U.S. and in Japan. Assume there is no trade between the two countries. If bad weather causes the supply curves in each country to shift leftward by the same amount, then

1.

A) the price will increase the same amount in both countries.

2.

B) the price will decrease the same amount in both countries.

3.

C) the price will increase more in Japan than in the U.S.

4.

D) the price will decrease more in Japan than in the U.S.

6) A vertical demand curve results in 1.

A) no change in quantity when the supply curve shifts.

2.

B) no change in price when the supply curve shifts.

3.

C) no change in the supply curve being possible.

4.

D) no change in quantity when the demand curve shifts.

7) A vertical demand curve for a particular good implies that consumers are

1.

A) sensitive to changes in the price of that good.

2.

B) not sensitive to changes in the price of that good.

3.

C) irrational.

4.

D) not interested in that good.

8) Consider the following products. Which of them has the flattest demand curve? 1.

A) insulin

2.

B) alcohol

3.

C) cigarettes

4.

D) butter

9) If the supply curve of cigarettes shifts to the left, quantity demanded for cigarettes 1.

A) will decrease substantially.

2.

B) will increase substantially.

3.

C) will slightly increase.

4.

D) will slightly decrease.

10) The current price floor in the agricultural lettuce market makes it such that price of lettuce is 25% higher than equilibrium price and 100 heads of lettuce are demanded. Assuming that the elasticity of demand for lettuce is -0.50, what would be the equilibrium price and quantity of lettuce if the government removed the current price floor? 1.

A) Price = 0.875, Quantity = 125

2.

B) Price = 1, Quantity = 112.5

3.

C) Price = $0.75, Quantity = 112.5

4.

D) Price = $0.75, Quantity = 125

11) As the supply curve shifts to the right, the increase in quantity demanded will not depend on the shape of the demand curve.

12) Explain why the shape of the demand curve will determine how a shock to the market equilibrium affects price and quantity. .

3.2 Sensitivity of Quantity Demanded to Price

1) The percentage change in the quantity demanded in response to a percentage change in the price is known as the 1.

A) slope of the demand curve.

2.

B) excess demand.

3.

C) price elasticity of demand.

4.

D) All of the above.

2) Suppose the inverse demand curve for a good is expressed as Q = 50 - 2p. If the good currently sells for $3, then the price elasticity of demand is 1.

A) -3 ∗ (2/50).

2.

B) -2 ∗ (50/3).

3.

C) -2 ∗ (3/44).

4.

D) -3 ∗ (44/2).

3) The above figure shows the demand curve for crude oil. If the market price is $10 a barrel, what is the price elasticity of demand? 1.

A) -.02

2.

B) -1

3.

C) -10

4.

D) -500

4) The above figure shows the demand curve for crude oil. The demand curve has unitary price elasticity when price equals 1.

A) $0.

2.

B) $1.

3.

C) $10.

4.

D) $20.

5) Suppose the demand function for a good is expressed as Q = 100 - 4p. If the good currently sells for $10, then the price elasticity of demand equals 1.

A) -1.5.

2.

B) -0.67.

3.

C) -4.

4.

D) -2.5.

6) If an increase in income results in a rightward parallel shift of the demand curve, then at any given price, the price elasticity of demand will have 1.

A) increased in absolute terms.

2.

B) decreased in absolute terms.

3.

C) remained unchanged.

4.

D) increased, decreased or stayed the same. It cannot be determined.

7) If the demand function for orange juice is expressed as Q = 2000 - 500p, where Q is quantity in gallons and p is price per gallon measured in dollars, then the demand for orange juice has a unitary elasticity when price equals 1.

A) $0.

2.

B) $1.

3.

C) $2.

4.

D) $4.

8) If the demand curve for orange juice is expressed as Q = 2000 - 500p, where Q is measured in gallons and p is measured in dollars, then at the price of $3, elasticity equals 1.

A) -0.33.

2.

B) -3.

3.

C) -9.

4.

D) -17.

9) If the demand for orange juice is expressed as Q = 2000 - 500p, where Q is measured in gallons and p is measured in dollars, then at the price of $3, the demand curve 1.

A) is elastic.

2.

B) has a unitary elasticity.

3.

C) is inelastic.

4.

D) is perfectly inelastic.

10) If the demand curve for comic books is expressed as Q = 10,000/p, then demand has a unitary elasticity 1.

A) only when p = 10,000.

2.

B) only when p = 100.

3.

C) always.

4.

D) never.

11) If the demand curve for a good always has unitary price elasticity, what does this imply about consumer behavior? 1.

A) Consumers do not react to a price change.

2.

B) Consumers will spend a constant total amount on the good.

3.

C) Consumers are irrational.

4.

D) Consumers do not obey the Law of Demand.

12) If the price elasticity of demand for a good is greater than one in absolute value, economists characterize that demand is 1.

A) elastic.

2.

B) inelastic.

3.

C) perfect.

4.

D) vertical.

13) If the price elasticity of demand for a good is less than one in absolute value, economists would characterize consumers of this good 1.

A) as not very sensitive to price.

2.

B) as not very sensitive to the quantity they demand.

3.

C) as very sensitive to price.

4.

D) as elastic.

14) If the price of orange juice rises 10%, and as a result the quantity demanded falls by 8%, the price elasticity of demand for orange juice is 1.

A) -1.25.

2.

B) inelastic.

3.

C) Both A and B above.

4.

D) Neither A nor B above.

15) If the price of orange juice rises 10%, and as a result the quantity demanded falls by 8%, the price elasticity of demand for orange juice is 1.

A) -1.25.

2.

B) elastic.

3.

C) Both A and B above.

4.

D) Neither A nor B above.

16) If the price of orange juice rises 10%, and as a result the quantity demanded falls by 8%, the price elasticity of demand for orange juice is 1.

A) -1.25.

2.

B) -80.0.

3.

C) -0.80.

4.

D) -10.0.

17) If the price of orange juice rises 10%, and as a result the quantity demanded falls by 10%, then one can conclude that the demand for orange juice 1.

A) is perfectly elastic.

2.

B) is inelastic.

3.

C) has a unitary elasticity.

4.

D) has a constant elasticity.

18) A horizontal demand curve for a good could arise because consumers 1.

A) are irrational.

2.

B) are not sensitive to price changes.

3.

C) view this good as identical to another good.

4.

D) have no equivalent substitutes for this good.

19) Which of the following is most likely to be true? 1.

A) Income elasticity of demand for fur coats exceeds that of oatmeal.

2.

B) Income elasticity of demand for oatmeal exceeds that of fur coats.

3.

C) Income elasticity of demand for fur coats equals that of oatmeal.

4.

D) It is not possible to make any prediction about relative income elasticities.

20) If a consumer doubles her quantity of ice cream consumed when her income rises by 25%, then her income elasticity of demand for ice cream is 8.

A) 8.0.

9.

B) 4.0.

10.

C) .25.

11.

D) .08.

21) If a good has an income elasticity of demand greater than 1, one might classify that good as 1.

A) a necessity.

2.

B) a luxury.

3.

C) unusual.

4.

D) inelastic.

22) The market demand for wheat is Q = 100 - 2p + 1pb, where pb is the price of barley. If the price of wheat is $2, the price elasticity of demand 1.

A) equals (-4/46).

2.

B) equals (-46).

3.

C) equals (-1).

4.

D) cannot be calculated without more information.

23) How will a decrease in price affect a firm's revenues? 1.

A) It depends on the price elasticity of demand.

2.

B) Revenues will stay the same.

3.

C) Revenues will decrease.

4.

D) Revenues will increase.

24) The cross price elasticity of demand for a good x is the percentage change in the quantity demanded of good x in response to a given percentage change in 1.

A) income.

2.

B) the price of good x.

3.

C) the price of good y.

4.

D) the quantity demanded of good y.

Question Status: Revised

25) The market demand for wheat is Q = 100 - 2p + 1pb, where pb is the price of barley. The cross price elasticity of demand for wheat with respect to barley 1.

A) cannot be calculated from just the information provided.

2.

B) is negative.

3.

C) suggests that wheat and barley are complements.

4.

D) equals 1.

26) The market demand for wheat is Q = 100 - 2p + 1pb + 2Y. If the price of wheat, p, is $2, and the price of barley, pb, is $3, and income, Y, is $1000, the income elasticity of wheat is 1.

A) 2 ∗ (1000/2099).

2.

B) 2.

3.

C) 1/2 ∗ (1000/2099).

4.

D) cannot be calculated from the information provided.

27) The cross price elasticity of demand between two goods will be positive if 1.

A) the two goods are complements.

2.

B) the two goods are substitutes.

3.

C) the two goods are luxuries.

4.

D) one of the goods is a luxury and the other is a necessity.

28) On a linear demand curve, the lower the price,

1.

A) the less elastic is demand.

2.

B) the more elastic is demand.

3.

C) the elasticity equals -1.

4.

D) the elasticity equals zero.

29) The price elasticity of demand 1.

A) depends on the units in which quantity is measured.

2.

B) depends on the units in which price is measured.

3.

C) depends on the units in which money is measured.

4.

D) is independent of the units in which quantity and price are measured.

30) If a one percent increase in the population leads to a five percent increase in the quantity sold, an economist would claim 1.

A) the good is elastic with respect to population.

2.

B) the good is inelastic with respect to population.

3.

C) the good is a fad.

4.

D) consumers are misinformed about the quality of the product.

31) The number of vehicle types available in the United States has increased dramatically over the past thirty years. Everything else equal, this would make 1.

A) the demand for individual vehicle types to become less elastic.

2.

B) the demand for individual vehicle types to become more elastic.

3.

C) the demand for all vehicle types to become unitary elastic.

4.

D) the demand for low quality vehicle types to become less elastic.

32) One reason the U.S. government might subsidize research of an alternative to crude-oil based gasoline is that

1.

A) more substitutes will reduce the price of crude-oil based gasoline.

2.

B) more substitutes will increase the elasticity of crude-oil based gasoline.

3. 4.

C) more substitutes will reduce the impact of supply shocks on the price of crudeoil based gasoline. D) Any or all of the above.

33) If the cross price elasticity of two goods is -3.5, then 1.

A) these two products are relatively elastic substitutes.

2.

B) these two products are relatively inelastic substitutes.

3.

C) these two products are relatively elastic complements.

4.

D) these two products are relatively inelastic complements.

34) The elasticity of demand for employees is -0.50. It is also estimated that the existing minimum wage (price floor) has increased the raise the wage by 25% above equilibrium wage. How much would the employment change if the price floor was eliminated?

12.

A) Employment would decrease by 12.5%.

13.

B) Employment would increase by 12.5%.

14.

C) Employment would decrease by 25%.

15.

D) Employment would increase by 25%.

35) The current price floor in the agricultural lettuce market makes it such that the price of lettuce is 25% higher than the equilibrium price and that 100 heads of lettuce are demanded. Assuming that the elasticity of demand for lettuce is -0.50, how much would revenue (P ∗ Q) change for the lettuce company if the government removed the current price floor? 15.

A) Revenue will increase by $15.60.

16.

B) Revenue will decrease by $15.60.

17.

C) Revenue will increase by $40.60.

18.

D) Revenue will increase by $ 9.40.

36) An agricultural corn market faces a positive supply shock due to a beneficial rainy season and the use of new genetically modified seeds. As a result, farmers face the largest crop harvest in decades. Which answer below explains how a farm could actually go bankrupt under this scenario. 1.

A) The elasticity of supply for corn is elastic such that a positive shock reduces total revenue.

2.

B) The demand for corn is inelastic such that a positive supply shock reduces total revenue.

3.

C) An inelastic demand curve will cause revenue to fall because price decreases by more than the increase in quantity demanded.

4.

D) B and C

37) In the case of a linear demand curve, demand becomes more price elastic as price increases.

38) When comparing elasticities between two different linear demand curves, the curve that is flatter has greater price elasticity at every given price.

39) Because demand curves slope downward according to the Law of Demand, the price elasticity of demand is a negative number.

40) Assume the market demand for wheat may be written as

Q = 45 - 2p + 0.3Y + 1pb where Y refers to income and pb refers to the price of barley. Assuming that wheat and barley both sell for $1, and income is $20, calculate the price elasticity, cross price elasticity and income elasticity for wheat.

41) Which good would you expect to have a greater price elasticity: a gallon of gasoline sold at a specific gasoline station on Main Street in Phoenix, a gallon of gasoline sold in Phoenix, or a gallon of gasoline sold in Arizona? Why?

42) The price elasticity of demand for gasoline is estimated to be -0.2. Two million gallons are sold daily at a price of $3. Use this information to calculate a demand curve for gasoline assuming it is linear.

43) The above figure shows three demand curves labeled D1, D2, and D3. Rank these three demand curves in terms of elasticity at a price of c.

44) Explain why when the demand curve for a good is elastic, a one percent reduction in the price of the good will increase a consumer's expenditure on the good.

45) Explain why the price elasticity of demand changes along a linear demand curve.

3.3 Sensitivity of Quantity Supplied to Price

1) The percentage change in the quantity supplied in response to a percentage change in the price is known as the 1.

A) slope of the supply curve.

2.

B) excess supply.

3.

C) price elasticity of supply.

4.

D) All of the above.

2) The supply curve for tickets for a sporting event 1.

A) is perfectly inelastic.

2.

B) is vertical.

3.

C) has a price elasticity of zero.

4.

D) All of the above.

3) As prices change, the elasticity of supply describes the movement 1.

A) of a shift in the supply curve.

2.

B) of the equilibrium price.

3.

C) along the supply curve.

4.

D) from a necessity to a luxury good.

4) In the late 1980s, the health benefits of oat bran were widely advertised. If the price of oats increased 50%, causing the quantity of oats supplied to increase by 40%, then the price elasticity of supply was 1.

A) 1.25.

2.

B) -1.25.

3.

C) -0.80.

4.

D) 0.80.

5) If the supply curve for orange juice is estimated to be Q = 40 + 2p, then, at a price of $2, the price elasticity of supply is 1.

A) .01.

2.

B) .09.

3.

C) 1.

4.

D) 11.

6) If the supply curve for orange juice is estimated to be Q = 40 + 2p, then 1.

A) supply is price elastic at all prices.

2.

B) supply is price inelastic at all prices.

3.

C) supply is elastic only at prices below 20.

4.

D) No general statements about price elasticity of supply can be made.

7) The supply of movie tickets at one theater's box office for this Saturday's 4:30 show of a new movie is

1.

A) perfectly elastic until all seats are filled.

2.

B) unit elastic.

3.

C) perfectly inelastic.

4.

D) elastic.

8) A vertical supply curve exhibits 1.

A) a constant elasticity of supply.

2.

B) a perfectly inelastic supply curve.

3.

C) Both A and B are true.

4.

D) None of the above.

9) A given supply curve has a zero intercept. At the current equilibrium price the price elasticity of supply equals

1.

A) 1.

2.

B) 0.

3.

C) 2.

4.

D) Not enough information.

10) The price elasticity of supply when the supply curve is Q = 5 is 5.

A) 5.

6.

B) perfectly inelastic.

7.

C) perfectly elastic.

8.

D) Cannot be calculated from the information provided.

11) As the demand for corn increases to provide input for ethanol production, what is expected to happen to the price elasticity of corn supply? 1.

A) It will decrease.

2.

B) It will become zero.

3.

C) It will increase.

4.

D) It will not change.

12) The duration of the "short-run" 1.

A) is one year.

2.

B) is the same for all goods.

3.

C) depends on the relative short-run elasticity of demand and supply for the good.

4.

D) depends on how long it takes consumers or firms to adjust for a particular good.

13) Electricity accounts for almost 20% of the cost of making steel. A 10% increase in electricity prices results in steel firms decreasing production and thereby demanding 5% less electricity. Over many years, technological innovations can change the way steel firms make steel and reduce the industry's energy requirements. This suggests that the steel industry's short-run elasticity of demand for electricity is probably

1.

A) less than one in absolute terms in the short run.

2.

B) less than its long-run elasticity of demand for electricity.

3.

C) Both A and B above.

4.

D) Neither A nor B above.

14) Why is the supply of oil more price elastic in the long run? 1.

A) New deposits are found.

2.

B) Better extraction technology is developed.

3.

C) Firms have the ability to change the amount of all inputs.

4.

D) All of the above.

15) In the mid-1980s, the salaries of accounting professors with Ph.D.s increased dramatically. This resulted in an increase in enrollments in Ph.D. accounting programs. Since a Ph.D. degree in accounting may take at least four years to complete, the shortrun elasticity of supply of accounting professors is

1.

A) greater than the long-run-elasticity of supply.

2.

B) less than the long-run elasticity of supply.

3.

C) equal to the long-run elasticity of supply.

4.

D) equal to the short-run elasticity of demand.

16) The rising price of oil has made it feasible to extract oil out of oily sand in Canada. Concerning the oil market this is an example of 1.

A) a higher price elasticity of supply in the long run.

2.

B) a higher price elasticity of supply in the short run.

3.

C) a higher price elasticity of demand in the short run.

4.

D) an inelastic long-run supply of oil.

17) Which of the following goods probably has the lowest (absolute value) short-run price elasticity of demand?

1.

A) fresh fruit

2.

B) frozen dinners

3.

C) cars

4.

D) refrigerators

18) The short-run elasticity of supply is less than the long-run elasticity of supply 1.

A) because consumers' tastes and preferences change in the long run but not in the short run.

2.

B) because producers can adjust the amount of machinery in the long run but not in the short run.

3.

C) only for durable goods.

4.

D) only for non-durable goods.

19) Relative to the short-run demand for gasoline, the long-run demand for gasoline is

1.

A) probably more elastic since people need time to change automobiles and driving habits.

2.

B) probably less elastic since people need time to change automobiles and driving habits.

3.

C) probably more elastic because people can hoard this good.

4.

D) probably less elastic because people cannot store this good.

20) As more people quit smoking in the United States, what is expected to happen to the price elasticity of supply of cigarettes? 1.

A) It will decrease.

2.

B) It will increase.

3.

C) It can increase or decrease.

4.

D) It will not change.

21) Some environmental groups are on record suggesting that the price of gasoline should be much higher than it was in the early 1990s. Why might they say this?

1.

A) They own stock in oil companies.

2.

B) They anticipate that the longer the price is high, the more elastic the response by consumers will be.

3.

C) They anticipate that the longer the price is high, the less elastic the response by consumers will be.

4.

D) They anticipate that higher prices will reduce the price elasticity of supply of oil.

22) Consider Sam and Linda both drive a relatively inefficient sport utility vehicle (SUV). Sam has a lease that doesn't expire for three years whereas Linda owns her sport utility vehicle free and clear. If the price of gasoline was to increase by fifty percent, which of these statements is most likely true? 1.

A) Linda will have a less elastic response than Sam.

2.

B) Sam will have a less elastic response than Linda.

3.

C) Sam and Linda will have identically elastic responses.

4.

D) Sam will have a more elastic response than Linda.

23) The elasticity of supply of rental units in New York City is estimated to be about 0.10. Current price restrictions (price floors) are estimated to decrease the price of rental units by 10% below equilibrium price. By how much would price and quantity supplied change if the price floors were removed from the rental unit market in New York City? 1.

A) Price will increase by 10% and quantity supplied will increase by 1%.

2.

B) Price will decrease by 10% and quantity supplied will increase by 1%.

3.

C) Price will increase by 10% and quantity supplied will decrease by 1%.

4.

D) Price and quantity supplied will increase by 10%.

24) Suppliers with a high supply elasticity will bear a ______ tax incidence, while suppliers with a low supply elasticity will bear a ______ tax incidence. 1.

A) lower; higher

2.

B) higher; lower

3.

C) lower or no; higher or full

4.

D) A and C

25) If a linear supply curve has a zero intercept, the elasticity of supply is always unitary.

26) For all goods, the long run demand curve is always more elastic than the short run demand curve.

27) The short-run price elasticity of demand for refrigerators is relatively inelastic.

28) Explain whether you would expect the elasticity of supply to be highly elastic or inelastic for fresh cut flowers and why.

29) The National Association of Business Schools recently required that all business schools must hire three additional people with Ph.D. degrees in English literature. What is the immediate effect on the salaries of people with Ph.D.s in English literature? What will be the effect after ten years? Answer:

30) Explain why short-run demand for frozen fish sticks may be more price elastic in the short run than in the long run.

3.4 Effects of a Sales Tax

1) Suppose the supply curve and the demand curve both have unitary elasticity at all prices. The price increase to consumers resulting from a specific tax of $1 imposed on sellers will be 1.

A) $1.

2.

B) 50 cents.

3.

C) zero.

4.

D) impossible to calculate without knowing the slope of the supply curve.

2) For a given positively sloped supply curve, the price increase to consumers resulting from a specific tax imposed on sellers will be 1.

A) greater the more price elastic demand is.

2.

B) greater the less price elastic demand is.

3.

C) equal to the entire tax when demand is perfectly elastic.

4.

D) equal to half of the tax whenever demand is unit elastic.

3) A specific tax on sellers will 1.

A) shift the demand curve to the right.

2.

B) shift the demand curve to the left.

3.

C) shift the supply curve to the right.

4.

D) shift the supply curve to the left.

4) Consumers will always pay the entire amount of a specific tax whenever 1.

A) demand is perfectly inelastic.

2.

B) supply is perfectly elastic.

3.

C) Both A and B above.

4.

D) Either A or B above but not at the same time.

5) If a government wants to maximize revenues from a tax it should 1.

A) impose it on sellers.

2.

B) impose it on consumers.

3.

C) choose a good with a relatively elastic demand.

4.

D) choose a good with a relatively inelastic demand.

6) If the demand curve for a good is unit price elastic and the supply curve is perfectly price elastic, a $1 specific tax imposed on the sellers of this good will 1.

A) shift the supply curve up vertically by $1.

2.

B) shift the demand curve down vertically by $1.

3.

C) not raise price at all.

4.

D) cause price to increase but by less than $1.

7) Suppose the demand curve for a good is downward sloping and the supply curve is upward sloping. At the market equilibrium, if demand is more elastic than supply in absolute value, a $1 specific tax will 1.

A) raise the price to consumers by 50 cents.

2.

B) raise the price to consumers by less than 50 cents.

3.

C) raise the price to consumers by more than 50 cents.

4.

D) raise the price to consumers by $1.

8) Suppose the demand curve is perfectly inelastic and the supply curve is upward sloping. The price sellers receive after a specific tax is imposed on sellers 1.

A) is less than before the tax.

2.

B) is higher than before the tax.

3.

C) is unchanged.

4.

D) depends on the supply elasticity.

9) Suppose the demand curve for movie tickets has unitary price elasticity and the supply curve is perfectly price elastic. If 3 million tickets are currently sold at a price of $5, approximately how much tax revenue could the government generate from a $1 specific tax? 1.

A) $18 million

2.

B) $3 million

3.

C) $2.7 million

4.

D) $1.5 million

10) In the case of a specific tax, tax incidence is independent of who pays 1.

A) only when supply and demand elasticities are not constant.

2.

B) only when the tax is collected from consumers.

3.

C) in most but not all cases.

4.

D) in all cases.

11) If the government decides to levy an ad valorem tax on product with a perfectly inelastic supply. The consumers tax incidence will be 1.

A) 0

2.

B) 1

3.

C) .5

4.

D) Cannot be determined.

12) In the case of a specific tax the resulting price received by producers depends on 1.

A) who pays the tax.

2.

B) the price elasticity of supply.

3.

C) the price elasticity of demand.

4.

D) All of the above.

13) The tax incidence of a specific tax or ad valorem tax is influenced by 1.

A) who pays the tax.

2.

B) the amount of the tax.

3.

C) the price elasticities of supply and demand.

4.

D) All of the above.

14) Which of the following is an example of an ad valorem tax? 1.

A) 5% of price.

2.

B) 5% of quantity sold.

3.

C) $0.50 per unit sold.

4.

D) Government regulation.

15) Who will bear the burden of a $0.05 tax placed on soda suppliers (consumer or seller) in a soda market where Qd = 225-10P and Qs = 50 + 15P? 1.

A) Consumers pay $0.30 of the tax, bearing the burden.

2.

B) Consumers pay $0.25 of the tax, bearing the burden.

3.

C) Sellers pay $0.20 of the tax and bear the burden.

4.

D) Sellers pay all of the tax and bear the burden.

16) To prevent obesity, the government may establish a tax on high caloric foods, such as twinkies. A twinkie tax will have the smallest impact on quantity demanded when the demand curve for twinkies is 1.

A) perfectly elastic.

2.

B) perfectly inelastic.

3.

C) more elastic than the supply curve.

4.

D) both A and B.

17) If the supply curve is perfectly inelastic and the demand curve is a downward sloping straight line, what is the effect of a consumer ad valorem tax on equilibrium price and quantity? 1.

A) Price remains unchanged and quantity increases.

2.

B) Price decreases and quantity increases.

3.

C) Price decreases and quantity remains unchanged.

4.

D) Price and quantity decrease.

18) The government heavily subsidizes the production of ethanol to encourage the purchase of ethanol over oil, a less environmentally friendly form of energy. Given that the supply elasticity of ethanol, η, is estimated to be about 0.13, what would the elasticity of demand, ε, have to be for consumers to receive at least half of the subsidy, and therefore encourage ethanol consumption? 1.

A) ε must be less than .

2.

B) ε must be greater than .

3.

C) ε must be more inelastic than the elasticity of supply.

4.

D) Both A and C.

19) Only in the case of perfectly inelastic demand will consumers pay the full amount of a specific tax or ad valorem tax. .

20) Government revenue from an excise tax of a given amount is greater when demand is relatively inelastic than when it is relatively elastic.

21) Explain why a tax increase on cigarettes in one state might not lead to a substantial price increase for all consumers in that state.

22) Suppose the market for grass seed can be expressed as

Demand: QD = 100 - 2p Supply: QS = 3p

At the market equilibrium, calculate the price elasticities of supply and demand. Use these numbers to predict the change in price resulting from a specific tax. t p = 20 Q = 60, e = -2 ∗ (20/60) = -0.67. n = 3 ∗ (20/60) = 1. The change in price resulting from a specific tax = [n/(n - e)] ∗ tax = [1/1.67] ∗ tax = 0.6 ∗ tax.

23) Suppose the market for grass seed can be expressed as

Demand: QD = 100 - 2p Supply: QS = 3p

If government imposes a $5 specific tax to be collected from sellers, what is the price consumers will pay? How much tax revenue is collected? What fraction is paid by sellers?

24) Suppose the market for grass seed can be expressed as

Demand: QD = 100 - 2p Supply: QS = 3p

If government imposes a 10% ad valorem tax to be collected from sellers, what is the price consumers will pay? How much tax revenue is collected?

25) Suppose the market for grass seed is expressed as

Demand: QD = 100 - 2p

Supply: QS = 3p

Price elasticity of supply is constant at 1. If the supply curve is changed to Q = 8p, price elasticity of supply is still constant at 1. Yet, with the new supply curve, consumers pay a larger share of a specific tax. Why?

26) Suppose the market for grass seed can be expressed as

Demand: QD = 100 - 2p Supply: QS = 3p

Price elasticity of supply is constant at 1. If the demand curve is changed to Q = 10 - . 2p, price elasticity of demand at any given price is the same as before. Yet, the incidence of a tax falling on consumers will be higher. Why?

27) Suppose the market for grass seed can be expressed as

Demand: QD = 200 - 5p Supply: QS = 40 + 5p

If the government collects a $5 specific tax from sellers, how much will the quantity demanded change from the amount demanded before the tax? What price will consumers pay after the tax? What price will sellers receive after the tax? What is the tax revenue?

Microeconomics, 7e (Perloff) Chapter 4 Consumer Choice

4.1 Preferences

1) An indifference curve represents bundles of goods that a consumer 1.

A) views as equally desirable.

2.

B) ranks from most preferred to least preferred.

3.

C) refers to any other bundle of goods.

4.

D) All of the above.

2) The assumption of completeness means that 1.

A) the consumer can rank all possible consumption bundles.

2.

B) more of a good is always better.

3.

C) the consumers can rank all affordable consumption bundles.

4.

D) all preferences conditions are met.

3) If a consumer prefers Apples to Bananas and prefers Bananas to Citrus Fruit, in order to satisfy assumptions about preferences she has to prefer 1.

A) Bananas to Apples.

2.

B) Citrus Fruit to Bananas.

3.

C) Apples to Citrus Fruit.

4.

D) Citrus Fruit to Apples.

4) The principle that "More is better" results in indifference curves 1.

A) sloping down.

2.

B) not intersecting.

3.

C) reflecting greater preferences the further they are from the origin.

4.

D) All of the above.

5) If two indifference curves were to intersect at a point, this would violate the assumption of 1.

A) transitivity.

2.

B) completeness.

3.

C) Both A and B above.

4.

D) None of the above.

6) Indifference curves that are thick violate

1.

A) the assumption of transitivity.

2.

B) the assumption that more is better.

3.

C) the assumption of completeness.

4.

D) none of the assumptions.

7) A consumer's willingness to trade one good for another can be expressed by the consumer's 1.

A) indifference curve.

2.

B) marginal rate of substitution.

3.

C) Both A and B above.

4.

D) None of the above.

8) Convexity of indifference curves implies that consumers are willing to 1.

A) give up more "y" to get an extra "x" the more "x" they have.

2.

B) give up more "y" to get an extra "x" the less "x" they have.

3.

C) settle for less of both "x" and "y".

4.

D) acquire more "x" only if they do not have to give up any "y".

9) Measuring "y" on the vertical axis and "x" on the horizontal axis, convexity of indifference curves implies that the MRS of "y" for "x" 1.

A) is decreasing as "x" increases.

2.

B) is increasing as "x" increases.

3.

C) is constant as "x" increases.

4.

D) cannot be calculated for large levels of "x".

10) Diminishing marginal rate of substitution can be seen when indifference curves 1.

A) cross.

2.

B) are convex.

3.

C) are downward sloping.

4.

D) become flatter as we move down and to the right.

11) If two goods are perfect substitutes, then the indifference curves for those two goods would be 1.

A) upward sloping and concave to the origin.

2.

B) downward sloping and convex to the origin.

3.

C) downward sloping and straight.

4.

D) L-shaped.

12) The indifference curves for left shoes and right shoes would most likely be 1.

A) upward sloping and concave to the origin.

2.

B) downward sloping and convex to the origin.

3.

C) downward sloping and straight lines.

4.

D) L-shaped.

13) The statement "There is no accounting for taste" implies 1.

A) individuals all have the same preferences.

2.

B) individuals all have different cardinal preferences but the same ordinal preferences.

3.

C) individuals all have different ordinal preferences but the same cardinal preferences.

4.

D) individuals all have different ordinal and cardinal preferences.

14) Economists use a preference map to illustrate that 1.

A) more is better than less.

2.

B) preferences are transitive.

3.

C) preferences are complete.

4.

D) All of the above.

15) Indifference curves cannot intersect. Answer:

16) Indifference curves cannot ever be concave for two goods.

17) Indifference curves for perfect substitutes must be parallel lines with a slope of negative one.

18) Indifference curves on the same indifference map can have different shapes.

19) Lisa views pizzas and burritos as goods. If she prefers a bundle of 4 burritos and 4 pizzas to a bundle of 4 burritos and 5 pizzas, which property of consumer preference is violated? What change in the assumptions could lead a rational consumer to prefer the first bundle?

20) Explain why most indifference curves are convex.

21) Draw the indifference curves for rock concerts and food for each of the following:

(a) a typical 17-year-old (b) a typical 75-year-old Answer: See the above figure. These graphs assume that a typical 17-year-old would enjoy both food and rock concerts. The 75-year-old might find the rock concerts neutral or even bad.

22) Draw the indifference curves for nickels and dimes. Would they ever have a nonconstant slope? Explain. Answer:

4.2 Utility

1) Utility is the set of numerical values that 1.

A) yields an absolute level of pleasure from a bundle of goods.

2.

B) reflects the relative ranking of various bundles of goods.

3.

C) describes how much more a consumer prefers one bundle to another.

4.

D) yields a cardinal ranking of bundles.

2) If two bundles are on the same indifference curve, then 1.

A) the consumer derives the same level of utility from each.

2.

B) the consumer derives the same level of ordinal utility from each but not the same level of cardinal utility.

3.

C) no comparison can be made between the two bundles since utility cannot really be measured.

4.

D) B and C.

3) If the utility function (U) between food (F) and clothing (C) can be represented as U = , the marginal utility of food equals 1.

A) .

2.

B) .

3.

C) 1/2.

4.

D) 1/2.

4) If the utility function (U) between food (F) and clothing (C) can be represented as U = , the marginal utility of food 1.

A) is not positive.

2.

B) does not diminish as food increases.

3.

C) is not affected by the amount clothing.

4.

D) increases as one obtains more clothing.

5) If the utility function (U) between food (F) and clothing (C) can be represented as U = , the marginal rate of substitution of clothing for food equals 1.

A) -C/F.

2.

B) -F/C.

3.

C) -.

4.

D) -.

6) If Fred's marginal utility of pizza equals 10 and his marginal utility of salad equals 2, then 1.

A) he would give up 5 pizzas to get the next salad.

2.

B) he would give up 5 salads to get the next pizza.

3.

C) he will eat five times as much pizza as salad.

4.

D) he will eat five times as much salad as pizza.

7) If Fred's marginal rate of substitution of salad for pizza equals 5, then 1.

A) he would give up 5 pizzas to get the next salad.

2.

B) he would give up 5 salads to get the next pizza.

3.

C) he will eat five times as much pizza as salad.

4.

D) he will eat five times as much salad as pizza.

8) Adrian's total utilities of two consumption bundles are 50 and 100. This implies that 1.

A) the consumer prefers the first bundle.

2.

B) the consumer prefers the second bundle.

3.

C) the consumer likes the second bundle twice as much.

4.

D) the consumer likes the first bundle twice as much.

9) If the utility for two goods "x" and "y" is measured as U = x + y, then it can be concluded that 1.

A) "x" and "y" are perfect substitutes.

2.

B) "x" and "y" are perfect complements.

3.

C) "x" and "y" are both bads.

4.

D) the indifference curves on the x,y graph will be upward sloping.

10) Assuming that good "x" is measured on the x-axis and good "y" is measured on the y-axis, if the utility for the two goods "x" and "y" can be measured as U = x, then it can be concluded that 1.

A) "x" and "y" are perfect complements.

2.

B) "y" is a "bad".

3.

C) the indifference curves on the x,y graph are upward sloping.

4.

D) the indifference curves on the x,y graph are vertical.

11) Assuming that good "x" is measured on the x-axis and good "y" is measured on the y-axis, if the utility for the two goods "x" and "y" can be measured as U = y, then it can be concluded that 1.

A) "x" and "y" are perfect complements.

2.

B) "x" is a "bad".

3.

C) the indifference curves on the x,y graph are upward sloping.

4.

D) the indifference curves on the x,y graph are horizontal.

12) If two goods, "x" and "y", are perfect substitutes, then which of the following best represents the utility function for the two goods? 1.

A) U = x + y

2.

B) U = x ∗ y

3.

C) U = x2+ y2

4.

D) Any of the above.

13) If Johnny likes homework (H) but hates exercise (E), which of the following might best represent his utility function for homework and exercise? 1.

A) U = H + E

2.

B) U = H/E

3.

C) U = H2+

4.

D) U = H2×

14) Clifford lives by the motto "Eat, drink and be merry today, for tomorrow doesn't matter." If today's consumption is measured on the horizontal axis and tomorrow's consumption is measured on the vertical axis, Clifford's indifference curves 1.

A) are horizontal straight lines.

2.

B) are vertical straight lines.

3.

C) show decreasing utility as one moves upward.

4.

D) cannot be determined from the information given.

15) Clifford lives by the motto "Eat drink and be merry today, for tomorrow doesn't matter." If today's consumption is represented by "x" and tomorrow's consumption is represented by "y", then which of the following best represents Clifford's utility function? 1.

A) U = x - y

2.

B) U = x/y

3.

C) U = x

4.

D) U = y

16) Some people view cosmetic surgery (c) and facials (f) as perfect substitutes as measured by the utility function U(c,f) = 5c + 10f. What is the marginal utility of cosmetic surgery? 5.

A) MU = 5.

6.

B) MU = 10.

7.

C) MU = 15.

8.

D) None of the above.

17) People view alcohol and marijuana as perfect substitutes. This means that 1. 2.

A) individuals will consume either alcohol or marijuana, but not both, regardless of price. B) as the price of alcohol decreases, marijuana use decreases.

3.

C) the marginal utility for alcohol and marijuana is constant.

4.

D) Both B and C.

18) Individuals derive utility from picnics, p, and kayak trips, k. Assuming that an individual's utility is U(p,k) = k0.5p0.5 and income is $100, what is the marginal rate of substitution (MRS) between picnics and kayak trips? 1.

A) MRS = 1.

2.

B) MRS = -1.

3.

C) MRS = -

4.

D) There is no substitution because picnics and kayak trips are perfect complements.

19) The absolute value of the slope of an indifference curve equals the ratio of the marginal utilities of the two goods involved.

20) What is the difference between ordinal and cardinal measurement?

21) Suppose Joe's utility for lobster (L) and soda (S) can be represented as U = L0.5 S0.5. Draw the indifference curve that yields a utility level of 9. Calculate the MUL, MUS, and MRS of L for S on that indifference curve when S = 3. Answer:

4.3 Budget Constraint

1) Joe's income is $500, the price of food (F) is $2 per unit, and the price of shelter (S) is $100. Which of the following represents his budget constraint? 1.

A) 500 = 2F + 100S

2.

B) F = 250 - 50S

3.

C) S = 5 - .02F

4.

D) All of the above.

2) Joe's income is $500, the price of food (F) is $2 per unit, and the price of shelter (S) is $100. Which of the following represents his marginal rate of transformation of food for shelter? 1.

A) -5

2.

B) -50

3.

C) -.02

4.

D) None of the above.

3) Joe's income is $500, the price of food (F) is $2 per unit, and the price of shelter (S) is $100. Which of the following represents his budget constraint? 1.

A) 500 = 100F + 2S

2.

B) 500 = 2F + 100S

3.

C) S = 500 - 2F

4.

D) All of the above.

4) Joe's budget constraint equals 500 = 2F + 100S, where $500 is Joe's income, $2 is the price of food (F) and $100 is the price of shelter (S). How much food can Joe buy if he buys one unit of shelter? 1.

A) 2 units

2.

B) 200 units

3.

C) 250 units

4.

D) 400 units

5) Joe's income is $500, the price of food (F) is $2, and the price of shelter (S) is $100. Which of the following bundles is in Joe's opportunity set? 1.

A) 50 units of food, 5 units of shelter

2.

B) 200 units of food, 2 units of shelter

3.

C) 100 units of food, 1 unit of shelter

4.

D) 150 units of food, 3 units of shelter

6) The marginal rate of transformation of y for x represents 1.

A) the slope of the budget constraint.

2.

B) the rate at which the consumer must give up y to get one more x.

3.

C) -Px/Py.

4.

D) All of the above.

7) Betty consumes good x and good y. If the price of x = $3 and the price of y = $4, then 1.

A) an extra unit of x costs 4/3 units of y.

2.

B) an extra unit of y costs 4/3 units of x.

3.

C) an extra unit of x costs 3/4 units of y.

4.

D) Both B and C.

8) If the price of one good increases while the price of the other good and the consumer's income remain unchanged, what will happen to the budget line? 1.

A) The budget line rotates inward from the intercept on the axis of the good that did not change in price.

2.

B) The budget line rotates outward from the intercept on the axis of the good that did not change in price.

3.

C) The budget line shifts inward without a change in slope.

4.

D) The budget line shifts outward without a change in slope.

9) Lisa eats both pizzas and burritos. If the price of a pizza increases, Lisa's opportunity set 1.

A) becomes larger.

2.

B) becomes smaller.

3.

C) is unchanged.

4.

D) cannot be determined without more information.

10) If the consumer's income increases while the prices of both goods remain unchanged, what will happen to the budget line? 1.

A) The budget line rotates inward from the intercept on the horizontal axis.

2.

B) The budget line rotates outward from the intercept on the vertical axis.

3.

C) The budget line shifts inward without a change in slope.

4.

D) The budget line shifts outward without a change in slope.

11) If the prices of both goods and income increase by the same percentage, what will happen to the budget line? 1.

A) The budget line rotates inward from the intercept on the axis of the good that did not change in price.

2.

B) The budget line rotates outward from the intercept on the axis of the good that did not change in price.

3.

C) The budget line shifts outward without a change in slope.

4.

D) Nothing.

12) A consumer buys food (F) and shelter (S). If the consumer's income rises and there is no change in the prices of F or S, the marginal rate of transformation of F for S will 1.

A) increase.

2.

B) decrease.

3.

C) stay the same.

4.

D) change, but there is not enough information to know how.

13) If both prices increases by 50%, 1.

A) budget constraint will be unchanged.

2.

B) slope of the budget constraint stay the same.

3.

C) slope of the budget constraint will decrease.

4.

D) budget constraint will shift outward in a parallel fashion.

14) If both prices decreases by 50%, 1.

A) budget constraint will be unchanged.

2.

B) slope of the budget constraint will increase.

3.

C) slope of the budget constraint will decrease.

4.

D) budget constraint will shift outward in a parallel fashion.

15) If a consumer's budget line for food (F) and shelter (S) is represented as F = 250 5S, we know that 250.

A) the consumer's income is 250.

251.

B) the price of shelter is 5.

252.

C) the price of shelter is 5 times the price of food.

253.

D) All of the above.

16) The intuition behind the budget constraint is that 1.

A) more options are preferred to less.

2.

B) money is the root of all happiness.

3.

C) information is power.

4.

D) scarcity is avoidable with prosperity.

17) Quotas, such as limiting the amount of residential water use during a drought, restricts an individual's preference set and

1.

A) reduces utility because an individual cannot consume as much as they would without the quota.

2.

B) increases utility because quotas restrict output and raise profits for the water company.

3.

C) reduces utility because an individual is forced to substitute to other goods.

4.

D) does not affect overall utility.

18) A consumer derives utility for "all other goods," g, and cigarettes, s. The price of "all other goods" equals $9 and the price of cigarettes is $10. Now suppose the government implements a $0.50 per-unit tax on cigarettes to encourage a decrease in smoking. How does the tax affect the marginal rate of transformation, MRT? 1.

A) The MRT will decrease to - .

2.

B) The MRT will increase to - .

3.

C) The MRT will increase to - + 0.05.

4.

D) The MRT will decrease to - - 0.05.

19) The slope of the budget line represents the rate at which the consumer is willing to trade one good for another at any given bundle.

20) Joe subscribes to an Internet provider that charges $2 per hour. Draw his budget line for Internet access on the horizontal axis and money spent on all other goods on the vertical axis assuming he has $100 per month to spend. Another company offers unlimited Internet access for a flat monthly fee of $20. Draw this budget line. Answer: See the above figure.

21) Lisa has an income of $100. She spends all of her income on pizza and burritos. A pizza costs $10 and a burrito costs $5. However, the store where Lisa buys her burritos has a special deal. After you've bought 6 burritos, then you can buy each burrito for $2.50. Draw Lisa's opportunity set. Answer: See the above figure.

22) Explain the difference between the marginal rate of substitution and the marginal rate of transformation.

4.4 Constrained Consumer Choice

1) Economists assume consumers select a bundle of goods that maximizes their wellbeing subject to 1.

A) their budget constraint.

2.

B) their income.

3.

C) relative prices.

4.

D) their marginal rate of substitution.

2) An optimum that occurs as a corner solution 1.

A) includes only one good.

2.

B) cannot be an equilibrium.

3.

C) cannot exhaust the budget constraint.

4.

D) includes the exact same amounts of each good.

3) The consumer is in equilibrium when 1.

A) MRT = MRS.

2.

B) Px/Py. = MUx/MUy.

3.

C) the budget line is tangent to the indifference curve at the bundle chosen.

4.

D) All of the above.

4) By selecting a bundle where MRS = MRT, the consumer is 1.

A) achieving a corner solution.

2.

B) reaching the highest possible indifference curve she can afford.

3.

C) not behaving in an optimal way.

4.

D) All of the above.

5) By selecting a bundle where MRS = MRT, the consumer is saying 1. 2.

A) "I value my last unit of each good equally." B) "I am willing to trade one good for the other at the same rate that I am required to do so."

3.

C) "I will equate the amounts spent on all goods consumed."

4.

D) All of the above.

6) Assume the price of beer is $4, the price of pizza is $10 and the consumer's income is $250. Which consumption bundle will NOT be the consumer's choice? 1.

A) 5 Beer, 5 Pizza

2.

B) 0 Beer, 25 Pizza

3.

C) 25 Beer, 15 Pizza

4.

D) None of the bundle will be chosen.

7) With respect to consuming food and shelter, two consumers face the same prices and both claim to be in equilibrium. We therefore know that 1.

A) they both have the same marginal utility for food.

2.

B) they both have the same marginal utility for shelter.

3.

C) they both have the same MRS of food for shelter.

4.

D) All of the above.

8) Johnny has allocated $30 toward coffee and tea and feels that coffee and tea are perfect substitutes. Due to differences in caffeine levels, his MRS of tea for coffee equals 2. If coffee and tea sell for the same price, Johnny will 1.

A) spend all $30 on tea.

2.

B) spend all $30 on coffee.

3.

C) spend $20 on coffee and $10 on tea.

4.

D) be indifferent between any bundle of coffee and tea costing $30.

9) Max has allocated $100 toward meats for his barbecue. His budget line and an indifference map are shown in the above figure. Which bundle will Max choose? 1.

A) a

2.

B) b

3.

C) c

4.

D) d

10) Max has allocated $100 toward meats for his barbecue. His budget line and an indifference map are shown in the above figure. What is the price of chicken? 1.

A) $0.80/lb

2.

B) $1.25/lb

3.

C) $4/lb

4.

D) $5/lb

11) Max has allocated $100 toward meats for his barbecue. His budget line and an indifference map are shown in the above figure. If the price of burger increases, 1.

A) Max will buy less burger and more chicken.

2.

B) Max will buy less burger and the same quantity of chicken.

3.

C) Max will buy less of both meats.

4.

D) More information is needed to answer the question.

12) Max has allocated $100 toward meats for his barbecue. His budget line and an indifference map are shown in the above figure. What happens if Max's mother gives him 10 pounds of burger? 1.

A) Max would have preferred receiving the dollar value of the burger.

2.

B) Max is indifferent between this gift and the dollar value of the burger.

3.

C) Max prefers this gift to the dollar value of the burger.

4.

D) None of the above.

13) Max has allocated $100 toward meats for his barbecue. His budget line and an indifference map are shown in the above figure. What happens if Max's mother gives him 30 pounds of burger? 1.

A) Max would have preferred receiving the dollar value of the burger.

2.

B) Max is indifferent between this gift and the dollar value of the burger.

3.

C) Max prefers this gift to the dollar value of the burger.

4.

D) None of the above.

14) Max has allocated $100 toward meats for his barbecue. His budget line and an indifference map are shown in the above figure. What happens if Max receives a $100 cash grant to buy either meat or chicken? 1.

A) Max will double his consumption of both meats.

2.

B) Max will spend it all on burger. Because of its lower price, he can buy more of it.

3. 4.

C) Max will take advantage of the gift by buying all chicken because it is the more expensive meat. D) There is not enough information to answer the question.

15) Max has allocated $100 toward meats for his barbecue. His budget line and an indifference map are shown in the above figure. Which of the following best describes Max's preferences? 1.

A) d > b > e

2.

B) d = b = e

3.

C) a = b > c

4.

D) a = b > e

16) Max has allocated $100 toward meats for his barbecue. His budget line and an indifference map are shown in the above figure. Which of the following bundles are in Max's opportunity set? 1.

A) a, b, c

2.

B) b, d, e

3.

C) a, b, d

4.

D) None of the above.

17) Max has allocated $100 toward meats for his barbecue. His budget line and indifference map are shown in the above figure. If the price of burger increases, which of the following bundles are in Max's opportunity set? 1.

A) b, d, e

2.

B) d, e

3.

C) a, b, c, d, e

4.

D) None of the labeled points are in Max's opportunity set.

18) Max has allocated $100 toward meats for his barbecue. His budget line and indifference map are shown in the above figure. If Max is currently at point e, 1.

A) the absolute value of his MRS is less than the trade-off offered by the market.

2.

B) he is willing to give up more burger than he has to given market prices.

3.

C) he is not maximizing his utility.

4.

D) All of the above.

19) Max has allocated $100 toward meats for his barbecue. His budget line and indifference map are shown in the above figure. If Max is currently at point d, 1.

A) the absolute value of his MRS is larger than the trade-off offered by the market.

2.

B) he is willing to give up more chicken than he has to given market prices.

3.

C) he is not maximizing his utility.

4.

D) All of the above.

20) Cash may be preferred to food stamps because additional cash 1.

A) rotates the budget constraint.

2.

B) shifts out the budget constraint at every point.

3.

C) provides a smaller opportunity set.

4.

D) allows the purchase of more food.

21) An interior solution to a consumer's utility maximization problem implies 1.

A) consuming a positive amount of all goods.

2.

B) consuming negative amounts of all goods.

3.

C) consuming less than optimal amounts of all goods.

4.

D) consuming more than an optimal amount of at least one good.

22) One reason more sport utility vehicles (SUVs) are driven in the United States than in Europe is 1.

A) different marginal rates of substitution.

2.

B) different marginal rates of transformation.

3.

C) the United States hasn't had a recession since 2000-2001.

4.

D) Either A or B

23) Those who criticize individuals who choose to drive a relatively inefficient sport utility vehicle (SUV) might mistakenly 1.

A) assume everyone faces the same relative prices.

2.

B) assume everyone has the same preference set.

3.

C) assume everyone has the same budget constraint.

4.

D) All of the above.

24) If the food stamp program in the United States moved from coupons redeemable for food to cash payments 1.

A) everyone would buy less food and more of other goods.

2.

B) everyone would buy more food and less of other goods.

3.

C) some people might buy less food and obtain a higher level of utility.

4.

D) some people might buy less food and obtain a lower level of utility.

25) Rank-order the following three gifts in terms of potentially providing the highest utility to the lowest: 1) A new video game; 2) A gift card to a video game store; 3) Cash. 1.

A) 1, 2, 3

2.

B) 2, 3, 1

3.

C) 3, 2, 1

4.

D) 1, 3, 2

26) One reason a gift card might be less desirable than cash? 1.

A) A gift card requires an individual to purchase from a particular store, which restricts the choice set.

2.

B) Cash requires an individual to purchase from a particular country, which restricts the choice set.

3.

C) Cash and the gift card are equally desirable, there is no problem here.

4.

D) Gift cards are easier to steal or lose relative to cash.

27) People derive utility in their lives from consuming goods (all other goods), g, and clean air (measured as the number of particulates removed per m3), a, as measured with the utility function U(g,a) = g0.6a0.4. The price index of consumer goods equals $20 per day. Income equals $1000. What is likely to happen to the number of asthma cases as the price of clean air (abatement cost) increases from $10 to $20? 1.

A) Asthma cases decrease because the particulants per m3increases by 2 units.

2.

B) Asthma cases increase because the particulant per m3decreases by 2 units.

3.

C) Asthma cases increase because the particulants per m3increase by 2 units.

4.

D) There is not enough information to answer this question.

28) If income increases due to a decrease in taxes, then 1. 2. 3. 4.

A) utility will increase because consumers can afford a larger bundle of goods. B) utility will fall because consumers will be forced to buy a smaller bundle of goods. C) utility will remain constant. D) utility will increase because consumers will be forced to buy a smaller bundle of goods.

29) Data shows that United States college students purchase more e-books than German college students. Assuming that all students have identical preferences for ebooks and textbooks, what is the likely explanation for this result? 1.

A) Taxes on textbooks are higher in the United States than taxes on textbooks in Germany.

2.

B) Taxes on textbooks are lower in the United States than taxes on textbooks in Germany.

3.

C) The price of textbooks is cheaper in the United States than textbooks in Germany.

4.

D) Both A and C.

30) Robinson Crusoe is stranded on an island. He finds that coconuts are freely available (zero harvest cost), but fish are difficult to harvest and require a lot of energy. As a result, harvesting fish has a high price. If coconuts and fish are imperfect substitutes, what is Robinson Crusoe likely to consume? 1.

A) He will consume more coconuts than fish.

2.

B) He will consume more fish than coconuts.

3.

C) He will consume equal amounts of both goods.

4.

D) Not enough information is given.

31) Consumers do not prefer gifts-in-kind to cash gifts.

32) If MRS > MRT, then the consumer is better off than at equilibrium.

33) Joe subscribes to an Internet provider that charges $2 per hour. He has $100 per month to spend and is at equilibrium by buying 10 hours of Internet access and $80 worth of other goods. Draw the indifference curve-budget line. If the company switches to a $20 monthly fee for unlimited Internet access, is Joe better off? Answer:

34) Suppose Joe's utility for lobster (L) and soda (S) can be represented as U = L0.5 S0.5. Joe walks into a restaurant with $72. Lobsters cost $18 each and sodas cost $2 each. How much lobster and soda will Joe consume if he intends to spend all his money? (There are no tax and no tips.) .

35) Joe's indifference map for lobster and soda is shown in the above figure along with his budget line. Will Joe choose point a? Explain your answer in terms both of MRS and the level of utility.

36) John is indifferent between canned soup and fresh soup. Canned soup sells for $1 per serving and fresh soup sells for $2 per serving. Assuming that John has allocated $4 toward soup, how will he spend it? Explain your answer by drawing John's budget line and indifference curves. Answer:

37) Suppose that left shoes and right shoes must be purchased separately. Ingrid needs an equal number of each type of shoe and has a budget of $100 for shoes. Left shoes always cost $1. If right shoes cost $19 each, how many of each will Ingrid buy? If the price of right shoes increases to $49 each, how will Ingrid react? Explain your answer by drawing the indifference curves-budget lines. Answer:

38) Johnny has $100 to spend on books and all other goods. Books cost $20 each and Johnny is at equilibrium consuming 3 books and $40 worth of other goods. Johnny's grandmom wants to give Johnny either a book or $20 for his birthday. Which gift does Johnny prefer? Explain using an indifference map and budget lines. Answer:

39) Lisa consumes only pizzas (P) and burritos (B). Her utility function is U = P0.5 B0.5. The price of per pizza is $10 and the price per burrito is $5. In equilibrium, Lisa consumes 4 pizzas. Using Lisa's utility function, calculate how many burritos she consumes.

40) Lisa consumes only pizzas and burritos. In equilibrium, her marginal utility of pizza is 20 and her marginal utility of a burrito is 10. The price of a pizza is $4. What is the price of a burrito?

4.5 Behavioral Economics

1) Behavioral economics extends traditional economic models by 1.

A) including insights from psychology and human cognition models.

2.

B) modeling behavior rather than prices.

3.

C) admitting that individuals are irrational.

4.

D) admitting that incentives are very important.

2) Behavioral economists use which technique to test economic theory? 1.

A) controlled experiments

2.

B) price theory

3.

C) environmental economics

4.

D) lie detector tests

3) Behavioral economists have discovered that 1.

A) transitivity of preferences always holds, even in animals.

2.

B) the law of demand does not hold in controlled experiments.

3.

C) transitivity of preferences does not always hold, especially for young people.

4.

D) reflexivity of preferences is not true.

4) Which is a behavioral economics justification for limiting advertising directed towards children? 1.

A) Children have no money.

2.

B) Children pester their parents too much.

3.

C) Children have no memory.

4.

D) Children do not always have transitive preferences.

5) In behavioral economics, the endowment effect refers to

1.

A) most people believe that most wealthy people inherit their wealth.

2.

B) many people would be indifferent between being endowed with money or knowledge.

3.

C) many people place a higher value on what they own than when they consider purchasing.

4.

D) most people respond to tax incentives to provide an endowment for their children.

6) Which of the following might explain the evidence of an endowment effect in behavioral economics? 1.

A) government regulation

2.

B) knowledge and experience

3.

C) the federal tax code

4.

D) class envy

7) Behavioral economists have discovered that

1.

A) experimental design is important.

2.

B) experimental design doesn't matter for most questions.

3.

C) experimental design only matters when people have bounded rationality.

4.

D) experimental design doesn't matter when people have bounded rationality.

8) In Spain, people are considered organ donors unless the explicitly indicate they do not want to be. In the United States, people are only considered organ donors if they explicitly indicate they wish to be. Behavioral economics would suggest that 1.

A) everything else equal, the opt-in system of Spain would generate more organ donors as a percentage of the adult population.

2.

B) everything else equal, the opt-in system of the United States would generate more organ donors as a percentage of the adult population.

3.

C) everything else equal, the opt-out system of Spain would generate more organ donors as a percentage of the adult population.

4.

D) everything else equal, the opt-out system of the United States would generate more organ donors as a percentage of the adult population.

9) In behavioral economics, the term salience refers to 1.

A) relevance to the problem being investigated.

2.

B) people only consider information when it is conveyed in a subtle manner.

3.

C) how an experiment is designed.

4.

D) people consider information when it is presented in an "eye grabbing" manner.

10) In behavioral economics, salience is best exemplified by 1.

A) consumers responding differently when posted prices increase rather than when prices increase because of sales tax increases.

2.

B) consumers responding the same regardless of how prices change.

3.

C) the end of a controlled experiment.

4.

D) consumers responding differently when income increases permanently rather than temporarily.

11) Bounded rationality suggests that 1.

A) individuals might make "incorrect" decisions because they are unable to consider all possible options.

2.

B) individuals would rather have less choice to more choice.

3.

C) rational decisions can only be made when choices are restricted.

4.

D) individuals are happier when their choices are restricted or "bounded."

Microeconomics, 7e (Perloff) Chapter 5 Applying Consumer Theory

5.1 Deriving Demand Curves

1) The above figure shows Bobby's indifference map for juice and snacks. Also shown are three budget lines resulting from different prices for snacks assuming he has $20 to spend on these goods. Which of the following points are on Bobby's price-consumption curve? 1.

A) 10 snacks and 20 juices

2.

B) 10 snacks and 0 juices

3.

C) 10 snacks and 5 juices

4.

D) 10 snacks and 15 juices

2) The above figure shows Bobby's indifference map for juice and snacks. Also shown are three budget lines resulting from different prices for snacks assuming he has $20 to spend on these goods. Which of the following points are on Bobby's demand curve for snacks? 1.

A) p = 2, q = 10

2.

B) p = 2, q = 13

3.

C) p = 2, q = 5

4.

D) p = 1, q = 20

3) The above figure shows Bobby's indifference map for juice and snacks. Also shown are three budget lines resulting from different prices for snacks. This information could be used to determine 1.

A) the slope of Bobby's demand curve for juice.

2.

B) the amount by which Bobby's demand curve for juice shifts when his income rises.

3.

C) the amount by which Bobby's demand curve for juice shifts when the price of snacks rises.

4.

D) All of the above.

4) The above figure shows Bobby's indifference map for juice and snacks. Also shown are three budget lines resulting from different prices for snacks. As the price of snacks rises, Bobby's utility 1.

A) stays the same.

2.

B) increases.

3.

C) decreases.

4.

D) might change, but there is not enough information to determine.

5) The above figure shows Bobby's indifference map for juice and snacks. Also shown are three budget lines resulting from different prices for snacks. Bobby's demand for snacks is 1.

A) unit elastic.

2.

B) elastic.

3.

C) inelastic.

4.

D) perfectly elastic.

6) The above figure shows Bobby's indifference map for juice and snacks. Also shown are three budget lines resulting from different prices for snacks. As the price of snacks rises, the price for juice 1.

A) stays the same.

2.

B) increases.

3.

C) decreases

4.

D) might change, but there is not enough information to determine.

7) An individual's demand curve for a good can be derived by measuring the quantities selected as 1.

A) the price of the good changes.

2.

B) the price of substitute goods changes.

3.

C) income changes.

4.

D) All of the above.

8) As the price of a good rises, the consumer will experience 1.

A) a desire to consume a different bundle.

2.

B) a decrease in utility.

3.

C) a southern or western movement on the indifference map.

4.

D) All of the above.

9) An increase in the price of a good causes 1.

A) a change in the slope of the budget line.

2.

B) an increase in the consumption of that good.

3.

C) a rightward shift of the demand curve for that good.

4.

D) a parallel rightward shift of the budget line.

10) Suppose a graph is drawn to show a consumer's preferences for football tickets and basketball tickets. The quantity of football tickets is measured on the horizontal axis. If the price-consumption curve is horizontal when the price of football tickets changes, then 1.

A) football tickets are an inferior good.

2.

B) the demand for football tickets is perfectly elastic.

3.

C) the demand for football tickets is unit elastic.

4.

D) the demand curve for football tickets will be horizontal.

11) A demand curve for a Giffen good would be 1.

A) upward sloping.

2.

B) downward sloping.

3.

C) horizontal.

4.

D) vertical.

12) Suppose the quantity of x is measured on the horizontal axis. If the price consumption curve is vertical when the price of x changes, then the demand for x is 1.

A) perfectly elastic.

2.

B) perfectly inelastic.

3.

C) unit elastic.

4.

D) There is not enough information to determine the price elasticity of demand for x.

13) Sandy derives utility from consuming "all other goods," g, and clean air (measured by particulate matter removed per m3), a, as measured by the utility function U(g,a) = g0.6a0.4. The price of "all other goods" is $20 and the price of clean air (abatement) equals $10. Brian is the only other consumer in the market for clean air and demands 10 units of clean air. What is the market demand for clean air? 1.

A) Total market demand is 14 units of clean air.

2.

B) Total market demand is 12 units of clean air.

3.

C) Total market demand is 10 units of clean air.

4.

D) Total market demand is 16 units of clean air.

14) Ten individuals have $100 and identical preferences for picnics, p, and kayak trips, k, where U(p, k) = k0.5p0.5. The price of picnics is $5 and the price per kayak trip is $ 10. What is the shortage/surplus in the market when the supply of picnics totals 120? 20.

A) There is a surplus of 20.

21.

B) There is a shortage of 20.

22.

C) The market is in equilibrium. Therefore, there is no surplus/shortage.

23.

D) There is not enough information to answer this question.

15) Three individuals have $1000 and identical preferences for gum, g, and cigarettes, s, as measured by the utility function U(g,s) = 10g0.9a0.1. The price of gum is $9 and the price of cigarettes is $12. What is the market surplus/shortage at a price of $12 when the supply of cigarettes is 5? 1.

A) There will be a shortage of 3 cigarettes.

2.

B) There will be a surplus of 3 cigarettes.

3.

C) There will be a shortage of 2/3 cigarettes.

4.

D) There will be a surplus of 2/3 cigarettes.

16) What is the slope of the price consumption curve for two goods, x and y, when preferences are measured by the utility function U(x,y) = x0.5y0.5, the price of good y is $10, income equals $100, and the price of good x increases from $5 to $10? 1.

A) Slope equals zero.

2.

B) Slope is infinite.

3.

C) Slope is - .

4.

D) Slope is .

17) If the price-consumption curve is upward sloping when the price of the good measured on the horizontal axis changes, then the demand curve for that good will be upward sloping.

18) Draw two graphs, one directly above the other. On the upper graph, label the vertical axis Good X and label the horizontal axis Good Y. On the lower graph, label the vertical axis the Price of good Y and label the horizontal axis Good Y. In the upper graph, show the income and substitution effects of a decrease in the Price of good Y when Y is a Giffen good. Draw the corresponding demand curve for Good Y in the lower graph.

See the above figure. Point A is the original consumption point. The movement from point A to point B is the substitution effect. The movement from point B to point C is the income effect.

19) The above figure shows a consumer's indifference curves for soda and all other goods. Assuming a budget of $100, derive the consumer's demand for soda for prices of $4 and $10 per case of soda. Estimate the price elasticity of demand for soda. t a price of $4, 15 cases are purchased, At a price of $10, 6 cases are purchased. In both cases, the same total amount, $60, is spent on soda. This implies unit elasticity.

5.2 How Changes in Income Shift Demand Curves

1) A movement upward along an upward sloping Engel curve corresponds to 1.

A) upward sloping indifference curves.

2.

B) crossing indifference curves.

3.

C) a rotation in the budget constraint.

4.

D) a parallel shift in the budget constraint.

2) When deriving an Engel curve, the prices of both goods 1.

A) are held constant.

2.

B) increase by the same percentage as income.

3.

C) decrease by the same percentage as income.

4.

D) can either decrease, increase or stay the same.

3) The above figure shows Larry's indifference map and budget lines for ham and pork. Which of the following statements is TRUE? 1.

A) Pork is an inferior good.

2.

B) Ham is an inferior good.

3.

C) Neither pork nor ham is an inferior good.

4.

D) Both ham and pork are inferior goods.

4) The above figure shows Larry's indifference map and budget lines for ham and pork. Which of the following statements is TRUE? 1.

A) Larry's Engel curve for pork will be upward sloping.

2.

B) Larry's Engel curve for pork will be downward sloping.

3.

C) Larry's Engel curve for pork will be backward bending.

4.

D) Larry's Engel curve for pork cannot be derived from the information provided.

5) The above figure shows Larry's indifference map and budget lines for ham and pork. Which of the following statements is TRUE? 1.

A) Larry's demand curve for pork shifts rightward when his income increases.

2.

B) Larry's income elasticity of demand for pork is greater than zero.

3.

C) Pork is a normal good.

4.

D) All of the above.

6) After Joyce and Larry purchased their first house, they made additional home improvements in response to increases in income. After a while, their income rose so much that they could afford a larger home. Once they realized they would be moving, they reduced the amount of home improvements. Their Engel curve for home improvements on their current home is 1.

A) negatively sloped.

2.

B) flat.

3.

C) positively sloped.

4.

D) backward bending.

7) Suppose the quantity of x is measured on the horizontal axis. If the income consumption curve is vertical, then the income elasticity of demand for x is 1.

A) 0.

2.

B) 1.

3.

C) -1.

4.

D) There is not enough information to determine the income elasticity of demand for x.

8) An inferior good exhibits 1.

A) a negative income elasticity.

2.

B) a downward sloping Engel curve.

3.

C) a decline in the quantity demanded as income rises.

4.

D) All of the above.

9) When John's income was low, he could not afford to dine out and would respond to a pay raise by purchasing more frozen dinners. Now that his income is high, a pay raise causes him to dine out more often and buy fewer frozen dinners. Which graph in the above figure best represents John's Engel curve for frozen dinners? 1.

A) Graph A

2.

B) Graph B

3.

C) Graph C

4.

D) Graph D

10) When John's income was low, he could not afford to dine out and would respond to a pay raise by purchasing more frozen dinners. Now that his income is high, a pay raise causes him to dine out more often and buy fewer frozen dinners. Which graph in the above figure best represents John's Engel curve for dining out? 1.

A) Graph A

2.

B) Graph B

3.

C) Graph C

4.

D) Graph D

11) Both Sally and Sam receive a 10% raise in a single year. Sally increases her demand for ground beef whereas Sam decreases his demand for ground beef. 1. 2. 3. 4.

A) This is impossible. B) This is only possible if Sally considers ground beef an inferior good, and Sam views it as a normal good. C) This is only possible if Sally has lower income than Sam. D) This is only possible if Sally considers ground beef a normal good and Sam views it as an inferior good.

12) Even though Mary's income is very low, she makes sure that she purchases enough milk for her family to drink. As her income rises, she does buy more milk. Which graph in the above figure best represents Mary's Engel curve for milk? 1.

A) Graph A

2.

B) Graph B

3.

C) Graph C

4.

D) Graph D

13) When John was in college and his income was low, he drank "Red Ribbon" beer. As his income increased, he purchased better-quality beer and less "Red Ribbon." Which graph in the above figure best represents John's Engel curve for "Red Ribbon" beer? 1.

A) Graph A

2.

B) Graph B

3.

C) Graph C

4.

D) Graph D

14) Which graph in the above figure best represents a good that is an inferior good at some income levels, and a normal good at other income levels? 1.

A) Graph A

2.

B) Graph B

3.

C) Graph C

4.

D) Graph D

15) If consumer income and prices increase by the same percentage, 1.

A) the consumer will buy more of both goods.

2.

B) the consumer will buy more of both goods if they are both normal goods.

3.

C) the consumer will buy less of both goods if they are both inferior goods.

4.

D) the consumer's utility maximizing bundle stays the same.

16) An individual derives utility from consuming "all other goods," g, and clean air (measured by the reduction in particulate matter per m3), a, as measured by the utility function U(g,a) = g0.6a0.4. The price of consumer goods equals $20 and the price of clean air (abatement) equals $10. What is the slope of the Engel curve when income increases from $100 to $200? 25.

A) The slope is 25.

26.

B) The slope is -25.

27.

C) The slope is zero.

28.

D) The slope is infinite.

17) An individual derives utility from games, g (y-axis), and toy airplanes, a (x-axis), described by the utility function U(g,a) = g0.6a0.4. The price per game is $20 and the price of toy airplanes is $10. Using the slope of the income consumption curve (ICC), determine whether games and toy airplanes are normal or inferior goods when income increases from $100 to $200. 1.

A) Both goods are normal goods with an ICC slope of .

2.

B) Both goods are normal goods with an ICC slope of .

3.

C) Both goods are inferior goods with an ICC slope of - .

4.

D) Both goods are inferior goods with an ICC slope of - .

18) An increase in income (all else equal) will ALWAYS lead to a parallel shift of the budget line.

19) Explain what the slope of the income consumption curve shows about the income elasticity of demand.

20) Why can't all goods be inferior?

21) The above figure shows three different Engel curves. Rank them in terms of income elasticity. Answer: Engel curve A implies that a certain level of income is required before any of the good is purchased. Engel curve B implies that the quantity demanded is proportional to income (unit elastic). Engel curve C implies that the good is a necessity since it would be consumed even if income were zero. Thus A > B > C in terms of income elasticity.

22) When income increases by 1%, the quantity demanded of a good decreases by 2%. What is the income elasticity of the good? Is the good normal or inferior? Why?

23) Why would you expect the demand for diamond jewelry to fall faster than plastic, costume jewelry when all incomes fall?

24) Draw budget constraints, indifference curves, and the income consumption curve for a good that has an income elasticity that is perfectly inelastic.

Answer: See the above figure.

5.3 Effects of a Price Change

1) Median household income is $50,000 per year. The typical household spends about $125 per year on milk, which has an income elasticity of about 0.07. From this information, we can conclude that 1.

A) milk is a luxury.

2.

B) milk is a Giffen good.

3.

C) the income effect from a change in the price of milk is very large.

4.

D) the income effect from a change in the price of milk is very small.

2) When the price of a good changes, the substitution effect can be found by comparing the equilibrium quantities purchased 1.

A) on the old budget line and the new budget line.

2.

B) on the original indifference curve when faced with the original prices and when faced with the new prices.

3.

C) on the new budget line and a hypothetical budget line that is a shift back to the original indifference curve parallel to the new budget line.

4.

D) on the new indifference curve.

3) When the price of a good changes, the income effect can be found by comparing the equilibrium quantities purchased 1.

A) on the old budget line and the new budget line.

2.

B) on the original indifference curve when faced with the original prices and when faced with the new prices.

3.

C) on the new budget line and a hypothetical budget line that is a shift back to the original indifference curve parallel to the new budget line.

4.

D) on the new indifference curve.

4) The substitution effect can be measured holding ________ constant.

1.

A) income

2.

B) utility

3.

C) the price of one good

4.

D) the price of all goods

5) Suppose that frozen dinners were once a normal good for John, but now frozen dinners are an inferior good for him. John's demand curve for frozen dinners 1.

A) has become steeper as a result.

2.

B) has become flatter as a result.

3.

C) has not changed as a result.

4.

D) has disappeared as a result.

6) One characteristic of a Giffen good is that it 1.

A) is a luxury good.

2.

B) is an inferior good.

3.

C) has an upward-sloping Engel curve.

4.

D) All of the above.

7) A Giffen good has 1.

A) a positive substitution effect.

2.

B) a negative income effect.

3.

C) a larger income effect than substitution effect.

4.

D) All of the above.

8) If a good is an inferior good, then its 1.

A) demand curve will be upward sloping.

2.

B) income effect reinforces the substitution effect.

3.

C) income elasticity is negative.

4.

D) Engel curve cannot be drawn.

9) Suppose Lisa spends all of her money on books and coffee. When the price of coffee decreases, the 1.

A) substitution effect on coffee is positive, and the income effect on coffee is positive.

2.

B) substitution effect on coffee is ambiguous, and the income effect on coffee is ambiguous.

3.

C) substitution effect on coffee is positive, and the income effect on coffee is ambiguous.

4.

D) substitution effect on coffee is ambiguous, and the income effect on coffee is positive.

10) In the case of a normal good, 1.

A) demand curves always slope downward.

2.

B) the income effect and substitution effect are in the same direction.

3.

C) the Engel curve slopes upward.

4.

D) All of the above.

11) The above figure shows Bobby's indifference map for soda and juice. B1 indicates his original budget line. B2 indicates his budget line resulting from a decrease in the price of soda. What change in quantity best represents his substitution effect? 1.

A) 3

2.

B) 10

3.

C) 15

4.

D) 7

12) The above figure shows Bobby's indifference map for soda and juice. B1 indicates his original budget line. B2 indicates his budget line resulting from a decrease in the price of soda. What change in quantity best represents his income effect?

1.

A) 3

2.

B) 10

3.

C) 15

4.

D) 7

13) The above figure shows Bobby's indifference map for soda and juice. B1 indicates his original budget line. B2 indicates his budget line resulting from an increase in the price of soda. From the graph, one can conclude that 1.

A) Bobby views soda as an inferior good.

2.

B) Bobby's demand for soda is perfectly inelastic.

3.

C) Bobby views soda as a normal good.

4.

D) the income elasticity of demand for soda is 1.

14) When measuring the substitution effect, one uses the change along 1.

A) the old indifference curve.

2.

B) the new indifference curve.

3.

C) either the old or the new indifference curve.

4.

D) the budget constraint.

15) The Slutsky equation shows that, holding the total effect constant, the income effect will be larger for goods that 1.

A) have a smaller substitution effect.

2.

B) make up a larger percentage of a household's budget.

3.

C) have perfectly inelastic demand curves.

4.

D) All of the above.

16) Suppose that the interest rate paid to savers increases. As a result, Tom wishes to save less. This suggests that, for Tom, 1.

A) the substitution effect is greater than the income effect.

2.

B) the income effect is greater than the substitution effect.

3.

C) utility maximization is not occurring.

4.

D) future consumption is a luxury.

17) Suppose that the interest rate paid to savers increases. As a result, Tom wishes to save more. This suggests that, for Tom, 1.

A) the substitution effect is greater than the income effect.

2.

B) the income effect is greater than the substitution effect.

3.

C) utility maximization is not occurring.

4.

D) future consumption is a luxury.

18) What is the primary difference between the substitution and the income effect of a price change? 1.

A) The substitution effect holds income constant and the income effect holds utility constant.

2.

B) The substitution effect is always positive and the income effect is always negative.

3.

C) The substitution effect holds utility constant and the income effect holds prices constant.

4.

D) The substitution effect is always negative and the income effect is always positive.

19) What is the benefit of understanding the income and the substitution effects? 1. 2.

A) The income effect might run in the opposite direction of the substitution effect. B) The income effect always runs in the opposite direction of the substitution effect.

3.

C) The income and substitution effects just offset each other, which explains a lot.

4.

D) The income and substitution effects are proportional to each other.

20) For an inferior good, if the income effect more than offsets the substitution effect, we call that good

1.

A) a Giffen good.

2.

B) a normal good.

3.

C) an inferior good.

4.

D) a neutral good.

21) The Affordable Care Act is intended to reduce the price of health care. A decrease in the price of health care may 1.

A) increase the number of doctors office visits if office visits are considered an inferior good and the substitution effect dominates the income effect.

2.

B) decrease the number of doctors office visits if office visits are considered an inferior good and the substitution effect dominates the income effect.

3.

C) increase the number of doctors office visits if office visits are a normal good.

4.

D) Both A and C.

22) If a good is considered a normal good, the demand curve will shift ________ when income increases because ________.

1.

A) right; the income and substitution effects move in the same direction.

2.

B) right; the income and substitution effects move in the opposite direction.

3.

C) left; the income and substitution effects move in the same direction.

4.

D) left; the income and substitution effects move in the opposite direction.

23) If a consumer is compensated for the income effect that occurs when the price of a good increases, then his demand curves can never slope upward.

24) A good may be inferior at some income levels and normal at others.

25) If the Engel curve for a good is upward sloping, the demand curve for that good must be downward sloping.

26) Suppose Joe earns $1,000 in Year 1 and $0 in Year 2. Any amount he saves will earn interest at a rate of 10%. Draw Joe's budget line. (Hint: He can either consume all $1000 this year or consume nothing this year and have $1,100 next year.) Assuming convex indifference curves, show that an increase in the rate of interest can cause Joe's savings to either increase or decrease. Explain in terms of income and substitution effect. Answer:

27) Many manufacturers sell products labeled as having imperfections at a discount at their factory outlets but do not ship these imperfect goods to regular retail outlets. Why?

5.4 Cost-of-Living Adjustments

1) Due to inflation, nominal prices are usually 1.

A) equal to real prices.

2.

B) smaller than real prices.

3.

C) larger than real prices.

4.

D) a constant proportion different from real prices.

2) A Consumer Price Index (CPI) adjustment overcompensates for inflation because it ignores 1.

A) the income effect when relative prices change.

2.

B) the substitution effect when relative prices change.

3.

C) that some goods are inferior.

4.

D) that the substitution effect may offset the income effect.

3) Employing a fixed-weight index like the Consumer Price Index to adjust a person's salary in response to inflation will overcompensate this person because doing so will allow this person to 1.

A) buy the same bundle of goods as he did before the inflation.

2.

B) achieve a higher level of utility than he did before the inflation.

3.

C) achieve the same level of utility as before the inflation.

4.

D) buy more of all goods.

4) Under which of the following conditions will there be no substitution bias in the CPI? 1.

A) Indifference curves are convex.

2.

B) Indifference curves are L-shaped.

3.

C) Indifference curves are linear.

4.

D) Indifference curves are downward sloping.

5) Under which of the following conditions will there be no substitution bias in the CPI? 1.

A) Lower-priced goods increase in price by a greater percentage than do higherpriced goods.

2.

B) Higher-price goods increase in price by a greater percentage than do lowerpriced goods.

3.

C) All goods change in price by the same amount.

4.

D) All goods change in price by the same percentage.

6) A true cost-of-living adjustment (COLA) in response to a change in prices would compensate consumers so that they would be able to 1.

A) purchase the same bundle they purchased before prices changed.

2.

B) achieve the same level of utility they did before prices changed.

3.

C) face the same choices they did before prices changed.

4.

D) achieve an increase in utility that is equal to the rate of inflation.

7) Richard receives government transfer payments and currently consumes 5 guns and 6 goose livers. Assume the price of guns decreases by 10% and the price of goose liver increases by 20%. The government raises Richard's transfer payments so he can still afford 5 guns and 6 goose livers. Does this constitute a true cost-of-living adjustment (COLA)? 1.

A) No. Richard is overcompensated.

2.

B) No. Richard is undercompensated.

3.

C) Yes. The payment just achieves the right level of compensation.

4.

D) Not enough information.

8) Before an uneven rise in prices Allan consumed 5 bread and 6 juice. After the price increase and with an increased welfare payment from the government Allan consumes 4 bread and 7 juice. Does the government payment represent a true cost-of-living adjustment (COLA)? 1.

A) Yes, if the two consumption bundles lie on the same indifference curve.

2.

B) Yes, if the second bundle yields more utility than the first.

3.

C) No, the first bundle is clearly preferred.

4.

D) Not enough information.

9) The Consumer Price Index (CPI) measures inflation for every individual. 1. 2. 3. 4.

A) True, the weights used in calculating the CPI are adjusted for every individual in the country. B) True, people all face the same prices and therefore face the same inflation. C) False, the CPI uses weights based on how much each product represents in the typical household budget. D) False, the CPI doesn't measure inflation.

10) Using a Laspeyres index to calculate the Consumer Price Index (CPI) 1.

A) weights quantities with current prices.

2.

B) weights prices with base-year quantities.

3.

C) weights quantities with base-year prices.

4.

D) weights prices with current year quantities.

11) Using a Paasche index to calculate the Consumer Price Index (CPI) 1.

A) weights quantities with current prices.

2.

B) weights prices with base-year quantities.

3.

C) weights quantities with base-year prices.

4.

D) weights prices with current year quantities.

12) What is one way to adjust the CPI for substitution bias? 1.

A) Use the Paasche index.

2.

B) Use the Laspeyres index.

3.

C) Multiply the Paasche Index and the Laspeyres index.

4.

D) Take the geometric mean of the Paasche index and the Laspeyres index.

13) The Fisher index 1.

A) uses the arithmetic mean of the Paasche index and the Laspeyres index.

2.

B) uses the standard deviation of the Paasche index and the Laspeyres index.

3.

C) uses the geometric mean of the Paasche index and the Laspeyres index.

4.

D) uses the harmonic mean of the Paasche index and the Laspeyres index.

14) Which of the following is the geometric mean of 4 and 9? 6.

A) 6.5

7.

B) 6

8.

C) 36

9.

D) 3.6

15) Inflation over time necessarily makes consumers worse off.

16) Using the CPI to compensate workers for inflation is appropriate because, in the face of a change in relative prices, people should be allowed to purchase the same bundle as they did before the price changes.

17) Suppose the typical consumer only purchases food and clothing, and her utility can be expressed as U = F ∗ C. Currently, food costs $5 per unit and clothing costs $2 per unit. Her income is $70. If the price of food increases to $6, compare the resulting Laspeyres price index with a true cost of living index. Answer: Maximizing utility subject to the initial constraint (5F + 2C = 70) yields C/F = 5/2 or F = 7 and C = 17.5. The Laspeyres price index calculates the ratio of the income necessary to achieve the original bundle relative to the original income. In this case, [(6 ∗ 7) + (2 ∗ 17.5)]/70 = 1.10. The true cost of living index calculates the ratio of the income necessary to achieve the original level of utility relative to the original income. Utility is held constant when C ∗ F = 17.5 ∗ 7 = 122.5. The consumer is on the new budget line when C/F = 3. Combining yields F = 6.39 and C = 19.17. At the new prices, this requires an income of 76.68 and a resulting cost of living index of 76.68/70 = 1.095.

5.5 Deriving Labor Supply Curves

1) The price of leisure 1.

A) is the same for everyone.

2.

B) depends on the number of hours worked.

3.

C) is measured as foregone earnings.

4.

D) is immeasurable.

2) If a person supplies more hours of labor in response to a wage increase, then 1.

A) the substitution effect is greater than the income effect.

2.

B) the income effect is greater than the substitution effect.

3.

C) the income effect equals the substitution effect.

4.

D) the person is not maximizing utility.

3) If a person supplies fewer hours of labor in response to a wage increase, then 1.

A) the substitution effect is greater than the income effect.

2.

B) the income effect is greater than the substitution effect.

3.

C) the income effect equals the substitution effect.

4.

D) the person is not maximizing utility.

4) Empirical studies have found that the labor supply curves for most parts of the population are 1.

A) backward-bending.

2.

B) upward-sloping.

3.

C) downward-sloping.

4.

D) nearly vertical.

5) A backward-bending labor supply curve implies that 1.

A) the substitution effect dominates the income effect at higher wage rates but not at lower wage rates.

2.

B) the substitution effect dominates the income effect at lower wage rates but not at higher wage rates.

3.

C) leisure is an inferior good.

4.

D) workers are irrational.

6) If Bobby thinks that leisure is an inferior good, then his labor supply curve 1.

A) is backward bending.

2.

B) is always negatively sloped.

3.

C) is always positively sloped.

4.

D) does not exist.

7) A tax cut that raises the after-tax wage rate will most likely result in more hours worked if 1.

A) tax rates were low already.

2.

B) the relevant portion of the labor supply curve is upward sloping.

3.

C) the relevant portion of the labor supply curve is downward sloping.

4.

D) workers can be easily fooled.

8) If workers are in the backward-bending section of their labor supply curves, than an increase in the income tax rate will 1.

A) increase the tax revenue and increase the number of hours worked.

2.

B) increase the tax revenue and decrease the number of hours worked.

3.

C) decrease the tax revenue and increase the number of hours worked.

4.

D) decrease the tax revenue and decrease the number of hours worked.

9) In response to an increase in the wage rate, the substitution effect will cause a person to 1.

A) supply fewer hours of labor.

2.

B) supply more hours of labor.

3.

C) supply the same hours of labor.

4.

D) have a backward bend in her labor supply curve.

10) In response to an increase in the wage rate, the income effect will usually cause a person to 1.

A) supply fewer hours of labor.

2.

B) supply more hours of labor.

3.

C) supply the same hours of labor.

4.

D) have a horizontal labor supply curve.

11) What might explain a professional baseball player having lower production the year after signing a multi-million dollar contract? 1.

A) the substitution effect

2.

B) the endowment effect

3.

C) bounded rationality

4.

D) the income effect

12) Newspaper accounts of the U.S. labor market often point out that many people are working more hours than their parents did. What might explain this phenomenon? 1.

A) the substitution effect

2.

B) the endowment effect

3.

C) bounded rationality

4.

D) the income effect

13) Consider two lottery winners, Tino who is 65 years old and Sasha who is 32 years old. Which of these two would be expected to have the larger income effect, all else equal? 1.

A) Sasha

2.

B) Tino

3.

C) Both would have no income effect.

4.

D) Both would have equal income effects.

14) Wealthy people will tend to have vertical labor supply curves 1.

A) only if their income effect just offsets their substitution effect.

2.

B) only if their income effect is greater than their substitution effect.

3.

C) only if their income effect is less than their substitution effect.

4.

D) only if they don't have an income effect.

15) Recent data has shown that income and volunteer time are positively related. Assuming that volunteer time is included in leisure time, what could explain this observation? 1.

A) Higher income individuals view leisure as a normal good such that the income effect dominates the substitution effect.

2.

B) Higher income individuals view leisure as a normal good such that the substitution effect dominates the income effect.

3.

C) Higher income individuals view leisure as an inferior good such that the income and substitution effects both increase leisure time.

4.

D) Both A and C.

16) A tax cut will unambiguously lower income-tax revenue.

17) An increase in unearned income always creates a disincentive to work. Answer: False. The effect on labor supply will also depend on the individual's preferences towards work (leisure).

18) Draw a graph with Goods Per Day on the vertical axis and Leisure Hours Per Day increasing from left to right on the horizontal axis. Show that a person who works can work fewer hours and increase utility when the wage rate increases. Answer: See the above figure.

19) Suppose a person's utility for leisure (L) and consumption (Y) can be expressed as U = Y + L0.5. Show what happens to the person's labor supply curve when the income tax is cut from 70 percent to 30 percent. Denote hours worked as H and wage per hour as w.

20) Suppose a person's utility for leisure (L) and consumption (Y) can be expressed as U = Y + L0.5. Assuming a wage rate of $10 per hour, show what happens to the person's labor supply curve when the person wins a lottery prize of $100 per day.

21) Suppose a person's utility for leisure (L) and consumption (Y) can be expressed as U = Y ∗ L and this person has no non-labor income. Assuming a wage rate of $10 per hour, show what happens to the person's labor supply when the person wins a lottery prize of $100 per day.

Microeconomics, 7e (Perloff) Chapter 6 Firms and Production

6.1 The Ownership and Management of Firms

1) Economists typically assume that the owners of firms wish to 1.

A) produce efficiently.

2.

B) maximize sales revenues.

3.

C) maximize profits.

4.

D) All of the above.

2) An organization that converts inputs (like Labor, Capital etc.) into output can be a 1.

A) firm.

2.

B) sole proprietorship.

3.

C) corporation.

4.

D) All of the above.

3) Efficient production occurs if a firm 1.

A) cannot produce its current level of output with fewer inputs.

2.

B) given the quantity of inputs, cannot produce more output.

3.

C) maximizes profit.

4.

D) All of the above.

4) Limited liability is a benefit to 1.

A) sole proprietorships.

2.

B) partnerships.

3.

C) corporations.

4.

D) All of the above.

5) What is one of the biggest differences between a sole proprietorship and a corporation? 1.

A) Sole proprietorships offer stock.

2.

B) Corporation shareholders elect the managers of the firm.

3.

C) Sole proprietorships have limited liability.

4.

D) Corporations are the only profitable firms.

6) Which entity produces the greatest proportion of U.S. gross national product?

1.

A) government

2.

B) non-profit organizations such as hospitals

3.

C) firms

4.

D) universities

7) Technological efficiency is 1.

A) a necessary and sufficient condition for profit maximization.

2.

B) a sufficient but not necessary condition for profit maximization.

3.

C) a necessary but not sufficient condition for profit maximization.

4.

D) a theoretical construct with little connection to the real world.

8) In China, firms owned by the government are known as 1.

A) state-owned enterprises (SOEs).

2.

B) government enterprises.

3.

C) corporations.

4.

D) government owned partnerships.

9) Since 1999, the number of small state-owned enterprises (SOEs) in China has 1.

A) decreased, but the large SOEs continue to hold a large proportion of industrialized assets.

2.

B) increased, but large SOEs has decreased.

3.

C) decreased along with the large SOEs.

4.

D) remained unchanged.

10) Firms maximize profit when 1.

A) the additional benefit from producing a good equals the additional cost of producing that good.

2.

B) MR = MC.

3.

C) the derivative of the profit function with respect to output is zero.

4.

D) All of the above.

11) Only corporations benefit from limited liability.

12) What do we mean by efficient production?

6.2 Production

1) Which of the following statements best describes a production function? 1.

A) the maximum profit generated from given levels of inputs

2.

B) the maximum level of output generated from given levels of inputs

3.

C) all levels of output that can be generated from given levels of inputs

4.

D) all levels of inputs that could produce a given level of output

2) With respect to production, the short run is best defined as a time period 1.

A) lasting about six months.

2.

B) lasting about two years.

3.

C) in which all inputs are fixed.

4.

D) in which at least one input is fixed.

3) In the long run, all factors of production are 1.

A) variable.

2.

B) fixed.

3.

C) materials.

4.

D) rented.

4) Which of the following is least likely to be considered a capital input? 1.

A) a sewing machine

2.

B) a tractor

3.

C) a telephone

4.

D) a ten dollar bill

5) A production function tells the firm

1.

A) the maximum it can expect to produce with a given mix of inputs.

2.

B) the average it can expect to produce with a given mix of inputs.

3.

C) the minimum it can expect to produce with a given mix of inputs.

4.

D) the average level of production for other firms in the industry.

6) Production functions only apply to advanced economies.

7) The length of the short run is the same for all firms.

8) Explain why labor might not always be a variable input.

6.3 Short-Run Production: One Variable and One Fixed Input

1) The above figure depicts a short-run production function for Albert's Pretzels. The marginal productivity of labor 1.

A) rises then falls as the amount of capital increases.

2.

B) falls then rises as the amount of labor increases.

3.

C) is greater than or equal to the average productivity of labor for all amounts of labor.

4.

D) is less than or equal to the average productivity of labor for all amounts of labor.

2) The above figure shows the short-run production function for Albert's Pretzels. The marginal productivity of labor for the third worker is 6.

A) 6.

7.

B) 8.

8.

C) 24.

9.

D) not known from the information provided.

3) The above figure shows the short-run production function for Albert's Pretzels. The marginal productivity of labor equals the average productivity of labor 1.

A) for all levels of labor.

2.

B) at none of the levels of labor.

3.

C) only for the first worker.

4.

D) only for the fifth worker.

4) The above figure shows the short-run production function for Albert's Pretzels. The law of diminishing marginal productivity

1.

A) appears with the second worker.

2.

B) has not yet appeared for any of the levels of labor.

3.

C) first appears with the fifth worker.

4.

D) is refuted by this evidence.

5) The above figure shows the short-run production function for Albert's Pretzels. The average product of labor 1.

A) increases first and then decreases.

2.

B) decreases first and then increases.

3.

C) decreases throughout.

4.

D) increases throughout.

6) If the average productivity of labor equals the marginal productivity of labor, then

1.

A) the average productivity of labor is at a maximum.

2.

B) the marginal productivity of labor is at a maximum.

3.

C) Both A and B above.

4.

D) Neither A nor B above.

7) The Average Product of Labor is 1.

A) the change in total product resulting from an extra unit of labor, holding other factors constant.

2.

B) the ratio of output to the number of workers used to produce that output.

3.

C) the amount of output that can be produced by a given amount of labor.

4.

D) equal to the marginal product of labor when the average product is increasing.

8) Total Product is 1.

A) the change in total product resulting from an extra unit of labor, holding other factors constant.

2.

B) the ratio of output to the number of workers used to produce that output.

3.

C) the amount of output that can be produced by a given amount of labor.

4.

D) equal to the marginal product of labor when the average product is increasing.

9) The Marginal Product of Labor is 1.

A) the change in total product resulting from an extra unit of labor, holding other factors constant.

2.

B) the ratio of output to the number of workers used to produce that output.

3.

C) the amount of output that can be produced by a given amount of labor.

4.

D) equal to the marginal product of labor when the average product is increasing.

10) Average productivity will fall as long as 1.

A) marginal productivity is falling.

2.

B) it exceeds marginal productivity.

3.

C) it is less than marginal productivity.

4.

D) the number of workers is increasing.

11) If the marginal productivity of labor is constant for all levels of output, then the average productivity of labor 1.

A) is constant.

2.

B) equals the marginal productivity of labor.

3.

C) Both A and B above.

4.

D) Either A or B above but not both.

12) Joey cuts lawns during the summer. Let q equal the number of acres mowed per day, and let L equal the number of hours worked per day. Joey never works more than eight hours per day, and during that time his short-run production function is q = 0.2 ∗ L. Which of the following statements is FALSE? 1.

A) Joey's marginal productivity equals his average productivity.

2.

B) Joey's marginal productivity diminishes by 0.2 for each additional hour worked.

3.

C) Joey's average productivity is constant.

4.

D) Joey's marginal productivity is constant.

13) The slope of the total product curve always equals 1.

A) the ratio of the marginal product and the average product.

2.

B) the change in input divided by the change in output.

3.

C) the average product of the input.

4.

D) the marginal product of the input.

14 q = 10K0.5 L0.5) Which of the following statements best summarizes the law of diminishing marginal returns?

1.

A) In the short run, as more labor is hired, output diminishes.

2.

B) In the short run, as more labor is hired, output increases at a diminishing rate.

3. 4.

C) In the short run, the amount of labor a firm will hire diminishes as output increases. D) As more labor is hired, the length of time that defines the short run diminishes.

15) Which situation is most likely to exhibit diminishing marginal returns to labor? 1.

A) a factory that obtains a new machine for every new worker hired

2.

B) a factory that hires more workers and never increases the amount of machinery

3.

C) a factory that increases the amount of machinery and holds the number of worker constant

4.

D) None of these situations will result in diminishing marginal returns to labor.

16) Thomas Malthus' prediction of mass starvation resulting from diminishing marginal returns has not been fulfilled because

1.

A) the law of diminishing marginal returns did not hold in this case.

2.

B) Malthus ignored other factors like technological change.

3. 4.

C) relative to Malthus' day, larger percentage of today's labor works in the agricultural sector. D) All of the above.

17) Jennifer is the only employee of her sole proprietorship. She is entertaining the idea of hiring an additional employee. She knows that on her own she can produce 100 units per day. Jennifer figures that Applicant A will help her produce 175 units per day whereas Applicant B will help her produce 155 units per day. Which of the following statements is most accurate? 1.

A) Applicant B has a marginal product of 75 units.

2.

B) Applicant B has an average product of 77.5 units.

3.

C) Applicant A has a marginal product of 75 units.

4.

D) Applicant A has an average product of 87.5 units.

18) A typical professional National Football League team has three quarterbacks on its roster. What is one reason why they might not have a fourth quarterback? 1.

A) The fourth quarterback's marginal product is approximately zero.

2.

B) The fourth quarterback's marginal product is approximately ten.

3. 4.

C) The fourth quarterback's marginal product is less than the first quarterback's marginal product. D) There is a law against carrying four quarterbacks on a team.

19) For a linear production function, q = f(L,K) = 4L + 2K, what is the short-run production function given that capital is fixed at = 50? 1.

A) q = 4L + 100

2.

B) q = 4L + 50

3.

C) q = 4L

4.

D) q = 104

20) Skateboards are produced according to the production function, q = 10K0.25L0.5 L. At what quantity of labor does total product begin to decline given that capital is fixed at 16 units in the short-run? 100.

A) L = 100.

101.

B) L = 10.

102.

C) L = 5.

103.

D) Not enough information is given.

21) Were it not for the law of diminishing marginal returns, we could grow the world's food supply from a flowerpot.

22) A firm operating with diminishing total returns cannot be profit maximizing.

23) The actual time length of the short run is determined by when diminishing marginal returns start.

24) If marginal productivity is decreasing as more labor is hired, then average productivity must be decreasing as well.

25) At low levels of production, marginal productivity of labor increases as labor increases. At high levels of production, marginal productivity of labor decreases as labor increases. Are these two statements contradictory? Explain.

26) Explain the difference between diminishing returns to labor and diminishing marginal returns to labor.

27) Each additional worker produces two extra units of output until the first three units are produced for some production function. After the third unit is produced each additional worker produces one extra unit of output. What are the total product of labor, average product of labor and marginal product of labor for the first five units of production associated with this production function? Answer:

28) Consider the following short-run production function: q = 5L2 - 1/3 L3. At what level of L do diminishing marginal returns begin? At what level of L do diminishing returns begin?

29) Describe the relationship between marginal productivity and average productivity. Use calculus or a graph to support your answer.

30) Suppose the production function for T-shirts can be represented as q = L0.25K0.75. Show that the marginal productivity of labor diminishes in the short run.

6.4 Long-Run Production: Two Variable Inputs

1) At Joey's Lawncutting Service, a lawn mower cannot cut grass without a laborer. A laborer cannot cut grass without a lawn mower. Which graph in the above figure best represents the isoquants for Joey's Lawncutting Service when capital per day is on the vertical axis and labor per day is on the horizontal axis? 1.

A) Graph A

2.

B) Graph B

3.

C) Graph C

4.

D) Graph D

2) In the above figure, Graph D with Capital on the vertical axis and labor on the horizontal axis implies that 1.

A) the marginal product of labor is increasing as more labor is employed.

2.

B) the marginal product of labor is decreasing as more labor is employed.

3.

C) the capital and labor are perfect substitutes.

4.

D) capital and labor have to be employed in fixed proportions.

3) Lectures in microeconomics can be delivered either by an instructor (labor) or a movie (capital) or any combination of both. Each minute of the instructor's time delivers the same amount of information as a minute of the movie. Which graph in the above figure best represents the isoquants for lectures in microeconomics when capital per day is on the vertical axis and labor per day is on the horizontal axis? 1.

A) Graph A

2.

B) Graph B

3.

C) Graph C

4.

D) Graph D

4) Lectures in microeconomics can be delivered either by an instructor (labor) or a movie (capital) or any combination of both. Yet, it gets harder and harder to substitute more movies for an instructor the more movies are already used. Which graph in the above figure best represents the isoquants for lectures in microeconomics when capital per day is on the vertical axis and labor per day is on the horizontal axis? 1.

A) Graph A

2.

B) Graph B

3.

C) Graph C

4.

D) Graph D

5) Which graph in the above figure represents the isoquants where, as the amount of labor used increases and the amount of capital used decreases, the marginal product of labor rises when capital per day is on the vertical axis and labor per day is on the horizontal axis? 1.

A) Graph A

2.

B) Graph B

3.

C) Graph C

4.

D) Graph D

6) To say that isoquants are convex is to say that 1.

A) the marginal rate of technical substitution falls as labor increases.

2.

B) capital and labor are perfect substitutes.

3.

C) labor, but not capital, is subject to the law of diminishing marginal returns.

4.

D) there are constant returns to scale.

7) One way to explain the convexity of isoquants is to say that 1.

A) as labor increases and capital decreases, MPLrises while MPK

2.

B) as labor increases and capital decreases, MPLfalls while MPK

3.

C) as labor increases and capital decreases, MPLand MPKboth fall.

4.

D) as labor increases and capital decreases, MPLand MPKboth rise.

8) An isoquant represents levels of capital and labor that 1.

A) have constant marginal productivity.

2.

B) yield the same level of output.

3.

C) incur the same total cost.

4.

D) All of the above.

9) Suppose the production of paved roadways can be represented as q = L0.5 + K0.5. Which of the following statements is (are) TRUE? 1.

A) Labor is subject to diminishing marginal productivity in the short run.

2.

B) Labor and capital are imperfect substitutes.

3.

C) The isoquants for paved roadways are convex.

4.

D) All of the above.

10) L-shaped isoquants imply that production requires that the inputs 1.

A) are perfect substitutes.

2.

B) are imperfect substitutes.

3.

C) cannot be used together.

4.

D) must be used together in a certain proportion.

11) Isoquants that are downward-sloping straight lines imply that the inputs 1.

A) are perfect substitutes.

2.

B) are imperfect substitutes.

3.

C) cannot be used together.

4.

D) must be used together in a certain proportion.

12) Isoquants that are downward-sloping straight lines exhibit 1.

A) an increasing marginal rate of technical substitution.

2.

B) a decreasing marginal rate of technical substitution.

3.

C) a constant marginal rate of technical substitution.

4.

D) a marginal rate of technical substitution that cannot be determined.

13) The slope of an isoquant tells us 1.

A) how much output increases when both inputs are increased.

2.

B) the increase in MPLwhen capital increases.

3.

C) the decrease in capital necessary to keep output constant when labor increases by one unit.

4.

D) the decrease in capital necessary to keep MPLconstant when labor increases by one unit.

14) The steeper an isoquant is, 1.

A) the greater is the marginal productivity of labor relative to that of capital.

2.

B) the greater is the substitutability between capital and labor.

3.

C) the greater is the need to keep capital and labor in fixed proportions.

4.

D) the greater is the level of output.

15) With capital on the vertical axis and labor on the horizontal axis, vertical isoquants imply that 1.

A) capital and labor are perfect substitutes.

2.

B) capital and labor must be used together in a certain proportion.

3.

C) capital is not productive.

4.

D) labor is not productive.

16) What is the MRTS for a firm with the following production function , q = 10L0.5K0.3? 1.

A) MRTS = -

2.

B) The firm will substitute a constant 3 units of capital for 5 units of labor.

3.

C) The MRTS does not exist because labor and capital are perfect complements.

4.

D) None of the above.

17) How much labor does a firm require to produce q = 1000 when capital is fixed at 5 and they have a production function equal to q = 200L0.5K0.5? 1.

A) L = 5

2.

B) L = 2.5

3.

C) L = 200

4.

D) L = 2.25

18) Unlike indifference curves, isoquants can intersect.

19) If inputs into production cannot be substituted for each other but have to be employed in fixed proportions isoquants are straight, downward-sloping lines.

20) Suppose that additional units of capital affect the marginal productivity of labor. This is not unrealistic. Word processors increase the marginal productivity of secretaries. On the other hand, robotic equipment can completely replace a worker and, thereby, lower the marginal productivity of labor. Given this possibility, determine the conditions under which isoquants will not be convex (i.e.: concave).

21) Suppose the production function for T-shirts can be represented as q = L0.25K0.75. Show that the isoquants for this function are convex.

22) Suppose the production function for T-shirts can be represented as q = L0.25K0.75. When K = 1 and q = 2, what is the slope of the isoquant? If there is insufficient information to answer the question, describe what information is needed.

23) During the "computer revolution" of the 1980s and 1990s, many firms replaced old technology with new technology. What might explain why firms don't change technology as quickly today?

6.5 Returns to Scale

1) Returns to scale refers to the change in output when 1.

A) all inputs increase proportionately.

2.

B) labor increases holding all other inputs fixed.

3.

C) capital equipment is doubled.

4.

D) specialization improves.

2) Decreasing returns to scale may occur as increasing the amount of inputs used 1.

A) increases specialization.

2.

B) always increases the amount of output produced.

3.

C) may cause coordination difficulties.

4.

D) increases efficiency.

3) The table in the above figure shows the levels of output resulting from different levels of inputs. Which of the following conclusions can be drawn from this information? 1.

A) Increasing returns to scale exist between 100 and 200 units of output.

2.

B) Constant returns to scale exist throughout all levels of production.

3.

C) Labor is subject to diminishing marginal productivity in the short run.

4.

D) No firm conclusions can be drawn.

4) The table in the above figure shows the levels of output resulting from different levels of inputs. Returns to scale are greatest at which level of output? 1.

A) 100-200 units

2.

B) 200-400 units

3.

C) 400-600 units

4.

D) There is insufficient information to answer the question.

5) The table in the above figure shows the levels of output resulting from different levels of inputs. At which level of input are there constant returns to scale? 1.

A) 400-600 units

2.

B) Constant returns to scale exist throughout all levels of production.

3.

C) Constant returns to scale do not exist at any level of production.

4.

D) No firm conclusions can be drawn.

6) Let the production function be q = ALaKb. Returns to scale are equal to 1.

A) a ∗

2.

B) a + b.

3.

C) La+ Kb.

4.

D) A ∗

7) Let the production function be q = ALaKb. The function exhibits increasing returns to scale if 1.

A) a + b = 1.

2.

B) a + b > 1.

3.

C) a + b < 1.

4.

D) Cannot be determined with the information given.

8) Let the production function be q = ALaKb. The function exhibits decreasing returns to scale if 1.

A) a + b = 1.

2.

B) a + b > 1.

3.

C) a + b < 1.

4.

D) Cannot be determined with the information given.

9) Let the production function be q = ALaKb. The function exhibits constant returns to scale if 1.

A) a + b = 1.

2.

B) a + b > 1.

3.

C) a + b < 1.

4.

D) Cannot be determined with the information given.

10) Suppose the production of mp3 players can be represented by the following production function: Which of the following statements is (are) TRUE? 1.

A) The production function has decreasing returns to scale.

2.

B) The marginal productivity of labor falls as labor increases in the short run.

3.

C) Capital and labor can be substituted for one another.

4.

D) All of the above.

11) Suppose the production of mp3 players can be represented by the following production function: q = L0.4K0.4. Which of the following statements is TRUE? 1.

A) The production function has decreasing returns to scale.

2.

B) The production function has increasing returns to scale.

3.

C) The production function has constant returns to scale.

4.

D) Returns to scale vary with the level of output.

12) Suppose the production of mp3 players can be represented by the following production function: q = L0.4K0.4. The firm currently produces q1 units. If all inputs doubled, the new level of output will equal 20.

A) 24q1.

21.

B) 28q1.

22.

C) 0.8 q1.

23.

D) 1.6 q1.

13) Returns to scale is a concept that operates 1.

A) only in the short run.

2.

B) only in the long run.

3.

C) in both the long run and the short run.

4.

D) in either the long run or the short run but never both.

14) The above figure shows the isoquants for producing steel. Increasing returns to scale are 1.

A) present when producing less than 10,000 tons.

2.

B) present when producing less than 20,000 tons.

3.

C) present when producing less than 30,000 tons.

4.

D) never present.

15) The above figure shows the isoquants for producing steel. Decreasing returns to scale are

1.

A) present when producing more than 10,000 tons.

2.

B) present when producing more than 20,000 tons.

3.

C) present when producing more than 30,000 tons.

4.

D) never present.

16) The above figure shows the isoquants for producing steel. Constant returns to scale are 1.

A) present when producing less than 10,000 tons.

2.

B) present when producing between 10,000 and 20,000 tons.

3.

C) present when producing more than 20,000 tons.

4.

D) never present.

17) The above figure shows the isoquants for producing steel. When producing more than 20,000 tons there are 1.

A) increasing returns to scale.

2.

B) decreasing returns to scale.

3.

C) constant returns to scale.

4.

D) economies of scale.

18) The above figure shows the isoquants for producing steel. When producing less than 10,000 tons there are 1.

A) increasing returns to scale.

2.

B) decreasing returns to scale.

3.

C) constant returns to scale.

4.

D) diseconomies of scale.

19) The above figure shows the isoquants for producing steel. When producing between 10,000 and 20,000 tons there are 1.

A) increasing returns to scale.

2.

B) decreasing returns to scale.

3.

C) constant returns to scale.

4.

D) economies of scale.

20) If a farmer produces 1000 bushels of corn using ten acres of land and one tractor and is able to produce 2000 bushels of corn using twenty acres of land and one tractor, the farmer has 1.

A) increasing returns to scale.

2.

B) constant returns to scale.

3.

C) decreasing returns to scale.

4.

D) no returns to scale.

21) What is one reason "micro-managers" might be less successful than so-called "delegators"? 1.

A) Delegators are smarter.

2.

B) Delegators enjoy increasing returns to scale.

3.

C) Micro-managers suffer decreasing returns to scale.

4.

D) Micro-managers have constant returns to scale.

22) In the Primary Metals industry, it is estimated that the elasticity of output with respect to labor is 0.51 and the elasticity of output with respect to capital is 0.73. These two measures indicate that the primary metals industry is characterized by 1.

A) decreasing returns to scale.

2.

B) constant returns to scale.

3.

C) increasing returns to scale.

4.

D) no returns to scale.

23) In the food and kindred products industry, it is estimated that the elasticity of output with respect to labor is 0.43 and the elasticity of output with respect to capital is 0.48. These two measures indicate that the primary metals industry is characterized by 1.

A) decreasing returns to scale.

2.

B) constant returns to scale.

3.

C) increasing returns to scale.

4.

D) no returns to scale.

24) Does this production function, q = 10L0.5K0.3, experience increasing, decreasing or constant returns to scale? 1.

A) Decreasing because a 100% increase in inputs increases outputs by 80%.

2.

B) Increasing because an 80% increase in inputs increases outputs by 100%.

3.

C) Decreasing because 0.5 + 0.3 < 1.

4.

D) A and C.

25) How much do outputs increase when labor and capital increase from 1 to 2 units for the following production function , q = 10L0.5K0.3? 7.

A) 7.4 units.

8.

B) 7 units.

9.

C) 8 units.

10.

D) None of the above.

26) Cobb-Douglas production functions can never possess varying returns to scale.

27) A firm can experience increasing, constant and decreasing returns to scale for various levels of output.

28) Show that increasing returns to scale can co-exist with diminishing marginal productivity.

The first three rows illustrate diminishing marginal returns. Comparing rows 1 and 4 shows increasing returns to scale.

29) Suppose the production function for T-shirts can be represented as q = L0.25K0.75. Show that the production function has constant returns to scale.

30) Provide a graph and an explanation to show that the production function Q = L0.5K0.5 has diminishing marginal product of labor but has constant returns to scale. Answer: See the above figure. The MPL = .5 ∗ L-0.5K0.5 so as the amount of labor increases the MPL falls. The convex shape of the isoquants illustrates the diminishing marginal product of labor. Doubling both capital and labor will result in output doubling.

31) The above figure shows the isoquants for the production of steel. In which regions of production are there increasing, decreasing, and constant returns to scale?

32) Production occurs using the fixed-proportion combination of one worker for every one unit of capital. Draw the isoquants for this production function. After one unit of output is produced, does this production function exhibit constant, increasing, or decreasing returns to scale? Answer: See the above figure. The production function exhibits constant returns to scale.

6.6 Productivity and Technical Change

1) Joey's lawncutting service recently traded in its push mowers for gasoline-powered mowers. Joey still requires one worker per lawnmower; however, more grass is now cut in the same amount of time as before. This is an example of

1.

A) labor-saving technical change.

2.

B) non-neutral technical change.

3.

C) neutral technical change.

4.

D) organizational change.

2) Suppose the production function for a certain device is q = L + K. If a labor-saving technical change has occurred, which of the following could be the new production function? 1.

A) q = L + 5K

2.

B) q = 5 ∗ (L + K)

3.

C) q = 5L + K

4.

D) All of the above are possible.

3) Suppose the production function for a certain device is q = L + K. If neutral technical change has occurred, which of the following could be the new production function? 1.

A) q = L + 5K

2.

B) q = 5 ∗ (L + K)

3.

C) q = 5L + K

4.

D) All of the above are possible.

4) Albert's Pretzel Baking Company used to have four workers who were each in charge of making their own pretzels from start to finish. Now, one worker mixes the dough, another shapes it, the third puts the unbaked pretzels into the oven and the fourth removes the finished product. Output has doubled. This is an example of 1.

A) neutral technical change.

2.

B) labor-saving technical change.

3.

C) division of labor.

4.

D) economies of scale.

5) Over a five-year span, the ABC Co. reduced the amount of labor it hired. At the same time, the marginal productivity of labor increased. Which of the following COULD explain this observation? 1.

A) the law of diminishing marginal returns

2.

B) labor saving technical change

3.

C) organizational innovation

4.

D) All of the above.

6) The implementation of the assembly line is an example of how 1.

A) changes in the organization of production improve productivity.

2.

B) neutral technical change improves productivity.

3.

C) non-neutral technical change can decrease productivity.

4.

D) labor saving technical change increases economy-wide unemployment.

7) Dell computers has increased production efficiency by

1.

A) producing output with fewer inputs.

2.

B) expanding the amount of inputs used.

3.

C) outsourcing production.

4.

D) relying on decreasing returns to scale.

8) Technical and organizational change 1.

A) typically reduces prices by increasing the supply of the product, ceteris paribus.

2.

B) typically reduces prices by decreasing the demand for the product, ceteris paribus.

3.

C) typically increases prices by increasing the demand for the product, ceteris paribus.

4.

D) typically increases prices by decreasing the supply for the product, ceteris paribus.

9) If a technological change occurs such that the production function shifts from q = 10K0.5 L0.5 to q = 10.5K0.3 L0.5, then the technological change is 1.

A) both neutral and non-neutral.

2.

B) neutral.

3.

C) non-neutral.

4.

D) Not enough information given.

10) Non-neutral technological change is more valuable to labor than neutral technological change.

11) Changing how production is organized cannot result in changes in productivity.

12) Suppose firms A and B each make T-shirts. Firm A's production function is q = L0.5K0.5. Firm B's production function is q = 1.2 ∗ L0.5K0.5. If the two firms each hire the same amounts of capital and labor, compare the two firms in terms of APL and MPL.

13) Explain how firms that each produce as efficiently as they can may not be equally productive.

Microeconomics, 7e (Perloff) Chapter 7 Costs

7.1 The Nature of Costs

1) Economic costs of an input include 1.

A) only implicit costs.

2.

B) only explicit costs.

3.

C) both implicit and explicit costs.

4.

D) whatever management wishes to report to the shareholders.

2) The cost of waiting two months for health care to address a debilitating problem in Canada is most accurately described as 1.

A) an explicit cost.

2.

B) an accounting cost.

3.

C) no real cost.

4.

D) an opportunity cost.

3) Sarah earns $40,000 per year working for a large corporation. She is thinking of quitting this job to work full time in her own business. She will invest her savings of $50,000 (which currently has an annual 10% rate of return) into the business. Her annual opportunity cost of this new business is 1.

A) $0.

2.

B) $40,000.

3.

C) $45,000.

4.

D) $90,000.

4) Johnny has worked as a CPA for five years and wants to open his own public accounting practice. The cost of his college degree in accounting represents 1.

A) the opportunity cost of this endeavor.

2.

B) a sunk cost.

3.

C) an expense.

4.

D) a variable cost.

5) Economists proclaim that competitive firms make zero economic profit in the long run. This shows how 1.

A) detached economists are from the real world.

2.

B) unrealistic economic theory is.

3.

C) firms cover all their cost, both monetary and non-monetary.

4.

D) firms cover only monetary cost when economic profits are zero.

6) If a firm buys a building so as to have office space for its workers, the monthly opportunity cost of the building is best measured as 1.

A) the monthly mortgage payment the firm must pay.

2.

B) the price the firm paid divided by twelve.

3.

C) zero.

4.

D) the rent the firm could earn if it rented the building to another firm.

7) Economic efficiency entails 1.

A) producing a given amount of output with the most expensive mix of inputs.

2.

B) producing a given amount of output with the least number of inputs.

3.

C) producing a given amount of output with the most inputs.

4.

D) producing a given amount of output with the cheapest mix of inputs.

8) If Option A costs $40 and yields 20 units of output and Option B costs $50 and yields 20 units of output, 1.

A) Option B and Option A are equally economically efficient.

2.

B) Option B is economically efficient relative to Option A.

3.

C) Option A is economically efficient relative to Option B.

4.

D) It is not possible to determine which option is more economically efficient.

9) If Option A costs $40 and yields 20 units of output and Option B costs $50 and yields 30 units of output, 1.

A) Option B and Option A are equally economically efficient.

2.

B) Option B is economically efficient relative to Option A.

3.

C) Option A is economically efficient relative to Option B.

4.

D) It is not possible to determine which option is more economically efficient.

10) Why might a police officer not pull over someone speeding two miles over the speed limit? 1.

A) The explicit costs of stopping the driver over are too high.

2.

B) The opportunity costs of stopping the driver are too high.

3.

C) The opportunity costs of topping he driver are too low.

4.

D) The explicit costs of stopping the driver are too low.

11) The high cost of advertising during the Super Bowl will 1.

A) not affect the efficient level of output because advertising is a sunk cost.

2.

B) will affect the efficient level of output because profits will fall significantly.

3.

C) not affect the efficient level of output because advertising is a fixed cost.

4.

D) Not enough information given.

12) When buying a piece of equipment, it is always best for the firm to pay cash instead of borrowing the funds since this renders the equipment less costly.

13) Four years after graduating from college you must decide if you want to go on as an accountant (your college major) or if you want to make a career change and become a singer. The cost of your education will matter for your decision.

14) An accountant may amortize the expense of a durable good by dividing the total amount spent on the good by the number of years the good is expected to last. An economist may amortize the expense of a durable and never fully account for the total expense.

15) Your company makes copper pipes. Over the years, you have collected a large inventory of raw copper. The production process involves melting the copper and shaping it into pipes. You also have a large stockpile of pennies. Suppose the price of copper rises so much that the copper in the penny becomes worth more than one cent. Should you melt down your pennies?

16) You have two career options. You can work for someone else for $50,000 a year, or, you can run your own business, with an annual revenue of $100,000, and explicit costs of $40,000 annually. Explain which career option a profit-maximizer would select and why.

7.2 Short-Run Costs

1) A firm's marginal cost can always be thought of as the change in total cost if 1.

A) the firm produces one more unit of output.

2.

B) the firm buys one more unit of capital.

3.

C) the firm's average cost increases by $1.

4.

D) the firm moves to the next highest isoquant.

2) Fixed costs are 1.

A) a production expense that does not vary with output.

2.

B) a production expense that changes with the quantity of output produced.

3.

C) equal to total cost divided by the units of output produced.

4.

D) the amount by which a firm's cost changes if the firm produces one more unit of output.

3) Variable costs are 1.

A) a production expense that does not vary with output.

2.

B) a production expense that changes with the quantity of output produced.

3.

C) equal to total cost divided by the units of output produced.

4.

D) the amount by which a firm's cost changes if the firm produces one more unit of output.

4) Which of the following statements is NOT true? 1.

A) AC = AFC + AVC

2.

B) C = F + VC

3.

C) AVC = wage/MPL

4.

D) AFC = AC - AVC

5) Joey's Lawncutting Service rents office space from Joey's dad for $300 per month. Joey's dad is thinking of increasing the rent to $400 per month. As a result Joey's marginal cost of cutting grass will 1.

A) increase by $100 divided by the amount of grass cut.

2.

B) increase by $100.

3.

C) decrease by $100.

4.

D) not change.

6) Suppose a firm can only vary the quantity of labor hired in the short run. An increase in the cost of capital will 1.

A) increase the firm's marginal cost.

2.

B) decrease the firm's marginal cost.

3.

C) have no effect on the firm's marginal cost.

4.

D) More information is needed to answer the question.

7) Suppose the total cost of producing T-shirts can be represented as TC = 50 + 2q. The marginal cost of the 5th T-shirt is 2.

A) 2.

3.

B) 10.

4.

C) 12.

5.

D) 60.

8) Suppose the total cost of producing T-shirts can be represented as TC = 50 + 2q. The average cost of the 5th T-shirt is 2.

A) 2.

3.

B) 12.

4.

C) 52.

5.

D) 60.

9) Suppose the total cost of producing T-shirts can be represented as TC = 50 + 2q. Which of the following statements is TRUE at all levels of production? 1.

A) MC = AVC

2.

B) MC = AC

3.

C) MC > AFC

4.

D) All of the above.

10) Suppose the short-run production function is q = 10 ∗ L. If the wage rate is $10 per unit of labor, then AVC equals 1.

A) q.

2.

B) q/10.

3.

C) 10/q.

4.

D) 1.

11) Suppose the short-run production function is q = 10 ∗ L. If the wage rate is $10 per unit of labor, then MC equals 1.

A) q.

2.

B) q/10.

3.

C) 10/q.

4.

D) 1.

12) If average cost is decreasing, 1.

A) marginal cost equals average cost.

2.

B) marginal cost exceeds average cost.

3.

C) marginal cost is less than average cost.

4.

D) Not enough information is given.

13) If average cost is positive, 1.

A) marginal cost equals average cost.

2.

B) marginal cost exceeds average cost.

3.

C) marginal cost is less average cost.

4.

D) Not enough information is given.

14) Suppose the short-run production function is q = L0.5. If the marginal cost of producing the tenth unit is $5, what is the wage per unit of labor?

1.

A) $1

2.

B) $0.5

3.

C) $0.25

4.

D) It cannot be determined without more information.

15) Suppose the short-run production function is q = 10 ∗ L. If the wage rate is $10 per unit of labor, then AFC equals 1.

A) 0.

2.

B) 1.

3.

C) 10/q.

4.

D) It cannot be determined from the information provided.

16) When a firm produces one unit, the variable cost is $3. When the firm produces two units, the variable cost is $6. What is the marginal cost associated with two units of production? 1.

A) $2

2.

B) $0.5

3.

C) $6

4.

D) $3

17) If the marginal cost of producing a good is increasing as a firm produces more of the good, then which of the following must be TRUE? 1.

A) AFC is rising.

2.

B) AVC is rising.

3.

C) MC > AVC.

4.

D) MPL is falling.

18) If the average cost of producing a good is increasing as a firm produces more of the good, then which of the following must be TRUE? 1.

A) AFC is falling.

2.

B) AVC is rising.

3.

C) MC > AVC.

4.

D) All of the above.

19) Suppose each worker must use only one shovel to dig a trench, and shovels are useless by themselves. In the short run, an increase in the price of shovels will result in 1.

A) fewer shovels being purchased.

2.

B) more workers being hired.

3.

C) a decrease in the firm's output.

4.

D) no change in the firm's output.

20) Which of the following will cause the marginal cost curve of making cigarettes to shift? 1.

A) a $5 million penalty charged to each cigarette maker

2.

B) a $1 per pack tax on cigarettes

3.

C) a $1 million advertising campaign by the American Cancer Society

4.

D) All of the above.

21) Which of the following will cause the average cost curve of making cigarettes to shift? 1.

A) a $5 million penalty charged to each cigarette maker

2.

B) a $1 per pack tax on cigarettes

3.

C) a $1 an hour wage increase paid to all cigarette production workers

4.

D) All of the above.

22) Which of the following will cause the average fixed cost curve of making cigarettes to shift?

1.

A) a $5 million penalty charged to each cigarette maker

2.

B) a $1 per pack tax on cigarettes

3.

C) a $3 per hour wage increase

4.

D) An increase in the demand for cigarettes.

23) A specific tax of $1 per unit of output will affect a firm's 1.

A) average total cost, average variable cost, average fixed cost, and marginal cost.

2.

B) average total cost, average variable cost, and average fixed cost.

3.

C) average total cost, average variable cost, and marginal cost.

4.

D) marginal cost only.

24) Assume baseball player salaries are fully determined before the season starts. From the point of view of the baseball team owner, player salaries during the course of the season are then 1.

A) variable costs.

2.

B) marginal costs.

3.

C) fixed costs.

4.

D) average costs.

25) A firm's cost curve is determined by 1.

A) congressional laws.

2.

B) whether the firm hires engineers or not.

3.

C) natural laws.

4.

D) the firm's production function.

26) A windfall profit tax imposed on oil companies would shift the firms'

1.

A) marginal tax rate.

2.

B) marginal cost curve.

3.

C) average cost curve.

4.

D) production function.

27) Assume Congress decides that oil companies are making too much profit and decides to tax oil companies for each gallon of gasoline produced. This would 1.

A) shift the marginal cost curve up.

2.

B) shift the marginal cost curve down.

3.

C) shift the average fixed cost curve up.

4.

D) shift the average fixed cost curve down.

28) Assume Congress decides that Social Security taxes must increase in order to fund the system. This would

1.

A) shift up the marginal cost curve for any firms that hire labor.

2.

B) guarantee a decrease in profits.

3.

C) shift up the average fixed cost curve for any firms that hire labor.

4.

D) guarantee an increase in tax revenues.

29) In the short run, marginal cost is increasing when 1.

A) MPLis decreasing.

2.

B) MPLis increasing.

3.

C) APLis increasing.

4.

D) APLis decreasing.

30) In the short run, marginal cost is minimized when 1.

A) MPLis maximized.

2.

B) MPLequals zero.

3.

C) APLis maximized.

4.

D) APLequals zero.

31) In the short run, average variable costs are minimized when 1.

A) MPLequals APL.

2.

B) APLis maximized.

3.

C) MPLis maximized and APLis increasing.

4.

D) Both A and B.

32) At what output level, q, is average cost minimized if the short-run costs for a company are C = 40 + 4q + q2? 2.

A) q = 2.

3.

B) q = 0.

4.

C) q = 40.

5.

D) q = 2.

33) The "Law of Diminishing Marginal Returns" could also be termed the "Law of Increasing Marginal Costs."

34) The marginal cost curve intersects the average fixed cost curve at its minimum.

35) A consumer purchases a book by driving across town to a bookstore, standing in line for five minutes to pay the cashier, and then pays $5. The same book is purchased

by another consumer who spends 2 minutes placing the order over the internet for $10. The book necessarily cost the first consumer less.

36) A firm's production function for pretzels is shown in the above figure. If the firm's fixed cost equals $100 per time period and the wage rate equals $1 per unit of labor per time period, calculate the firm's MC, AVC, and AC schedules. Do these cost functions follow the general rules concerning the relationships between MC, AVC and AC? Answer:

MC > AVC, AVC is rising. MC < AC, AC is falling.

37) Explain why the marginal cost curve intersects a U-shaped average cost curve at its minimum point. t low quantities, the average cost curve declines as the quantity increases. The marginal cost is below the average cost. The marginal cost represents the cost of an additional unit of production. Thus, as the marginal cost curve declines, this pulls the average cost down from its previous level. Then, the marginal cost curve will begin to rise. However, the marginal cost is still below the average cost, and will continue to lower the average

cost. When the two costs are equal the marginal cost will leave the average cost unchanged. Then, the marginal cost will be above the average cost so it will start to pull up the average cost. Thus, the marginal cost curve will intersect the average cost curve at its minimum point.

7.3 Long-Run Costs

1) In the long run, fixed costs are 1.

A) sunk.

2.

B) avoidable.

3.

C) larger than in the short run.

4.

D) not included in production decisions.

2) The slope of the isocost line tells the firm how much

1.

A) capital must be reduced to keep total cost constant when hiring one more unit of labor.

2.

B) capital must be increased to keep total cost constant when hiring one more unit of labor.

3.

C) more expensive a unit of capital costs relative a unit of labor.

4.

D) the isocost curve will shift outward if the firm wishes to produce more.

3) The slope of the isoquant tells the firm how much 1.

A) output increases when labor increases by one unit.

2.

B) output increases when capital and labor are doubled.

3. 4.

C) capital must decrease to keep output constant when labor increases by one unit. D) a unit of capital costs relative to the cost of labor.

4) If an isocost line crosses the isoquant twice, a cost minimizing firm will

1.

A) use a different isocost line to select the bundle of inputs.

2.

B) use the input bundle associated with the intersection on the higher point of the isoquant.

3.

C) use the input bundle associated with the intersection on the lower point of the isoquant.

4.

D) Both B and C.

5) When the isocost line is tangent to the isoquant, then 1.

A) MRTS = -w/r.

2.

B) the firm is producing that level of output at minimum cost.

3. 4.

C) the last dollar spent on capital yields as much extra output as the last dollar spent on labor. D) All of the above.

6) A firm can minimize cost by 1.

A) picking the bundle of inputs where the lowest isocost line touches the isoquant.

2.

B) picking the bundle of inputs where the isoquant is tangent to the isocost line.

3.

C) picking the bundle of inputs where the last dollar spent on one input gives as much extra output as the last dollar spent on any other input.

4.

D) All of the above.

7) When the isocost line is tangent to the isoquant, then 1.

A) MPL = MPK.

2.

B) the firm is producing that level of output at minimum cost.

3.

C) the firm has achieved the right economies of scale.

4.

D) All of the above.

8) If the cost of labor increases, the isocost line will

1.

A) stay the same.

2.

B) shift outward in parallel fashion.

3.

C) rotate inward around the point where only capital is employed in production.

4.

D) shift inward in parallel fashion.

9) If the isoquants are straight lines or L-shaped, then a cost-minimizing firm will 1.

A) not be able to minimize costs.

2.

B) find the lowest isocost line touching the relevant isoquant.

3.

C) find the highest isocost line touching the relevant isoquant.

4.

D) choose not to produce any output.

10) If the marginal rate of technical substitution for a cost minimizing firm is 10, and the wage rate for labor is $5, what is the rental rate for capital in dollars? 1.

A) .5

2.

B) 1

3.

C) 2

4.

D) 10

11) Suppose MPL = 0.5 ∗ (q/L) and MPK = 0.5 ∗ (q/K). In the long run, the firm will hire equal amounts of capital and labor 1.

A) all of the time.

2.

B) only when w = r.

3.

C) only when w = 0.5 ∗

4.

D) at no point in time.

12) Suppose that each worker must use only one shovel to dig a trench, and shovels are useless by themselves. In the long run, an increase in the price of shovels will result in 1.

A) fewer shovels being purchased to produce the same number of trenches.

2.

B) more workers being hired to produce the same number of trenches.

3.

C) the firm wishing to produce more trenches.

4.

D) no change in the firm's input mix.

13) Suppose that each worker must use only one shovel to dig a trench, and shovels are useless by themselves. In the long run, the firm will experience 1.

A) increasing returns to scale.

2.

B) constant returns to scale.

3.

C) decreasing returns to scale.

4.

D) The returns to scale cannot be determined from the information provided.

14) Suppose that each worker must use only one shovel to dig a trench, and shovels are useless by themselves. In the long run, the firm's cost function is 1.

A) TC = (w/r) ∗

2.

B) TC = (w + r)/q.

3.

C) TC = (w + r).

4.

D) TC = (w + r) ∗

15) At the XYZ Co., a unit of capital costs 3 times as much as a unit of labor. If the isoquants are convex, and the firm does not change its input mix in the long run, we can conclude that 1.

A) MPK= 3 ∗ MPL.

2.

B) the firm will not hire any capital.

3.

C) the firm will hire 3 times as much labor as capital.

4.

D) the firm will hire 3 times as much capital as labor.

16) The production of cigarettes is highly automated; however, a worker is required to monitor each machine. Machines and workers do not interact with one another. Given this information, there are most likely 1.

A) economies of scale.

2.

B) economies of scope.

3.

C) constant returns to scale.

4.

D) increasing returns to scale.

17) A change in relative factor prices will always result in 1.

A) a change in the slope of the isoquants.

2.

B) a tangency between the new isocost line and a new isoquant.

3.

C) a rotation of the isocost lines.

4.

D) All of the above.

18) Assuming that w and r are both positive, if the long-run expansion path is horizontal, then 1.

A) MPK= 0.

2.

B) MRTS is a function of capital only.

3.

C) w = r.

4.

D) All of the above.

19) The above figure shows the long-run expansion path. The long-run average cost curve will be 1.

A) horizontal.

2.

B) downward sloping.

3.

C) upward sloping.

4.

D) vertical.

20) The long-run average cost curve may initially slope downward due to 1.

A) decreasing average fixed costs.

2.

B) increasing marginal returns.

3.

C) economies of scale.

4.

D) All of the above.

21) If a production function is represented as q = LaKb, the long-run average cost curve will be horizontal as long as 1.

A) a + b = 0.

2.

B) a + b = 1.

3.

C) q > 0.

4.

D) L = K.

22) If there are diseconomies of scale within a given range of output, which of following is(are) TRUE?

1.

A) The short-run average cost curve must be upward sloping within that range of output.

2.

B) The long-run average cost curve must be upward sloping within that range of output.

3.

C) Long-run average cost must equal short-run average cost.

4.

D) All of the above.

23) The total cost of producing one unit is $50. The total cost of producing two units is $75. At a production level of two units, the cost function exhibits 1.

A) economies of scale.

2.

B) rising average costs.

3.

C) increasing marginal costs.

4.

D) constant returns to scale.

24) Economies of scale in nuclear power plants exist because of 1.

A) more efficient plant management.

2.

B) a better understanding of the plant's idiosyncrasies, or learning-by-doing.

3.

C) nuclear power technology changes.

4.

D) both A and B.

25) A U.S. firm currently produces 200 units of output according to the production function q = L0.5K0.5 and faces input prices equal to wU.S. = rU.S = $11. Should the U.S. firm move their company abroad where they will face input prices equal to wabroad = $6.50 and rabroad = $15.00? 810.

A) Yes, because the total costs will fall from $3,859 to $2,810.

811.

B) No, because the total costs will increase from $2,810 to $3,859.

812.

C) No, because the firm has decreasing returns to scale.

813.

D) Not enough information is given to answer this problem.

26) A firm produces output according to the production function, q = L4/3K1/2 and faces input prices equal to w = $20 and r = $80. What is the minimum cost of producing 1140 units of output? 780.

A) Cost = $780.

781.

B) Cost = $694

782.

C) Cost = $2,071.

783.

D) Not enough information is given to answer this problem.

27) If increasing returns to scale are present, the long-run average cost increases as more output is produced.

28) Economies of scale and Increasing Returns to Scale are the same thing looked at from either the production or cost perspective.

29) Explain the difference between fixed costs in the short run and fixed costs in the long run.

30) What are the functions for MC and AC if TC = 100q + 100q2? Are the returns to scale increasing, decreasing, or constant?

31) Explain how a firm can have constant returns to scale in production and economies of scale in cost.

32) Explain why the long-run total cost curve, not the short-run total cost curve, shows the lowest cost of producing any level of output. Is there an exception?

33) A local non-profit group prints a weekly newsletter. Professional typists earn $10 per hour and can type 2 pages per hour. Unpaid volunteers can type only 1 page per hour. Measuring hours of professional typist services on the vertical axis and hours of unpaid volunteer typist services on the horizontal axis, draw the relevant isoquant and isocost curves if the newsletter is 10 pages long. What input mix is chosen by the non-profit group if they wish to minimize the cost of the newsletter? If the group will reimburse volunteers for expenses (lunch, driving), how much must the reimbursement be for your answer to change? Answer: See the above figure. The isoquant is a straight line with a slope of 1/2. The group can use either 5 pros, 10 volunteers, or some combination of the two. The isocost curves are horizontal since volunteers are free. As a result, the lowest isocost curve is achieved by hiring all volunteers. If the group reimbursed volunteers for expenses (lunch, driving, etc.) then the group will still hire all volunteers as long as they do not cost more than $5 per hour.

34) A firm pays $5 for each unit of capital. Labor costs $5 per hour for the first 10 hours and $10 per hour for every hour thereafter. Draw the isocost curves for total costs of $50 and $100.

Answer: See the above figure.

35) To dig a trench, each worker needs a shovel. Workers can use only one shovel at a time. Workers without shovels do nothing, and shovels cannot operate on their own. Graphically determine the number of shovels and workers used by a firm to dig 2 trenches when: (a) w = 10 and r = 10 (b) w = 10 and r = 5 Answer: See the above figure. Because the production process requires fixed proportions of K and L, the firm cannot change input mix when the relative factor costs change.

36) Suppose the production function is q = 12 L0.25K0.75. Determine the long-run capital-to-labor ratio (K/L) if the cost a unit of capital (r) is three times the cost of a unit of labor (w).

37) "If the wage rate paid to one form of labor is twice the cost of another form of labor, the first type of labor must be twice as productive." Comment.

38) Suppose capital and labor are perfect substitutes resulting in a production function of q = K + L. That is, the isoquants are straight lines with a slope of -1. Derive the longrun total cost function TC = C(q) when the wage rate is w and the rental rate on capital is r.

7.4 Lower Costs in the Long Run

1) Long-run average cost is never greater than short-run average cost because in the long run, 1.

A) capital costs equal zero.

2.

B) the firm can move to the lowest possible isocost curve.

3.

C) wages always increase over time.

4.

D) wages always decrease over time.

2) Which of the following statements best explains why long-run average cost is never greater than short-run average cost? 1.

A) In the long run, tangency of the isocost and isoquant is attainable. This is not necessarily true in the short run.

2.

B) In the long run, diseconomies of scale might not occur, but in the short run diminishing marginal returns do.

3.

C) In the long run, the cost of capital declines because the firm is able to pay down some of its debts.

4.

D) In the long run, the average cost curve need not be U-shaped, but in the short run it is.

3) In the short run, the expansion path is 1.

A) horizontal.

2.

B) vertical.

3.

C) diagonal.

4.

D) indeterminate.

4) In the long run, the expansion path is 1.

A) horizontal.

2.

B) vertical.

3.

C) diagonal.

4.

D) Not enough information.

5) Learning by doing will result in 1.

A) an upward sloping long-run average cost curve.

2.

B) a larger long-run marginal cost than long-run average cost.

3.

C) a rotation in the isocost curves.

4.

D) lower long-run costs than short-run costs.

6) Learning by doing is represented by 1.

A) a decrease in the average total cost curve.

2.

B) an increase in the average total cost curve.

3.

C) no change in the average total cost curve.

4.

D) an increase in the average total cost curve and a decrease in the marginal cost curve.

7) Why do many people choose to not read the manuals included with their new computer? 1.

A) They perceive that learning by doing decreases costs faster than learning by reading.

2.

B) They perceive that learning by doing is more enjoyable than learning by reading.

3.

C) They perceive that learning by reading is not sophisticated.

4.

D) They perceive that learning by doing is better than learning by reading.

8) A firm is currently producing1140 units of output according to the production function q = L4/3K1/2 and faces input prices equal to w = $20 and r = $80. In the short run, capital is fixed at 5 units. In the long run, the firm's costs are 1.

A) lower because the firm substitutes towards more labor and away from capital.

2.

B) lower because the firm substitutes towards more capital and away from labor.

3.

C) higher because the firm substitutes towards more labor and away from capital.

4.

D) higher because the firm substitutes towards more capital and away from labor.

9) Short-run average cost exceeds long-run average cost only when there are economies of scale.

10) Short-run costs are never equal or lower than long-run cost.

11) At Albert's Pretzel Company, MPL = 1/L, and MPK = 1/K. The isoquant for 100 pounds of pretzels daily is shown in the above figure. Albert minimizes the cost of producing 100 pounds of pretzels daily by hiring 5 units of labor and 10 units of capital when w = 50 and r = 25. When r rises to 100, what is the minimum cost of producing 100 pounds of pretzels daily in the short run? in the long run?

7.5 Cost of Producing Multiple Goods

1) Many universities have either a top football program OR a top basketball program. Very few have both. These results suggest the presence of 1.

A) economies of scope.

2.

B) diseconomies of scope.

3.

C) returns to scale.

4.

D) the law of diminishing marginal returns.

2) Economies of scope exist between book publishing and magazine publishing if 1.

A) the cost of publishing a magazine is lower for book publishers than for other firms.

2.

B) the cost of publishing a magazine is lower for firms that publish many magazines than for firms that publish only one magazine.

3.

C) the cost of publishing a book falls over time as the publisher acquires more experience.

4.

D) the cost of a publishing a book is not subject to diminishing marginal returns.

3) Suppose the cost of producing two goods, x and y, can be represented as C = ax + by + cxy. If the measure of economies of scope, SC, is zero, then which of the following must be true? 1.

A) a = b

2.

B) a + b = -c

3.

C) c = 0

4.

D) a = -b

4) Suppose the cost of producing two goods, x and y, can be represented as C = ax + by + cxy. If there are economies of scope, then which of the following must be true? 1.

A) c > 0

2.

B) a + b = -c

3.

C) c = 0

4.

D) c < 0

5) Suppose the cost of producing two goods, x and y, can be represented as C = ax + by + cxy. If there are diseconomies of scope, then which of the following must be true? 1.

A) a = b

2.

B) a + b = -c

3.

C) c > 0

4.

D) c < 0

6) A production possibilities frontier that is a downward-sloping straight line implies 1.

A) economies of scale.

2.

B) diseconomies of scale.

3.

C) economies of scope.

4.

D) no economies of scope.

7) A production possibilities frontier that is bowed-inward implies 1.

A) economies of scale.

2.

B) diseconomies of scope.

3.

C) economies of scope.

4.

D) no economies of scope.

Question Status: Revised

8) What is one possible reason a firm that produces balloons might outsource its accounting department? 1.

A) There is no longer economies of scale.

2.

B) There are now economies of scope.

3.

C) There is no longer economies of scope.

4.

D) There are now economies of scale.

9) Travel websites such as Travelocity tend to offer reservation services for multiple travel modes. This is because 1.

A) the firms have contractual obligations to offer reservations for airlines and railroads, for example.

2.

B) the firms have statutory obligations to offer reservations for airlines and railroads, for example.

3.

C) once the firm has the reservation technology for airlines, there are economies of scale in offering the same service for railroads.

4.

D) once the firm has the reservation technology for airlines, there are economies of scope in offering the same service for railroads.

10) Wood Working Company (WWC) can produce tables for a cost equal to C = 30 + qt or chairs for a cost equal to C = 30 + qc. However, the firm can also produce tables and chairs simultaneously for a cost equal to C = 50 + qc + qt. Does WWC experience economies of scope? 1.

A) Yes, with SC = .

2.

B) Yes, with SC = .

3.

C) No, with SC = - .

4.

D) No, with SC = 0.

11) If a pharmaceutical firm is researching ways to improve its heartburn medicine and discovers a technique that will improve its allergy medicine, one could conclude that economies of scope exist in that industry.

12) Explain why in the case of economies of scope the production possibility frontier is bowed outward.

Microeconomics, 7e (Perloff) Chapter 8 Competitive Firms and Markets

8.1 Perfect Competition

1) Economists define a market to be competitive when the firms 1.

A) spend large amounts of money on advertising to lure customers away from the competition.

2.

B) watch each other's behavior closely.

3.

C) are price takers.

4.

D) All of the above.

2) A market's structure is described by 1.

A) the number of firms in the market.

2.

B) the ease with which firms can enter and exit the market.

3.

C) the ability of firms to differentiate their product.

4.

D) All of the above.

3) Firms that exhibit price-taking behavior 1. 2.

A) wait for other firms to set price, take it as given, and charge a higher price. B) have outputs that are too small to influence market price and thus take it as given.

3.

C) take pricing behavior in their own hands.

4.

D) are independently capable of setting price.

4) If consumers view the output of any firm in a market to be identical to the output of any other firm in the market, the demand curve for the output of any given firm

1.

A) will be identical to the market demand curve.

2.

B) will be horizontal.

3.

C) will be vertical.

4.

D) cannot be determined from the information given.

5) A horizontal demand curve for a firm implies that 1.

A) the firm is a monopoly.

2.

B) the market the firm is operating in is not competitive.

3.

C) the firm is selling in a competitive market.

4.

D) the products of that firm are very different from other firms' products.

6) In the absence of any government regulation on price, if a firm has no power to set price on its own, one can safely conclude 1.

A) the demand curve for the firm's product is horizontal.

2.

B) there aren't many firms in the industry.

3.

C) the market is in long-run equilibrium.

4.

D) the firms in this industry are not profitable.

7) In a perfectly competitive market, 1.

A) firms can freely enter and exit.

2.

B) firms sell a differentiated product.

3.

C) transaction costs are high.

4.

D) All of the above.

8) If all conditions for a perfectly competitive market are met, 1.

A) firms face sunk cost when entering the market.

2.

B) firms' demand curves are horizontal.

3.

C) the market demand curve is horizontal.

4.

D) the firms' demand curves are downward-sloping.

9) In a competitive market, if buyers did not know all the prices charged by the many firms, 1.

A) all firms still face horizontal demand curves.

2.

B) firms sell a differentiated product.

3.

C) demand curves can be downward sloping for some or all firms.

4.

D) the number of firms will most likely decrease.

10) Many used car owners and used car dealers describe their different cars for sale in the local newspapers and list their asking price. Many people shopping for a used car consider the different choices listed in the paper. The market for used cars could be described as 1.

A) relatively competitive.

2.

B) perfectly competitive.

3.

C) non-competitive.

4.

D) having high transaction costs.

11) Many car owners and car dealers describe their different cars for sale in the local newspapers and list their asking price. Many people shopping for a used car consider the different choices listed in the paper. The absence of which condition prohibits this market from being described as perfectly competitive? 1.

A) Buyers and sellers know the prices.

2.

B) Firms freely enter and exit.

3.

C) Transaction costs are low.

4.

D) Consumers believe all firms sell identical products.

12) A special license is required to operate a taxi in many cities. The number of licenses is restricted. More drivers want licenses than are issued. This describes a non-perfectly competitive market because

1.

A) taxi services are very different.

2.

B) firms cannot freely enter and exit the market.

3.

C) transaction costs are high.

4.

D) the government generates revenue from the licenses.

13) If a firm operates in a perfectly competitive market, then it will most likely 1.

A) advertise its product on television.

2.

B) take the price of its product as determined by the market.

3.

C) have a difficult time obtaining information about the market price.

4.

D) have an easy time keeping other firms out of the market.

14) If a firm operates in a perfectly competitive market, then 1.

A) all firms will advertise.

2.

B) no firms will advertise.

3.

C) the market leader will advertise.

4.

D) new firms will advertise.

15) The "Got Milk?" advertising campaign is a good example of 1.

A) advertising in a competitive market.

2.

B) how advertising in a competitive market does not pay off for a single firm.

3.

C) interest groups financed by the industry advertise for the whole industry.

4.

D) All of the above.

16) The perfectly competitive model makes a lot of fairly unrealistic assumptions. Why do economics text books still talk a lot about this model? 1. 2. 3.

A) Many markets are close to being perfectly competitive. B) It is an important model to use as a benchmark to compare other markets structures to. C) Perfectly competitive markets maximize societal welfare.

4.

D) All of the above.

17) If a firm happened to be the only seller of a particular product, it might behave as a price taker as long as 1.

A) buyers have full information about the firm's price.

2.

B) the transaction costs of doing business with this firm are low.

3.

C) there are many buyers.

4.

D) there is free entry and exit.

18) The demand curve that an individual competitive firm faces is known as its 1.

A) excess demand curve.

2.

B) market demand curve.

3.

C) residual demand curve.

4.

D) leftover demand curve.

19) Even if two products have different characteristics, such as color, the products are only considered heterogeneous if consumers 1.

A) consider the two products as perfect complements.

2.

B) consider the two products as perfect substitutes.

3.

C) consider the two products as imperfect substitutes.

4.

D) consider the two products as imperfect complements.

20) Which of the following are not characteristics of a competitive market? 1.

A) There is freedom of entry and exit.

2.

B) There are zero transaction costs.

3.

C) There are only one or two sellers.

4.

D) Buyers and sellers have complete information.

21) Which of the following characteristics of a competitive market make auction sites such as eBay so popular? 1.

A) There is freedom of entry and exit.

2.

B) There are very low transaction costs.

3.

C) There are only one or two sellers.

4.

D) Buyers and sellers have complete information.

22) Many auction sites, such as eBay, provide a reputation score by which previous customers can rate a seller. Which of the following characteristics of a competitive market is this policy trying to emulate? 1.

A) There is freedom of entry and exit.

2.

B) There are very low transaction costs.

3.

C) There are only one or two sellers.

4.

D) Buyers have more complete information.

23) In a competitive market, one would expect to see 1.

A) no advertising.

2.

B) false advertising.

3.

C) advertising only in the Sunday papers.

4.

D) minimal advertising.

24) How can the market demand for a product be inelastic but the demand for a particular firm is elastic? 1.

A) There is no advertising.

2.

B) There is a sufficiently large number of sellers.

3.

C) There is only one or two sellers.

4.

D) Buyers do not have complete information.

25) The model of perfect competition is valuable for 1.

A) prediction.

2.

B) comparison to other markets.

3.

C) Either A or B

4.

D) None of the above.

26) In a competitive market where the elasticity of the market demand curve is -0.5, there are 100 identical firms, and the elasticity of the supply curve to the other 99 firms is 4. What is the elasticity of the demand curve of the 100th firm? 1.

A) -446

2.

B) -489

3.

C) -50

4.

D) -0.5

27) A market is perfectly competitive even if firms have the ability to set their own price as long as the price difference reflects differences in the product.

28) If transaction costs are high, then it is more likely a firm's demand curve is downward sloping.

29) Explain why individual firms in competitive markets face more elastic demand curves than the market as a whole.

8.2 Profit Maximization

1) If a firm makes zero economic profit, then the firm 1.

A) has no incentive to stay in the industry.

2.

B) is better off exiting the industry.

3.

C) is indifferent between staying and exiting the industry.

4.

D) will shut down.

2) If a firm makes zero economic profit, then the firm

1.

A) has total revenues greater than its economic costs.

2.

B) must shut down.

3.

C) can be earning positive business profit.

4.

D) must have no fixed costs.

3) A small business owner earns $50,000 in revenue annually. The explicit annual costs equal $30,000. The owner could work for someone else and earn $25,000 annually. The owner's business profit is ________ and the economic profit is ________. 1.

A) $20,000, $5,000

2.

B) $20,000, -$5,000

3.

C) $25,000, -$5,000

4.

D) $45,000, -$5,000

4) A small business owner earns $60,000 in revenue annually. The explicit annual costs equal $40,000. The owner could work for someone else and earn $25,000 annually. The owner's accounting profit is ________ and owner's economic profit is ________.

1.

A) $20,000, $5,000

2.

B) $20,000, -$5,000

3.

C) $25,000, -$5,000

4.

D) $45,000, -$5,000

5) If a competitive firm's marginal profit is positive at an output of 1000 units, 1.

A) at 1000 units, MR = MC.

2.

B) it should produce more than 1000 units.

3.

C) it should produce less than 1000 units.

4.

D) at 1000 units, MR < MC.

6) If marginal revenue equals marginal cost, the firm is maximizing profits as long as

1.

A) the resulting profits are positive.

2.

B) marginal cost exceeds marginal revenue for greater levels of output.

3.

C) the average cost curve lies above the demand curve.

4.

D) All of the above are required.

7) If a firm is operating at an output level where losses are minimized, the firm 1.

A) has no incentive to stay in the industry.

2.

B) is better off exiting the industry.

3.

C) is maximizing profits.

4.

D) will shut down.

8) The above figure shows the cost curves for a competitive firm. If the firm is to earn economic profit, price must exceed 1.

A) $0.

2.

B) $5.

3.

C) $10.

4.

D) $11.

9) The above figure shows the cost curves for a competitive firm. The firm will incur economic losses if the price is less than 1.

A) $0.

2.

B) $5.

3.

C) $10.

4.

D) $11.

10) The above figure shows the cost curves for a competitive firm. If the market price is $15 per unit, the firm will earn profits of 1.

A) $0.

2.

B) $4.

3.

C) $40.

4.

D) $160.

11) If a competitive firm maximizes short-run profits by producing some quantity of output, which of the following must be true at that level of output? 1.

A) p = MC

2.

B) MR = MC

3.

C) p ≥ AVC

4.

D) All of the above

12) If a competitive firm maximizes short-run profits by producing some quantity of output, which of the following must be true at that level of output? 1.

A) p > MC

2.

B) MR > MC

3.

C) p ≥ AVC

4.

D) All of the above

13) If a profit-maximizing firm finds that, at its current level of production, MR > MC, it will 1.

A) earn greater profits than if MR = MC.

2.

B) increase output.

3.

C) decrease output.

4.

D) shut down.

14) If a profit-maximizing firm finds that, at its current level of production, MR < MC, it will 1.

A) decrease output.

2.

B) increase output.

3.

C) shut down.

4.

D) operate at a loss.

15) If a firm traded on the New York Stock Exchange posts an accounting profit of $10 million, then the firm is making a positive economic profit 1.

A) only if the Securities and Exchange Commission (SEC) approves the accounting report.

2.

B) only if the firm's opportunity cost is less than $10 million.

3.

C) only if the firm's opportunity benefit is more than $10 million.

4.

D) only if the firm's management receives stock compensation.

16) If a firm goes out of business because of negative economic profits, its books 1.

A) might indicate a positive accounting profit.

2.

B) might indicate that opportunity costs were zero.

3.

C) might indicate that taxes are too high.

4.

D) might suggest a mistaken value of explicit costs.

17) Even though fixed costs do not affect the output decision, an increase in fixed costs results in a wider range of prices for which the firm operates at a loss.

18) If a firm sets marginal revenue equal to marginal cost, it will make an economic profit.

19) If a firm doesn't make an economic profit, it will shut down.

20) Suppose there are 20 competitive firms in a market. The supply curve of each firm is q = 2p. The market demand is Q = 200 - 2p. What is the residual demand curve facing a typical firm?

21) Explain why shutting down and going out-of-business are different concepts.

22) Suppose a firm has the following total cost function: TC = 100 + 4q2. What is the minimum price necessary for the firm to earn profit? Below what price will the firm shut down?

23) The above figure shows the cost curves for a typical firm in a competitive market. Note that if p = 10, then MC = p at both q = 5 and q = 60. Can they both yield maximum profit? Explain.

8.3 Competition

1) A firm should always shut down if its revenue is 1.

A) declining.

2.

B) less than its average fixed costs.

3.

C) less than its total costs.

4.

D) less than its avoidable costs.

2) Suppose a competitive firm's total revenue is $1,000,000 where MR = MC, its explicit variable costs are $900,000, its fixed costs are $90,000 of which $60,000 are sunk.. If its implicit opportunity costs are $50,000, the firm should 1.

A) produce because its economic profit is positive.

2.

B) produce because its economic profit is zero.

3.

C) produce even though its economic profit is negative.

4.

D) shut down.

3) If a competitive firm finds that it maximizes short-run profits by shutting down, which of the following must be true? 1.

A) p < AVC for all levels of output.

2.

B) p < AVC only for the level of output at which p = MC.

3.

C) p < AVC only if the firm has no fixed costs.

4.

D) The firm will earn zero profit.

4) In a graph of a firm's short-run total costs and total revenue, the total cost and the total revenue curves, respectively, will intersect the vertical axis 1.

A) above the origin, above the origin.

2.

B) above the origin, at the origin.

3.

C) at the origin, at the origin.

4.

D) below the origin, below the origin.

5) The above figure shows the cost curves for a competitive firm. If the firm is to operate, price must exceed 1.

A) $0.

2.

B) $5.

3.

C) $10.

4.

D) $11.

6) The above figure shows the cost curves for a competitive firm. If the profit-maximizing level of output is 40, price is equal to 1.

A) $0.

2.

B) $15.

3.

C) $10.

4.

D) $11.

7) A firm will shut down if 1.

A) total fixed costs are too high.

2.

B) total revenue from operating would not cover all costs.

3.

C) total revenue from operating would not cover variable costs.

4.

D) total revenue from operating would not cover fixed costs.

8) The reasons why a competitive firm's short-run supply curve is upward sloping are 1.

A) the law of diminishing marginal returns and profit maximization.

2.

B) constant returns to scale and profit maximization.

3.

C) decreasing returns to scale and profit maximization.

4.

D) Both B and C.

9) The competitive firm's supply curve is equal to 1.

A) its marginal cost curve.

2.

B) the portion of its marginal cost curve that lies above AC.

3.

C) the portion of its marginal cost curve that lies above AVC.

4.

D) the portion of its marginal cost curve that lies above AFC.

10) If a firm is a price taker, then its marginal revenue will always equal

1.

A) price.

2.

B) total cost.

3.

C) zero.

4.

D) one.

11) An increase in the cost of an input will result in 1.

A) a leftward shift in the firm's supply curve.

2.

B) an upward shift of the firm's marginal cost curve.

3.

C) a leftward shift of the market supply curve.

4.

D) All of the above.

12) When the production of a good involves several inputs, an increase in the cost of one input will usually cause total costs to

1.

A) rise more than in proportion.

2.

B) rise less than in proportion.

3.

C) remain unchanged.

4.

D) rise by the exact amount of the input price increase.

13) When the production of a good involves several inputs and inputs are used in fixed proportions, an increase in the cost of one input will usually cause total costs to 1.

A) rise more than in proportion.

2.

B) rise less than in proportion.

3.

C) remain unchanged.

4.

D) rise by the exact amount of the input price increase.

14) If a competitive firm is in short-run equilibrium, then 1.

A) profits equal zero.

2.

B) it will not operate at a loss.

3.

C) an increase in its fixed cost will have no effect on profit.

4.

D) an increase in its fixed cost will have no effect on output.

15) If a competitive firm is in short-run equilibrium, then 1.

A) economic profits equal zero.

2.

B) economic profits will be positive.

3.

C) economic profits will be negative.

4.

D) All of the above are possible in the short-run.

16) Suppose a firm's costs are F + v ∗ q2 where F and v are positive real numbers and the firm sells its product at the market determined price p. Profits are calculated using 2.

A) p ∗ q - F - v ∗ q2.

3.

B) [p -(F/q + v ∗ q)] ∗

4.

C) [(p ∗ q)/q -(F + v ∗ q)/q] ∗

5.

D) Both A and B.

17) Suppose TC = 10 + (0.1 ∗ q2). If p = 10, the firm's profit maximizing level of output is 40.

A) 40.

41.

B) 50.

42.

C) 60.

43.

D) 0, since the firm will shut down.

18) Suppose TC = 10 + (0.1 ∗ q2). If p = 10, the firm's profits will be 240.

A) 240.

241.

B) 250.

242.

C) 260.

243.

D) -10 because the firm will shut down.

19) Suppose TC = 10 + (0.1 ∗ q2). If there are 100 identical firms in the market, the market supply curve is 1.

A) Q = 1000 ∗

2.

B) Q = 500 ∗

3.

C) Q = 100 ∗

4.

D) Q = 10.

20) The above figure shows the cost curves for a typical firm in a market and three possible market supply curves. If there are 100 identical firms, the market supply curve is best represented by 1.

A) curve A.

2.

B) curve B.

3.

C) curve C.

4.

D) either curve A or B, but definitely not C.

21) If a firm is currently in short-run equilibrium earning a profit, what impact will a lumpsum tax have on its production decision? 1.

A) The firm will decrease output to earn a higher profit.

2.

B) The firm will increase output but earn a lower profit.

3.

C) The firm will not change output but earn a lower profit.

4.

D) The firm will not change output and earn a higher profit.

22) Suppose that once a well is dug, water flows out of it continuously without any additional effort. Customers collect their water and pay a per gallon fee when they leave the site of the well., the competitive firm in this market 1.

A) will not shut down because variable costs are zero.

2.

B) has no fixed costs.

3.

C) faces diminishing marginal returns.

4.

D) can act as a price setter.

23) Suppose that once a well is dug, water flows out of it continuously without any additional effort. Customers collect their water and pay a per gallon fee when they leave the site of the well., the competitive firm in this market 1.

A) has no variable costs.

2.

B) has no fixed costs.

3.

C) will shut down.

4.

D) can produce water at no cost.

24) If a competitive firm cannot earn profit at any level of output during a given short-run period, then which of the following is LEAST likely to occur? 1.

A) It will shut down and wait until the price increases sufficiently.

2.

B) It will exit the industry.

3.

C) It will operate at a loss.

4.

D) It will minimize its loss by decreasing output so that price exceeds marginal cost.

25) In deciding whether to operate, the firm must be concerned with the relationship between price of the output and 1.

A) total cost.

2.

B) average variable cost.

3.

C) total fixed cost.

4.

D) the number of buyers.

26) There are currently N identical firms in a market. If it is a perfectly competitive market, the short-run market supply curve at any given price is

1.

A) N times the supply of an individual firm.

2.

B) N - 1 times the supply of an individual firm.

3.

C) N plus the supply of an individual firm.

4.

D) It cannot be determined from the information provided.

27) Suppose there are two perfectly competitive industries with similar numbers of firms but where one industry consists of N identical firms while the second consists of N firms with differing costs. Compared to the short-run supply curve of the industry with identical firms, the short-run supply curve of the differing cost industry will tend to be 1.

A) steeper at higher prices.

2.

B) flatter at higher prices.

3.

C) steeper at lower prices.

4.

D) flatter at lower prices.

28) If a firm cannot earn profits, it will shut down.

29) If the market price in a competitive market is below the minimum of average variable cost, the firm will shut down.

30) If a competitive firm has to pay a lump sum tax, it will produce less. .

31) A competitive firm's supply curve is identical to its marginal cost curve.

32) The above figure shows the cost curves for a typical firm in a competitive market. From the graph, estimate the firm's profits when price equals $10 per unit.

33) The above figure shows the cost curves for a typical firm in a competitive market. If there are 200 identical firms, estimate the market quantity supplied when p = 4, 8, and 10.

34) If a firm operates at a loss, the loss is equal to TC - TR. If the firm shuts down instead, its loss is equal to FC. Given this, show that price must exceed AVC for the firm to operate at a loss and not shut down.

35) Suppose a firm has the following total cost function TC = 100 + 2q2. If price equals $20, what is the firm's output decision? What are its short-run profits?

36) Draw a graph that shows how the short-run shutdown price changes when an input price increases. Answer: See the above figure.

37) Suppose there are 1000 identical wheat farmers. For each, TC = 10 + q2 Derive the market supply curve.

38) Suppose there are 1000 identical wheat farmers. For each, TC = 10 + q2. Market demand is

Q = 600,000 – 100p. Derive the short-run equilibrium Q, q, and p. Does the typical firm earn a short-run profit?

8.4 Competition

1) Markets with hit-and-run entry and exit experience 1. 2.

A) barriers to entry. B) firms entering whenever they can make a profit and exiting when they cannot make a profit.

3.

C) steady long-run economic profit.

4.

D) a very steady number of firms.

2), profits will equal zero in a competitive market because of 1.

A) constant returns to scale.

2.

B) identical products being produced by all firms.

3.

C) the availability of information.

4.

D) free entry and exit.

3) Assuming a horizontal long-run market supply curve, which of the following statements is (are) TRUE about competitive firms? 1.

A) p = MC

2.

B) p = AC

3.

C) profit = 0

4.

D) All of the above.

4) Long-run market supply curves are upward sloping if 1.

A) firms are identical.

2.

B) the number of firms is restricted.

3.

C) input prices fall as the industry expands.

4.

D) All of the above.

5) If the long-run supply curve in a perfectly competitive industry is upward sloping, this is because 1.

A) firms are different.

2.

B) firms are identical.

3.

C) input prices rise as the industry expands.

4.

D) Either A or C.

6) Long-run market supply curves are downward sloping if 1.

A) firms are identical.

2.

B) the number of firms is restricted.

3.

C) input prices fall as the industry expands.

4.

D) All of the above.

7) If firms in a competitive market are not identical, then the long-run market supply curve will be 1.

A) horizontal.

2.

B) upward sloping.

3.

C) downward sloping.

4.

D) undetermined.

8) If firms in a competitive market are not identical, then an increase in cost will 1.

A) shift marginal cost to the right.

2.

B) push the most inefficient firms out of the market.

3.

C) push the most efficient firms out of the market.

4.

D) Need more information.

9) Suppose that for each firm in the competitive market for potatoes, long-run average cost is minimized at $0.20 per pound when 500 pounds are grown. If the long-run supply curve is horizontal, then 1.

A) some firms will enjoy long-run profits because they operate at minimum average cost.

2.

B) the long-run price will be $0.20 per pound.

3.

C) each consumer will purchase $100 worth of potatoes.

4.

D) the long-run price will be set just above $0.20 per pound.

10) Suppose that for each firm in the competitive market for potatoes, long-run average cost is minimized at $0.20 per pound when 500 pounds are grown. The demand for potatoes is Q = 10,000/p. If the long-run supply curve is horizontal, then how many firms will this industry sustain? 1.

A) 0

2.

B) 100

3.

C) 50,000

4.

D) There is not enough information to answer.

11) Suppose that for each firm in the competitive market for potatoes, long-run average cost is minimized at $0.20 per pound when 500 pounds are grown. The demand for potatoes is Q = 10,000/p. If the long-run supply curve is horizontal, then how much will consumers spend, in total, on potatoes? 1.

A) $0

2.

B) $500

3.

C) $10,000

4.

D) $50,000

12) Suppose that for each firm in the competitive market for potatoes, long-run average cost is minimized at $0.20 per pound when 500 pounds are grown. The demand for potatoes is Q = 10,000/p. If the long-run supply curve is horizontal, then how many pounds of potatoes will be consumed in total?

1.

A) 0

2.

B) 500

3.

C) 10,000

4.

D) 50,000

13) Baseball teams shut down in the winter. This is an example of 1.

A) permanently leaving the industry because price is less than average fixed cost.

2.

B) temporarily leaving the industry because price is less than average variable cost.

3. 4.

C) temporarily leaving the industry because price is less than average fixed cost. D) permanently leaving the industry because price is less than average variable cost.

14) Which is an important aspect of the perfectly competitive market that leads to long run equilibrium?

1.

A) perfect information

2.

B) freedom of entry and exit

3.

C) price taking behavior

4.

D) homogeneous products

15) Which of the following markets would reach long-run equilibrium fastest? 1.

A) online retail

2.

B) auto dealers

3.

C) oil extraction

4.

D) World Series tickets

16) Firms in long-run perfect competition produce at 1.

A) increasing returns to scale.

2.

B) decreasing returns to scale.

3.

C) constant returns to scale.

4.

D) no returns to scale.

17) Downward sloping long-run supply curves occur in markets 1.

A) with learning-by-doing.

2.

B) with increasing returns to scale.

3.

C) with constant returns to scale.

4.

D) Either A or B

18) Under what circumstances will the residual supply curve for a country be upward sloping? 1.

A) when it does not import any of the good from the rest of the world

2.

B) when it imports a small portion of the rest of the world's supply of the good

3.

C) when it imports a large portion of the rest of the world's supply of the good

4.

D) Either A or B

19) Suppose the long-run supply curve for a perfectly competitive industry is horizontal at a price of $12, and the minimum short-run average variable cost for each of the identical N firms in the industry is $8. If the demand curve for the industry decreases so that it intersects the short-run supply curve of the industry at $10, 1.

A) the price will decrease to $10, and the number of firms will still be N. the price will return to $12, and the number of firms will be less than N.

2.

B) the price will decrease to $10, and the number of firms will be less than N. the price will return to $12, and the number of firms will return to N.

3.

C) the price will remain at $12, and the number of firms will still be N. the price will fall to $8, and the number of firms will be less than N.

4.

D) the price will decrease to $10, and the number of firms will be less than N. the price will return to $12, and the number of firms will return to N.

20) If firms in a competitive market are identical, the long-run market supply curve is horizontal.

21) The long-run supply curve in a competitive market is upward-sloping.

22) If the shutdown rule, p < AVC, is the same and the long run, explain why the shutdown prices may be different.

23) The above figure shows the long-run cost curves for a typical firm in a competitive market. If the number of firms is unrestricted and input costs are constant, derive the long-run market supply curve.

24) All the supply of peppermint oil is produced from mint plants grown in one county by several competitive growers (the number of growers is not limited). The quality of land in the county varies greatly. Would you expect the long-run market supply curve to slope upward, downward, or remain constant? Why?

25) Suppose an industry has no fixed costs. Draw two graphs side by side for the industry. In the left graph draw a U-shaped average cost curve and the corresponding marginal cost curve. In the right graph, draw a downward sloping market demand curve. Also in the right graph, draw a short-run supply curve that would generate positive profit, and the long-run supply curve that would result in. Answer: See the above figure.

26) Suppose all firms in a competitive market are currently in both short-run and longrun equilibrium. What impact will a lump sum tax have on each firm??

27) Suppose market demand is Q = 1000 - 4p. If all firms have LRAC = 50 - 5q + q2, how many identical firms will there be when this industry is in long-run equilibrium? Microeconomics, 7e (Perloff) Chapter 9 Applying the Competitive Model

9.1 Zero Profit for Competitive Firms in the Long Run

1) A firm that generates zero economic profit usually has 1.

A) negative business profit.

2.

B) zero business profit.

3.

C) positive business profit.

4.

D) business profit equal to half the total revenue.

2) In the long run, competitive firms MUST be profit maximizers because if they do not maximize profits, 1.

A) they will not survive.

2.

B) they will not be price takers.

3.

C) they will attract entry.

4.

D) the profits that they do earn will only cover variable costs.

3) Long-run economic profit does not exist for fixed factors like land because 1.

A) bidding drives up the price of the factor until no economic profit exists.

2.

B) there is no market for such factors.

3.

C) these factors have L-shaped isoquants.

4.

D) these factors will earn economic profits.

4) Suppose a farmer in a perfectly competitive agricultural industry rents land that is uniquely productive in the production of a certain crop. In the long run 1.

A) the owner of the land receives economic rent while the farmer earns zero economic profit.

2.

B) the owner of the land earns zero economic profit while the farmer receives economic rent.

3.

C) both the farmer and the owner of the land receive economic rent.

4.

D) neither the farmer nor the owner of the land receive economic rent.

5) Does a competitive long-run equilibrium require cost-minimization? 1.

A) Yes, if firms fail to be as efficient as their competitors, they are driven out of the market.

2.

B) No, in the long run, firms make zero profits.

3.

C) Yes, if they didn't, even less efficient firms would enter the industry.

4.

D) No, because competition ensures their survival.

6) Firms are ________ with an economic profit of zero, they will ________ in the industry since they ________ be better off in another industry. 1.

A) satisfied, stay, won't

2.

B) unsatisfied, leave, will

3.

C) satisfied, leave, will

4.

D) unsatisfied, stay, won't

7) Which of the following characterizes long-run equilibrium in perfect competition? 1.

A) P = MC = ATC

2.

B) P = MC < ATC

3.

C) P > MC = ATC

4.

D) P = MC > ATC

8) What is one reason firms might lobby to prevent entry into their market? 1.

A) The long run equilibrium might be characterized by P = MC = ATC.

2.

B) The long run equilibrium might be characterized by P = MC < ATC.

3.

C) The long run equilibrium might be characterized by P > MC = ATC.

4.

D) The long run equilibrium might be characterized by P = MC > ATC.

9) Survivability in a perfectly competitive world requires that 1.

A) firms minimize average total cost.

2.

B) firms produce new and different products.

3.

C) firms maximize profit.

4.

D) firms maximize revenue.

10) If a firm is in a perfectly competitive world but decides to charge a higher price than its competitors, 1.

A) the firm's profits will be zero or negative, and the firm will fail in the long run.

2.

B) the firm's profits will be zero or negative, and the firm will fail in the short run.

3.

C) the firm's profits will be positive or negative, and the firm will fail in the short run.

4.

D) the firm's profits will be positive or negative, and the firm will fail in the long run.

11) The owners of sports franchises often complain that free-agency (open bidding for player services) threatens their profitability and thus their long-run viability. Given your knowledge of perfect competition, which of the following is correct? 1.

A) Team owners might be correct in as much as free-agency bids up the price of players so that economic profits from those players equal zero.

2.

B) Team owners might be correct in as much as free-agency bids up the price of players so that economic profits from those players is negative.

3.

C) Team owners are lying, as free-agent salaries are still much too low.

4.

D) Team owners are telling the truth, as free-agent salaries are much too high.

12) What is one reason perfectly competitive firms wish to be ever more efficient? 1.

A) Individual firms are awarded by the tax code to be more efficient.

2.

B) Individual firms can better control their costs than the price they can charge.

3.

C) Individual firms can better control their costs than their output levels.

4.

D) Individual firms don't need to be efficient; government policies do not reward efficiency.

13) Suppose when a market has four firms, average economic profit is $1,000 per month. When the market has five firms, the average economic profit is -$50 per month. This suggests that 1.

A) the long-run equilibrium number of firms is between four and five.

2.

B) the long-run equilibrium number of firms is four.

3.

C) the long-run equilibrium number of firms is five.

4.

D) there is no long-run equilibrium in this market as profits can never be zero.

14) Before the DVD, the VCR was a popular format for taping and replaying video. When the DVD was introduced, which of the following most accurately describes the long-run adjustment process in the VCR industry? 1.

A) Costs increased, price increased, demand decreased, quantity decreased, profit decreased.

2.

B) Demand increased, costs increased, price increased, quantity increased, profit decreased.

3.

C) Demand decreased, quantity decreased, price decreased, profit decreased.

4.

D) Demand decreased, price decreased, quantity decreased, profit decreased.

15) If entry is limited due to a limited input, firms in that market earn long run economic profit.

16) In the long run, firms in a competitive market make zero economic profit. This induces most firms to leave the industry.

17) Even if two competitive firms in the same market have different production technologies, they will each earn long-run zero profits. Why?

9.2 Consumer Welfare

1) Sheri is currently purchasing 10 units of a normal good and her indifference curves exhibit diminishing marginal rate of substitution. Suppose there is a decrease in the market price of this good. Then 1.

A) both her utility and her consumer surplus will increase.

2.

B) her consumer surplus will increase, but her utility will remain the same.

3.

C) her utility will increase, but her consumer surplus will remain the same.

4.

D) her consumer surplus will increase, but the change in her utility is unknown without more information.

2) Mister Jones was selling his house. The asking price was $220,000, and Jones decided he would take no less than $200,000. After some negotiation, Mister Smith purchased the house for $205,000. Smith's consumer surplus is 1.

A) $5,000.

2.

B) $15,000.

3.

C) $20,000.

4.

D) not able to be calculated from the information given.

3) You enter a store and buy a bottle of soda. Do you usually receive consumer surplus? 1.

A) Yes, because you wouldn't buy the soda if your willingness to pay would be less than the price.

2.

B) Yes, because you are thirsty.

3.

C) No, because you value other drinks more.

4.

D) No, because you have less money after the transaction.

4) Mary purchased a stuffed animal toy for $5. After a few weeks, someone offered her $100 for the toy. Mary refused. One can conclude that Mary's consumer surplus from the toy is 5.

A) less than $5.

6.

B) at least $95.

7.

C) at least $100.

8.

D) $105.

5) Joe's demand for spring water can be represented as p = 10 - Q (where p is measured in $/gallon and Q is measured in gallons). He recently discovered a spring where water can be obtained free of charge. His consumer surplus from this water is 1.

A) $0.

2.

B) $50.

3.

C) $100.

4.

D) unknown based upon the information provided.

6) Assume a consumer has a horizontal demand curve for a product. His consumer surplus from buying the product 1.

A) is maximized.

2.

B) can't be calculated.

3.

C) equals zero.

4.

D) Need more information.

M

7) The above figure shows the market demand curve for telecommunication while driving one's car (time spent on the car phone). The current price is $0.35 per minute. If the price were to increase by ten cents per minute, consumer surplus would 820.

A) fall to $820.

821.

B) fall by $84.

822.

C) fall by $58.

823.

D) fall to $369.

8) The above figure shows the market demand curve for telecommunication while driving one's car (time spent on the car phone). At the current price of $0.35 per minute, consumer surplus equals 301.

A) $301.00.

302.

B) $924.50.

303.

C) $1,225.50.

304.

D) $1,250.00.

9) The above figure shows the market demand curve for telecommunication while driving one's car (time spent on the car phone). If the price were zero, consumer surplus equals 301.

A) $301.00.

302.

B) $924.50.

303.

C) $1,225.50.

304.

D) $1,250.00.

10) The above figure shows the market demand curve for telecommunication while driving one's car (time spent on the car phone). If the price were $2.50, consumer surplus equals 301.

A) $301.00.

302.

B) $924.50.

303.

C) $1,225.50.

304.

D) $0

11) As the price of a good increases, the loss in consumer surplus is larger, 1.

A) the more elastic demand is.

2.

B) the more money previously spent on the good.

3.

C) the less money previously spent on the good.

4.

D) the smaller the price increase.

12) If lower-income households spend a greater share of their income on cigarettes than do higher-income households, then a tax that raises the price of cigarettes will 1.

A) cause lower-income households to incur a greater loss of consumer surplus than that incurred by higher-income households.

2.

B) cause higher-income households to incur a greater loss of consumer surplus than that incurred by lower-income households.

3. 4.

C) raise consumer surplus among higher-income households. D) cause consumer surplus to decline among smokers, but the relative impact cannot be determined from the given information.

13) Suppose consumers of cigarettes can be classified into two groups: heavy users and light users. Heavy users purchase more cigarettes and are less sensitive to price changes relative to light users. To determine whether a heavy user suffers a greater loss of consumer surplus than a light user does when the price of cigarettes increases, one would need to know 1.

A) each group's average income.

2.

B) the actual quantities purchased by each.

3.

C) each individual's price elasticity of demand.

4.

D) no additional information.

14) Sarah's demand curve for shoes has the same slope as Pete's; however, it lies to the right of Pete's. An increase in the price of shoes will cause 1.

A) Sarah to incur a greater loss of consumer surplus than Pete will.

2.

B) Pete to incur a greater loss of consumer surplus than Sarah will.

3.

C) Sarah and Pete to incur the same loss of consumer surplus.

4.

D) Sarah's demand curve to shift closer to Pete's.

15) Sarah and David both have linear demand curves for lemonade. Sarah's demand curve for lemonade intersects David's demand curve at a price of 50 cents per glass. Sarah's demand curve is more inelastic than David's. A change in the price of lemonade from 50 cents to 25 cents per glass will 1.

A) decrease Sarah's consumer surplus more than David's.

2.

B) decrease David's consumer surplus more than Sarah's.

3.

C) increase Sarah's consumer surplus more than David's.

4.

D) increase David's consumer surplus more than Sarah's.

16) Sally is shopping for textbooks at the beginning of the semester. What is one reason she might decide to not purchase a textbook? 1.

A) Her expected producer surplus is positive.

2.

B) Her expected consumer surplus is negative.

3.

C) Her expected consumer surplus is positive.

4.

D) Her expected profits are positive.

17) Consumers seek to 1.

A) maximize profits.

2.

B) maximize expected consumer surplus.

3.

C) minimize expenditures.

4.

D) maximize choice.

18) Consumers often purchase products that, afterward, they regret purchasing. This can be explained by 1.

A) consumers trying products to determine if their consumer surplus increases.

2.

B) consumers trying products to determine if firm advertising is honest.

3.

C) consumers trying to minimize expenditures.

4.

D) consumers trying to maximize choice.

19) What is one reason it might be difficult to dissuade people from pirating music off the Internet? 1.

A) Consumer surplus is very high when music is pirated.

2.

B) In general, consumers get a rush from "stealing."

3.

C) Consumers mistakenly believe that the Internet is public domain.

4.

D) Producer surplus is very low when music is pirated.

20) In the long-run equilibrium in perfect competition, consumer surplus is 1.

A) positive.

2.

B) negative.

3.

C) zero.

4.

D) less than producer surplus.

21) Consumer surplus from a given purchase is the difference between what one was willing to pay for that purchase and what was actually paid.

22) Consumers who are more sensitive to changes in price suffer a greater loss of consumer surplus from any given price increase.

23) The change in total welfare from a 10% increase in price will depend only on the elasticity of demand.

24) Ann and Bill each spend $30 per month on cigarettes when the price is $1 per pack. Draw a graph to illustrate that the consumer with the less elastic demand will suffer the greater loss of consumer surplus when the price of cigarettes increases. Explain and label the figure. Answer:

25) Figure 9.6 shows an individual's demand curve for time per month spent telecommunicating while driving (talking on the car phone.) A car phone is useless except for talking with somebody who is not in the car. If calls are priced at ten cents per minute, what is the consumer surplus derived from talking? What is the most this person would pay for the car phone? Explain.

9.3 Producer Welfare

1) Producer surplus is equal to 1.

A) the area under the supply curve.

2.

B) the difference between price and average cost for all units sold.

3.

C) the difference between price and marginal cost for all units sold.

4.

D) the firm's profit when fixed costs exist.

2) Producer surplus equals 1.

A) total revenue minus total variable cost.

2.

B) total revenue minus the sum of all marginal cost.

3.

C) profit plus fixed cost.

4.

D) All of the above.

3) Mister Jones was selling his house. The asking price was $220,000, and Jones decided he would take no less than $200,000. After some negotiation, Mister Smith purchased the house for $205,000. Jones' producer surplus is 1.

A) $5,000.

2.

B) $15,000.

3.

C) $20,000.

4.

D) not able to be calculated from the information given.

4) Suppose the market supply curve is p = 5 + Q. At a price of 10, producer surplus equals 50.

A) 50.

51.

B) 25.

52.

C) 12.50.

53.

D) 10.

5) Suppose the market supply curve is p = 5Q. At a price of 10, producer surplus equals 50.

A) 50.

51.

B) 25.

52.

C) 12.50.

53.

D) 10.

6) In a perfectly competitive market the long-run demand and supply curves are Q = 12 - P and Q = 5P respectively. Producer surplus in this market equals 1.

A) 0.

2.

B) 5.

3.

C) 10.

4.

D) It cannot be determined without more information.

7) The difference between producer surplus and profit is always the associated with 1.

A) opportunity costs.

2.

B) total costs.

3.

C) variable costs.

4.

D) fixed costs.

8) In the short run, if a firm operates, it earns a profit of $500. The fixed costs of the firm are $100. This firm has a producer surplus of 500.

A) $500.

501.

B) $100.

502.

C) $400.

503.

D) $600.

9) Assume the price of tomatoes increases. Which of the following causes would correspond to greater producer surplus? 1.

A) an increase in supply

2.

B) an increase in costs

3.

C) a decrease in supply

4.

D) an increase in demand

10) In the long-run equilibrium in perfect competition, 1.

A) producer surplus is positive.

2.

B) producer surplus is negative.

3.

C) producer surplus is greater than consumer surplus.

4.

D) producer surplus is less than consumer surplus.

11) If a firm enjoys producer surplus in perfectly competitive Market A of $1000 and would enjoy producer surplus in perfectly competitive Market B of $1200, the firm would consider moving to Market B if 1.

A) fixed costs are greater than $100 in Market A.

2.

B) fixed costs are less than $200 in Market B.

3.

C) fixed costs are less than $300 but greater than $200 in Market B.

4.

D) fixed costs in Market B are less than the fixed costs in Market A plus $200.

12) Assume government policy increases the demand for corn. 1.

A) The consumer surplus of corn buyers will increase.

2.

B) The producer surplus of corn growers will decrease.

3.

C) The producer surplus of corn growers will increase.

4.

D) The producer surplus of corn growers will not change.

13) Producer surplus is the sum of the profits earned by all firms in a market.

14) Producer surplus equals total revenue minus the sum of all marginal cost.

15) When is the profit a firm earns equal to the producer surplus? Explain.

16) Suppose the market supply curve for wheat is shown in the above figure. Calculate the producer surplus when price is $2 per bushel. If legislation mandates that the price be $1 per bushel, what is the resulting loss in producer surplus?

9.4 Competition Maximizes Welfare

1) Economists claim that measuring society's welfare as CS + PS 1.

A) is inappropriate since ultimately everyone is a consumer.

2.

B) is valid only when the same person could be either a consumer or a producer.

3.

C) treats the gains to consumers and producers equally.

4.

D) is not commonly accepted.

2) Advocates of steel tariffs to protect U.S. steel firms realize that when imposing such tariffs, the gains of firms are outweighed by the losses to consumers.. This implies that 1.

A) such advocates value producer surplus more than consumer surplus.

2.

B) such advocates want to help consumers.

3.

C) such advocates value consumer surplus more than producer surplus.

4.

D) such advocates value producer surplus and consumer surplus equally.

3) If a market produces a level of output below the competitive equilibrium, then 1.

A) social welfare is not maximized.

2.

B) consumer surplus might still be maximized.

3.

C) the actual price will be below the equilibrium price.

4.

D) social welfare might still be enhanced if a price ceiling keeps price below the competitive price.

4) If in a market the last unit of output was sold at a price higher than marginal cost, 1.

A) producers are better off producing more.

2.

B) consumers are better off if less of the product is sold.

3.

C) social welfare is not maximized.

4.

D) the unit increased total profit.

5) A competitive market maximizes social welfare because in a competitive market, 1.

A) profits are zero.

2.

B) price equals marginal cost of the last unit produced.

3.

C) price equals average cost of the last unit produced.

4.

D) there is free entry and exit.

6) If a market produces a level of output that exceeds the competitive equilibrium output, then 1.

A) social welfare will be higher.

2.

B) producer surplus will be higher.

3.

C) marginal cost will exceed price.

4.

D) All of the above.

7) If an economist states that not enough of a good is being produced, she usually means that 1.

A) not everyone can afford the good.

2.

B) price exceeds marginal cost.

3.

C) consumer surplus equals zero.

4.

D) at equilibrium, some people who still wish to sell the good cannot find a buyer.

8) Deadweight loss occurs when 1.

A) producer surplus is greater than consumer surplus.

2.

B) the maximum level of total welfare is not achieved.

3.

C) consumer surplus is reduced.

4.

D) an inferior good is consumed.

9) In a competitive market, the demand and supply curves are Q = 12 - P and Q = 5P, respectively. If output is fixed at Q = 5, what is the amount of the resulting deadweight loss? 1.

A) 0

2.

B) 5

3.

C) 10

4.

D) It cannot be determined without more information.

10) In a competitive market, the demand and supply curves are Q = 12 - P and Q = 5P, respectively. If output is fixed at Q = 11, what is the amount of the resulting deadweight loss? 1.

A) 0

2.

B) 0.6

3.

C) 11.4

4.

D) 15

11) Giving presents at Christmas does NOT generate a deadweight loss if 1.

A) all gift are money.

2.

B) everybody gets exactly what they want.

3. 4.

C) nobody can be made better off by returning the gift and purchasing a different one. D) All of the above.

12) The deadweight loss associated with output less than the competitive level can be determined by 1.

A) subtracting the competitive level producer surplus from the producer surplus associated with less output.

2.

B) subtracting the consumer surplus from the producer surplus associated with less output.

3.

C) summing the consumer and producer surplus associated with less output.

4.

D) summing the change in the total consumer and producer surplus from moving from the competitive level of output to less output.

13) If activists successfully lobbied government to force firms to produce more output than they normally would in a perfectly competitive market, 1.

A) consumer surplus would decline.

2.

B) producer surplus would increase.

3.

C) taxation would solve the problem.

4.

D) total surplus in the market would decline.

14) What is one reason activists might lobby the government for regulation limiting the production of a product to less than would normally be in a perfectly competitive market? 1.

A) They value consumer surplus more than producer surplus.

2.

B) They value producer surplus more than consumer surplus.

3.

C) They seek to avoid future regulation.

4.

D) They seek to minimize total surplus.

15) What is one reason activists might lobby the government to force firms to produce more output than they normally would in a perfectly competitive market? 1.

A) They value consumer surplus more than producer surplus.

2.

B) They value producer surplus more than consumer surplus.

3.

C) They seek to avoid future regulation.

4.

D) They seek to minimize total surplus.

16) As the quantity produced of a good increases, the social welfare generated by that good increases.

17) While producing less than the competitive output decreases social welfare, the same cannot be said about producing more than the competitive output.

18) Explain why the competitive output maximizes welfare.

19) Suppose a consumer advocacy group has convinced legislators that vitamin pills should be free to consumers. Such a policy would enhance the health of the citizenry, they argue. Assuming a downward-sloping linear demand curve and a horizontal longrun supply curve, determine the resulting output and social welfare from such a policy. Compare this result to the competitive equilibrium. onsumer surplus is maximized when price equals zero. Output will be the quantity where the demand curve hits the quantity axis. The social welfare is less than the competitive result because for quantities beyond the competitive equilibrium, producers incur a cost for which they are not reimbursed. Some of this loss represents a transfer to consumers (the area under the demand curve to the right of the equilibrium quantity). The area above the demand curve and below the supply curve for quantities beyond the equilibrium quantity represents the deadweight loss.

20) Suppose an industry trade group has convinced legislators that a price floor should be used so that producer surplus is maximized in the market for milk. The group argues that such a policy would save the "family farm." Assuming a downward-sloping linear demand curve and a horizontal long-run supply curve, determine the resulting price, output and social welfare from such a policy. Compare this result to the competitive equilibrium.

9.5 Policies That Shift Supply Curves

1) The services of real estate brokers are provided in a competitive market. If the state Board of Realtors enacts several requirements that limit the number of real estate brokers, which of the following is most likely to occur? 1.

A) The supply curve of real estate brokers will shift to the left.

2.

B) The supply curve of real estate brokers will shift to the right.

3.

C) Social welfare will remain unchanged.

4.

D) The supply curve will remain unchanged.

2) The services of real estate brokers are provided in a competitive market. If the state Board of Realtors enacts several requirements that limit the number of real estate brokers, which of the following is most likely to occur? 1.

A) Consumer surplus will increase.

2.

B) Producer surplus will increase.

3.

C) Entry of new brokers will increase.

4.

D) Social welfare will increase.

3) The services of real estate brokers are provided in a competitive market. If the state Board of Realtors enacts several requirements that limit the number of real estate brokers, then consumer surplus will most likely 1.

A) increase.

2.

B) decrease.

3.

C) remain unchanged.

4.

D) There is not enough information to answer.

4) The services of real estate brokers are provided in a competitive market. If the state Board of Realtors enacts several requirements that limit the number of real estate brokers, then social welfare will most likely 1.

A) not change, but there will be a transfer from consumer to producer.

2.

B) not change, but there will be a transfer from producer to consumer.

3.

C) decrease, although producers are made better off.

4.

D) decrease, although consumers are made better off.

5) A new law applied to a competitive market that requires laid off workers be paid a large severance payment will 1.

A) not generate a deadweight loss.

2.

B) increase total welfare.

3.

C) increase consumer surplus in the market.

4.

D) decrease consumer surplus in the market.

6) If a city decides to restrict the number of pizza parlors, 1.

A) the price of pizza will increase.

2.

B) pizza parlors will make higher profits.

3.

C) total welfare will decrease.

4.

D) All of the above.

7) Without restrictions, the market supply curve is horizontal at P = 5, and the inverse demand curve for taxi cab rides is P = 20 - Q in a competitive market. Subsequently, only 10 taxi cabs are allowed in the market. This results in a deadweight loss of 1.

A) 0.

2.

B) 5.5.

3.

C) 12.5.

4.

D) 25.

8) Without restrictions, the market supply curve is horizontal at P = 5, and the inverse demand curve for taxi cab rides is P = 20 - Q in a competitive market. Subsequently, only 10 taxi cabs are allowed in the market. After the market adjusts to the restricted supply, 1.

A) there will be a decrease in consumer surplus.

2.

B) the remaining taxi cab drivers will make positive economic profits.

3.

C) there will be an increase in producer surplus.

4.

D) All of the above.

9) In economics, welfare analysis focuses on 1.

A) income transfer programs.

2.

B) food stamp programs.

3.

C) international aid programs.

4.

D) None of the above.

10) In economics, welfare analysis is useful to 1.

A) gauge the validity of "mend it, don't end it."

2.

B) determine who should receive income transfers.

3.

C) determine who gains and who loses in a particular policy option.

4.

D) prove which policy option is "best."

11) What is one reason existing firms might lobby the government to increase regulation in their industry? 1.

A) It increases entry and exit costs, thereby reducing producer surplus to existing firms.

2.

B) It increases entry and exit costs, thereby potentially increasing producer surplus to existing firms.

3.

C) It increases entry and exit costs, but has no impact on producer surplus.

4.

D) Firms cannot be trusted to treat their customers fairly and ethically.

12) In 2007, the National Collegiate Athletic Association put a moratorium on new Football Bowl Series (formerly Division IA) teams. This policy will 1.

A) protect the producer surplus of existing football programs.

2.

B) ensure that when entry occurs, producer surplus will not be zero.

3.

C) ensure that when entry occurs, producer surplus will be positive.

4.

D) ensure that consumer surplus is greater in the future.

13) Policies that restrict supply could generate an increase in social welfare because the increase in producer surplus could exceed the decrease in consumer surplus.

14) If a city decides to lift restrictions of how many taxi cabs can operate, social welfare will increase.

15) Suppose anyone with a driver's license is capable of supplying one trip from the airport to the downtown business center on any given day. The long-run supply curve of such trips is horizontal at p = $50, which is the average cost of such trips. Suppose daily demand is Q = 1000 10p. Calculate the change in consumer surplus, producer surplus and social welfare if the city government requires those people supplying such trips to possess a special license, and the government will issue only 300 licenses.

16) The above figure shows the demand and supply curves in the market for milk. If the government imposes a quota at 500 gallons, calculate the deadweight loss. WL = .5(4 - 2)(1000 - 500) = 500

9.6 Policies That Create a Wedge Between Supply and Demand

1) The total welfare associated with a market that includes a government sales tax equals 1.

A) consumer surplus plus producer surplus.

2.

B) consumer surplus plus producer surplus minus government tax revenue.

3.

C) consumer surplus plus producer surplus plus government tax revenue.

4.

D) the government tax revenue.

2) If a city government enacts a maximum price on rent, 1.

A) quantity supplied will decrease.

2.

B) quantity demanded will increase.

3.

C) allocational problems develop.

4.

D) All of the above.

3) The above figure shows supply and demand curves for apartment units in a large city. If the city government passes a law that establishes $350 per month as the legal maximum rent, the loss in social welfare equals

1.

A) b + c.

2.

B) f.

3.

C) a.

4.

D) f + g.

4) The above figure shows supply and demand curves for apartment units in a large city. If the city government passes a law that establishes $350 per month as the legal maximum rent, deadweight loss occurs because 1.

A) consumers place a greater value on the last apartment unit than the cost to supply it.

2.

B) the supplier of the last apartment unit receives a rental price that is less than the marginal cost of supplying it.

3.

C) the quantity of apartments supplied has decreased.

4.

D) All of the above.

5) The above figure shows supply and demand curves for apartment units in a large city. If the city government passes a law that establishes $350 per month as the legal maximum rent, the consumer's net gain in surplus equals 1.

A) c - f.

2.

B) b - f.

3.

C) d - f.

4.

D) The answer cannot be determined from the information given.

6) The above figure shows supply and demand curves for apartment units in a large city. If the city government passes a law that establishes $350 per month as the legal maximum rent, producer surplus 1.

A) increases.

2.

B) decreases.

3.

C) stays the same.

4.

D) changes in a direction that cannot be determined from the information given.

7) The above figure shows supply and demand curves for apartment units in a large city. If the city government passes a law that establishes $350 per month as the legal maximum rent, producer surplus decreases by 1.

A) d.

2.

B) b + f.

3.

C) c + g.

4.

D) i.

8) The above figure shows supply and demand curves for apartment units in a large city. If the city government passes a law that establishes $350 per month as the legal maximum rent, producer surplus will be 1.

A) d.

2.

B) d + e.

3.

C) d + g.

4.

D) d + c + g.

9) The above figure shows supply and demand curves for apartment units in a large city. At the unregulated equilibrium, producer surplus will be 1.

A) d.

2.

B) d + e.

3.

C) d + g.

4.

D) d + c + g.

10) The above figure shows supply and demand curves for apartment units in a large city. The area "e" represents 1.

A) the loss in producer surplus if a rent ceiling of $350 is imposed.

2.

B) the total variable cost of supplying Q1 units.

3.

C) the marginal cost of supplying Q1 units.

4.

D) the total revenue received by supplying Q1 units.

11) The above figure shows supply and demand curves for apartment units in a large city. The area "c" represents 1.

A) the loss in consumer surplus if a rent ceiling of $350 is imposed.

2.

B) a transfer from producers to consumers if a rent ceiling of $350 is imposed.

3.

C) a transfer from consumers to producers if a rent ceiling of $350 is imposed.

4.

D) the total revenue received by supplying Q1 units.

12) The above figure shows supply and demand curves for milk. In an effort to help farmers, the government passes a law that establishes a $3 per gallon price support. To maintain the price support, government expenditures must equal 1.

A) k + i.

2.

B) f + g + h + i + j.

3.

C) f + g + h + i + j + k.

4.

D) f + g + h + i + j + k + e.

13) The above figure shows supply and demand curves for milk. If amount Q2 is produced in the market, 1.

A) producer surplus is maximized.

2.

B) consumer surplus is minimized.

3.

C) a deadweight loss is generated.

4.

D) All of the above.

14) The above figure shows supply and demand curves for milk. In an effort to help farmers, the government passes a law that establishes a $3 per gallon price support. To maintain the price support, government must purchase 1.

A) Q1

2.

B) Q2

3.

C) Q1- Q2

4.

D) Q2- Q1

15) The above figure shows supply and demand curves for milk. In an effort to help farmers, the government passes a law that establishes a $3 per gallon price support. As a result, consumer surplus falls by 1.

A) a.

2.

B) b + f.

3.

C) f + g.

4.

D) b + f - c.

16) The above figure shows supply and demand curves for milk. In an effort to help farmers, the government passes a law that establishes a $3 per gallon price support. The loss in social welfare resulting from this price support equals 1.

A) k + i.

2.

B) j.

3.

C) [$3 ∗ (Q2- Q1)] - h.

4.

D) $3 ∗

17) The above figure shows supply and demand curves for milk. If the government passes a $2 per gallon specific tax, the loss in social welfare will equal 1.

A) b + c + f + g.

2.

B) f + g.

3.

C) b + f.

4.

D) c + g.

18) The above figure shows supply and demand curves for milk. If the government passes a $2 per gallon specific tax, the loss in consumer surplus will equal 1.

A) b + c + f + g.

2.

B) f + g.

3.

C) b + f.

4.

D) c + g.

19) The above figure shows supply and demand curves for milk. If the government passes a $2 per gallon specific tax, the loss in producer surplus will equal 1.

A) b + c + f + g.

2.

B) f + g.

3.

C) b + f.

4.

D) c + g.

20) The above figure shows supply and demand curves for milk. If the government passes a $2 per gallon specific tax, the tax revenue is 1.

A) $2 ∗ Q1.

2.

B) $2 ∗ Q2.

3.

C) $2 ∗ (Q2- Q1).

4.

D) $2.

21) The above figure shows supply and demand curves for milk. In an effort to help farms, the government passes a law that establishes a $3 per gallon price support, but allows farmers to decide how much milk to produce. The government then provides a deficiency payment to guarantee that the farmers receive $3 per gallon. The required deficiency payment equals 1.

A) b + c + f + g + h + i + j.

2.

B) b + c + f + g + h + i.

3.

C) f + g.

4.

D) b + c.

22) The above figure shows supply and demand curves for milk. In an effort to help farms, the government passes a law that establishes a $3 per gallon price support, but allows farmers to decide how much milk to produce. The government then provides a deficiency payment to guarantee that the farmers receive $3 per gallon. The welfare loss of this price support is 1.

A) j.

2.

B) f + g.

3.

C) b + c.

4.

D) f + g + h + i + j.

23) The above figure shows supply and demand curves for milk. If the government passes a $2 per gallon specific tax, the welfare loss will equal 1.

A) j.

2.

B) f + g.

3.

C) b + c + f + g.

4.

D) c + g.

24) The above figure shows supply and demand curves for milk. Suppose that the government passes a $2 per gallon subsidy. The deadweight loss resulting from this policy will be 1.

A) j.

2.

B) f + g.

3.

C) b + c + f + g.

4.

D) c + g.

25) Which of the following is not a potential result of a price floor? 1.

A) excess supply

2.

B) price greater than free-market equilibrium price

3.

C) Lower quality inputs are used, which increases marginal cost.

4.

D) all of the above

26) Which of the following is a potential result of a price ceiling? 1.

A) excess supply

2.

B) long lines

3.

C) higher quality output

4.

D) higher marginal costs

27) A minimum wage is an example of a 1.

A) price floor.

2.

B) price ceiling.

3.

C) quantity quota.

4.

D) free market equilibrium.

28) A per unit subsidy increases both consumer and producer surplus, but results in a deadweight loss.

29) The tax revenue that is generated by a government tax is counted towards total welfare.

30) The above figure shows the demand and supply curves in the market for milk. Currently the market is in equilibrium. If the government imposes a $2 per gallon tax to be collected from sellers, estimate the change in p, Q, and social welfare.

31) The above figure shows the demand and supply curves in the market for milk. Currently, the market is in equilibrium. If the government imposes a $2 per gallon tax to be collected from sellers, calculate the dead weight loss associated with the tax, and explain why the dead weight loss occurs.

32) The above figure shows the demand and supply curves in the market for milk. Currently the market is in equilibrium. If the government establishes a $4 per gallon price support, estimate the change in p, Q, and social welfare. .

33) The above figure shows the demand and supply curves in the market for milk. Currently the market is in equilibrium. If the government establishes a $2 per gallon price ceiling to ensure that children are nourished, estimate the change in p, Q, and social welfare.

9.7 Comparing Both Types of Policies: Imports

1) The larger the U.S. imposed per unit import tariff on a good imported and produced in the United States, 1.

A) the smaller the U..S consumer surplus.

2.

B) the larger the U.S. producer surplus.

3.

C) the larger the government revenue.

4.

D) All of the above.

2) A ban on imports, a tariff, or a quota raises the price to domestic consumers. This means that consumers will buy less of the product at a higher price. The loss associated with this is called 1.

A) production associated loss.

2.

B) productive consumption loss.

3.

C) consumption distortion loss.

4.

D) consumer misperception loss.

3) The above figure shows the market for rice in Japan. S represents the domestic supply curve, and the horizontal line at P =1 represents the world supply curve. If imported rice is banned, the loss in social welfare is 1.

A) a + b + c + d + e.

2.

B) a.

3.

C) c + e.

4.

D) i.

4) The above figure shows the market for rice in Japan where price is expressed in dollars. S represents the domestic supply curve, and the horizontal line at P = $1 represents the world supply curve. Currently Q1 units are imported. The Consumption distortion loss is equal to 1.

A) c + e.

2.

B) c.

3.

C) c + g + i.

4.

D) a + c + d + e.

5) The above figure shows the market for rice in Japan where price is expressed in dollars. S represents the domestic supply curve, and the horizontal line at P = $1 represents the world supply curve. Currently Q1 units are imported. The loss from shifting production from foreign to domestic producers equals 1.

A) c + e

2.

B) i.

3.

C) e.

4.

D) a + c + d + e.

6) The above figure shows the market for rice in Japan where price is expressed in dollars. S represents the domestic supply curve, and the horizontal line at P = $1 represents the world supply curve. If a $1 tariff is imposed on imported rice, the loss in social welfare is 1.

A) b + c + d + e.

2.

B) a.

3.

C) i.

4.

D) a + c + d + e.

7) The above figure shows the market for rice in Japan where price is expressed in dollars. S represents the domestic supply curve, and the horizontal line at P = 1 represents the world supply curve. Suppose a free market exists. If a $1 per unit tariff is imposed on imported rice, the quantity of imported rice will decrease by 1.

A) Q1

2.

B) Q2- Q1

3.

C) Q2

4.

D) Q1- Q2

8) The above figure shows the market for rice in Japan where price is expressed in dollars. S represents the domestic supply curve, and the horizontal line at P = 1 represents the world supply curve. Suppose a free market exists. An import quota of Q2 units would 1.

A) cause consumer surplus to fall by "e."

2.

B) cause social welfare to fall by "j."

3.

C) increase producer surplus by "d."

4.

D) have no effect.

9) The above figure shows the market for rice in Japan where price is expressed in dollars. S represents the domestic supply curve, and the horizontal line at P = 1 represents the world supply curve. Suppose a free market exists. The smallest tariff necessary to completely eliminate imported rice is 1.

A) $0.50 per unit.

2.

B) $1.00 per unit.

3.

C) $1.50 per unit.

4.

D) $2.00 per unit.

10) The above figure shows the market for rice in Japan where price is expressed in dollars. S represents the domestic supply curve, and the horizontal line at P = 1 represents the world supply curve. A $1 per unit tariff has the same effect on producer and consumer surplus as a quota of 1.

A) Q1

2.

B) Q2

3.

C) Q2- Q1

4.

D) Q1 - Q2

11) Tariffs and quotas create a loss in social welfare because 1.

A) producer surplus declines.

2.

B) revenues from tariffs are misspent.

3.

C) consumer surplus declines.

4.

D) All of the above.

12) The welfare loss from an import quota is greater than that of an equivalent tariff because 1.

A) tariff revenues can be used to society's benefit.

2.

B) the loss in consumer surplus is not as large.

3.

C) domestic producers gain more from a quota than from a tariff.

4.

D) tariff revenues represent an additional deadweight loss.

13) The cost of lobbying for an import quota in a perfectly competitive market 1.

A) increases the welfare loss of the quota.

2.

B) decreases the deadweight loss of the quota.

3.

C) shifts the supply curve of the good to the left.

4.

D) increases the consumer surplus.

14) Rent seeking in the form of lobbying for an increase in import tariffs by domestic producers 1.

A) increases consumer surplus.

2.

B) increases total welfare.

3.

C) increases the deadweight loss.

4.

D) None of the above.

15) The United States and many other countries often impose trade sanctions on other countries. These sanctions 1.

A) decrease producer and consumer surplus in both the sanctioned and sanctioning countries.

2.

B) tend to increase total welfare.

3.

C) tend to decrease the deadweight loss.

4.

D) tend to decrease consumer and producer surplus only in the sanctioned country.

16) The welfare loss of a tariff equals that of an import quota that leads to the same level of imports.

17) "Supporters of import restrictions and protectionist policies place greater weight on producer welfare than on consumer welfare." Comment.

18) The domestic demand curve, domestic supply curve, and world supply curves for a good are given in the above figure. All the curves are linear. Initially, the country allows imports. Then imports are banned. Calculate how consumer and producer surplus change because of the ban. Is the country better off with the ban on imports? Why?

19) Explain why a government would impose an import tariff when domestic consumers suffer more than producers gain.

Microeconomics, 7e (Perloff) Chapter 10 General Equilibrium and Economic Welfare

10.1 General Equilibrium

1) General equilibrium analysis is the study of 1.

A) how an equilibrium is determined in all markets simultaneously.

2.

B) how an equilibrium is determined in all closely related markets.

3. 4.

C) the effects of a change in a market, and all spillover effects in all related markets. D) Any of the above.

2) As opposed to general equilibrium analysis, partial equilibrium analysis looks 1.

A) at an equilibrium and changes to it in a single, isolated market.

2.

B) at how changes in all other markets effect a particular market.

3.

C) at how equilibrium is determined in all markets simultaneously.

4.

D) at either price or quantity movements.

3) A general equilibrium analysis of a price change in the corn chip market would include an investigation of the impacts in 1.

A) the television market.

2.

B) the coffee market.

3.

C) the salsa market.

4.

D) All of the above.

4) The general equilibrium analysis of a minimum wage applied to only some sectors of the economy suggests that 1.

A) workers in all sectors will face increased wages.

2.

B) some workers in the covered sectors will lose their jobs and remain unemployed.

3.

C) some workers originally employed in the covered sectors will move to the uncovered sectors, driving down wages in the uncovered sectors.

4.

D) all workers will be worse off.

5) When comparing partial equilibrium effects to general equilibrium effects one can conclude that 1.

A) general equilibrium effects are always larger.

2.

B) partial equilibrium effects are always larger.

3.

C) the effects are of equal size.

4.

D) one cannot determine before the fact which effect is greater.

6) Reparations for slavery in the United States would 1.

A) be consistent with the Pareto principle.

2.

B) be inconsistent with the Pareto principle.

3.

C) have nothing to do with the Pareto principle.

4.

D) be unconstitutional.

7) If firms are producing efficiently, but consumers can reallocate goods amongst themselves, 1.

A) the equilibrium is not efficient.

2.

B) the equilibrium is efficient.

3.

C) the consumers are behaving irrationally.

4.

D) the firms are too greedy.

8) Assume Congress holds a hearing on the impact of gasoline prices on the price of corn. Most likely, this hearing will be

1.

A) a partial equilibrium analysis.

2.

B) a general equilibrium analysis.

3.

C) about consumer rather than producer surplus.

4.

D) an analysis of efficiency.

9) If two or more markets are closely related, 1.

A) a partial equilibrium analysis will tend to overstate the price impact of a supply shock.

2.

B) a partial equilibrium analysis will tend to accurately predict the price impact of a supply shock.

3.

C) a partial equilibrium analysis will tend to understate the price impact of a supply shock.

4.

D) they should be analyzed concurrently but using partial equilibrium analysis alone.

10) The saying "what's that got to do with the price of tea?" reflects 1.

A) two markets where general equilibrium analysis would be most useful.

2.

B) two markets where general equilibrium analysis likely won't be very useful.

3.

C) two markets where the products are clearly closely related.

4.

D) two markets where firms are incredibly greedy.

11) There are two closely related crops, X and Y, with the following demand functions QX = 180 - 2PX + PY and QY = 150 + PX - PY where QX is the quantity of X, PX is the price of X, QY is the quantity of Y, and PY is the price of Y. These two crops are grown in two widely separated countries so there is no interrelationship between the supply curves. The short-run perfectly inelastic supply for X is 150 while the short-run perfectly inelastic supply for Y is 100. In equilibrium, the prices are 1.

A) PX= 80, PY= 130

2.

B) PX= 40, PY= 65

3.

C) PX= 60, PY= 120

4.

D) PX= 30, PY= 80

12) There are two closely related crops, X and Y, with the following demand functions QX = 180 - 2PX + PY and QY = 150 + PX - PY where QX is the quantity of X, PX is the price of X, QY is the quantity of Y, and PY is the price of Y. These two crops are grown in two widely separated countries so there is no interrelationship between the supply curves. The short-run perfectly inelastic supply for X is 200 while the short-run perfectly inelastic supply for Y is 100. In equilibrium, the prices are 1.

A) PX= 30, PY= 80

2.

B) PX= 40, PY= 60

3.

C) PX= 60, PY= 120

4.

D) PX= 80, PY= 130

13) Employers in a city must pay a specific tax of $t per hour worked by their employees while employers in the suburbs of the city do not have an employment tax. What does a general equilibrium approach predict regarding the wages and employment of both the city and suburban workers if the city decides to substantially reduce their employment tax rate? 1.

A) Wages will increase in the city, but not in the suburbs, and employment will increase in both.

2.

B) Wages will increase in both the city and the suburbs, but employment will fall in both.

3.

C) Wages will increase in both the city and suburbs, employment will increase in the city, but decrease in the suburbs.

4.

D) Wages will increase in both the city and the suburbs, employment will decrease in the city, but increase in the suburbs.

14) The effects of a price change are always understated by a partial-equilibrium analysis when compared to a general-equilibrium analysis.

15) Employing a general equilibrium approach, describe the effect of a new law that prohibits steel imports.

16) Suppose that the minimum wage covers all sectors of the economy; however, for unionized laborers, the minimum wage is ineffective. That is, the union wage is already above the minimum wage. Analyze the impact of an increase in the minimum wage on both the unionized and non-unionized labor markets. (Assume that the higher minimum

wage is still ineffective in the unionized sector and that union and nonunion labor are substitutable.) Answer:

10.2 Trading Between Two People

1) Joe and Rita each have some cookies and milk. Joe is willing to trade 2 cookies for an additional ounce of milk. Rita is willing to trade 4 cookies for an additional ounce of milk. If trading is possible, which of the following is most likely to occur? 1.

A) Joe will give some milk to Rita in exchange for cookies.

2.

B) Rita will give some milk to Joe in exchange for cookies.

3. 4.

C) No trade will take place since they both prefer to have more milk and fewer cookies. D) There is not enough information to make any predictions.

2) Joe and Rita each have some milk and cookies (Milk on the horizontal axis). Joe's MRS of cookies for milk is 2. Rita's MRS of cookies for milk is 4. Which of the following statements is TRUE? 1. 2. 3. 4.

A) No gains from trade are possible. B) Both Rita and Joe can be made better off if Rita gives Joe some cookies in exchange for milk. C) Rita and Joe are on the contract curve. D) Both Rita and Joe can be made better off if Joe gives Rita some cookies in exchange for milk.

3) Gains from trade can only occur when 1.

A) marginal rates of substitutions differ across people.

2.

B) marginal rates of substitution are equal across people.

3.

C) indifference curves are convex.

4.

D) people find themselves on the contract curve.

4) Moving away from the contract curve will 1.

A) harm both parties.

2.

B) harm only one of the parties.

3.

C) harm at least one of the parties.

4.

D) harm neither of the parties.

5) If only two people are trading their endowments and no production is possible, then the equilibrium they reach will 1.

A) be on their contract curve.

2.

B) result in unequal marginal rates of substitution for the two people.

3.

C) result in one person being worse off than with his or her endowment.

4.

D) All of the above.

6) The above figure depicts the Edgeworth box for two individuals, Al and Bruce. Part of the contract curve can be found by connecting points 1.

A) a and b.

2.

B) a and c.

3.

C) b and d.

4.

D) c and d.

7) The above figure depicts the Edgeworth box for two individuals, Al and Bruce. Points a and b 1.

A) are most likely to reflect the final allocations after trading.

2.

B) are least likely to reflect the final allocations after trading.

3. 4.

C) are equally likely to reflect the final allocations after trading than other points on the contract curve. D) are definitely not the final allocations after trading.

8) The above figure depicts the Edgeworth box for two individuals, Al and Bruce. If the endowment is at point a, and trade is possible, which of the following points are possible equilibria? 1.

A) a and b

2.

B) a and c

3.

C) b and d

4.

D) c and d

9) The above figure depicts the Edgeworth box for two individuals, Al and Bruce. Considering only the labeled points, point c is a possible equilibrium 1.

A) only if it is the endowment.

2.

B) only if point a is the endowment.

3.

C) if either point a or b is the endowment.

4.

D) only if point d is the endowment.

10) The above figure depicts the Edgeworth box for two individuals, Al and Bruce. If the endowment is at point a, and Al has no ability to bargain, the final allocation will be at point 1.

A) a.

2.

B) b.

3.

C) c.

4.

D) d.

11) The above figure depicts the Edgeworth box for two individuals, Al and Bruce. Point a is NOT Pareto efficient because 1.

A) Al's MRS exceeds Bruce's MRS.

2.

B) the point is not near the center of the box.

3.

C) Al's indifference curve is not far enough away from the origin.

4.

D) All of the above.

12) The above figure depicts the Edgeworth box for two individuals, Al and Bruce. Point c is Pareto efficient because 1.

A) the MRS's are equal.

2.

B) the indifference curves are tangent.

3.

C) no mutual gains from trade exist.

4.

D) All of the above.

13) Only individuals A and B live on a desert island where no production is possible. A is endowed with 12 units of good X and 18 units of good Y. B is endowed with 6 units of good X and 42 units of good Y. In the Edgeworth box, good X is measured on the horizontal axis. Individual A's utility function is UA = 3XY so her MUX = 3Y and her MUY = 3X. Individual B's utility function is UB = 10 so his and his MUY = 5. Which of the following allocations are on the contract curve? 1.

A) (XA= 12, YA= 24), (XB = 6, YB = 36)

2.

B) (XA= 10, YA= 28), (XB = 8, YB = 32)

3.

C) (XA= 9, YA= 30), (XB = 9, YB = 30)

4.

D) None of the above.

14) Only individuals A and B live on a desert island where no production is possible. A is endowed with 15 units of good X and 17 units of good Y. B is endowed with 13 units of good X and 23 units of good Y. In the Edgeworth box, good X is measured on the horizontal axis. Individual A's utility function is UA = 3XY so her MUX = 3Y and her MUY = 3X. Individual B's utility function is UB = 10 so his and his MUY = 5. Which of the following allocations are on the contract curve? 1.

A) (XA= 14, YA= 20), (XB = 14, YB = 20)

2.

B) (XA= 15, YA= 17), (XB = 13, YB = 23)

3.

C) (XA= 18, YA= 22), (XB = 10, YB = 18)

4.

D) None of the above.

15) When considering trade of two goods between two people, if one person has all the endowment of both goods, this allocation 1.

A) is never on a contract curve.

2.

B) will result in trade so each person has all of one good.

3.

C) will result in trade to a equal division of goods between the two people.

4.

D) is Pareto efficient.

16) When two people are on the contract curve, the allocation of goods 1.

A) cannot be improved.

2.

B) is pareto efficient.

3. 4.

C) is such that neither individual can be made better off without making the other worse off. D) All of the above.

17) Gains from trade will be possible as long as 1.

A) people have different endowments.

2.

B) people place different values on some goods.

3.

C) marginal rates of substitution are equal across individuals.

4.

D) excess supply equals excess demand.

18) Gains from trade will be possible as long as 1.

A) levels of utility differ.

2.

B) utility functions differ.

3.

C) marginal rates of substitution differ.

4.

D) endowments differ.

19) The term "born with a silver spoon in his mouth" mistakenly implies 1.

A) only monetary endowments allow one to trade with others.

2.

B) only the wealthy are strong negotiators in trade.

3.

C) endowments are physical.

4.

D) endowments differ.

20) Any point on the contract curve is Pareto efficient regardless of the initial endowment.

21) If two grade-school children willingly trade their lunches with one another, we can conclude that at least one of them preferred the other's lunch to his own.

22) When two people trade their initial endowments to a point on the contract curve, only the level of the endowments will determine the new allocation.

23) Explain why having different marginal rates of substitution is necessary for trade to occur.

24) Consider a society consisting of just a farmer and a tailor. The farmer has 10 units of food but no clothing. The tailor has 20 units of clothing but no food. Suppose each has the utility function U = F ∗ C. Derive the contract curve.

25) Robinson starts out with 10 lobsters and 5 coconuts. Friday starts out with 10 lobsters and 15 coconuts. After trading, Robinson ends up with 8 lobsters and 10 coconuts. Robinson feels neither better nor worse off than when he started but cannot get Friday to agree to any more trades. Friday feels better off than when he started. Draw the Edgeworth box consistent with this story. Answer: See the above figure.

10.3 Competitive Exchange

1) The First Theorem of Welfare Economics can be expressed as 1.

A) the competitive equilibrium results only when no transactions costs exist.

2.

B) the competitive equilibrium does not involve reallocation of endowments.

3.

C) any efficient allocations can be achieved by competition.

4.

D) the competitive equilibrium is efficient.

2) An initial allocation of goods is called a(n) 1.

A) endowment.

2.

B) inheritance.

3.

C) pareto set.

4.

D) general equilibrium goods set.

3) Suppose two people start with an initial endowment and trade until they obtain a Pareto-efficient allocation with the corresponding price line. What happens when more people who have the same tastes and endowments as the original two traders are included in the Edgeworth box analysis? 1.

A) The price line does not change.

2.

B) The price line becomes flatter.

3.

C) The price line becomes steeper.

4.

D) The price line shifts up or down depending upon how many of each type of trader is included in the analysis.

4) In a competitive marketplace, prices adjust until 1.

A) MRS's are equal to zero.

2.

B) excess supply equals excess demand equals zero in all markets.

3.

C) each consumer has maximized utility subject to his budget constraint.

4.

D) all firms earn zero profit.

5) A competitive equilibrium is Pareto-efficient because at the competitive equilibrium, 1.

A) prices have been allowed to adjust.

2.

B) there are no further gains from trade.

3.

C) the final outcome is different from the original inefficient endowment.

4.

D) all members of society can be made better off.

6) The fact that at the competitive equilibrium nobody can be made better off without making someone else worse off implies that 1.

A) the equilibrium is Pareto-efficient.

2.

B) the equilibrium is not Pareto-efficient.

3.

C) the prices need to adjust further.

4.

D) further gains from trade are possible.

7) Any competitive equilibrium is Pareto-efficient because, with a competitive equilibrium, 1.

A) the marginal rates of substitution are equal for all consumers.

2.

B) the price line is the contract curve.

3.

C) mutual gains from trade exist.

4.

D) the slope of the price line equals the ratio of the MRS for all consumers.

8) The fact that any Pareto-efficient equilibrium can be achieved through competition by adjusting endowments is called 1.

A) the Second Welfare Theorem.

2.

B) the First Welfare Theorem.

3.

C) the Third Welfare Theorem.

4.

D) That is not possible.

9) For a given set of prices, two consumers choose bundles that are off the contract curve. In a competitive market, 1.

A) prices will adjust until the consumers choose bundles that are on the contract curve.

2.

B) the indifference curves will shift back to the contract curve.

3.

C) the contract curve will shift to connect these bundles.

4.

D) no adjustments need to be made.

10) In a competitive market, prices adjust until all consumers find themselves 1.

A) maximizing utility.

2.

B) on the contract curve.

3.

C) happy with their original endowment.

4.

D) with many opportunities to gain from additional exchange.

11) The so-called "death tax" might 1.

A) aim to alter endowments so as to attain an inefficient outcome.

2.

B) aim to alter endowments consistent with the First Theorem of Welfare Economics.

3.

C) aim to alter endowments consistent with the Second Theorem of Welfare Economics.

4.

D) aim to alter prices consistent with the First Theorem of Welfare Economics.

12) A government policy of providing free public K-12 education is most consistent with 1.

A) Pareto-efficiency.

2.

B) the First Theorem of Welfare Economics.

3.

C) the Second Theorem of Welfare Economics.

4.

D) the contract curve.

13) True or false? The Edgeworth box version of inter-personal trade requires the individuals to be in close proximity of one another. 1.

A) True, that way they can see each other's endowments and prices.

2.

B) False, begin in close proximity is not required for mutually beneficial trade to occur in the Edgeworth box.

3.

C) True, the Edgeworth box only works when there folks see "eye-to-eye."

4.

D) False, although prices are only valid if they are communicated in person.

14) A competitive equilibrium is not Pareto efficient if some members of society are unable to afford a necessary good.

15) Assume a government likes a particular equilibrium along the contract curve. It can achieve that equilibrium through competition and income redistribution.

16) Explain why Robin Hood's practice of stealing from the rich to give to the poor is never Pareto efficient.

17) Explain the logic behind the First Theorem of Welfare Economics.

18) Consider a society consisting of just a farmer and a tailor. The farmer has 10 units of food but no clothing. The tailor has 20 units of clothing but no food. Suppose each has the utility function U = F ∗ C. The price of clothing is always $1. If the price of food is $3, does a competitive equilibrium exist? If not, what will happen to the price of food?

19) The above figure depicts the Edgeworth box for two consumers, Al and Bruce. Explain why point "a" cannot be a competitive equilibrium.

10.4 Production and Trading

1) If pizza is measured on the horizontal axis and pretzels are measured on the vertical axis, the slope of the production possibility frontier at a given combination reflects 1.

A) the total cost of producing that combination.

2.

B) the total cost of producing that quantity of pizza in terms of pretzels.

3.

C) the cost of making the last pizza in terms of pretzels.

4.

D) the cost of making the last pretzel in terms of pizza.

2) The ability to produce a good at a lower opportunity cost than someone else is called 1.

A) competitive production.

2.

B) comparative advantage.

3.

C) selective advantage.

4.

D) absolute advantage.

3) Suppose the production possibilities for two countries, producing either food or clothing, are shown in the above figure. They can each produce any linear combination as well. Measuring food on the horizontal axis, the joint production possibility frontier 1.

A) will kink away from the origin at 20 units of food.

2.

B) will kink toward the origin at 20 units of food.

3.

C) will kink toward the origin at 10 units of food.

4.

D) will kink away from the origin at 10 units of food.

4) Suppose the production possibilities for two countries, producing either food or clothing, are shown in the above figure. The United States has a comparative advantage in producing

1.

A) food.

2.

B) clothing.

3.

C) food and clothing.

4.

D) neither good.

5) Suppose the production possibilities for two countries, producing either food or clothing, are shown in the above figure. They can each produce any linear combination as well. Measuring food on the horizontal axis, the joint production possibility frontier 1.

A) will have a slope of -3/4 over the entire frontier.

2.

B) will have a slope of -2 when less than 20 units of food are produced.

3.

C) will have a slope of -1 when less than 20 units of food are produced.

4.

D) will have a slope of -1/2 when less than 20 units of food are produced.

6) Suppose the production possibilities for two countries, producing either food or clothing, are shown in the above figure. They can each produce any linear combination as well. Assuming free trade between these two countries, Canada will produce food

1.

A) as long as any positive amount of food is demanded.

2.

B) as long as more than 10 units of food are demanded.

3.

C) as long as people are willing to give up at least 1 unit of clothing to get a unit of food.

4.

D) as long as people are willing to give up more than 1/2 unit of clothing to get a unit of food.

7) Suppose the production possibilities for two countries, producing either food or clothing, are shown in the above figure. They can each produce any linear combination as well. If production occurs at the kink on the joint production possibility frontier, 1.

A) the U.S. will specialize in food and Canada will specialize in clothing.

2.

B) the U.S. will specialize in clothing and Canada will specialize in food.

3.

C) each country will devote half of its resources to each industry.

4.

D) joint output is minimized.

8) Suppose the production possibilities for two countries, producing either food or clothing, are shown in the above figure. They can each produce any linear combination as well. Suppose each country is in competitive equilibrium prior to free trade being allowed. Once free trade is allowed, the price of food will be 1.

A) two times the price of clothing.

2.

B) equal to the price of clothing.

3.

C) one-half the price of clothing.

4.

D) somewhere between one-half the price of clothing and the price of clothing, depending upon the relative bargaining power of the two countries.

9) Suppose the production possibilities for two countries, producing either food or clothing, are shown in the above figure. They can each produce any linear combination as well. Once free trade is allowed, Canada will produce 1.

A) no clothing.

2.

B) 10 units of clothing.

3.

C) 20 units of clothing.

4.

D) 5 units of clothing.

10) Suppose the production possibilities for two countries, producing either food or clothing, are shown in the above figure. They can each produce any linear combination as well. Assuming that consumers place equal value on both of these goods and that free trade between these two countries is possible, gains from trade 1.

A) are not possible.

2.

B) exist because food is more expensive to produce in Canada than in the U.S.

3.

C) exist because food is more expensive to produce in the U.S. than in Canada.

4.

D) do not exist because the consumers in each country have set their marginal rate of substitution equal to their country's marginal rate of transformation.

11) Suppose the production possibilities for two countries, producing either food or clothing, are shown in the above figure. They can each produce any linear combination as well. Measuring food on the horizontal axis, the joint production possibility frontier has a horizontal intercept of 10.

A) 10.

11.

B) 20.

12.

C) 30.

13.

D) 50.

12) Suppose the production possibilities for two countries, producing either food or clothing, are shown in the above figure. They can each produce any linear combination as well. Measuring food on the horizontal axis, the joint production possibility frontier has a vertical intercept of 10.

A) 10.

11.

B) 20.

12.

C) 30.

13.

D) 50.

13) Suppose the production possibilities for two countries, producing either food or clothing, are shown in the above figure. They can each produce any linear combination as well. Measuring food on the horizontal axis, the joint production possibility frontier will have a slope of -1 1.

A) only when 20-30 units of food are produced.

2.

B) only when 10-20 units of food are produced.

3.

C) only when 10-20 units of clothing are produced.

4.

D) throughout the entire PPF.

14) Suppose the production possibilities for two countries, producing either food or clothing, are shown in the above figure. They can each produce any linear combination as well. Measuring food on the horizontal axis, the joint production possibility frontier has a kink because 1.

A) each country has a constant marginal rate of transformation that differs from the other country.

2.

B) each country has a marginal rate of transformation that varies with the product mix.

3.

C) all production possibility frontiers have kinks.

4.

D) the two countries have the same marginal rate of transformation.

15) Every point on the joint production possibilities frontier represents 1.

A) an initial endowment.

2.

B) inefficient production.

3.

C) the marginal rate of substitution of goods for each producer.

4.

D) at least one producer specializing in production.

16) At a perfectly competitive equilibrium with production and trade, the slope of the production possibility curve will be 1.

A) equal to the slope of the price line faced by the consumers.

2.

B) steeper than the slope of the price line faced by consumers.

3.

C) flatter than the slope of the price line faced by consumers.

4.

D) either steeper or flatter than the price line faced by the consumers, depending upon the relative preferences of the consumers.

17) As more producers with differing marginal rates of transformation are added, the joint production possibility curve becomes 1.

A) steeper.

2.

B) flatter.

3.

C) more convex to the origin.

4.

D) smoother.

18) In a perfectly competitive equilibrium with production and trade, which of the following results occur(s)? 1.

A) Pareto-efficiency is obtained.

2.

B) The First Welfare Theorem is satisfied.

3.

C) There is efficiency in production.

4.

D) All of the above.

19) Competition results in the efficient product mix because 1. 2. 3.

A) producers are setting MRT equal to minus the price ratio while consumers are setting MRS equal to minus the price ratio ensuring that MRT will equal MRS. B) consumers are on the contract curve. C) the slope of the production possibility frontier will equal the slope of the contract curve.

4.

D) the distribution of the final output is Pareto efficient.

20) By saying that MRS = MRT, an economist means that 1.

A) for all goods, the value that society places on the last unit produced equals the cost of making that unit.

2.

B) for all goods, the value that a consumer places on the last unit consumed equals the value that all consumers would place on that unit.

3.

C) society is able to produce more output of one good without reducing the output of any other good.

4.

D) no other output mix is technically feasible.

21) Economic growth can be depicted as 1.

A) a change in the country's MRS.

2.

B) a change in the country's MRT.

3.

C) a shift of the country's production possibilities frontier to the right.

4.

D) a shift of the country's production possibilities frontier to the left.

22) Economic growth can be depicted as a 1.

A) shift in the contract curve.

2.

B) a change in the dimensions of the Edgeworth box.

3.

C) a change in the preference curves of individuals.

4.

D) a change in the number of people in the Edgeworth box.

23) Economic growth can be obtained 1.

A) by government legislation.

2.

B) by a change in the level of inputs or an improvement in technology.

3.

C) only through a change in the level of inputs, hence the desire for conquest throughout the history of the world.

4.

D) only through a change in technology, which explains the explosive growth of the United States.

24) If MRS's are equal across all consumers, then the efficient product mix has been achieved.

25) At the competitive equilibrium quantity supplied equals quantity demanded in all markets.

26) Explain how it is possible for one of two people in a two-good economy to have an absolute advantage in producing both goods, but trade can still benefit both people.

27) Suppose a society's PPF for food (F) and clothing (C) can be written as 25 = F2 + C2. If all consumers have the same endowment and the same utility function Ui = Fi ∗ Ci, what is the efficient product mix of food and clothing?

28) Can consumption efficiency be achieved even if the efficient product mix is not achieved?

29) How does competition ensure that the efficient product mix is attained?

30) The above figure shows a production possibility frontier for a society with two members, Al and Bruce. If point "a" is the efficient product mix, draw a possible Edgeworth box and indifference curves.

31) Suppose the U.S. can produce 10 units of food and 5 units of clothing (or any linear combination) and Canada can produce 6 units of food and 3 units of clothing (or any linear combination). What type of trade will occur between these two countries? Explain.

32) Suppose the U.S. can produce 10 units of food and 5 units of clothing (or any such linear combination) and Canada can produce 6 units of food and 4 units of clothing (or any such linear combination). If trade occurs between these two countries, which should produce more food and which more clothing?

10.5 Efficiency and Equity

1) Any policy change that results in a Pareto-superior allocation 1.

A) will increase welfare under certain conditions.

2.

B) must increase welfare.

3.

C) will leave welfare unchanged.

4.

D) will have an unpredictable effect on welfare.

2) A cake is to be shared by two people. Both desire the largest piece possible. One of the two will cut the cake. Under which of the following situations will the cutter adopt a Rawlsian social welfare function? 1.

A) The person cutting the cake chooses the first piece.

2.

B) The person not cutting the cake chooses the first piece.

3.

C) The two individuals will bid for the right to cut the cake and choose first.

4.

D) The two individuals will toss a coin for the right to cut the cake and choose first.

3) Suppose in a democratic society, all voters prefer choice G over choice B; however, when the two choices are presented along with a third choice, R, B wins the election. This violates the assumption of 1.

A) transitivity.

2.

B) non-dictatorship.

3.

C) independence of irrelevant alternatives.

4.

D) completeness.

4) No clearly defined socially preferred outcome may result when majority voting on outcomes because 1.

A) often voters don't understand the outcomes.

2.

B) voting may violate the independence of irrelevant alternatives.

3.

C) voting may lead to incomplete preferences.

4.

D) voting may lead to non-transitive preference.

5) If everyone's utility is given equal weight and a change in resource allocation results in one person's gain exceeding another person's loss, we can say that the new allocation 1.

A) is Pareto superior to the original one.

2.

B) increases social welfare.

3.

C) decreases social welfare.

4.

D) is efficient.

6) If a society only cares about efficiency and not equity, then 1.

A) all points on the contract curve yield the same level of social welfare.

2.

B) it will not rely on competitive markets to allocate goods.

3.

C) it will maximize the utility of its worst-off member.

4.

D) an equitable outcome is impossible.

7) If a society relies on competitive markets to allocate goods, then 1.

A) an equitable distribution is assured.

2.

B) an equitable distribution is certain to not occur.

3.

C) the competitive equilibrium will be Pareto-superior to any other.

4.

D) social welfare as measured by consumer surplus plus producer surplus will equal zero.

8) A dictator is most likely to 1.

A) adopt a Rawlsian social welfare function.

2.

B) maximize her own utility.

3.

C) place equal weight on everyone's utility function.

4.

D) have nontransitive preferences.

9) If society were to maximize the utility of its worst-off member, the final allocation would most likely be 1.

A) relatively egalitarian.

2.

B) on the contract curve.

3.

C) Pareto-efficient.

4.

D) one in which one person gets everything.

10) If society were to maximize the utility of its best-off member, the final allocation would be 1.

A) perfect equity.

2.

B) on the contract curve.

3.

C) Pareto-efficient.

4.

D) one in which one person gets everything.

11) For most commonly used social welfare functions, an efficient allocation is 1.

A) always preferred over any inefficient allocation.

2.

B) not possible.

3.

C) usually preferred.

4.

D) never preferred.

12) The end of slavery in the United States represented 1.

A) a large wealth transfer from the South to the North.

2.

B) a large wealth transfer from the future to the present.

3.

C) a large wealth transfer from Southern slave-holders to newly freed slaves.

4.

D) a nominal wealth transfer from Southern slave-holders to newly freed slaves.

13) Comparing the distribution of wealth of the wealthiest 1% of the population in the United States before and after the recent Great Recession to what occurred before and after the Great Depression, 1.

A) the percentage of the wealth of the wealthiest 1% increased after the recent Great Recession, unlike what happened after the Great Depression

2.

B) the percentage of the wealth of the wealthiest 1% increased after the recent Great Recession, similar to what happened after the Great Depression.

3.

C) the percentage of the wealth of the wealthiest 1% declined after the recent Great Recession, unlike what happened after the Great Depression.

4.

D) the percentage of the wealth of the wealthiest 1% declined after the recent Great Recession, similar to what happened after the Great Depression.

14) The Arrow impossibility theorem suggests 1.

A) democracies are doomed to fail in the long-run.

2.

B) dictatorships are impossible in the long-run.

3.

C) there is no universally applicable decision rule in a majority-rule democracy.

4.

D) there is no way to make democracy better than a dictatorship.

15) Most of the debates in the U.S. Congress center on 1.

A) efficiency concerns.

2.

B) equity concerns.

3.

C) both efficiency and equity equally.

4.

D) market inefficiency.

16) At a given point in time, the Rawlsian welfare function gives equal weight to each individual's utility.

17) Equity and efficiency can be achieved simultaneously through competition.

18) The United States is the most inequitable country in the developed world.

19) If a market is controlled by one perfect price discriminator who is able to charge each consumer the highest price that consumer is willing to pay, the seller will produce output until the price paid by the last consumer is equal to the marginal cost of making the good. That is, the price of the last good equals the marginal cost of making the good. If welfare is measured as consumer surplus plus producer surplus, compare this market structure to a competitive market in terms of efficiency and equity. perfect price discriminator is Pareto efficient. Producer surplus plus consumer surplus is the same as in a competitive market. From an equity standpoint, however, there is much less equity than in a competitive market. A perfect price discriminator can capture all of the consumer surplus for herself, leaving consumer surplus equal to zero.

20) Explain why a government may select an inefficient allocation.

Microeconomics, 7e (Perloff) Chapter 11 Monopoly

11.1 Monopoly Profit Maximization

1) For a monopoly, marginal revenue is less than price because 1.

A) the firm is a price taker.

2.

B) the firm must lower price if it wishes to sell more output.

3.

C) the firm can sell all of its output at any price.

4.

D) the demand for the firm's output is perfectly elastic.

2) For a monopoly, marginal revenue is less than price because 1.

A) the demand for the firm's output is downward sloping.

2.

B) the firm has no supply curve.

3.

C) the firm can sell all of its output at any price.

4.

D) the demand for the firm's output is perfectly elastic.

3) Marginal Revenue is 1.

A) the increase in total revenue from selling one more unit of output.

2.

B) equal to P(1+1/ε).

3.

C) equal to P when the price elasticity of demand is infinite.

4.

D) All of the above.

4) At the current level of output, a firm's marginal cost equals 16 and marginal revenue equals 10. The firm 1.

A) is producing the profit-maximizing amount.

2.

B) should produce more.

3.

C) should produce less.

4.

D) Not enough information.

5) At an output level of 100, a monopolist faces MC = 15 and MR = 17. At output level q = 101, the monopolist's MC = 16 and MR = 15. To maximize profits, the firm 1.

A) should produce 100 units.

2.

B) should produce 101 units.

3.

C) cannot maximize profits.

4.

D) is not a monopoly.

6) If a firm is able to influence its price, 1.

A) it is a monopoly.

2.

B) it has constant marginal revenue.

3.

C) it sells its output at a constant price.

4.

D) it faces a downward-sloping demand curve.

7) One difference between a monopoly and a competitive firm is that 1. 2.

A) a monopoly is a price taker. B) a monopoly maximizes profit by setting marginal revenue equal to marginal cost.

3.

C) a monopoly faces a downward sloping demand curve.

4.

D) None of the above.

8) If the inverse demand function for a monopoly's product is p = a - bQ, then the firm's marginal revenue function is 1.

A) a.

2.

B) a - (1/2)bQ.

3.

C) a - bQ.

4.

D) a - 2bQ.

9) If the inverse demand function for a monopoly's product is p = 100 - 2Q, then the firm's marginal revenue function is 2.

A) -2.

3.

B) 100 - 4Q.

4.

C) 200 - 4Q.

5.

D) 200 - 2Q.

10) If the inverse demand curve a monopoly faces is p = 100 - 2Q, then profit maximization 1.

A) is achieved when 25 units are produced.

2.

B) is achieved by setting price equal to 25.

3.

C) is achieved only by shutting down in the short run.

4.

D) cannot be determined solely from the information provided.

11) If the inverse demand curve a monopoly faces is p = 100 - 2Q, and MC is constant at 16, then profit maximization 1.

A) is achieved when 21 units are produced.

2.

B) is achieved by setting price equal to 21.

3.

C) is achieved only by shutting down in the short run.

4.

D) cannot be determined solely from the information provided.

12) If the inverse demand curve a monopoly faces is p = 100 - 2Q, and MC is constant at 16, then profit maximization is achieved when the monopoly sets price equal to 16.

A) 16.

17.

B) 21.

18.

C) 25.

19.

D) 58.

13) If the inverse demand curve a monopoly faces is p = 100 - 2Q, and MC is constant at 16, then maximum profit 336.

A) equals $336.

337.

B) equals $882.

338.

C) equals $1,218.

339.

D) cannot be determined solely from the information provided.

14) The monopoly maximizes profit by setting 1.

A) price equal to marginal cost.

2.

B) price equal to marginal revenue.

3.

C) marginal revenue equal to marginal cost.

4.

D) marginal revenue equal to zero.

15) The above figure shows the demand and cost curves facing a monopoly. The monopoly maximizes profit by selling 1.

A) 0 units.

2.

B) 25 units.

3.

C) 50 units.

4.

D) 75 units.

16) The above figure shows the demand and cost curves facing a monopolist. The monopoly maximizes profit by setting price equal to 100.

A) $100.

101.

B) $200.

102.

C) $300.

103.

D) $400.

17) The above figure shows the demand and cost curves facing a monopoly. Maximum profit equals 1.

A) $0.

2.

B) $100.

3.

C) $1,000.

4.

D) $2,500.

18) A profit-maximizing monopolist will never operate in the portion of the demand curve with price elasticity equal to 3.

A) -3.

4.

B) -1.

5.

C) -1/3.

6.

D) Any of the above—the price elasticity does not matter.

19) A monopolist faces the inverse demand curve P = 60 - Q. It has variable costs of Q2 so that its marginal costs are 2Q, and it has fixed costs of 30. The monopoly's profit maximizing output is 5.

A) 5.

6.

B) 10.

7.

C) 15.

8.

D) 20.

20) A monopolist faces the inverse demand curve P = 60 - Q. It has variable costs of Q2 so that its marginal costs are 2Q, and it has fixed costs of 30. The monopoly's profit maximizing price is 55.

A) 55.

56.

B) 50.

57.

C) 45.

58.

D) 40.

21) A monopolist faces the inverse demand curve P = 60 - Q. It has variable costs of Q2 so that its marginal costs are 2Q, and it has fixed costs of 30. At its profit maximizing output level, the monopoly's average cost is 11.

A) 11.

12.

B) 13.

13.

C) 17.

14.

D) 21.5.

22) A monopolist faces the inverse demand curve P = 60 - Q. It has variable costs of Q2 so that its marginal costs are 2Q, and it has fixed costs of 30. The monopoly's maximum profit is 220.

A) 220.

221.

B) 370.

222.

C) 420.

223.

D) 510.

23) Which of the following markets is closest to a monopoly? 1.

A) a firm with a 90% market share

2.

B) the only gas station for 100 miles

3.

C) cable television

4.

D) garbage disposal

24) The monopolist's marginal revenue curve 1.

A) doesn't exist.

2.

B) lies below the demand curve.

3.

C) is identical to the demand curve.

4.

D) lies above the demand curve.

25) A monopolist changes price from $1 to $2 and sells 10 fewer units. The marginal revenue is 1.

A) $10

2.

B) -$10

3.

C) $0

4.

D) impossible to determine with the information provided.

26) When the marginal revenue curve cuts the horizontal axis, 1.

A) demand is relatively elastic.

2.

B) demand is relatively inelastic.

3.

C) demand is perfectly elastic.

4.

D) demand is unitary elastic.

27) If a firm is a profit maximizer and faces positive marginal costs, 1.

A) there is a natural limit to the size of the firm, where MR = 0.

2.

B) there is no natural limit to the size of the firm; it can be as large as it wants to be.

3. 4.

C) there is a natural limit to the size of the firm, where MR > 0. D) there is no natural limit to the size of the firm, hence the need for government regulation.

28) A monopolist that chooses price 1. 2.

A) necessarily produces less than a monopolist that chooses quantity, hence the laws against price fixing. B) produces the same amount as a monopolist that chooses quantity.

3.

C) produces more than a monopolist that chooses quantity, thus the irony of laws against price fixing.

4.

D) could produce more or less than a monopolist that chooses quantity since the demand curve is not specified.

29) Consider a monopoly who posts an economic profit of $10,000,000. All else equal, this monopolist should expect 1.

A) entry into its market, prices to fall, profits to fall.

2.

B) no entry into its market, prices to remain the same, profits to remain the same.

3.

C) exit from its market, prices to rise, profits to rise.

4.

D) entry into its market, the need to increase price, profits to remain the same.

30) A profit maximizing monopolist 1.

A) is guaranteed to lose money because of a lack of competition.

2.

B) is not guaranteed to make a positive profit.

3.

C) is guaranteed to make a positive profit, hence the desire to be a monopolist.

4.

D) is guaranteed to make a non-negative profit, otherwise government would step in to assist.

31) In general, the quantity that maximizes revenue for the monopolist 1.

A) is greater than the quantity that maximizes profit.

2.

B) is less than the quantity that maximizes profit.

3.

C) is the same as the quantity that maximizes profit.

4.

D) is illegal according to anti-trust statutes.

32) Which of the following is most necessary for a monopolist to survive in the long run? 1.

A) a legal protection from entry

2.

B) a perfect product

3.

C) a brilliant Chief Operating Officer

4.

D) an excellent marketing campaign

33) If a monopoly's demand curve shifts to the right, the 1.

A) monopoly will charge a higher price.

2.

B) monopoly will charge a lower price.

3.

C) monopoly will sell more.

4.

D) monopoly's decision cannot be determined.

34) The fact that a monopoly has to take the shapes of marginal cost AND marginal revenue into account when making decisions is reflected in the fact that

1.

A) monopolies don't have a supply curve.

2.

B) monopolies don't have a demand curve.

3.

C) monopolies have the same supply curve as perfectly competitive firms.

4.

D) monopolies maximize profit.

35) If the demand for a monopoly's output shifts rightward, the change in quantity produced is 1.

A) positive.

2.

B) negative.

3.

C) zero.

4.

D) not predictable.

36) If the demand for a monopoly's output shifts rightward, the change in quantity produced is not predictable because 1.

A) the monopoly is a profit maximizer.

2.

B) the monopoly is a price taker.

3.

C) the monopoly has no supply curve.

4.

D) the monopoly's marginal cost curve might not be upward sloping.

37) If the demand for a monopoly's output shifts rightward, the change in quantity produced is not predictable because 1.

A) the monopoly is a profit maximizer.

2.

B) the monopoly is a price taker.

3.

C) the monopoly has no supply curve.

4.

D) the monopoly's marginal cost curve might not be upward sloping.

38) If the monopoly's demand curve intersects the AVC curve at minimum AVC, the firm will shut down.

39) Since there are no close substitutes for the monopoly's product, the monopoly can charge any price it wishes.

40) Since a monopoly can set any price it wants, it always makes a profit.

41) A monopoly always operates in the inelastic portion of its demand curve.

42) A monopoly does not have a supply curve.

43) A monopoly's output decision depends only on the shape of its marginal cost curve. ve.

44) In a recent court case, an expert witness defined a monopoly as a firm that can "raise price without reducing its total revenue." What does this imply about the elasticity of demand? Would this definition hold for a profit-maximizing monopoly? Explain.

45) Suppose a monopolist has TC = 100 + 10Q + 2Q2, and the demand curve it faces is p = 90 - 2Q. What will be the price, quantity, and profit for this firm?

46) Explain why a monopolist has no supply curve.

11.2 Market Power

1) The ability of a monopoly to charge a price that exceeds marginal cost depends on 1.

A) the price elasticity of supply.

2.

B) price elasticity of demand.

3.

C) slope of the demand curve.

4.

D) shape of the marginal cost curve.

2) The Lerner Index is 1.

A) the ratio of the difference between price and marginal cost to price.

2.

B) equal to (Price - MC)/Price.

3.

C) a measure of market power.

4.

D) All of the above.

3) The above figure shows the demand and cost curves facing a monopoly. At the profitmaximizing price, the elasticity of demand equals 1.

A) -1.

2.

B) zero.

3.

C) infinity.

4.

D) -3.

4) The above figure shows the demand and cost curves facing a monopoly. If the firm is a profit maximizer, its Lerner Index will equal

1.

A) 1.

2.

B) 1/3.

3.

C) 1.5.

4.

D) 3.

5) If the demand for a firm's output is perfectly elastic, then the firm's Lerner Index equals 1.

A) zero.

2.

B) one.

3.

C) infinity.

4.

D) one-half.

6) If the demand curve a monopolist faces is perfectly elastic, then the ratio of the firm's price to the marginal cost is

1.

A) 0.

2.

B) 1.

3.

C) 2.

4.

D) None of the above—the answer cannot be determined.

7) The more elastic the demand curve, a monopoly 1.

A) will have a larger Lerner Index.

2.

B) will face a lower marginal cost.

3.

C) will earn more profit.

4.

D) will lose more sales as it raises its price.

8) The introduction of satellite television systems would cause the Lerner Index for cable television to 1.

A) become smaller.

2.

B) increase.

3.

C) change in accordance to the increase in market power of cable TV providers.

4.

D) be unchanged.

9) If the inverse demand curve a monopoly faces is p = 100 - 2Q, and MC is constant at 16, then the firm's Lerner Index equals 16.

A) 58/16.

17.

B) 16/42.

18.

C) 58/42.

19.

D) 42/58.

10) The introduction of satellite television systems would cause the demand curve for cable television to be

1.

A) more elastic.

2.

B) less elastic.

3.

C) perfectly inelastic.

4.

D) unchanged.

11) Humana Hospital's price/marginal cost ratio of 2.3 is most likely to decline if 1.

A) the number of nearby hospitals increases.

2.

B) the number of nearby hospitals decreases.

3.

C) the demand curve for hospital services shifts rightward.

4.

D) the demand curve for hospital services becomes steeper.

12) As other firms enter a monopoly's market, the monopoly's market power 1.

A) is unaffected.

2.

B) declines.

3.

C) increases.

4.

D) increases according to the Lerner Index but decreases according to the price/marginal cost ratio.

13) If a monopoly can produce a good at zero marginal cost, then its Lerner Index is 1.

A) zero.

2.

B) one.

3.

C) infinity.

4.

D) undetermined.

14) A monopoly sets a price of $50 per unit for an item that has a marginal cost of $10. Assuming profit maximization, the implicit demand elasticity is 1.

A) -0.2.

2.

B) -0.8.

3.

C) -1.25.

4.

D) -5.0.

15) A monopoly incurs a marginal cost of $1 for each unit produced. If the price elasticity of demand equals -2.0, the monopoly maximizes profit by charging a price of 1.

A) $1.00.

2.

B) $1.50.

3.

C) $2.00.

4.

D) $3.00.

16) If a monopoly's Lerner Index exceeds 1, then 1.

A) it is earning maximum profit.

2.

B) it has ultimate market power.

3.

C) it must be pricing below marginal cost.

4.

D) marginal revenue is negative.

17) If a monopoly discovers that the demand for its output has become more elastic at the original output level, then it will respond by 1.

A) producing more and setting a higher price.

2.

B) setting a lower price.

3.

C) setting a higher price.

4.

D) producing more while leaving price unchanged.

18) Which of the following DOES NOT contribute to the market power of a firm? 1.

A) number of available substitutes

2.

B) the color of the product

3.

C) legal protections

4.

D) the number of firms in the market

19) Market power guarantees profit. 1.

A) True, which is why firm's locate as far away from each other as possible.

2.

B) False, market power guarantees price greater than marginal cost.

3.

C) True, market power guarantees price greater than average cost.

4.

D) False, market power guarantees price equal to average cost.

20) Market power is illegal. 1.

A) True, no one is allowed to charge a price greater than marginal cost.

2.

B) False.

3.

C) True, no one is allowed to charge a price greater than average cost.

4.

D) False, because market power guarantees price equal to average cost.

21) The less elastic is the demand for a firm's product, the greater is that firm's market power.

22) The Lerner Index is derived from the profit-maximizing condition of a firm. e.

23) When would a profit-maximizing monopolist that operates with no government intervention choose to produce the competitive level of output?

24) It is a conventional practice among apparel retailers to set the retail price of clothing at twice the cost paid to the manufacturer. For example, if the retailer pays $7 for a pair of jeans, the jeans will retail for $14. What must the price elasticity of demand be for this practice to be profit maximizing?

25) For profit-maximizing monopolies, explain why the boundaries on the Lerner Index are 0 and 1.

11.3 Market Failure Due to Monopoly Pricing

1) The loss associated with the fact that at the profit-maximizing quantity consumers value the goods more than it cost to produce them is called 1.

A) deadweight loss.

2.

B) comparative loss.

3.

C) Lerner Loss.

4.

D) Consumer Value Loss.

2) The above figure shows the demand and cost curves facing a monopoly. The deadweight loss of this monopoly is 100.

A) $100.

101.

B) $250.

102.

C) $1,250.

103.

D) $2,500.

3) The above figure shows the demand and cost curves facing a monopoly. A $100 per unit tax would raise price by 100.

A) $100.

101.

B) $50.

102.

C) $25.

103.

D) $0.

4) The above figure shows the demand and cost curves facing a monopoly. If a $100 per unit tax is charged, what is the incidence of the tax on consumers? 1.

A) 100%

2.

B) 50%

3.

C) 25%

4.

D) 0%

5) The above figure shows the demand and cost curves facing a monopoly. If a $100 per unit tax is charged, the loss in welfare resulting from the tax is 250.

A) $250.

251.

B) $312.50.

252.

C) $1,250.

253.

D) $1,562.50.

6) The above figure shows the demand and marginal cost curves for a monopoly. The deadweight loss of this monopoly equals 1.

A) h.

2.

B) c.

3.

C) c + f.

4.

D) c + d + e + f.

7) The above figure shows the demand and marginal cost curves for a monopoly. Under monopoly, consumer surplus equals 1.

A) a + b.

2.

B) a + b + c.

3.

C) a + b + c + d + e + f.

4.

D) None of the above.

8) The existence of a deadweight loss associated with a monopoly can be seen because 1.

A) consumers are willing to pay more for the last unit of output than it costs to produce.

2.

B) the cost of the last unit produced is more than consumers are willing to pay for it.

3.

C) the producer surplus is larger than in a competitive market.

4.

D) None of the above.

9) If the government desires to raise a certain amount of revenue by taxing a monopoly, an ad valorem tax will 1.

A) generate the same loss of consumer surplus as a specific tax.

2.

B) generate a greater loss of consumer surplus than a specific tax.

3.

C) generate a smaller loss of consumer surplus than a specific tax.

4.

D) generate no loss of consumer surplus.

10) The government prefers an ad valorem tax to a specific tax that reduces the monopoly output by the same amount because 1.

A) consumers are not harmed by the ad valorem

2.

B) the monopoly prefers the ad valorem

3.

C) consumers prefer the ad valorem

4.

D) the ad valorem tax transfers more revenue from the monopoly to the government.

11) If the inverse demand curve a monopoly faces is p = 100 - 2Q, and MC is constant at 16, then the deadweight loss from monopoly equals 21.

A) $21.

22.

B) $441.

23.

C) $882.

24.

D) $1,764.

12) If the inverse demand curve a monopoly faces is p = 100 - 2Q, MC is constant at 16, and the government imposes an $8 per unit specific tax on the monopoly, the deadweight loss due to both the monopoly and the tax is 529.

A) $529.

530.

B) $1332.

531.

C) $1764.

532.

D) $441.

13) If the inverse demand curve a monopoly faces is p = 100 - 2Q, MC is constant at 16, and the government imposes an $8 per unit specific tax on the monopoly, the deadweight loss solely due to the tax is

88.

A) $88.

89.

B) $152.

90.

C) $361.

91.

D) $441.

14) A monopolist faces the inverse demand curve P = 60 - Q. It has variable costs of Q2 so that its marginal costs are 2Q, and it has fixed costs of 30. If a governmental agency imposes an $8 per unit specific tax on output, the deadweight loss from both the monopoly and the tax is 37.

A) $37.50.

38.

B) $73.00

39.

C) $526.50.

40.

D) $562.50.

15) If the government imposes a specific tax on a monopoly, the consumer's tax incidence

1.

A) can exceed 100%.

2.

B) will always be between 0-100%.

3.

C) may be negative.

4.

D) will be the same as when the tax is imposed on a perfectly competitive firm.

16) The producer surplus to a monopolist must be 1.

A) less than zero or the firm is in violation of anti-trust statutes.

2.

B) at least as great as the producer surplus in a competitive market.

3.

C) positive, otherwise why would the monopoly produce?

4.

D) the same as for a competitive market.

17) In spring 2008, the U.S. Congress proposed to tax oil companies because of their near-monopoly status. This could have the unintended consequence of 1.

A) increasing the equilibrium price by more than the tax.

2.

B) destroying the oil companies and leaving the United States without oil.

3.

C) increasing the profit of the best oil company.

4.

D) decreasing the power of the U.S. Congress.

18) If the government's goal is to generate a certain amount of tax revenue, a specific tax and an ad valorem tax on a monopoly have the same impact on social welfare.

19) The deadweight loss represents the sum of additional consumer and producer surplus should the firm produce the quantity where P = MC rather than where MR = MC.

20) Suppose that market demand for a good is Q = 480 - 2p. The marginal cost is MC = 2Q. Calculate the deadweight loss resulting from a monopoly in this market.

21) Why is the monopoly total welfare lower than the competitive total welfare?

22) If the government sets a specific tax and an ad valorem tax so that they raise the same amount of tax revenue, why does the ad valorem tax reduce output less than the specific tax?

11.4 Causes of Monopoly

1) A flour mill holding exclusive contracts to 95% of the wheat in a large geographic area may operate as a flour producing monopoly locally because 1.

A) the mill has a very inelastic supply curve.

2.

B) the mill is a natural monopoly.

3.

C) the mill controls a key input.

4.

D) the government will declare it a monopoly.

2) Which of the following total cost functions suggests the presence of a natural monopoly? 1.

A) TC = 2Q

2.

B) TC = 100 + 2Q

3.

C) TC = 100 + 2Q2

4.

D) All of the above.

3) Which of the following average cost functions suggests the presence of a natural monopoly? 1.

A) AC = 2

2.

B) AC = 100/Q + 2

3.

C) TC = 100/Q + 2Q

4.

D) All of the above.

4) The situation in which one firm can produce the total output of the market at lower cost than several firms is called a 1.

A) natural monopoly.

2.

B) pure monopoly.

3.

C) ruling monopoly.

4.

D) cost monopoly.

5) Which of the following is most likely the most beneficial form of monopoly advantage? 1.

A) better production methods

2.

B) input hoarding

3.

C) decreasing returns to scale

4.

D) government protection

6) Limited government licenses that create a monopoly do so because 1.

A) the license generates a marginal cost advantage.

2.

B) the monopoly will become a natural monopoly.

3.

C) a barrier to enter the market exists.

4.

D) All of the above.

7) An exclusive right to sell a new and useful product, process, substance, or design for a fixed period of time is called a 1.

A) patent.

2.

B) barrier to entry.

3.

C) monopoly.

4.

D) research disincentive.

8) A justification for patents is that without patents consumer surplus would be 1.

A) larger than with the patent.

2.

B) zero since the product would not be invented.

3.

C) only slightly smaller than with the patent.

4.

D) zero since the monopoly would be a revenue maximizer.

9) Patents 1.

A) will create a profit incentive to do research.

2.

B) might be welfare reducing if granted for too long a period.

3.

C) serve as a barrier to entry.

4.

D) All of the above.

10) Why does a patent stimulate research? 1. 2.

A) Patents give firms time to do research. B) Patents give firms the opportunity to recover research costs and thus serve as a profit motive.

3.

C) Without patents firms would not research as much.

4.

D) They don't.

11) Government actions that create monopolies 1.

A) spur product innovation by the monopoly.

2.

B) create deadweight loss.

3.

C) result in lower average costs of production.

4.

D) ensure that firms price at marginal cost.

12) The longer a patent lasts,

1.

A) the greater the potential economic profits available to an innovator.

2.

B) the lower the deadweight loss.

3.

C) the lower the average costs of production.

4.

D) the less innovation there will be.

13) An alternative to patent protection might be 1.

A) dollar prizes for innovation.

2.

B) an appeal to altruism.

3.

C) granting of citizenship for innovation.

4.

D) forcing innovators to cede their innovations.

14) What is one reason patents are required? 1.

A) Information concerning innovation is private and secure.

2.

B) Human nature is corrupt.

3.

C) Information concerning innovation is generally not private and secure.

4.

D) The patent only makes things better in the short-run.

15) Suppose a patent is granted for a product that has the linear demand curve P = a b Q. The constant marginal cost of producing this product is $50 per unit, a unit sells for $150, and consumers purchase 100 units of the good at that price. If the monopoly is maximizing profit, b equals 1.

A) 1.

2.

B) 1.5.

3.

C) 2.

4.

D) 2.5.

16) Suppose a patent is granted for a product that has the linear demand curve P = a b Q. The constant marginal cost of producing this product is $50 per unit, a unit sells for $150, and consumers purchase 100 units of the good at that price. If the monopoly is maximizing profit, the deadweight loss is 5000. A) $5000.

5001. B) $2500. 5002. C) $7500. 5003. D) $10000.

17) If a firm in an industry experiences very high fixed costs and constant marginal cost, it is a good candidate for a natural monopoly.

18) The optimal patent length is equal to 20 years.

19) The average cost for a typical electric-power-production firm is AC = 100 - 10Q + Q2 where Q is measured in billion kilowatt hours per day. At the current regulated price, consumers demand 4 billion kilowatt hours per day. Is this market a natural monopoly? If demand increases to 10 billion kilowatt hours, is this market a natural monopoly? Explain.

20) The SSS Co. has a patent on a particular medication. The medication sells for $1 per daily dose and marginal cost is estimated to be a constant at $0.20. Assuming linear demand and marginal cost curves, use this information to estimate the deadweight loss from monopoly pricing if the firm currently sells 1,000 doses per day. Can this loss be justified?

11.5 Government Actions That Reduce Market Power

1) Optimal price regulation sets price equal to 1.

A) marginal cost.

2.

B) average variable cost.

3.

C) average cost.

4.

D) minimum average cost.

2) If the government regulates the price a monopoly can charge, and the price ceiling is set below what the competitive market price would be, then 1.

A) a shortage will exist.

2.

B) a surplus will exist.

3.

C) producer surplus is maximized.

4.

D) consumer surplus is maximized.

3) The government forcing a monopoly telecommunications company to allow other firms to use its cables is an attempt to 1.

A) regulate prices.

2.

B) decrease the monopoly market power by eliminating a natural monopoly.

3.

C) decrease the monopoly market power by increasing competition.

4.

D) None of the above.

4) When attempting price regulation, a government faces what problem(s)? 1.

A) limited information

2.

B) bribes

3.

C) uncooperative firms

4.

D) All of the above.

5) If the government attempts to force a natural monopoly to charge a price equal to marginal cost, 1.

A) the natural monopoly will shut down.

2.

B) the natural monopoly will still make high profits.

3.

C) the natural monopoly's marginal cost curve will shift up.

4.

D) total welfare is maximized.

6) If the government wants to regulate a natural monopoly, it will force the firm to set price equal to 1.

A) average cost.

2.

B) marginal cost.

3.

C) marginal revenue.

4.

D) None of the above.

7) Suppose a monopolist's demand curve is P = 60 - Q, and its cost function is C = 10Q + 50 so its marginal cost is 10. If a governmental agency wished to set the price that maximized social welfare, that price would be 10.

A) $10.00.

11.

B) $11.02.

12.

C) $14.57.

13.

D) $35.00.

8) Suppose a monopolist's demand curve is P = 60 - Q, and its cost function is C = 10Q + 50 so its marginal cost is 10. If a governmental agency wished to set the price so that it created the smallest deadweight loss without causing the monopolist to have negative economic profits (thus shutting down), that price would be 10.

A) $10.00.

11.

B) $11.02.

12.

C) $14.57.

13.

D) $35.00.

9) A dominant firm's residual demand curve is 1.

A) the horizontal difference between the market demand curve and the supply curve of the fringe firms.

2.

B) the vertical difference between the market demand curve and the supply curve of the fringe firms.

3.

C) the demand curve left for the fringe firms after the dominant firm has determined an output level.

4.

D) None of the above.

10) In the dominant firm model as evidenced by the production of iPods by Apple, the entrance of the competitive fringe firms has what effect on the dominant firm? 1.

A) Its price is lower, but it produces more output.

2.

B) Its price is lower, and it produces less output.

3.

C) Its price is the same, but it produces less output.

4.

D) Its price is higher, but it produces more output.

11) What is one problem with trying to regulate a monopoly's price? 1.

A) The government needs information on the monopoly's marginal cost.

2.

B) The government needs information on the price people are willing to pay.

3.

C) The government needs to identify which firm is a monopolist.

4.

D) Anything that the government does is problematic.

12) Regulation is guaranteed to be more efficient than a monopoly. 1.

A) True, the government is able to internalize the dead weight loss of the monopoly.

2.

B) True, the consumers are better off if government provides the product rather than a private firm.

3.

C) False, the government does not always have sufficient information to provide a more efficient market outcome.

4.

D) False, the consumers are worse off under government regulation.

13) Which is an ironic solution to the government protected monopoly? 1.

A) The government might try to "force" less competition in the market.

2.

B) The government might "do more" by "doing less," i.e., by removing the monopoly's protection.

3.

C) The dead weight loss goes to the government.

4.

D) The inherent unfairness of monopoly can only be solved by dictatorship.

14) Forcing a natural monopoly to charge P = MC will not work.

15) A monopoly faces an inverse demand curve of P = 100 - 2Q. The marginal cost curve is MC = .5Q. What government price ceiling would represent optimal price regulation?

16) In the early 1900s, U.S. Steel acted as a dominant firm with over sixty percent (60%) of the market. Many, including the Supreme Court in 1920, viewed U.S. Steel as a "good" monopolist since competitive fringe firms prospered under this arrangement. Graph the market for steel as it might have existed at this time showing that being a member of the competitive fringe can be profitable. Assume a linear market demand and linear MC and AC that slope upward.

Answer:

11.6 Networks, Dynamics, and Behavioral Economics

1) The situation where one person's demand for a good depends on the consumption of the good by others is called a 1.

A) network externality.

2.

B) network internality.

3.

C) consumption externality.

4.

D) production externality.

2) The situation in which a person places greater value on a good as more and more people possess it is called the 1.

A) Bandwagon Effect.

2.

B) Greater Value Effect.

3.

C) Snob Effect.

4.

D) Behavioral Effect.

3) The situation in which a person places greater value on a good as fewer and fewer people possess it is called the 1.

A) Bandwagon Effect.

2.

B) Greater Value Effect.

3.

C) Snob Effect.

4.

D) Behavioral Effect.

4) A monopoly can be formed by a bandwagon effect. Which of the following products would most closely match the bandwagon effect? 1.

A) the Atlanta Braves baseball club

2.

B) the Dell Computer

3.

C) the Apple iPod

4.

D) the Chevrolet Corvette

5) The use of "introductory prices" suggests 1.

A) firms engaged in multi-period decision making.

2.

B) firms engaged in price gouging.

3.

C) firms engaged in anti-competitive behavior.

4.

D) firms engaged in single-period decision making.

6) The two-period dynamic monopoly model is more useful than the static monopoly model in analyzing monopoly behavior when

1.

A) the product produced requires a bandwagon effect.

2.

B) the product produced generates a positive network externality.

3.

C) the monopoly initially uses a lower introductory price.

4.

D) All of the above situations.

7) When generic drugs enter the market after the patent for a brand name drug expires, the price of the brand name drug often increases. This is usually due to 1.

A) the demand curve for the brand name drug shifting in but becoming more inelastic.

2.

B) the demand curve for the brand name drug not shifting, but the marginal cost of producing the good increases.

3.

C) the demand curve for the brand name drug shifting in but becoming more elastic.

4.

D) the demand curve for the brand name drug not shifting, but loyal customers are willing to pay a higher price for the brand name drug.

8) The telephone is an example of a product with network externalities.

9) Explain Microsoft Windows' monopoly positions in terms of network externalities.

Microeconomics, 7e (Perloff) Chapter 12 Pricing and Advertising

12.1 Conditions for Price Discrimination

1) Which of the following conditions must be true so that a firm can price discriminate? 1.

A) There are no other firms in the market.

2.

B) The good is a non-durable.

3.

C) The good cannot be easily resold.

4.

D) All of the above.

2) Why do firms engage in price discrimination? 1.

A) to decrease cost

2.

B) to increase profits

3.

C) to increase consumer surplus

4.

D) to prohibit the resale of their products

3) When firms price discriminate they turn ________ into ________. 1.

A) producer surplus, revenue

2.

B) consumer surplus, profit

3.

C) total cost, profit

4.

D) producer surplus, consumer surplus

4) Theatres charge lower prices for a matinee and usually don't accept coupons for the night showing of movies because 1.

A) consumers that attend the matinee have a higher price elasticity of demand.

2.

B) consumers that attend the night show have a lower price elasticity of demand.

3.

C) it increases profits compared to charging a single price.

4.

D) All of the above.

5) At many municipal golf courses, local residents pay a lower fee to play than other golfers do. One necessary condition for the golf course to be able to successfully price discriminate according to residency is that 1.

A) they can check the identification cards of golfers.

2.

B) local resident golfers and other golfers have the same price elasticity of demand to play at the municipal course.

3.

C) there are many golf courses nearby from which golfers can choose.

4.

D) they require all golfers to rent a cart.

6) If consumers are identical, then 1.

A) price discrimination is impossible.

2.

B) price discrimination can occur if each consumer has a downward-sloping demand curve for the product.

3.

C) perfect price discrimination is the only form of price discrimination that can increase a monopoly's profit.

4.

D) tie-in sales cannot increase a monopoly's profit.

7) Charging a higher price for a motel room to customers with dogs or cats than to customers with no pets is most likely an example of 1.

A) first-degree price discrimination.

2.

B) second-degree price discrimination.

3.

C) third-degree price discrimination.

4.

D) actual cost differences.

8) The case where a firm sells each unit at the maximum amount each customer is willing to pay for it is called 1.

A) first-degree price discrimination.

2.

B) second-degree price discrimination.

3.

C) third degree price discrimination.

4.

D) nonlinear price discrimination.

9) Price discrimination is welfare reducing. 1.

A) False, price discrimination can increase the coverage of a market thereby increasing welfare.

2.

B) False, price discrimination limits the coverage of a market thereby increasing welfare.

3.

C) True, price discrimination limits the coverage of a market thereby increasing welfare.

4.

D) True, price discrimination can increase the coverage of a market thereby increasing welfare.

10) Historically, price discrimination was considered illegal in all instances. More recently, antitrust authorities have discovered that 1. 2.

A) price discrimination can increase the coverage of a market thereby increasing welfare. B) price discrimination limits the coverage of a market thereby increasing welfare.

3.

C) price discrimination limits the coverage of a market thereby decreasing welfare.

4.

D) price discrimination can increase the coverage of a market thereby decreasing welfare.

11) Mouthwash is sold in 24 oz bottles for $2.40 and in 12 oz. bottles for $1.20. This represents 1.

A) price differentiation.

2.

B) price discrimination.

3.

C) marginal cost pricing.

4.

D) None of the above.

12) If a firm faces a flat demand curve, 1.

A) it cannot engage in price discrimination.

2.

B) it can only engage in two-part tariffs.

3.

C) it can only engage in perfect price discrimination.

4.

D) None of the above.

13) If two markets have the same price elasticity of demand at every price, a monopoly will not practice multimarket price discrimination.

14) Firms price discriminate to maximize total revenue.

15) Explain why a firm can earn more profit by price discrimination than from setting a uniform price.

16) Suppose two countries, A and B, are at war with each other. Country A is very wealthy; country B is very poor. The XYZ Co. produces tanks. Is XYZ able to set a different price for the tank sold to country A than the price for the tank sold to country B? Explain.

12.2 Perfect Price Discrimination

1) A perfect price discriminator 1. 2.

A) charges each buyer her reservation price. B) charges different prices to each customer based upon different costs of delivery.

3.

C) generates a deadweight loss to society.

4.

D) charges lower prices to customers who buy greater quantities.

2) A perfect-price-discriminating monopoly's marginal revenue curve 1.

A) lies below the demand curve.

2.

B) is the demand curve.

3.

C) varies for each consumer.

4.

D) is the same as the monopolist's marginal revenue curve.

3) Which of the following sellers is most able to perfectly price discriminate? 1.

A) a college or university

2.

B) the post office

3.

C) a clothing store

4.

D) a grocery supermarket

4) The deadweight loss generated by a perfect-price-discriminating monopoly 1.

A) equals the deadweight loss of a single-price monopoly.

2.

B) is greater than the deadweight loss of a single-price monopoly.

3.

C) equals zero.

4.

D) equals the sum of all lost consumer surplus.

5) A perfect-price-discriminating equilibrium maximizes 1.

A) consumer surplus.

2.

B) the associated deadweight loss.

3.

C) the market inefficiency.

4.

D) total welfare.

6) A good example of perfect price discrimination is 1.

A) selling concert tickets to individuals on the street corner.

2.

B) buying concert tickets at the ticket window.

3.

C) selling concert tickets at the ticket window.

4.

D) buying a concert ticket on the street corner.

7) When a firm has a monopoly in a market and also perfectly price discriminates, total welfare

1.

A) is maximized.

2.

B) is lower than in a perfectly competitive market.

3.

C) is higher than in a perfectly competitive market.

4.

D) is minimized.

8) If a market is controlled by a perfect-price-discriminating monopoly, then 1.

A) a deadweight loss is generated.

2.

B) there is no consumer surplus.

3.

C) consumer surplus is the same as under perfect competition.

4.

D) output is less than that of a single-price monopoly.

9) The above figure shows the market for a particular good. If the market is controlled by a perfect-price-discriminating monopoly, consumer surplus equals 1.

A) A.

2.

B) A + B + C.

3.

C) C.

4.

D) zero.

10) The above figure shows the market for a particular good. If the market is controlled by a perfect-price-discriminating monopoly, social welfare equals 1.

A) A.

2.

B) A + B + C.

3.

C) A + B + C + D + E.

4.

D) zero.

11) The above figure shows the market for a particular good. If the market is controlled by a perfect-price-discriminating monopoly, producer surplus equals 1.

A) A + B + C + D + E.

2.

B) D + E.

3.

C) E.

4.

D) zero.

12) The above figure shows the market for a particular good. If the market is controlled by a perfect-price-discriminating monopoly, the deadweight loss equals 1.

A) C + E.

2.

B) A + B + C.

3.

C) C.

4.

D) zero.

13) A monopoly will not be able to perfectly price discriminate if

1.

A) obtaining information about each buyer's reservation price is too costly.

2.

B) demand is very elastic.

3.

C) demand is very inelastic.

4.

D) resale is impossible.

14) A monopoly will not be able to perfectly price discriminate if 1.

A) each consumer does not reveal her reservation price.

2.

B) demand is very elastic.

3.

C) the firm's marginal cost curve is upward sloping.

4.

D) All of the above.

15) Which of the following helps a monopoly perfectly price discriminate? 1.

A) unit demand by each consumer

2.

B) the product is perishable

3.

C) the product is personalizable

4.

D) All of the above.

16) Assume a firm organizes all individuals by their willingness to pay (least to most). If the firm starts to perfectly price discriminate, what is likely to happen? 1.

A) Consumers start to arbitrage amongst themselves.

2.

B) The firm's profits will be maximized.

3.

C) The firm's costs will be minimized.

4.

D) The firm starts to arbitrage with consumers.

17) Assume you have four tickets to a U2 concert. You decide to sell each of them separately on an auction site such as eBay. Your auctions represent 1.

A) price differentiation.

2.

B) perfect price discrimination amongst those who bid for your tickets.

3.

C) perfect price discrimination amongst all people who buy tickets for the concert.

4.

D) an illegal act, subject to state and federal prosecution.

18) What is one reason car dealerships might move away from perfect price discrimination to uniform pricing? 1.

A) Perfect price discrimination doesn't work.

2.

B) Transaction costs erode the profit of perfect price discrimination.

3.

C) Consumers are ill-informed and tend to complain too much.

4.

D) Uniform pricing is always more profitable and more fair as well.

19) Suppose a monopoly's inverse demand curve is P = 100 -Q, it produces a product with a constant marginal cost of 20, and it has no fixed costs. Compared to the consumer surplus if the market were perfectly competitive, consumer surplus is how much less when the monopolist practices perfect price discrimination? 1.

A) 3200

2.

B) 1600

3.

C) 800

4.

D) 0

20) Suppose a monopoly's inverse demand curve is P = 100 -Q, it produces a product with a constant marginal cost of 20, and it has no fixed costs. How much more or less is the deadweight loss if the monopoly can practice perfect price discrimination compared to it practicing uniform pricing? 800.

A) The deadweight loss is smaller by 800.

801.

B) The deadweight loss is greater by 800.

802.

C) The deadweight loss is smaller by 1600.

803.

D) The deadweight loss is greater by 1600.

21) A perfect-price-discriminating monopoly maximizes social welfare as measured by the sum of producer surplus plus consumer surplus.

22) A perfect price discriminator receives a price equal to marginal revenue for each unit.

23) The above figure shows the market for a given product. Defining welfare as consumer surplus plus producer surplus, calculate the social welfare associated with perfect competition, single-price monopoly, and a perfect-price-discriminating monopoly. Which market structure(s) maximize social welfare? .

24) Explain using welfare measures whether consumers prefer a single price monopoly or a perfect-price-discriminating monopoly.

12.3 Group Price Discrimination

1) Bob is the only carpet installer in a small isolated town. The above figure shows the demand curves of two distinct groups of customers-residential and business. If the marginal cost of installing carpet is a constant $1 per sq yard, what price does Bob charge each segment? 1.

A) $1 in each market

2.

B) $5.50 in the residential market and $8 in the business market

3.

C) $1 in the residential market and $5 in the business market

4.

D) $10 in the residential market and $15 in the business market

2) Bob is the only carpet installer in a small isolated town. The above figure shows the demand curves of two distinct groups of customers-residential and business. Bob is likely to price discriminate because 1.

A) elasticities differ across markets.

2.

B) the installation of carpets cannot be resold.

3.

C) Bob can probably identify which consumers belong to which segment.

4.

D) All of the above.

3) Which of the following is an example for group price discrimination? 1.

A) a BMW selling for more than a VW

2.

B) local residents receiving a discount at the local golf course

3.

C) the fact that a razor is cheap and blades are expensive

4.

D) a hotel charging more for a room if the customers bring pets

4) If the price of business broadband is greater than that of residential broadband, all else equal, 1.

A) business has greater price elasticity than residential.

2.

B) residential has greater price elasticity than business.

3.

C) both have positive income elasticity.

4.

D) generally speaking, broadband is equally priced.

5) If two identifiable markets differ with respect to their price elasticity of demand and resale is impossible, a firm with market power will 1.

A) set a higher price in the market that is more price elastic.

2.

B) set a lower price in the market that is more price elastic.

3.

C) set price so as to equate the elasticity of demand across markets.

4.

D) set price equal to marginal cost in both markets.

6) A group price discriminator sells its product in Florida for three times the price it sets in New York. Assuming the firm faces the same constant marginal cost in each market and the price elasticity of demand in New York is -2.0, the demand in Florida 6.

A) has an elasticity of -6.0.

7.

B) is more price elastic than the demand in New York.

8.

C) has an elasticity of -1.2.

9.

D) has an elasticity of -0.67.

7) Suppose group price discrimination is possible; however, a firm sets the same price in each market. As a result, 1.

A) price elasticity of demand is the same in each market.

2.

B) the price-inelastic market will buy zero units.

3. 4.

C) marginal revenue in the more price-elastic market exceeds marginal revenue in the less price-elastic market. D) the deadweight loss is less than if the firm price discriminated.

8) If the demand for air travel were to change so that business travelers and vacationers have the same price elasticity of demand for air travel, 1.

A) airlines would charge the same price to each type of flyer.

2.

B) airlines would still charge business flyers a higher fare since the traveler's employer pays anyway.

3.

C) airlines would be driven out of business.

4.

D) airlines would counter by charging vacationers a higher fare.

9) If somebody posing as a vacationer were able to purchase large numbers of airline tickets from the airlines and later resell them to business travelers, 1.

A) group price discrimination on the part of airlines would no longer be profitable.

2.

B) group price discrimination on the part of airlines would no longer be profit maximizing.

3.

C) the airlines would respond by raising further the price charged to business flyers.

4.

D) this person would not earn any economic profit.

10) Relative to a single-price monopoly, the effect of group price discrimination on social welfare is

1.

A) beneficial.

2.

B) detrimental.

3.

C) neutral.

4.

D) ambiguous.

11) Coupons represent a form of price discrimination because they offer a low-cost way for firms to 1.

A) identify customers with apparently more elastic demand and offer them a lower price.

2.

B) retain loyal customers who are not price sensitive.

3.

C) offer discounts to consumers who buy larger quantities.

4.

D) perfectly price discriminate.

12) Airlines offer lower prices to vacationers than to business travelers because 1.

A) of government regulations requiring them to do so.

2.

B) business travelers do not care at all about costs.

3.

C) business travelers are less flexible in their travel plans than vacationers are.

4.

D) airlines know that business travelers enjoy flying more than vacationers do.

13) Why doesn't a firm price discriminate based on income levels? 1.

A) It would be nearly impossible to conveniently confirm any individual's income level.

2.

B) It is illegal to ask someone their income levels.

3.

C) It is immoral to price discriminate based on income levels.

4.

D) It is common practice for firms to price discriminate based on income.

14) Price discrimination reveals 1.

A) the inherent greed of Western culture.

2.

B) the inability for regulators to stop unethical practices.

3.

C) that individuals have different willingness to pay.

4.

D) that individuals have the same willingness to pay.

15) Suppose a profit-maximizing monopoly is able to employ multimarket price discrimination. The marginal cost of providing the good is constant and the same in both markets. The marginal revenue the firm earns on the last unit sold in the market with the lower price will be 1.

A) greater than the marginal revenue the firm earns on the last unit sold in the market with the higher price.

2.

B) less than the marginal revenue the firm earns on the last unit sold in the market with the higher price.

3.

C) equal to the marginal revenue the firm earns on the last unit sold in the market with the higher price.

4.

D) greater than the marginal cost of the last unit.

16) Pizza joints often offer substantially lower prices for pizza picked up at the shop compared to delivered pizza prices. This may be an attempt at 1.

A) perfect price discrimination.

2.

B) group price discrimination.

3.

C) quantity discrimination.

4.

D) second-degree price discrimination.

17) What is a primary difference between rebates and coupons? 1. 2. 3. 4.

A) Coupons allow individuals to sort themselves into the high-elasticity group after the sale. B) Neither coupons nor rebates are redeemed in high numbers. C) Rebates allow individuals to sort themselves into the high-elasticity group after the sale. D) Coupons are legal and rebates are illegal.

18) If a firm offers a senior citizen discount, 1.

A) the firm expects the average senior citizen to have a lower price elasticity of demand.

2.

B) the firm expects the average senior citizen to have a higher price elasticity of demand.

3.

C) senior citizens may be offended.

4.

D) it may be prosecuted for discrimination.

19) What is one reason online prices might be considerably lower than brick-and-mortar prices? 1.

A) Online retailers engage in more price discrimination.

2.

B) Brick-and-mortar retailers engage in more price discrimination.

3.

C) Brick and mortar retailers may have higher costs.

4.

D) Online retailers are more likely to have steep demand curves.

20) If the marginal cost of production is $10, the elasticity of demand for group 1 is -1.5, the elasticity of demand for group 2 is -2.5, and the price paid by group 1 is $15, the price for group 2 is 8.

A) $8.33.

9.

B) $27.

10.

C) $15.

11.

D) Impossible to tell.

21) Suppose a monopoly sells to two identifiably different types of customers, A and B, who are unable to practice arbitrage. The inverse demand curve for group A is PA = 10 QA, and the inverse demand curve for group B is PB = 18 - QB. The monopolist is able to produce the good for either type of customer at a constant marginal cost of 2, and the monopolist has no fixed costs. If the monopolist practices group price discrimination, the profit maximizing prices charged to each type of customer are 10.

A) PA= 6, and PB= 10.

11.

B) PA= 4 and PB= 8.

12.

C) PA= 10, and PB= 6.

13.

D) PA= 8, and PB= 4.

22) Suppose a monopoly sells to two identifiably different types of customers, A and B, who are unable to practice arbitrage. The inverse demand curve for group A is PA = 10 QA, and the inverse demand curve for group B is PB = 18 - QB. The monopolist is able to produce the good for either type of customer at a constant marginal cost of 2, and the monopolist has no fixed costs. If the monopolist is able to practice group price discrimination, the values of the elasticities of the two groups at the profit maximizing prices are 1.

A) εA= -1.25, and εB= -1.5.

2.

B) εA= -1.5, and εB= -1.25.

3.

C) εA= -0.67, and εB= -0.8.

4.

D) εA= -0.8, and εB= -0.67.

23) Suppose a monopoly sells to two identifiably different types of customers, A and B, who are unable to practice arbitrage. The inverse demand curve for group A is PA = 10 QA, and the inverse demand curve for group B is PB = 18 - QB. The monopolist is able to produce the good for either type of customer at a constant marginal cost of 2, and the monopolist has no fixed costs. If the monopoly uses uniform pricing, the deadweight loss is 36. If the monopolist is able to practice group price discrimination, the deadweight loss is 30.

A) 30.

31.

B) 32.

32.

C) 34.

33.

D) 36.

24) A firm that practices multimarket price discrimination will set the lower price in the market that has the most elastic demand.

25) While price discrimination is possible between two markets, it is not possible in more than two.

26) Bob is the only carpet installer in a small isolated town. The above figure shows the demand curves of two distinct groups of customers-residential and business. If the marginal cost of installing carpet is a constant $1 per sq yard, what price does Bob charge each segment?

27) A specialized rice grower sells rice in two markets, the United States and Japan, and the marginal cost the same in both markets. The price elasticity of demand in the United States is -2.0, and the price elasticity of demand in Japan is -1.5. If the grower practices multimarket price discrimination, which country's consumers will pay a higher price and by how much?

28) A weapons producer sells guns to two countries that are at war with each other. The guns can be produced at a constant marginal cost of $10. The demand for guns from the two countries can be represented as:

QA = 100 - 2p QB = 80 - 4p

Why is the weapons producer able to price discriminate? What price will it charge to each country? Answer: The firm can price discriminate because there are two identifiable segments with different elasticities. Since the two countries are at war, resale is doubtful. To solve, find marginal revenue for each and set equal to marginal cost of $10.

MRB = 20 - (1/2)QB = 10 or QB = 20 and pB = 15. MRA = 50 - QA = 10 or QA = 40 and pA = 30. Section: Multimarket Price Discrimination

29) If a firm sells to two distinct identifiable markets and resale is impossible, why is price discrimination more profitable than setting a single price?

30) A hotel with market power charges customers who check in before 5:00 pm more than those who check in after 5:00 pm. Those who check in early are much more likely to use the hotel's pool. Explain why this price difference may not be price discrimination.

12.4 Nonlinear Price Discrimination

1) If a monopoly charges higher prices to consumers who buy smaller quantities than to consumers who buy larger quantities, then 1.

A) consumer surplus is larger than under single-price monopoly.

2.

B) social welfare is larger than under perfect competition.

3.

C) the monopoly's profits are larger than under single-price monopoly.

4.

D) the monopoly's profits are larger than under perfect price discrimination.

2) Nonlinear price discrimination is 1.

A) perfect price discrimination.

2.

B) quantity price discrimination.

3.

C) group price discrimination.

4.

D) two-part pricing.

3) If a monopoly charges higher prices to consumers who buy smaller quantities than to consumers who buy larger quantities, then 1.

A) consumers who buy larger quantities have a higher price elasticity of demand.

2.

B) consumers who buy larger quantities have a lower price elasticity of demand.

3. 4.

C) consumers who buy smaller quantities have a lower price elasticity of demand. D) Both A and C.

4) Declining-block quantity discrimination makes sense if 1.

A) buyers of smaller quantities are more price sensitive than buyers of larger quantities.

2.

B) buyers of smaller quantities are less price sensitive than buyers of larger quantities.

3. 4.

C) demand for the good is perfectly elastic. D) the lower price for larger quantities encourages all consumers to purchase the larger quantity.

5) The more block prices a monopoly can set instead of setting a single price, 1.

A) the smaller the deadweight loss.

2.

B) the more producer surplus.

3.

C) the larger the total welfare.

4.

D) All of the above.

6) If firms that practice second degree price discrimination use more block prices, 1.

A) both consumer surplus and welfare will decrease.

2.

B) both consumer surplus and welfare will increase.

3.

C) consumer surplus will decrease but welfare will increase.

4.

D) consumer surplus will increase but welfare will decrease.

7) Purchasing a season pass to the local symphony 1.

A) is an example of first degree price discrimination.

2.

B) is an example of second degree price discrimination.

3.

C) is an example of third degree price discrimination.

4.

D) All of the above.

8) If you purchase one pound of apples the price is $1.50 per pound. If you buy a five pound bag of apples, the cost is $5.00. This is most likely an example of 1.

A) quantity discounts.

2.

B) lower marginal cost.

3.

C) lower marginal benefit.

4.

D) price gouging.

9) What is one reason consumers might demand a discount for quantity purchases? 1.

A) higher storage costs

2.

B) lower marginal cost

3.

C) lower marginal benefit

4.

D) price gouging

10) What is one reason suppliers might offer a discount for quantity purchases? 1.

A) reduced storage costs

2.

B) lower marginal cost

3.

C) lower marginal benefit

4.

D) price gouging

11) A mail-order clothing company offers a discount if customers purchase two shirts instead of only one. This is necessarily an example of quantity discrimination.

12) When a firm uses a form of quantity discrimination that charges large purchasers less it is the high quantity purchasers that generate most profit.

13) An electric utility is going to use a block-pricing schedule. They plan to charge P1 for the first Q1 units and P2 for the subsequent units. The units sold at P2 are the total units sold, Q2, minus the total units sold at P1. The inverse demand curve is P = $100 Q, and the marginal and average cost is $40. Use calculus to solve for P1, P2, Q1, Q2.

12.5 Two-Part Pricing

1) Two-part pricing offers a mechanism whereby the firm can 1.

A) charge two different prices to distinct groups of customers.

2.

B) collect two times as much from consumers as a single-price monopoly can.

3.

C) capture some or all of the consumer surplus.

4.

D) reduce some of its fixed costs.

2) At the current price of a good, Al's consumer surplus equals 8, and Ben's consumer surplus equals 15. By using two-part pricing, a monopolist could increase his profit by 8.

A) 8.

9.

B) 16.

10.

C) 15.

11.

D) 30.

3) At the current price of a good, Al's consumer surplus equals 15 and Ben's consumer surplus equals 15. By using two-part pricing, a monopolist could increase his profit by 8.

A) 8.

9.

B) 16.

10.

C) 15.

11.

D) 30.

4) Many theme parks charge an entrance fee and a per-ride fee equal to zero. This is an example of 1.

A) bundling.

2.

B) two-part pricing.

3.

C) group price discrimination.

4.

D) perfect price discrimination.

5) Suppose all individuals are identical, and their monthly demand for Internet access from a certain leading provider can be represented as p = 5 - (1/2)q where p is price in $ per hour and q is hours per month. The firm faces a constant marginal cost of $1. Potential consumer surplus equals 4.

A) $4.

5.

B) $8.

6.

C) $16.

7.

D) $32.

6) Suppose all individuals are identical, and their monthly demand for Internet access from a certain leading provider can be represented as p = 5 - (1/2)q where p is price in $ per hour and q is hours per month. The firm faces a constant marginal cost of $1. Profitmaximizing two-part pricing yields total revenue of 24.

A) $24.

25.

B) $40.

26.

C) $16.

27.

D) $32.

7) Suppose all individuals are identical, and their monthly demand for Internet access from a certain leading provider can be represented as p = 5 - (1/2)q where p is price in $ per hour and q is hours per month. The firm faces a constant marginal cost of $1. Profit maximizing two-part pricing results in the firm selling 4.

A) 4.5 hours.

5.

B) 10 hours.

6.

C) 5 hours.

7.

D) 8 hours.

8) Suppose all individuals are identical, and their monthly demand for Internet access from a certain leading provider can be represented as p = 5 - (1/2)q where p is price in $ per hour and q is hours per month. The firm faces a constant marginal cost of $1. If the firm will charge a monthly access fee plus a per hour rate, the monthly access fee will equal 1.

A) $1.

2.

B) $5.

3.

C) $8.

4.

D) $16.

9) Consider a car dealership advertises a three-year lease at $250 per month. When you arrive to apply, you discover that the lease requires a downpayment of $3600 dollars. You will undertake the lease if 1.

A) you value the lease at least $350 per month.

2.

B) you value the lease at least $250 per month, the $3600 is a sunk cost.

3.

C) you value the lease less than $350 per month.

4.

D) you value buying a new car at $400 per month.

10) Many college football teams require a "donation" in order to purchase season tickets. This is 1.

A) price gouging.

2.

B) a tie-in sale.

3.

C) two part pricing.

4.

D) anti-competitive behavior.

11) Stores such as Costco and Sam's Club require an annual membership before you can shop there. This is a form of 1.

A) two part pricing.

2.

B) price gouging.

3.

C) a tie-in sale.

4.

D) anti-competitive behavior.

12) Recent research suggests that if Apple switched from uniform pricing of its iTunes to two-part pricing, 1.

A) total welfare would increase, but consumer surplus would decrease.

2.

B) total welfare and consumer surplus would increase.

3.

C) total welfare would decrease, but consumers surplus would increase.

4.

D) total welfare and consumer surplus would decrease.

13) Two-part tariffs allow the monopoly firm to capture all of the potential consumer surplus generated by the sale of its product.

14) For a theme park a two-tier tariff can include a positive admission price and a zero per-ride fee.

15) Each identical consumer has the following demand for golf, q = 100 - p, where q is the number of rounds of golf played per year and p is the price per round. The only golf course in an isolated town incurs a marginal cost of $10 per round of golf. It wishes to charge a membership fee and a fee per round of golf. What price will it set for each fee?

16) Explain why a monopoly that knows the demand curve of identical consumers can set a two-part tariff with the lump sum tariff equal to the total amount of potential consumer surplus.

12.6 Tie-In Sales

1) The sales of shoes that include shoelaces is a tie-in sale that most likely 1.

A) greatly increases the shoe producer's profit.

2.

B) increases transactions costs.

3.

C) increases efficiency.

4.

D) None of the above.

2) If Ben values good X more than good Y, and Catherine values good Y more than good X, a firm can increase its profits by 1.

A) charging the same price for both goods.

2.

B) bundling the goods.

3.

C) selling the goods in a competitive market.

4.

D) charging one price per good.

3) Tie-in sales are most advantageous to the seller when 1.

A) the demands for the two goods are negatively correlated.

2.

B) the demands for the two goods are positively correlated.

3.

C) the demands for the two goods are unrelated.

4.

D) there are economies of scope.

4) A photograph processing machine company requiring customers who buy a processing machine to purchase chemicals and photographic paper from it is an example of 1.

A) bundling.

2.

B) a requirement tie-in sale.

3.

C) nonlinear pricing.

4.

D) two-part pricing.

5) A firm that holds a monopoly in both goods stands to gain from a tie-in sale. 1.

A) True.

2.

B) False.

3.

C) True, which is why anti-trust laws exist.

4.

D) False, there are no dual monopolies.

6) Why don't we see firms tie-in the sales of fish filets with the sales of pencils? 1.

A) because the demands for these two goods are positively correlated

2.

B) because the demands for these two goods are negatively correlated

3.

C) because the demands for these two goods are independent

4.

D) It violates the Clean Food Act of 1908.

7) What is one way firms can enforce tie-in sales? 1.

A) One of the goods has no close substitutes.

2.

B) contractual arrangements

3.

C) information asymmetry

4.

D) Any of the above.

8) What is the primary difference between bundling and tie-ins? 1.

A) Bundling is typically a one-off purchase.

2.

B) Contractual arrangements.

3.

C) Tie-ins are one-off purchases.

4.

D) Bundling is illegal and tie-ins are legal.

9) Assume a company can offer customers cable television and Internet service at essentially zero marginal and average cost. The following table shows each customer's marginal willingness to pay for television, Internet services, and for a bundle containing both. If television and Internet services are sold separately, the profit maximizing prices are

Alex Rebecca

Television $100 $80

Internet $60 $100

Bundle $160 $180

60.

A) television $100 and Internet services $60.

61.

B) television $80 and Internet services $100.

62.

C) television $80 and Internet services $60.

63.

D) television $100 and Internet services $100.

10) Assume a company can offer customers cable television and internet service at essentially zero marginal and average cost. The following table shows each customer's marginal willingness to pay for television, internet services, and for a bundle containing both. If television and internet services are sold in a pure bundle, the profit maximizing price is

Alex Rebecca

Television $100 $80

Internet $60 $100

Bundle $160 $180

140.

A) $140.

141.

B) $160.

142.

C) $180.

143.

D) None of the above since pure bundling is not profit maximizing in this case.

11) Assume a company can offer customers cable television and internet service at essentially zero marginal and average cost. The following table shows each customer's marginal willingness to pay for television, internet services, and for a bundle containing both. Which strategy yields the maximum profit, and what maximum profit is obtained?

Alex Rebecca

Television $100 $80

Internet $60 $100

Bundle $160 $180

340.

A) Sell separately and make a profit of $340.

341.

B) Sell separately and make a profit of $280.

342.

C) Bundle and make a profit of $320.

343.

D) Bundle and make a profit of $360.

12) If both Ben and Catherine value good X more than good Y, a firm can increase profits by bundling the two products.

12.7 Advertising

1) The monopoly can shift the demand for its product rightward by 1.

A) accommodating entry.

2.

B) advertising new uses for its product.

3.

C) moving along the learning curve.

4.

D) All of the above.

2) If a monopoly can advertise and as a result the demand curve will become more inelastic, the monopoly 1. 2.

A) should always engage in the advertising. B) should engage in the advertising until the demand curve becomes more elastic.

3.

C) will earn higher gross profit if it advertises.

4.

D) None of the above.

3) A firm will increase its spending on advertising until 1.

A) it has monopolized the market.

2.

B) it has deterred all future entry.

3.

C) the marginal benefit of advertising is zero.

4.

D) the marginal benefit of advertising equals the marginal cost of advertising.

4) Advertising for Milk and Beef is usually done by 1.

A) interest groups that represent the whole industry.

2.

B) a single firm in the market.

3.

C) a small set of firms in a market.

4.

D) It is not done because it doesn't pay to advertise homogeneous products.

5) If identical firms sell an undifferentiated product, advertising is likely to be 1.

A) used to attack the rivals' products.

2.

B) collectively undertaken by the industry group.

3.

C) strategically aimed at deterring entry.

4.

D) focused on secret ingredients.

6) A firm's advertising can help rivals 1.

A) if it focuses on a general problem that the product addresses.

2.

B) if it focuses on a secret ingredient that only this firm possesses.

3.

C) if rivals do not advertise.

4.

D) if rivals advertise.

7) The effect on total market demand from the advertising of a specific brand will 1.

A) always be positive.

2.

B) always be negative.

3.

C) often be positive.

4.

D) often be negative.

8) A firm advertising using an expensive, famous spokesperson is often 1.

A) aimed to raise rivals' costs.

2.

B) used to increase the total market demand.

3.

C) used to steal customers from rivals.

4.

D) used to focus on general problems the product addresses.

9) Advertising that has the fine print "professional driver on a closed course," 1.

A) is likely engaged in persuasive advertising.

2.

B) is likely engaged in informative advertising.

3.

C) is likely engaged in strategic advertising.

4.

D) is likely engaged in false advertising.

10) Since a monopoly faces no competitors, it need not advertise.

11) Cigarette companies favored a ban on cigarette advertising.

12) A monopoly faces the following demand function: Q = 100 - p + sqrt(A), where A equals the dollar amount spent on advertising. If the cost function is A + 10 + 2Q, what are the profit-maximizing levels of price, output, and advertising? Compare this outcome to the case where the firm does not advertise at all.

13) Under what conditions would firms be likely to support an industry-wide advertising ban?

14) Explain why advertising that results in spuriously differentiated products may result in a deadweight loss to total welfare.

Microeconomics, 7e (Perloff) Chapter 13 Oligopoly and Monopolistic Competition

13.1 Market Structures

1) Perfect competition and monopolistic competition are similar in that both market structures include

1.

A) price-taking behavior by firms.

2.

B) a homogeneous product.

3.

C) no barriers to entry.

4.

D) very few firms.

2) Perfect competition and monopolistic competition are similar in that firms in both types of market structure will 1.

A) act as price takers.

2.

B) produce a level of output where price equals marginal cost.

3.

C) earn zero profit in the long run.

4.

D) act as price setters.

3) Which of the following market structures is (are) capable of earning positive economic profits in the long run? 1.

A) monopoly

2.

B) oligopoly

3.

C) monopolistic competition

4.

D) Both A and B.

4) A competitive market structure differs from the monopoly, oligopoly, and monopolistic competition structures in the 1.

A) producers' ability to set price.

2.

B) profit maximization condition.

3.

C) amount of long run profit.

4.

D) entry conditions.

5) Oligopoly differs from monopolistic competition in that an oligopoly includes 1.

A) product differentiation.

2.

B) barriers to entry.

3.

C) no barriers to entry.

4.

D) downward-sloping demand curves facing the firm.

6) Monopolistic competition and monopoly have all of the following in common EXCEPT 1.

A) P > MC.

2.

B) firms are price setters.

3.

C) barriers to entry.

4.

D) MR = MC.

7) Monopolistic competition and perfect competition differ because 1.

A) only monopolistically competitive firms will set MR = MC.

2.

B) only perfectly competitive firms will set MR = MC.

3.

C) only monopolistic competition allows for entry of other firms in the long run.

4.

D) only competitive firms take the price as given.

8) Which of the following market structures have market power? 1.

A) oligopoly

2.

B) monopolistic competition

3.

C) perfect competition

4.

D) Both A and B.

9) Regardless of market structure, all firms 1.

A) consider the actions of rivals.

2.

B) maximize profit by setting marginal revenue equal to marginal cost.

3.

C) produce a differentiated product.

4.

D) have the ability to set price.

10) What is the aspect of imperfect competition that is most distinct from perfect competition? 1.

A) free entry/exit

2.

B) perfect information

3.

C) differentiated products

4.

D) zero profits

11) In comparing monopolistic competition to perfect competition, one can conclude that the lack of free entry is the key to having the ability to set price.

12) Firms in all types of market structures pay attention to current rival firms' behavior.

13) Explain why a monopoly or a perfectly competitive firm does not consider a rival firm's behavior, but an oligopoly and a monopolistically competitive firm do.

14) Explain how long-run economic profits are linked to entry in monopolistic competition and perfect competition.

13.2 Cartels

1) A cartel is a group of firms that attempts to 1.

A) maximize joint revenue.

2.

B) maximize joint profit.

3.

C) behave independently.

4.

D) increase consumer surplus.

2) The Organization of Petroleum Exporting Countries (OPEC) is an example of a(n) 1.

A) oil monopoly.

2.

B) cartel.

3.

C) competitive arrangement.

4.

D) monopolistically competitive arrangement.

3) If a cartel is unable to monitor its members and punish those firms that violate the agreement, then 1.

A) the member firms will each act as price setters.

2.

B) the cartel will prosper in the long run.

3.

C) the market will become a monopoly.

4.

D) the cartel will fail.

4) In a sense, a cartel is self-destructive because 1.

A) it reduces consumer surplus.

2.

B) it sets price above marginal cost.

3.

C) each cartel member has the incentive to cheat on the cartel.

4.

D) each cartel member earns economic profit.

5) Which of the following conditions can help prolong the life of a cartel? 1.

A) There are only a few firms in the market and they all belong to the cartel.

2.

B) There are many firms in the market that are not members of the cartel.

3.

C) It is difficult to know what price any cartel member is actually charging.

4.

D) The cartel has no ability to punish members who cheat on the cartel.

6) Mergers may result in 1.

A) anticompetitive behavior.

2.

B) more efficient production.

3.

C) fewer firms in a market.

4.

D) All of the above.

7) Mergers are closely scrutinized by the government because 1.

A) they might allow the firms involved to dominate the market and act as a legalized cartel (monopoly).

2.

B) they always result in a more efficient market.

3.

C) the always result in lower joint profits of the firms involved.

4.

D) all mergers are undesirable.

8) The evidence on mergers occurring within the hospital industry suggests that 1. 2. 3. 4.

A) efficiency gains that resulted have been passed on to customers through lower prices. B) the mergers have tended to not have any significant effect on hospital prices. C) prices have risen slightly as some efficiency gains are passed on to customers after mergers. D) prices have risen after mergers.

9) A typical firm in a cartel will hold which of the following attitudes? 1.

A) If everyone cheats, I'm better off, and so is everyone in the cartel.

2.

B) If I alone cheat, I'm better off; if everyone cheats, I'm worse off.

3.

C) I can never do better for myself than following agreed-upon cartel rules.

4.

D) If I suspect others are planning to cheat, I'll do best for myself by deciding not to cheat.

10) Television stations have seemingly synchronized their commercial breaks. This is likely an example of 1.

A) tacit collusion.

2.

B) explicit collusion.

3.

C) mixed strategies.

4.

D) pure strategies.

11) There are only two firms in an industry with demand curves q1 = 30 - P and q2 = 30 - P. Both have no fixed costs and each has a marginal cost of 10 per unit produced. If they behave as profit maximizing price takers, each produces 10 units and sells them at a price of 10 so that each firm makes zero economic profits. If they form a cartel, their inverse demand curve is 1.

A) Q = 30 - P.

2.

B) Q = 60 - 2P.

3.

C) P = 60 - 2Q.

4.

D) P = 30 - Q/2.

12) There are only two firms in an industry with demand curves q1 = 30 - P and q2 = 30 - P. Both have no fixed costs and each has a marginal cost of 10 per unit produced. If they behave as profit maximizing price takers, each produces 20 units and sells them at a price of 10 so that each firm makes zero economic profits. If they formed a cartel, the profit maximizing price is 10.

A) 10.

11.

B) 15.

12.

C) 20.

13.

D) 25.

13) There are only two firms in an industry with demand curves q1 = 30 - P and q2 = 30 - P. Both have no fixed costs and each has a marginal cost of 10 per unit produced. If they behave as profit maximizing price takers, each produces 20 units and sells them at a price of 10 so that each firm makes zero economic profits. If they formed a cartel and

split the production of the output evenly, the profit maximizing quantity produced by each firm is 5.

A) 5.

6.

B) 10.

7.

C) 15.

8.

D) 20.

14) There are only two firms in an industry with demand curves q1 = 30 - P and q2 = 30 - P. Both have no fixed costs and each has a marginal cost of 10 per unit produced. If they behave as profit maximizing price takers, each produces 20 units and sells them at a price of 10 so that each firm makes zero economic profits. If they formed a cartel and split the production of the output evenly, the economic profit of each firm would be 1.

A) 0.

2.

B) 50.

3.

C) 100.

4.

D) 200.

15) There are only two firms in an industry with demand curves q1 = 30 - P and q2 = 30 - P. Both have no fixed costs and each has a marginal cost of 10 per unit produced. If they behave as profit maximizing price takers, each produces 20 units and sells them at a price of 10 so that each firm makes zero economic profits. Suppose the two firms form a cartel. While firm 1 produces one-half of the profit maximizing cartel output, firm 2 cheats and produces 5 units more. What would happen to the two firms' economic profits? 10.

A) Firm 1's profits remain the same, but firm 2's profits increase by 10.5

11.

B) Firm 1's profits decrease by 5.5, but firm 2's profits increase by 11.

12.

C) Firm 1's profits decrease by 25, but firm 2's profits increase by 12.5.

13.

D) Firm 1's profits decrease by 12.5, but firm 2's profits increase by 25.

16) Cartels are inherently self-destructive because each member firm has the incentive to cheat on the cartel agreement.

17) Collusion is more successful in a game that will continue forever or in a game with an uncertain ending time than in a game with a known ending time.

18) Suppose that market demand can be represented as p = 100 - 2Q. There are 10 identical firms producing an undifferentiated product, each with the total cost function TC = 50 + q2. Compare the competitive outcome with the cartel outcome. What is the individual firm's incentive to cheat on the cartel?

19) Explain why gasoline stations across the street from each other with large signs displaying their prices may "legally" jointly set monopoly prices.

13.3 Cournot Oligopoly

1) Two firms sell 100% orange juice in 10 ounce bottles. The juice is only good for one week. The two firms have contracts for all the oranges produced in a large geographic area. Each firm decides how many bottles of juice to produce at the same time. This market is best described with a 1.

A) Bertrand model.

2.

B) Stackelberg model.

3.

C) monopolistic competition model.

4.

D) Cournot model.

2) The Cournot Model of Oligopoly assumes that 1.

A) firms decide what quantity to produce.

2.

B) firms make their decisions simultaneously.

3.

C) firms do not cooperate.

4.

D) All of the above.

3) Compared to a cartel, firms in a Cournot Oligopoly 1.

A) make more joint profit.

2.

B) sell less output.

3.

C) make less joint profit.

4.

D) act independently.

4) The above figure shows the reaction functions for two pizza shops in a small isolated town. The Cournot equilibrium is at point 1.

A) a.

2.

B) b.

3.

C) c.

4.

D) d.

5) The above figure shows the reaction functions for two pizza shops in a small isolated town. Collusion would result in 1.

A) each firm producing 25 pizzas.

2.

B) each firm producing 40 pizzas.

3.

C) the firms splitting the production of 100 making 50 pizzas each.

4.

D) firm A monopolizing the market by selling 50 pizzas.

6) The above figure shows the reaction functions for two pizza shops in a small isolated town. The perfect competitive outcome is that 1.

A) each firm produces 40 pizzas.

2.

B) each firm produces 50 pizzas.

3.

C) the firms split the production of 200 pizzas.

4.

D) each firm produces 200 pizzas.

7) The above figure shows the reaction functions for two pizza shops in a small isolated town. Firm B producing 100 pizzas and firm A producing 50 pizzas is not a Cournot equilibrium because 1.

A) Cournot duopolists agree to share the market equally.

2.

B) firm B is not on its best-response function.

3.

C) firm A is not on its best-response function.

4.

D) neither firm is on its best-response function.

8) Suppose two Cournot duopolist firms operate at zero marginal cost. The market demand is p = a - bQ. Firm 1's best-response function is 1.

A) q1= (a - bq2)/2b.

2.

B) q1= (a - 2bq2)/2b.

3.

C) q1= a/b.

4.

D) q1= a/2b.

9) Suppose two Cournot duopolist firms operate at zero marginal cost. The market demand is p = a - bQ. Each firm will produce 1.

A) a/b.

2.

B) a/2b.

3.

C) a/3b.

4.

D) a/4b.

10) Suppose Cournot duopolist firms operate with each having a cost of 30qi (i = 1,2) so that each firm's marginal cost is 30. The inverse market demand curve is P = 120 - Q where Q = q1 + q2. At the Nash-Cournot equilibrium, the total quantity, Q, is 30.

A) 30.

31.

B) 45.

32.

C) 60.

33.

D) 90.

11) Suppose Cournot duopolist firms operate with each having a cost of 30qi (i = 1,2) so that each firm's marginal cost is 30. The inverse market demand curve is P = 120 - Q where Q = q1 + q2. At the Nash-Cournot equilibrium, the market price, P, is 30.

A) 30.

31.

B) 45.

32.

C) 60.

33.

D) 90.

12) Suppose Cournot duopolist firms operate with each having a cost of 30qi (i = 1,2) so that each firm's marginal cost is 30. The inverse market demand curve is P = 120 - Q where Q = q1 + q2. Suppose there were no barriers to entry and firms continued to enter so long as there were positive economic profits. At the Nash-Cournot equilibrium, the total output, Q, is 30.

A) 30.

31.

B) 45.

32.

C) 60.

33.

D) 90.

13) Suppose Cournot duopolist firms operate with each having a cost of 30qi (i = 1,2) so that each firm's marginal cost is 30. The inverse market demand curve is P = 120 - Q where Q = q1 + q2. Suppose there were no barriers to entry and firms continued to enter so long as there were positive economic profits. At the Nash-Cournot equilibrium, the price, P, is 30.

A) 30.

31.

B) 45.

32.

C) 60.

33.

D) 90.

14) In the Cournot model, a firm maximizes profit by selecting 1.

A) its output, assuming that other firms keep their output constant.

2.

B) its price, assuming that other firms keep their price constant.

3.

C) its output, assuming that other firms will retaliate.

4.

D) its price, assuming that other firms will retaliate.

15) In the Cournot model, the output that a firm chooses to produce increases as 1.

A) the total output of other firms increases.

2.

B) the number of firms in the market increases.

3.

C) the number of firms in the market decreases.

4.

D) its marginal cost increases.

16) Suppose Cournot duopolists firms face the same market demand curve, but have differing costs. At the Nash-Cournot equilibrium, the firm with the lower cost will 1.

A) have a lower price for its product than its competitor.

2.

B) produce a smaller output than its competitor.

3.

C) have a higher price for its product than its competitor.

4.

D) produce a larger output than its competitor.

17) In the Cournot model, if the products are differentiated, 1.

A) this reduces the pressure of one firm's decisions on the other.

2.

B) this increases the pressure of one firm's decisions on the other.

3.

C) there is no difference between this model and one with homogeneous goods.

4.

D) marginal costs are necessarily different.

18) As compared to the basic Nash-Cournot equilibrium for duopolists where the firms face the same market demand curve and have identical costs, in the situation where the firms produce products which are viewed by consumers as not being identical, 1.

A) there will generally be different prices charged by the two firms.

2.

B) there will generally be different quantities produced by the two firms.

3.

C) one or both of the firms may practice spurious differentiation.

4.

D) All of the above.

19) Suppose a market with a Cournot structure has five firms and a market price elasticity of demand equal to -2. What is a Cournot firm's Lerner Index? 1.

A) .1

2.

B) .2

3.

C) .5

4.

D) 1

20) The Cournot model assumes that firm A maximizes its profit, holding firm B's output constant.

21) Suppose the demand for pizza in a small isolated town is p = 10 - Q. There are only two firms, A and B, and each has a cost function TC = 2 + q. Determine the Cournot equilibrium.

22) Explain why the intersection of the best-response functions is the Cournot equilibrium.

23) Assuming Cournot behavior, what happens to the market output, the price of the output, and each firm's output as the number of firms in a market increases?

24) Draw a graph that shows the effect on the equilibrium quantities if the government subsidizes one firm in a Cournot duopoly with a per-unit subsidy. Assume that the bestresponse functions are linear. Explain the new equilibrium quantities. Answer:

25) If two firms playing Cournot are identical with decreasing average costs, how should the firms divide production to maximize joint profits?

13.4 Stackelberg Oligopoly

1) The Stackelberg model is more appropriate than the Cournot model in situations where 1.

A) there are more than two firms.

2.

B) all firms enter the market simultaneously.

3.

C) one firm makes its output decision before the other.

4.

D) firms will be likely to collude.

2) The outcome of the Stackelberg model is 1.

A) a Nash equilibrium.

2.

B) the same as the Cournot outcome.

3.

C) that the follower earns zero profit.

4.

D) that the follower cannot be on its best-response curve.

3) Which of the following is a necessary condition for government subsidies to influence a firm to choose an output level as if it were a Stackelberg leader? 1.

A) The subsidy must be announced before the firms choose output levels.

2.

B) The subsidy must be equal to the firm's marginal cost.

3.

C) The subsidy must be equal to the firm's rival's marginal cost.

4.

D) The firm does not have any fixed costs.

4) The above figure shows the reaction functions for two pizza shops in a small isolated down. The Stackelberg leader will produce 1.

A) 25 pizzas.

2.

B) 50 pizzas.

3.

C) 66.7 pizzas.

4.

D) 100 pizzas.

5) Suppose two duopolists operate at zero marginal cost. The market demand is p = a bQ. If firm 1 is the Stackelberg leader, what level of output will it choose? 1.

A) q1= (a - bq2)/2b

2.

B) q1= (a - 2bq2)/2b

3.

C) q1= a/b

4.

D) q1= a/2b

6) Suppose duopolists face the market inverse demand curve P = 100 - Q, Q = q1 + q2, and both firms have a constant marginal cost of 10. If firm 1 is a Stackelberg leader and firm 2's best response function is q2 = (100 – q1)/2, at the Nash-Stackelberg equilibrium firm 1's output is 30.

A) 30.

31.

B) 40.

32.

C) 60.

33.

D) 70.

7) Suppose duopolists face the market inverse demand curve P = 100 - Q, Q = q1 + q2, and both firms have a constant marginal cost of 10. If firm 1 is a Stackelberg leader and firm 2's best response function is q2 = (100 – q1)/2, at the Nash-Stackelberg equilibrium the prices the two firms charge are 40.

A) P1= 40, P2= 40.

41.

B) P1= 30, P2= 30.

42.

C) P1= 30, P2= 40.

43.

D) P1= 40, P2= 30.

8) In which of the following market structures with 2 identical firms do both firms produce more than the Cournot outcome? 1.

A) Stackelberg Oligopoly

2.

B) Cartel

3.

C) Perfect Competition

4.

D) None of the above.

9) What strategic advantage compared to a Cournot Oligopoly results in the Stackelberg outcome? 1.

A) the ability to move first

2.

B) the ability to set price

3.

C) the ability to set quantity

4.

D) the ability to make independent decisions by the Stackelberg leader

10) If only two identical firms operate in a market, consumers prefer 1.

A) a Cournot equilibrium.

2.

B) a Stackelberg equilibrium.

3.

C) a collusive equilibrium.

4.

D) any equilibrium since they all result in the same consumer surplus.

11) The profitability of the second mover in a Stackelberg model is 1.

A) guaranteed to be negative.

2.

B) smaller than that of the first mover.

3.

C) greater than that of the first mover.

4.

D) greater than the Cournot profits.

12) Strategic trade policy might work if 1.

A) trade decisions are made simultaneously by two countries.

2.

B) trade decisions are made sequentially by two countries.

3.

C) trade decisions entail response by a trading partner.

4.

D) trade decisions are made in a random fashion.

13) Strategic trade policy requires ALL of the following EXCEPT 1.

A) subsidies set before output levels.

2.

B) non-retaliation of the trading partner.

3.

C) credible actions on the part of the subsidizing government.

4.

D) incomplete information on the part of the government.

14) Which of the following models results in the highest level of output assuming a fixed number of firms with identical costs and a given demand curve?

1.

A) Cournot

2.

B) Stackelberg

3.

C) Monopoly

4.

D) Cartel

15) Which of the following market models results in the highest price assuming a fixed number of firms with identical costs and a given demand curve? 1.

A) Cournot

2.

B) Stackelberg

3.

C) Monopoly

4.

D) Price is the same in all three markets.

16) Which of the following market models results in the highest level of consumer surplus assuming a fixed number of firms with identical costs and a given demand curve? 1.

A) Cournot

2.

B) Stackelberg

3.

C) Monopoly

4.

D) Cartel

17) Which of the following models results in the greatest deadweight loss assuming a fixed number of firms with identical costs and a given demand curve? 1.

A) Cournot

2.

B) Stackelberg

3.

C) Monopoly

4.

D) Perfect competition

18) Which of the following models results in the greatest total profit, assuming a fixed number of firms with identical costs and a given demand curve?

1.

A) Cournot

2.

B) Stackelberg

3.

C) Monopoly

4.

D) Perfect competition

19) As the number of firms increases in a market, the differences between the Cournot, Stackelberg, and price-taking market structures 1.

A) decreases.

2.

B) increases.

3.

C) remains the same.

4.

D) cannot be determined.

20) Firms A and B are identical, produce identical products, and are the only firms in a market. Firm A's output is higher than Firm B's. This means that Firm B is the

1.

A) Cartel leader.

2.

B) Stackelberg leader.

3.

C) Stackelberg follower.

4.

D) Cournot leader.

21) If there are 2 identical firms in a market that choose the quantity they produce, total welfare is the highest when there is a cartel.

22) If two firms behave as Cournot duopolists, the level of social welfare is lower than if the same firms act as a cartel.

23) Suppose the demand for pizza in a small isolated town is p = 10 - Q. There are only two firms, A and B. Each has a cost function TC = 2 + Q. Determine the equilibrium quantities of each if firm A is the Stackelberg leader.

24) Suppose the demand for pizza in a small isolated town is p = 10 - Q. The only two firms, A and B, behave as Cournot duopolists. Each has a cost function TC = 2 + Q. If the government wants to subsidize firm A to raise its output to that of a Stackelberg leader, how large should the subsidy be?

25) What happens in a duopoly if both firms try to act as the Stackelberg leader?

26) Suppose the demand for pizza in a small isolated town is p = 10 - Q. There are only two firms, A and B, and each has a cost function TC = 2 + Q. Compare the firms' profits if they behave as Cournot duopolists with their profits if they form a cartel and share the market.

27) The above figure shows the reaction functions for two pizza shops in a small isolated town. Compare the Cournot, Stackelberg, cartel, and competitive equilibria in terms of total output and welfare.

13.5 Bertrand Oligopoly

1) The Bertrand model is a more plausible model of firm behavior than the Cournot model 1.

A) when firms set the quantity to be sold.

2.

B) when firms sell a differentiated product.

3. 4.

C) because firms that sell a non-differentiated product typically act as price takers. D) because the Bertrand model predicts that firms will price at marginal cost.

2) The Bertrand model of price setting assumes that a firm chooses its price 1.

A) independently of what price other firms charge.

2.

B) subject to what price rival firms are charging.

3.

C) so that joint profits are maximized.

4.

D) without considering the shape of the demand curve.

3) Assuming a homogeneous product, the Bertrand duopoly equilibrium price is 1.

A) the same as the Cournot equilibrium price.

2.

B) less than the Cournot equilibrium price.

3.

C) greater than the Cournot equilibrium price.

4.

D) equal to the monopoly price.

4) Assuming a homogeneous product, the Bertrand equilibrium price is 1.

A) independent of the number of firms.

2.

B) independent of the firm's marginal costs.

3.

C) equal to the Cournot equilibrium price.

4.

D) equal to the monopoly price.

5) One criticism of the Bertrand pricing model is that 1.

A) the model is implausible when there is product differentiation.

2.

B) when there is an oligopoly with no product differentiation, the model's prediction is inconsistent with reality.

3.

C) the model's predicted price is solely a function of demand conditions.

4.

D) the model's predicted price is dependent on the number of firms.

6) In a Bertrand model, graphically, the intersection of all firms' best-response curves determines 1.

A) the Nash equilibrium prices.

2.

B) the dominant strategy for each firm.

3.

C) the degree of product differentiation.

4.

D) the price of the market leader.

7) In a Bertrand model, market power is a function of 1.

A) marginal cost.

2.

B) the number of firms.

3.

C) price elasticity of supply.

4.

D) product differentiation.

8) In a Bertrand model with identical firms and a non-differentiated product, price will increase in response to 1.

A) an increase in the number of firms.

2.

B) a decrease in the number of firms.

3.

C) an increase in marginal cost.

4.

D) a decrease in marginal cost.

9) In a Bertrand model with differentiated products, 1.

A) firms can set price above marginal cost.

2.

B) firms set price at marginal cost.

3.

C) price is independent of marginal cost.

4.

D) firms set price independently of one another.

10) Product differentiation allows a firm to charge a higher price because the residual demand curve facing the firm 1.

A) is more elastic than the residual demand curve without product differentiation.

2.

B) is less elastic than the residual demand curve without product differentiation.

3.

C) is horizontal.

4.

D) shifts to the left.

11) Product differentiation 1.

A) may allow firms to price above a competitive level.

2.

B) generates value as consumers value more choices.

3.

C) depends on perceived differences between products.

4.

D) All of the above.

12) Two identical firms that share a market and produce a homogeneous good will find which of the following market outcomes LEAST desirable? 1.

A) Bertrand Oligopoly

2.

B) Cournot Oligopoly

3.

C) Cartel

4.

D) All are equally preferable.

13) Two identical firms that share a market and produce a homogenous good will find the Bertrand Oligopoly LEAST attractive because 1.

A) Cartels generate the highest joint profit.

2.

B) a Cournot Oligopoly will generate more profit than a Bertrand Oligopoly.

3.

C) they want to avoid a price war that leads to profit erosion and P = MC.

4.

D) All of the above.

14) Because firms selling a homogeneous product set price in response to the (perceived) pricing decision of other firms in the Bertrand Model of oligopoly in equilibrium price exceeds marginal cost.

15) Suppose the demand for Pepsi-Cola is qp = 54 – 2pp + 1pc. The demand for CocaCola is qc = 54 - 2pc + 1pp. Each firm faces a constant marginal cost of zero. Determine the Bertrand equilibrium prices. What happens to the Bertrand equilibrium prices and profits if increased differentiation causes the demand for Pepsi-Cola to become qp = 104 2pp + 1pc while the demand for Coca-Cola remains unchanged?

16) Suppose the demand for Pepsi-Cola is qp = 50 - 2pp + 1pc. The firm faces a constant marginal cost of m, and pc denotes the price of Coca-Cola. Assuming Bertrand behavior, derive Pepsi-Cola's best-response function and explain how the firm changes price in response to changes in its own marginal cost and changes in Coca-Cola's price.

17) In a Bertrand duopoly with product differentiation, explain how a change in one firm's marginal cost can have an effect on the price charged by the other firm.

13.6 Monopolistic Competition

1) Monopolistically competitive firms face downward sloping residual demand curves because these firms 1.

A) have relatively few rivals (compared to competition).

2.

B) sell differentiated products.

3.

C) A and/or B.

4.

D) None of the above.

2) In the long run, a monopolistically competitive firm 1.

A) earns zero economic profit.

2.

B) produces at minimum average cost.

3.

C) operates at full capacity.

4.

D) All of the above.

3) Monopolistically competitive firms 1.

A) have market power because they can set price above marginal cost.

2.

B) have no market power because they earn zero economic profit.

3.

C) have no market power because of free entry.

4.

D) have no market power because price equals marginal cost.

4) In the short run, a monopolistic competitor

1.

A) produces at minimum efficient scale.

2.

B) produces where P = AC.

3.

C) sets P = MC.

4.

D) sets MR = MC.

5) In the long run, a monopolistic competitor 1.

A) sets MR = MC.

2.

B) produces where P = AC.

3.

C) sets P > MC.

4.

D) All of the above.

6) Minimum efficient scale refers to the lowest level of output at which

1.

A) the firm can earn a profit.

2.

B) average cost is minimized.

3.

C) the firm will operate.

4.

D) the average cost curve is downward sloping.

7) The number of firms in a monopolistically competitive market will be smaller if 1.

A) the market demand curve shifts rightward.

2.

B) the minimum efficient scale is lower.

3.

C) fixed costs are smaller.

4.

D) fixed costs are larger.

8) Suppose a monopolistically competitive industry evolved into a perfectly competitive industry. Which of the following statements is correct? 1. 2.

A) The industry would produce more output and charge a lower price after the change. B) The industry would produce at decreasing returns to scale.

3.

C) Elasticity of demand for the firm's product would remain the same after this change occurred.

4.

D) This industry would produce the same level of output at lower prices in the long run than before the change.

9) Brand loyalty in monopolistically competitive markets manifests itself as 1.

A) upward sloping marginal cost curves.

2.

B) downward sloping demand curves.

3.

C) downward sloping marginal cost curves.

4.

D) upward sloping demand curves.

10) A firm with a flat demand curve 1.

A) has no brand loyalty.

2.

B) has weak brand loyalty.

3.

C) has strong brand loyalty.

4.

D) isn't really worried about brand loyalty; flat demand curves guarantee zero profit.

11) In monopolistically competitive markets 1.

A) price is greater than it would be in perfect competition.

2.

B) price is less than it would be in perfect monopoly.

3.

C) quantity is greater than it would be in perfect monopoly.

4.

D) All of the above.

12) Product differentiation 1. 2.

A) is possibly welfare enhancing if new products match consumer preferences better. B) is welfare reducing even if new products match consumer preferences better.

3.

C) is welfare enhancing even if new products do not match consumer preferences better.

4.

D) is welfare reducing even if new products do not match consumer preferences better.

13) Product differentiation 1.

A) can be actual or perceived.

2.

B) is only legal if it is perceived.

3.

C) is only important if it is perceived.

4.

D) is only important if it is actual.

14) Firms seek to differentiate their product 1.

A) to avoid state and federal regulation.

2.

B) to create an illusion of value.

3.

C) to strengthen their demand and to make it more inelastic.

4.

D) to strengthen their demand and to make it more elastic.

15) The ability to set a price greater than marginal cost guarantees an economic profit for the monopolistic competitor (assuming P > AC).

16) Is it true that in the long run, a monopolistically competitive firm has market power but earns no profit? Explain.

Microeconomics, 7e (Perloff) Chapter 14 Game Theory

14.1 Static Games

1) Which of the following is a simultaneous decision game? 1.

A) Tic-tac-toe

2.

B) Chess

3.

C) Poker

4.

D) Rock-paper-scissors

2) A game in economics is defined as 1.

A) something that is shown on ESPN.

2.

B) competition in which strategic decision making is integral.

3.

C) competition in general.

4.

D) an actual strategy chosen by one or more economic agents.

3) In a two-player simultaneous game, if player A has a dominant strategy and player B does not, player B will 1.

A) employ a mixed strategy.

2.

B) choose his best strategy assuming that player A plays her dominant strategy.

3.

C) not achieve a Nash equilibrium.

4.

D) assume that player A does not choose her dominant strategy.

4) In a two-player simultaneous game where neither player has a dominant strategy, 1.

A) there is never a Nash equilibrium.

2.

B) there is only one Nash equilibrium.

3.

C) the actual outcome is unpredictable.

4.

D) the actual outcome will not be a Nash equilibrium.

5) After analyzing his opponent, a tennis player decides to serve 10% of his serves to the left, 50% of his serves to the right, and 40% of his serves at the body of his opponent. This illustrates a 1.

A) deterministic strategy.

2.

B) dominant strategy.

3.

C) mixed strategy.

4.

D) non-game theoretic problem.

6) A mixed strategy may 1.

A) be part of a Nash equilibrium.

2.

B) be a set of probabilities of selecting each possible action.

3.

C) lead identical firms to choose different actions.

4.

D) All of the above.

7) When neither player has a dominant strategy, 1.

A) game theory will not provide information.

2.

B) no Nash-Equilibrium exists.

3.

C) at least one Nash-Equilibrium exists.

4.

D) the game cannot be analyzed.

8) The above figure shows the payoff to two airlines, A and B, of serving a particular route. If the two airlines must decide simultaneously, which one of the following statements is true? 1.

A) Firm A does not have a dominant strategy.

2.

B) Firm B does not have a dominant strategy.

3.

C) Neither firm entering is a Nash equilibrium.

4.

D) The outcome of the game is unpredictable.

9) The above figure shows the payoff to two airlines, A and B, of serving a particular route. If the two airlines must decide simultaneously, which one of the following statements is true? 1.

A) Firm A has a dominant strategy.

2.

B) Firm B has a dominant strategy.

3.

C) Neither firm entering is a Nash equilibrium.

4.

D) The outcome of the game is unpredictable.

10) The above figure shows the payoff to two airlines, A and B, of serving a particular route. If the two airlines must decide simultaneously, which one of the following statements is true? 1.

A) Only firm A will enter the market.

2.

B) Only firm B will enter the market.

3.

C) Neither firm entering is a Nash equilibrium.

4.

D) The outcome of the game is unpredictable.

11) The above figure shows the payoff to two airlines, A and B, of serving a particular route. If the two airlines must decide simultaneously, which one of the following statements is true? 1.

A) Since firm B has no dominant strategy, its decision is unpredictable.

2.

B) Since firm B's decision is unpredictable, firm A's decision is unpredictable.

3.

C) Neither firm entering is a Nash equilibrium.

4.

D) Firm B will not enter because it knows firm A will.

12) The above figure shows the payoff to two airlines, A and B, of serving a particular route. If the two airlines must decide simultaneously, what will happen if the government offers a $30 subsidy to airlines that serve this route? 1.

A) Both firms will enter profitably.

2.

B) Firm A will decide not to enter since firm B will.

3.

C) Firm B is still better off not entering.

4.

D) Neither firm will have a dominant strategy.

13) The above figure shows the payoff to two airlines, A and B, of serving a particular route. If the two airlines must decide simultaneously, how many pure Nash equilibria are there? 1.

A) 0

2.

B) 1

3.

C) 2

4.

D) It cannot be determined.

14) The above figure shows the payoff to two airlines, A and B, of serving a particular route. If the two airlines must decide simultaneously, what happens if the government imposes a $20 per firm tax on firms that service this route? 1.

A) Neither firm has a dominant strategy.

2.

B) Not entering is a dominant strategy for both firms.

3.

C) Neither firm entering is a Nash equilibrium.

4.

D) Only firm A will enter.

15) The above figure shows the payoff to two airlines, A and B, of serving a particular route. If the two airlines must decide simultaneously, and the government imposes a $20 per firm tax on firms that service this route, which of the following maximizes the firms' joint profits? 1.

A) Neither firm services the route.

2.

B) Firm A offers firm B $20 to not enter.

3.

C) Both firms will service this route.

4.

D) Firm B offers firm A $30 to not enter.

16) The game rock-paper-scissors has 1.

A) a mixed strategy equilibrium.

2.

B) a dominant strategy equilibrium.

3.

C) no equilibrium.

4.

D) a pure strategy equilibrium.

17) The above figure shows a payoff matrix for two firms, A and B, that must choose between a high-price strategy and a low-price strategy. For firm B, 1.

A) setting a high price is the dominant strategy.

2.

B) setting a low price is the dominant strategy.

3.

C) there is no dominant strategy.

4.

D) doing the opposite of firm A is always the best strategy.

18) The above figure shows a payoff matrix for two firms, A and B, that must choose between a high-price strategy and a low-price strategy. The Nash equilibrium in this game 1.

A) does not exist.

2.

B) occurs when both firms set a low price.

3.

C) occurs when both firms set a high price.

4.

D) occurs when firm A sets a high price and firm B sets a low price.

19) The above figure shows a payoff matrix for two firms, A and B, that must choose between a high-price strategy and a low-price strategy. Both firms setting a high price is not a Nash equilibrium because 1. 2. 3. 4.

A) setting a high price is the dominant strategy for each firm. B) neither firm can improve its payoff by setting a low price given that the other firm is setting a high price. C) there is no dominant strategy for either firm. D) both firms can improve their payoff by setting a low price given that the other firm is setting a high price.

20) The above figure shows a payoff matrix for two firms, A and B, that must choose between selling basic computers or advanced computers. Firm B's dominant strategy 1.

A) is to make basic computers.

2.

B) is to make advanced computers.

3.

C) is to adopt firm A's strategy.

4.

D) does not exist in this game.

21) The above figure shows a payoff matrix for two firms, A and B, that must choose between selling basic computers or advanced computers. How many Nash equilibria are there? 1.

A) 0

2.

B) 1

3.

C) 2

4.

D) 4

22) The above figure shows a payoff matrix for two firms, A and B, that must choose between selling basic computers or advanced computers. Which of the following is a Nash equilibrium? 1.

A) Both firms make advanced computers.

2.

B) Both firms make basic computers.

3.

C) Firm A makes basic computers and firm B makes advanced computers.

4.

D) There are no Nash equilibria.

23) A single-period duopoly firm can choose output level A or B. The firm decides it will produce level A regardless of what the other firm produces. This decision may occur because 1.

A) producing the output level A is a dominant strategy.

2.

B) this firm has simply decided to always produce at level A.

3.

C) Both A and B are possible.

4.

D) None of the above.

24) In a non-cooperative, imperfect information, simultaneous-choice, one-period game, a Nash equilibrium 1.

A) will never exist.

2.

B) will always include dominant strategies.

3.

C) will always result in both players taking the same action.

4.

D) may not maximize the sum of the firms' profits.

25) The term prisoners' dilemma refers to a game in which 1.

A) there are no Nash equilibria.

2.

B) there are no dominant strategies.

3.

C) the payoff from playing the dominant strategy is the same for each player.

4.

D) the payoff from playing the dominant strategy is not the highest payoff possible.

26) Collusion is more likely to occur when 1.

A) there is fear of punishment for not colluding.

2.

B) there is a known finite time horizon.

3.

C) there are large gains to be made by cheating on an agreement.

4.

D) the game lasts only one period.

27) A Nash equilibrium occurs when 1. 2.

A) players choose their best strategy given the strategies chosen by others. B) the efficient allocation of resources is achieved by setting marginal revenue equal to marginal cost.

3.

C) a monopolist is forced to produce the efficient level of output.

4.

D) oligopolists cooperate with each other.

28) A strategy is dominant if 1.

A) it yields a greater payoff than any other player receives.

2.

B) it yields a payoff at least as large as that from any other strategy, regardless of the actions of other players.

3.

C) the player cannot gain by changing strategy, assuming that no other player changes strategy.

4.

D) it is part of a Nash equilibrium.

29) A surprising outcome of the Rock-Paper-Scissors game is that 1.

A) it is a clear example of a first mover advantage.

2.

B) there is no pure-strategy Nash equilibrium.

3.

C) it is best not to play the game.

4.

D) it is a good way to determine who goes first in a sequential move game.

30) If the payoff to the United States to pursuing nuclear weapons is 100 if the USSR does not pursue nuclear weapons and 50 if they do, and the payoff to the USSR to pursuing nuclear weapons is 80 if the USA doesn't pursue nuclear weapons and 30 if they do, what is the non-cooperative equilibrium? 1.

A) The USA pursues nuclear weapons, the USSR does not.

2.

B) The USA pursues nuclear weapons, the USSR pursues nuclear weapons.

3.

C) The USA does not pursue nuclear weapons, the USSR does not pursue nuclear weapons.

4.

D) The USA does not pursue nuclear weapons, the USSR does pursue nuclear weapons.

31) In the 1980s, the USA and the USSR negotiated a reduction in nuclear arms; this is an example of a 1.

A) non-cooperative equilibrium.

2.

B) cooperative outcome that may not be a Nash equilibrium.

3.

C) cooperative outcome that was a Nash equilibrium.

4.

D) sub-game perfect equilibrium.

32) If a strategy is a best response, then 1.

A) it will never be played.

2.

B) it will never be part of a Nash equilibrium.

3.

C) A and B are correct.

4.

D) it could be part of a Nash equilibrium.

33) Which of the following constitutes a mixed strategy Nash equilibrium of the Odds and Evens game? 1.

A) Play Even and Odd with 50% probability each.

2.

B) Play Even with 75% probability and Odd with 25% probability.

3.

C) Always play Even.

4.

D) This game has no mixed strategy Nash equilibrium.

34) A player can choose among three strategies: T, M, and B. Nevertheless, strategy B is dominated by strategy T. This means that 1.

A) strategy T is always played.

2.

B) strategy B is never played.

3.

C) strategy B will be part of a Nash equilibrium.

4.

D) strategy M is never played.

35) What is the primary difference between a mixed strategy and a pure strategy? 1.

A) Pure strategies are always dominated strategies.

2.

B) Mixed strategies call for randomizing over possible actions, pure strategies do not.

3.

C) Pure strategies are much more common than mixed strategies.

4.

D) Mixed strategies are not optimal whereas pure strategies are.

36) Which of the following is LEAST likely characterized by mixed strategies? 1.

A) the choice of whether to go to war or not

2.

B) the choice of which pitch to throw in baseball

3.

C) the choice of which play to run in football

4.

D) the choice of where to kick a soccer penalty shot

37) What is the counter-intuitive solution to a mixed strategy? 1.

A) Player A makes Player B indifferent between its strategy choices.

2.

B) Player A makes itself indifferent between its strategy choices.

3.

C) Player A changes the rules of the game.

4.

D) There are no mixed strategies in most games.

38) Mutually Assured Destruction was a standing policy during the Cold War, in which the United States and the U.S.S.R. maintained and expanded nuclear arsenals beyond practical levels. What could explain such a phenomenon? 1.

A) insane public officials who were bent on world domination

2.

B) a prisoners' dilemma

3.

C) a leader-follower type game

4.

D) tacit collusion

39) Which of the following is NOT part of solving a game? 1.

A) Write down all possible combinations of strategies.

2.

B) Write down all possible payoffs and eliminate dominated strategies.

3.

C) Solve for any Nash Equilibrium.

4.

D) None of the above.

40) In a simultaneous game where both players prefer doing the opposite of what the opponent does, a Nash equilibrium does not exist.

41) If neither firm has a dominant strategy, a Nash equilibrium cannot exist.

42) The above figure shows the payoffs to two airlines, A and B, of serving a particular route. Is there a Nash equilibrium? What is it? Explain.

43) Suppose two firms, A and B, are simultaneously considering entry into a new market. If neither enters, both earn zero. If both enter, they both lose 100. If one firm enters, it gains 50 while the other earns zero. Set up the payoff matrix for this game and determine if any Nash equilibria exist. Can you predict the outcome? What if firm A gets to decide first?

44) Why is collusion more likely in a repeated game?

45) The above figure shows the payoff for two firms, A and B, that must each choose to sell either at a high or low price. Determine the dominant strategies for each firm (if any) and the Nash equilibria (if any).

14.2 Repeated Dynamic Games

1) Which of the following is a dynamic game? 1.

A) rock-paper-scissors

2.

B) flipping pennies

3.

C) chess

4.

D) the card game "war"

2) One interesting feature of a prisoner's dilemma game is that 1.

A) non-cooperative behavior leads to lower payoffs than cooperative behavior.

2.

B) it was only valid before the industrial revolution.

3.

C) individuals behave irrationally when they behave non-cooperatively.

4.

D) cooperative behavior leads to lower payoffs than non-cooperative behavior.

3) Repeated games are conducive to 1.

A) explicit cooperation.

2.

B) tacit cooperation.

3.

C) corruption.

4.

D) failing to have a Nash equilibrium.

4) One way to ensure cooperation in an infinitely repeated simultaneous game is 1.

A) to always play the dominant strategy.

2.

B) to punish the player that reneges on agreements.

3.

C) to never punish any player.

4.

D) to always punish all players.

5) Two identical firms are considering entering a new market that currently has no suppliers. The demand is large enough for both firms to make a positive profit. There are no fixed costs to enter. Explain how a simultaneous decision to enter on the part of the two firms will lead to a different outcome than a sequential entry decision.

14.3 Sequential Dynamic Games

1) The above figure shows the payoff to two gasoline stations, A and B, deciding to operate in an isolated town. If firm A chooses its strategy first, then 1.

A) firm A will not enter.

2.

B) firm B's entry is blockaded.

3.

C) both firms will enter.

4.

D) firm A will enter and firm B will not.

2) The above figure shows the payoff to two gasoline stations, A and B, deciding to operate in an isolated town. Suppose a $60 fee is required to enter the market. If firm A chooses its strategy first, then 1.

A) firm A will not enter.

2.

B) neither firm will enter.

3.

C) both firms will enter.

4.

D) firm A will enter and firm B will not.

3) The above figure shows the payoff to two gasoline stations, A and B, deciding to operate in an isolated town. Suppose a $30 fee is required to enter the market. If firm A chooses its strategy first, then 1.

A) firm A will not enter.

2.

B) neither firm will enter.

3.

C) both firms will enter.

4.

D) firm A will enter and firm B will not.

4) If only one firm operates in a market, and a potential entrant is blockaded from entering the market, then the incumbent firm must 1.

A) have acted to prevent entry.

2.

B) be pricing where price equals marginal cost.

3.

C) be a natural monopoly.

4.

D) be the Stackelberg leader.

5) An incumbent's threat to retaliate after a potential competitor enters the market will be taken seriously by potential competitors if 1. 2.

A) the incumbent can still earn a profit after carrying out the threat. B) the incumbent earns greater profit carrying out the threat than by accommodating entry.

3.

C) the potential entrant cannot earn a profit if the threat is carried out.

4.

D) the potential entrant's profit exceeds the incumbent's if the threat is carried out.

6) With regard to preventing entry, if identical firms act simultaneously, 1.

A) they cannot credibly threaten each other.

2.

B) they will all incur losses.

3.

C) only one firm will enter the market.

4.

D) none of them will enter the market.

7) In the Stackelberg model, the leader has a first-mover advantage because it 1.

A) has lower costs than the follower.

2.

B) commits to producing a larger quantity.

3.

C) reacts to the follower's decision.

4.

D) differentiates its output.

8) If a Cournot duopolist announced that it will double its output 1.

A) it becomes the leader.

2.

B) the other firm does not view the announcement as credible.

3.

C) the other firm will shut down.

4.

D) the other firm will double output also.

9) If a Cournot duopolist announced that it will double its output, the other firm does not view the announcement as credible because 1.

A) the announcing firm's profits will fall if it carries out the threat.

2.

B) the other firm's profits will fall if the announcing firm carries out the threat.

3.

C) the other firm's profits will rise if the announcing firm carries out the threat.

4.

D) the other firm will double output also.

10) If an incumbent faces an identical potential entrant with no costs of entry, the incumbent will 1.

A) produce the Cournot duopolist level of output.

2.

B) produce the Stackelberg leader level of output.

3.

C) set price equal to marginal cost.

4.

D) shut down.

11) If an incumbent cannot commit and faces an identical potential entrant with relatively high fixed costs that are below the level where entry is blockaded, the incumbent will 1.

A) produce the Cournot duopolist level of output.

2.

B) produce the Stackelberg leader level of output.

3.

C) set price equal to marginal cost.

4.

D) produce a level of output that is greater than the Stackelberg leader level of output.

12) One firm previously operated as a monopoly. Now, one potential entrant exists. Consumers would prefer 1. 2.

A) entry, and the firms to split the output equally. B) no entry, and for the incumbent to produce the Stackelberg leader level of output.

3.

C) entry, and for the incumbent to produce the Stackelberg leader level of output.

4.

D) no entry, and the monopoly to continue.

13) Incumbents are unaffected by fixed costs of entry while potential entrants are affected by them because 1.

A) for potential entrants the cost is avoidable, while for the incumbent, it is not.

2.

B) fixed costs will be greater for the potential entrant than for the incumbent.

3.

C) fixed costs are zero for the incumbent.

4.

D) incumbents will act to prevent entry at all costs.

14) Before entering, fixed cost associated with the industry in question are sunk costs for 1.

A) the incumbent firm.

2.

B) the outside firm.

3.

C) both firms.

4.

D) neither firm.

15) Assume an industry, currently dominated by one firm, experiences a large decline in fixed costs. This will 1.

A) make entry of other firms more likely.

2.

B) make entry of other firms less likely.

3.

C) serve as higher barrier to entry.

4.

D) induce the incumbent firm to exit the industry.

16) An incumbent announces it will significantly increase output in the next period, but only has contracts for the amount produced this period. The announcement is a 1.

A) credible threat.

2.

B) non-credible threat.

3.

C) commitment.

4.

D) mixed strategy.

17) The above figure shows the payoff matrix facing an incumbent firm and a potential entrant. The potential entrant cannot earn a profit if the incumbent 1.

A) chooses the Cournot level of output.

2.

B) chooses the Stackelberg leader level of output.

3.

C) shuts down.

4.

D) deters entry.

18) The above figure shows the payoff matrix facing an incumbent firm and a potential entrant. Assuming a fixed cost of entry, the incumbent will deter entry because 1.

A) it is more profitable than accommodating entry.

2.

B) it increases consumer surplus.

3.

C) the potential entrant winds up with zero profit.

4.

D) the incumbent would earn zero profit if it accommodated entry.

19) The above figure shows the payoff matrix facing an incumbent firm and a potential entrant. Assuming a fixed cost of entry, the outcome will be that the incumbent 1.

A) deters entry.

2.

B) chooses the Stackelberg leader level of output but the potential entrant does not enter anyway.

3.

C) chooses the Stackelberg leader level of output and the potential entrant enters.

4.

D) deters entry and earns zero profit.

20) The above figure shows the payoff matrix facing an incumbent firm and a potential entrant. If the fixed cost of entry were to increase, which of the following would occur? 1.

A) The incumbent chooses the Cournot level of output.

2.

B) The incumbent shuts down.

3.

C) The entry-deterring level of output rises.

4.

D) The entry-deterring level of output falls.

21) The above figure shows the payoff matrix facing an incumbent firm and a potential entrant. What policy could government adopt to prevent entry deterrence by the incumbent? 1.

A) production quotas

2.

B) price ceiling

3.

C) safety standards

4.

D) None of the above is correct.

22) The ability to deter entry requires 1.

A) a credible threat that if entry occurs the firm is willing to produce more than they would otherwise.

2.

B) a credible threat that if entry occurs the firm will not produce more than they would otherwise.

3.

C) a good lawyer.

4.

D) a clever accounting department.

23) Deterring entry might require a firm to 1.

A) price their product closer to the competitive price than to the monopoly price.

2.

B) price their product closer to the monopoly price than to the competitive price.

3.

C) drop output almost to zero to show the consumers "who's boss."

4.

D) drop price almost to zero to get price below marginal cost.

24) Assume a firm lowers price below marginal cost to deter entry. 1.

A) This strategy is not credible.

2.

B) This strategy is credible.

3.

C) This strategy is illegal.

4.

D) This strategy is immoral.

25) Assume a firm is a monopoly and enjoys $10,000,000 profits per year. The firm lobbies to have a moratorium passed by Congress on new firms in its market for the next 25 years. If there is no discount rate, how much would the firm be willing to pay to deter entry? 1.

A) $250 million

2.

B) $25 million

3.

C) $100 million

4.

D) $250 billion

26) Assume a firm is a monopoly and enjoys $10,000,000 profits per year. The firm lobbies to have a moratorium passed by Congress on new firms in its market for the next 25 years. If there is no discount rate, how much would any firm(s) arguing against the moratorium be willing to spend to block it? 1.

A) something less than $250 million

2.

B) $250 million

3.

C) $251 million

4.

D) $250 billion

27) A subgame perfect Nash equilibrium 1.

A) can be solved by backward induction.

2.

B) is a set of strategies that are a Nash equilibrium in every subgame.

3.

C) is a stronger form of Nash equilibrium.

4.

D) All of the above are correct.

28) A sub-game perfect Nash equilibrium is defined as 1.

A) a set of strategies that are a Nash equilibrium in every subgame of a static game.

2.

B) a set of strategies that are a Nash equilibrium in every subgame of a dynamic game.

3.

C) a set of strategies that are a Nash equilibrium in a single subgame of a dynamic game.

4.

D) the game within the game.

29) A firm producing a relatively large quantity before any rivals have entered the market, is an example of first-mover advantage. s.

30) If an incumbent threatens to retaliate against entry, but its profits are greater under accommodated entry than under the proposed threat, potential entrants will ignore the threat.

31) Fixed costs of entry create an advantage for potential entrants since incumbents have already made these expenditures while potential entrants can avoid these costs.

32) The above figure shows the payoffs to two firms deciding to open a gasoline station in an isolated town. If firm A decides first, what will happen? If there is a $60 fee to enter this market, what will happen?

33) The above figure shows the payoff matrix facing an incumbent firm. Assuming a fixed cost of entry, will the incumbent deter entry? Why?

34) How can a firm be made better off by limiting its options?

35) Suppose market demand is p = 10 - Q. Firms have a fixed entry cost of 5 and no marginal cost. If firm A is the incumbent, can it deter the entry of its rival, firm B?

36) Suppose market demand is p = 10 - Q. Firms incur no cost of production. If firm A is the incumbent, can it deter the entry of its rival, firm B?

14.4 Auctions

1) A sale in which property or a service is sold to the highest bidder is called a(n) 1.

A) auction.

2.

B) bidder sale.

3.

C) competitive market.

4.

D) Austrian bundle.

2) An auction in which the price announced by the auctioneer DESCENDS is called a(n) 1.

A) Dutch Auction.

2.

B) English Auction.

3.

C) Sealed Bid Auction.

4.

D) Descending Option Auction.

3) A private auction is an auction in which 1.

A) individuals know their own value of the good and everyone else's valuation, too.

2. 3.

B) individuals have their own valuation of the good but don't know everyone else's. C) many auctions are auctioned off at the same time.

4.

D) only one good is auctioned off.

4) The individual with the highest valuation of the good will win in which of the following auctions? 1.

A) English Auction

2.

B) Dutch Auction

3.

C) Sealed Bid Auction

4.

D) All of the above.

5) The internet auction site eBay is an example of a(n) 1.

A) Sealed Bid Auction.

2.

B) Second-Price Auction.

3.

C) English Auction.

4.

D) Both A and B.

6) The result that different auction styles in which the good goes to the winner with the highest valuation of the good generate the same amount of revenue is called 1.

A) Revenue Equivalence Theorem.

2.

B) Marginal Revenue Theory.

3.

C) Auction Revenue Theory.

4.

D) First Bid Revenue Theorem.

7) Winner's curse is likely to happen in which of the following auctions? 1.

A) unexplored oil reserves

2.

B) corn stored in a warehouse

3.

C) 1,000 ton of iron ore

4.

D) U.S. Treasury Bonds

8) Which auctioned good is more likely to have different private values across potential bidders?

1.

A) a truckload of sand

2.

B) a Monet painting

3.

C) a brand new car

4.

D) a gold bar

9) What is an example of the bidder's curse? 1.

A) addiction to auctions

2.

B) paying less than the auctioned good value

3.

C) Bid a value that is higher than the price of the good at a retail store.

4.

D) Never win an auction.

10) In auctions, the winner always pays a price equal to the highest (his) bid.

11) In Dutch or first-price sealed-bid auctions, participants will bid less than their highest valuation.

12) Explain why it is unwise to bid more than your valuation of the good in a sealed bid second-price auction.

14.5 Behavioral Game Theory

1) Which of the following factors affecting decision-making is studied by Behavioral Game Theory? 1.

A) psychological biases

2.

B) limited power of calculation

3.

C) weather forecast

4.

D) A and B are correct

Microeconomics, 7e (Perloff) Chapter 18 Externalities, Open-Access, and Public Goods

18.1 Externalities

1) If children go to school and become productive members of society, 1.

A) a negative externality is created by the schools.

2.

B) a positive externality is created by the schools.

3.

C) no externality is created by the schools.

4.

D) an externality is created that may be positive or negative.

2) Students who talk loudly with each other in class 1.

A) create an externality because other students cannot follow the lecture as well.

2.

B) disturb nobody.

3.

C) benefit the other students in class because they engage in conversation.

4.

D) only create an externality if they talk about something unrelated to class.

3) Positive externalities are created when 1. 2.

A) other consumers reduce their demand for coffee and price thereby declines. B) farmers spray pesticide in their fields and it washes into the local river after the first rainstorm.

3.

C) your neighbor plants beautiful trees and flowers in her yard.

4.

D) you purchase the "Mona Lisa" and lock it in a vault.

4) Negative externalities are created when 1. 2.

A) an increase in the price of butterfat drives up the price of ice cream. B) a driver leaves his car in a parking space after the meter expires and receives a ticket.

3.

C) a driver drives recklessly on a busy highway.

4.

D) a driver pulls over to help a stranded motorist fix a flat tire.

5) The price of pie increases. Some people who purchased pie before the price increase no longer purchase pie. This is 1.

A) a positive externality.

2.

B) a negative externality.

3. 4.

C) a positive externality for some consumers and a negative externality for others. D) not an externality.

6) In general, an externality is created when 1.

A) people are affected (other than by price) by a transaction which they were not part of.

2.

B) firms produce a product of low quality and consumers don't like it.

3.

C) firms have to pay for polluting the environment.

4.

D) the government subsidizes education.

7) Consider a housing development built near an existing airport. After the houses are occupied, homeowners complain that the airport imposes a negative externality on them and it should be moved or otherwise limited. Is the airport a negative externality? 1.

A) No, the airport was there first.

2.

B) No, if the original property values reflect the costs imposed by the airport.

3.

C) No, airports are government entities and therefore don't impose costs on individuals.

4.

D) Yes, the airport's noise should be curtailed for the well-being of the homeowners.

8) What is one reason drunk driving is held in such disrepute? 1. 2.

A) Drunk driving imposes high potential costs on non-drunk drivers. B) Drunk driving destroys cars and telephone poles, causing disruption in essential services.

3.

C) Only drunk drivers cause automobile fatalities.

4.

D) Drunk drivers impose high potential costs on other drunk drivers.

9) The productivity of the employees of a bakery is reduced because of the excessive noise coming from a next door car repair shop. This is an example of 1.

A) synergy.

2.

B) a positive externality.

3.

C) a negative externality.

4.

D) happy coexistence.

10) A student that asks interesting questions during the lecture generates 1.

A) positive externalities.

2.

B) no externalities.

3.

C) negative externalities.

4.

D) an excludable good.

11) Your new neighbor does not mow his lawn or prune the trees in his front yard. As a result, your house value decreases. Your neighbor generated 1.

A) a negative externality.

2.

B) a positive externality.

3.

C) no externality.

4.

D) a non-rival good.

For the following, please answer "True" or "False" and explain why.

12) Changing the price of a good will usually result in a negative externality.

13) You are having a party and one of your guests lights up a cigar without asking. Explain why this creates an externality.

18.2 The Inefficiency of Competition with Externalities

1) If a production process creates pollution, a competitive market produces excessive pollution because 1.

A) the firms do not include the social cost of the pollution in their profitmaximizing decisions.

2.

B) the firms place too high a price on society's cost of inflation.

3.

C) people are not injured by the pollution.

4.

D) zero pollution is optimal.

2) Which of the following statements about private and social costs is TRUE? 1.

A) Social costs include externalities.

2.

B) Private cost do not include externalities.

3.

C) Social costs are never smaller than private costs.

4.

D) All of the above.

3) If a production process creates pollution, a competitive market produces excessive pollution because 1.

A) private marginal cost of pollution exceeds its social marginal cost.

2.

B) social marginal cost of pollution exceeds its private marginal cost.

3.

C) the marginal benefit of pollution to the firm is zero.

4.

D) zero pollution is optimal.

4) If a production process creates positive externalities, a competitive market produces too few positive externalities because the producer 1.

A) does not pay all the costs of the externalities.

2.

B) does not receive compensation for the externalities.

3.

C) Both A and B.

4.

D) None of the above.

5) In the presence of no externalities, 1.

A) social marginal cost exceeds private marginal cost.

2.

B) social marginal cost is less than private marginal cost.

3.

C) social marginal cost equals private marginal cost.

4.

D) social marginal cost and private marginal cost cannot be compared.

6) If a production process generates pollution, then a competitive market will 1.

A) produce more of the good than is socially optimal.

2.

B) produce less of the good than is socially optimal.

3.

C) produce the socially optimal quantity of that good.

4.

D) produce zero output.

7) If a production process generates pollution, then a competitive market will produce more of the good than is socially optimal because 1.

A) firms take all costs into consideration.

2.

B) firms incur all costs of production but ignore some of them.

3.

C) firms ignore the costs of production that they do not incur.

4.

D) firms set price equal to social marginal cost.

8) If a market is subject to a positive externality, 1. 2. 3.

A) the demand curve reflecting social benefit will be to the right of the demand curve representing private benefit. B) there is only one demand curve. C) the demand curve reflecting social benefit will be to the left of the demand curve representing private benefit.

4.

D) private benefit will exceed social benefit.

9) The above figure shows the market for steel ingots. If the market is competitive, then 1.

A) the socially optimal quantity of steel is zero.

2.

B) the socially optimal quantity of steel of 50 units is produced.

3.

C) the socially optimal quantity of steel of 100 units is produced.

4.

D) more than the socially optimal quantity of 50 units of steel is produced.

10) The above figure shows the market for steel ingots. An externality can be seen because 1.

A) the social marginal cost exceeds the private marginal cost.

2.

B) the private marginal cost exceeds the social marginal cost.

3.

C) the optimal quantity of steel is zero.

4.

D) not enough steel gets produced by the competitive market.

11) The above figure shows the market for steel ingots. If the market is competitive, then the deadweight loss to society is 1.

A) a.

2.

B) b.

3.

C) c.

4.

D) zero.

12) The above figure shows the market for steel ingots. The socially optimal quantity of steel is 1.

A) 0 units.

2.

B) 50 units.

3.

C) 100 units.

4.

D) the amount produced if the market were competitive.

13) The above figure shows the market for steel ingots. The optimal quantity of pollution 1.

A) is 0 units.

2.

B) is 50 units.

3.

C) is 100 units.

4.

D) cannot be determined from the information provided.

14) In a competitive market, a negative externality creates a deadweight loss because 1.

A) the cost of the externality is double counted.

2.

B) a harm is generated.

3.

C) price equals social marginal cost.

4.

D) price equals private marginal cost.

15) In a market with positive externalities, 1.

A) the efficient level of production is less than what competition will obtain.

2.

B) the efficient level of production is equal to what competition will obtain.

3.

C) the efficient level of production is more than what competition will obtain.

4.

D) there cannot be an efficient level of production.

16) Assume a country agrees to a free-trade act with another country. In the process, some individuals are displaced from their jobs, thus the free-trade act results in a negative externality. 1.

A) False.

2.

B) True.

3.

C) Only if those who were displaced are not compensated with another job or income transfer.

4.

D) Only if those who were displaced were compensated with another job or income transfer.

17) The above figure shows the market for steel ingots. If the market is competitive, then the competitive market level of output is 1.

A) 100 units.

2.

B) 150 units.

3.

C) 50 units.

4.

D) 300 units.

18) A zero pollution level is not achievable because 1.

A) corporations would lobby against it.

2.

B) it implies no production and no consumption.

3. 4.

C) it implies an output level larger than that from a competitive market equilibrium. D) consumption would be larger than production.

For the following, please answer "True" or "False" and explain why.

19) To maximize welfare in a competitive market that has a negative externality in production, government should tax a pollution-generating good at a specific tax equal to the marginal cost of producing the good.

20) In the presence of a negative externality generated by producing a good, a competitive market will produce more of that good than is socially optimal.

21) Suppose that in the market for paper, demand is p = 100 - Q. The private marginal cost is MCp = 10 + Q. Pollution generated during the production process creates external marginal harm equal to MCe = Q. What specific tax would result in a competitive market producing the socially optimal quantity of paper?

22) Suppose that in the market for paper, demand is p = 100 - Q. The private marginal cost is MCp = 10 + Q. Pollution generated during the production process creates external marginal harm equal to MCe = Q. Is social welfare greater under monopoly or under competition?

23) Suppose that the market for steel is shown in the above figure. What specific tax would result in a competitive market producing the socially optimal quantity of steel?

24) In terms of cost-benefit analysis, explain why a competitive market with an externality produces too much pollution.

25) Explain why the optimal amount of pollution is often not zero.

18.3 Regulating Externalities

1) In the presence of a negative externality, a specific tax can achieve the social optimum because 1.

A) output is reduced to zero as a result.

2.

B) it internalizes the external cost.

3.

C) it directly charges the producer for polluting.

4.

D) the price of the good rises by the full amount of the tax.

2) The above figure shows the market for steel ingots. If the market is competitive, then to achieve the socially optimal level of pollution, the government can 1.

A) outlaw the production of steel.

2.

B) institute a specific tax of $25.

3.

C) institute a specific tax of $50.

4.

D) institute a specific tax equal to area b.

3) The above figure shows the market for steel ingots. If the market is competitive, and the government institutes a $100 specific tax on steel, then 1.

A) less than the socially optimal quantity of steel is produced.

2.

B) the socially optimal quantity of steel of 50 units is produced.

3.

C) the socially optimal quantity of steel of 100 units is produced.

4.

D) more than the socially optimal quantity of steel is produced.

4) The above figure shows the market for steel ingots. If the market is competitive, then to achieve the socially optimal level of pollution, the government can 1.

A) institute a standard and allow firm to produce at most 50 ingots per period.

2.

B) institute a standard and allow firm to produce at most 25 ingots per period.

3.

C) institute a standard and allow firm to produce at most 100 ingots per period.

4.

D) institute a standard and allow firm to produce at most 10 ingots per period.

5) Which of the following policies address the the problem posed by positive externalities? 1.

A) a subsidy to the agent that generates the positive externality

2.

B) a tax on the agent that generates the positive externality

3.

C) limit the activity that generates the positive externality

4.

D) a subsidy to the agents that benefit from the positive externality

For the following, please answer "True" or "False" and explain why.

6) To maximize welfare in a competitive market that has a negative externality in production, government should tax a pollution-generating good at a specific tax equal to the marginal cost of producing the good.

7) Suppose that in the market for paper, demand is p = 100 - Q. The private marginal cost is MCp = 10 + Q. Pollution generated during the production process creates external marginal harm equal to MCe = Q. What specific tax would result in a competitive market producing the socially optimal quantity of paper?

8) Suppose that the market for steel is shown in the above figure. What specific tax would result in a competitive market producing the socially optimal quantity of steel?

18.4 Market Structure and Externalities

1) In the presence of a negative externality in production, a monopoly will produce 1.

A) more than the social optimum.

2.

B) less than the social optimum.

3.

C) the social optimum.

4.

D) All of the above are possible.

2) Because a monopoly ignores external costs, it is possible that it will 1.

A) produce the socially optimal quantity of a good.

2.

B) produce more than the socially optimal quantity of a good.

3.

C) produce less than the socially optimal quantity of a good.

4.

D) All of the above.

3) A monopoly might produce less than the socially optimal amount of pollution because 1.

A) it likes to be a good citizen.

2.

B) it sets price above marginal cost.

3.

C) it earns economic profit.

4.

D) it internalizes the external costs.

4) If the social marginal cost of a good is very high relative to the private marginal cost, then a monopoly will most likely 1.

A) produce more than the social optimum.

2.

B) produce less than the social optimum.

3.

C) produce the social optimum.

4.

D) produce zero pollution.

5) Production of a good produces pollution that is very damaging with each additional unit. A monopoly facing a very elastic demand curve will most likely produce 1.

A) less than the social optimum of the good.

2.

B) more than the social optimum of the good.

3.

C) the social optimum of the good.

4.

D) no externality.

6) If both a monopoly and a competitive market with the same marginal cost would produce a quantity that is greater than the social optimum in a market because of externalities, then 1.

A) welfare is greater under monopoly.

2.

B) welfare is greater under competition.

3.

C) welfare is the same for both market structures.

4.

D) the social optimum must be zero.

7) A specific tax in a monopoly market equal to the marginal harm of pollution 1.

A) will increase welfare.

2.

B) will decrease welfare.

3.

C) will leave welfare unchanged.

4.

D) All of the above are possible.

8) Monopolizing the sale of liquor 1.

A) can lead to an increase total welfare.

2.

B) will decrease total welfare.

3.

C) results in a deadweight loss.

4.

D) is anti-competitive and thus lowers total welfare.

9) A tax on a previously untaxed monopoly-produced good will necessarily lower total welfare if 1.

A) the demand curve is relatively inelastic.

2.

B) the demand curve is relatively elastic.

3.

C) less than the socially optimum is produced before the tax.

4.

D) more than the socially optimum is produced before the tax.

10) If a product provides a positive externality, a duopoly 1.

A) will provide more social welfare than a monopoly.

2.

B) will provide less social welfare than a monopoly.

3.

C) will provide the same social welfare as a monopoly.

4.

D) introduces an interesting and untested twist to the externality story.

For the following, please answer "True" or "False" and explain why.

11) Because a monopoly will produce less of a good than a competitive market will, welfare is always greater under monopoly than under competition in the presence of a negative externality.

12) When negative externalities from production exist, the deadweight loss from a competitive market may be larger than with a monopoly.

13) Suppose that the market for steel is shown in the above figure. Is social welfare greater under monopoly or under competition?

14) Explain how a specific tax equal to the marginal harm of pollution can increase or decrease total welfare in a monopoly market.

18.5 Allocating Property Rights to Reduce Externalities

1) The existence of externalities is due mainly to the fact that 1.

A) monopolies tend to produce too little of a good anyway.

2.

B) the optimal level of pollution is zero.

3.

C) pollution is not a serious problem.

4.

D) property rights are poorly defined.

2) The exclusive privilege to use an asset is called a(n)

1.

A) property right.

2.

B) exclusive use agreement.

3.

C) property privilege.

4.

D) right to work privilege.

3) The result that, under certain circumstances, no government action is needed to control an externality because it can be eliminated by bargaining between the affected parties is called 1.

A) a Nash Equilibrium.

2.

B) Coase Theorem.

3.

C) Bargaining Theorem.

4.

D) English Bargaining.

4) Suppose two neighbors share a park. One neighbor, Al, leaves trash in the park. This bothers the other neighbor, Bert. According to Coase's Theorem, the optimal level of trash in the park can be achieved if 1.

A) Al is fined by the government.

2.

B) Al has the right to leave trash and Bert cannot do anything about it.

3.

C) Al has the right to leave trash and Bert can pay him to limit his dumping.

4.

D) Bert moves.

5) Suppose two neighbors share a park. One neighbor, Al, leaves trash in the park. This bothers the other neighbor, Bert. According to Coase's Theorem, one necessary condition to alleviate the externality is that 1.

A) Al is fined by the government.

2.

B) Al has the right to leave trash and Bert cannot do anything about it.

3.

C) Bert has the right to a clean park and Al cannot leave trash.

4.

D) Either Al or Bert owns the park.

6) Suppose two neighbors share a park. One neighbor, Al, leaves trash in the park. This bothers the other neighbor, Bert. According to Coase's Theorem, the optimal level of trash in the park can be achieved if 1.

A) someone is assigned property rights to the park.

2.

B) government limits the use of the park.

3.

C) nobody catches Al leaving the trash.

4.

D) Bert moves.

7) Suppose twenty neighbors share a park. One of the neighbors, Al, leaves trash in the park. This bothers the other neighbors. According to Coase's Theorem, assigning the property rights to the park to Al 1.

A) will achieve the socially optimal quantity of trash.

2.

B) will result in zero trash being dumped in the park.

3. 4.

C) might still not achieve the social optimum since coordinating the other nineteen neighbors can be costly. D) is unfair.

8) The above figure shows the marginal benefit to a firm of polluting in the local river while producing its output, and the marginal cost to the surrounding neighbors. The marginal cost of production is zero for the firm. If property rights are not defined, how much pollution will occur? 1.

A) 0 units

2.

B) 500 units

3.

C) 1000 units

4.

D) more than 1000 units

9) The above figure shows the marginal benefit to a firm of polluting in the local river while producing its output, and the marginal cost to the surrounding neighbors. The marginal cost of production is zero for the firm. If the firm owns the river and there are thousands of surrounding neighbors, how much pollution is likely to occur? 1.

A) 0 units

2.

B) 500 units

3.

C) 1000 units

4.

D) more than 1000 units

10) The above figure shows the marginal benefit to a firm of polluting in the local river while producing its output, and the marginal cost to the firm's neighbor. The marginal cost of production is zero for the firm. If there is just one neighbor who owns the river, how much pollution is likely to occur? 1.

A) 0 units

2.

B) 500 units

3.

C) 1000 units

4.

D) more than 1000 units.

11) The above figure shows the marginal benefit to a firm of polluting in the local river while producing its output, and the marginal cost to the surrounding neighbors. The marginal cost of production is zero for the firm. If the firm owns the river and there is just one neighbor affected by the pollution, how much pollution is likely to occur? 1.

A) 0 units

2.

B) 500 units

3.

C) 1000 units

4.

D) more than 1000 units

12) The above figure shows the marginal benefit to a firm of polluting in the local river while producing its output, and the marginal cost to the surrounding neighbors. The marginal cost of production is zero for the firm. According to Coase's Theorem, which of the following scenarios is most likely to lead to the socially optimal level of pollution? 1.

A) The firm owns the river and there are a thousand surrounding neighbors.

2.

B) The firm owns the river and there is just one nearby neighbor.

3.

C) The river is jointly owned by one thousand surrounding neighbors.

4.

D) The firm owns the river, and therefore produces the social optimum no matter what.

13) The above figure shows the marginal benefit from pollution for two firms. If each firm receives a marketable permit to produce 25 units of pollution, which one of the following is most likely to happen? 1.

A) Firm B will sell some pollution rights to firm A.

2.

B) Firm A will sell some pollution rights to firm B.

3.

C) Firm A will produce all 50 units of pollution.

4.

D) Both firms will produce 25 units of pollution.

14) The above figure shows the payoff matrix for two firms. A chemical firm must choose between a low level of production which yields one ton of pollution into a nearby lake and a high level of production which yields two tons of pollution into the nearby lake. A private beach on the lake must decide whether to operate or not. Increased pollution reduces the number of people who wish to visit the beach. If nobody owns the lake, then 1.

A) the beach shuts down and the chemical firm produces 1 ton of pollution.

2.

B) the beach shuts down and the chemical firm produces 2 tons of pollution.

3.

C) the beach operates and the chemical firm produces 1 ton of pollution.

4.

D) the beach operates and the chemical firm produces 2 tons of pollution.

15) The above figure shows the payoff matrix for two firms. A chemical firm must choose between a low level of production which yields one ton of pollution into a nearby lake and a high level of production which yields two tons of pollution into the nearby lake. A private beach on the lake must decide whether to operate or not. Increased pollution reduces the number of people who wish to visit the beach. If the beach owner also owns the lake, and the chemical firm must pay $10 per ton to pollute, then 1.

A) the beach shuts down and the chemical firm produces 1 ton of pollution.

2.

B) the beach shuts down and the chemical firm produces 2 tons of pollution.

3.

C) the beach operates and the chemical firm produces 1 ton of pollution.

4.

D) the beach operates and the chemical firm produces 2 tons of pollution.

16) The above figure shows the payoff matrix for two firms. A chemical firm must choose between a low level of production which yields one ton of pollution into a nearby lake and a high level of production which yields two tons of pollution into the nearby lake. A private beach on the lake must decide whether to operate or not. Increased pollution reduces the number of people who wish to visit the beach. If the chemical firm owns the lake, and the beach owner must pay $10 to keep the chemical firm at just one ton of pollution, then 1.

A) the beach shuts down and the chemical firm produces 1 ton of pollution.

2.

B) the beach shuts down and the chemical firm produces 2 tons of pollution.

3.

C) the beach operates and the chemical firm produces 1 ton of pollution.

4.

D) the beach operates and the chemical firm produces 2 tons of pollution.

17) The above figure shows the payoff matrix for two firms. A chemical firm must choose between a low level of production which yields one ton of pollution into a nearby lake and a high level of production which yields two tons of pollution into the nearby lake. A private beach on the lake must decide whether to operate or not. Increased pollution reduces the number of people who wish to visit the beach. As long as someone owns the lake and the two parties can negotiate, then 1.

A) the beach shuts down and the chemical firm produces 1 ton of pollution.

2.

B) the beach shuts down and the chemical firm produces 2 tons of pollution.

3.

C) the beach operates and the chemical firm produces 1 ton of pollution.

4.

D) the beach operates and the chemical firm produces 2 tons of pollution.

18) The above figure shows the payoff matrix for two firms. A chemical firm must choose between a low level of production which yields one ton of pollution into a nearby lake and a high level of production which yields two tons of pollution into the nearby lake. A private beach on the lake must decide whether to operate or not. Increased pollution reduces the number of people who wish to visit the beach. Joint profits are maximized when 1.

A) the beach shuts down and the chemical firm produces 1 ton of pollution.

2.

B) the beach shuts down and the chemical firm produces 2 tons of pollution.

3.

C) the beach operates and the chemical firm produces 1 ton of pollution.

4.

D) the beach operates and the chemical firm produces 2 tons of pollution.

19) The above figure shows the payoff matrix for two firms. A chemical firm must choose between a low level of production which yields one ton of pollution into a nearby lake and a high level of production which yields two tons of pollution into the nearby lake. A private beach on the lake must decide whether to operate or not. Increased pollution reduces the number of people who wish to visit the beach. If nobody owns the lake, then 1.

A) joint profits are zero.

2.

B) joint profits are maximized.

3.

C) joint profits are not maximized.

4.

D) the beach will not operate because of too much pollution.

20) If the government wanted to efficiently limit the emission of carbon monoxide by all firms to exactly 4 million tons it could

1.

A) issue rights to pollute worth 4 million tons and let the firms trade those rights in a market.

2.

B) appeal to firms' environmental conscience to pollute less.

3.

C) subsidize production.

4.

D) rely on the Coase Theorem.

For the following, please answer "True" or "False" and explain why.

21) Firms that are most likely to buy marketable pollution rights are those that produce the most pollution per unit of output produced.

22) A firm operates and produces pollution that only harms an individual, Bob. The firm and Bob both know the costs and benefits of reducing pollution. Neither the firm nor Bob acts strategically while bargaining, and there are no transaction costs associated with bargaining. Explain how the efficient level of pollution occurs no matter whether the firm or Bob owns the property right to pollution.

23) The above figure shows the marginal benefit from pollution for two firms. If both firms receive a marketable permit to pollute 25 units of pollution each, how much will each firm pollute and how much will a permit for one unit of pollution be worth?

24) The above figure shows the payoff matrix for two firms. A chemical firm must choose between a low level of production which yields one ton of pollution into a nearby lake and a high level of production which yields two tons of pollution into the nearby lake. A private beach on the lake must decide whether to operate or not. Increased pollution reduces the number of people who wish to visit the beach. Determine the Nash equilibrium without property rights.

25) The above figure shows the payoff matrix for two firms. A chemical firm must choose between a low level of production which yields one ton of pollution into a nearby lake and a high level of production which yields two tons of pollution into the nearby lake. A private beach on the lake must decide whether to operate or not. Increased pollution reduces the number of people who wish to visit the beach. If the chemical firm owns the lake and the beach owner must pay the chemical firm $10 to produce only one ton of pollution, what is the outcome? If the beach owner owns the lake and the chemical firm

must pay $10 per ton of pollution, what is the outcome? Compare this result to the case where nobody owns the lake.

18.6 Rivalry and Exclusion

1) A common resource is best described as a resource where 1.

A) there is a positive externality in consumption.

2.

B) there is a negative externality in consumption.

3.

C) there is a positive externality in production.

4.

D) there is a negative externality in production.

2) Which of the following is not a common property? 1.

A) a city park

2.

B) a main street

3.

C) a public beach

4.

D) a movie screening

3) To alleviate the commons problem, the government can 1.

A) apply a tax.

2.

B) set a quota.

3.

C) assign property rights.

4.

D) All of the above.

4) Fishermen on the East Coast are using lobster traps out of which most of the lobsters that enter can escape. Why? 1.

A) It will make over-fishing less likely.

2.

B) They can't come up with better traps.

3.

C) They are not educated enough to maximize profit.

4.

D) They catch more lobsters this way.

5) The Commons Problem arises because 1.

A) firms don't maximize profits.

2.

B) social and private incentives are not aligned and property rights are missing.

3.

C) social cost equals private cost and property rights are missing.

4.

D) social benefit equals private benefit and property rights are missing.

6) The U.S. Federal government limits the ability for private firms to harvest timber on much government land. This, it is argued, increases the amount of fuel for wildfires which often burn out of control and cost money and manpower to control. 1. 2.

A) This suggests that the policy addressing one positive externality might have created another positive externality. B) This suggests that those who harvest timber are prone to starting wildfires.

3. 4.

C) This suggests that the policy addressing timber harvesting created a negative externality. D) This suggests that government policy is destined to fail.

7) The Commons Problem could be addressed by 1.

A) charging a fee proportional to the use of the common property.

2.

B) charging a fee unrelated to the use of the common property.

3.

C) charging no fee.

4.

D) prohibiting the use of the common property.

8) In the case of a good that has no exclusion and no rivalry, private markets fail because 1.

A) of free-ridership.

2.

B) this is a natural monopoly.

3.

C) profit is driven down to zero.

4.

D) the quantity produced will exceed the social optimum.

9) A commodity or service whose consumption by one person does not preclude others from also consuming it is called a 1.

A) private good.

2.

B) public good.

3.

C) Giffen Good.

4.

D) Coase Good.

10) Which of the following goods has the property of rivalry? 1.

A) national defense

2.

B) a highway

3.

C) air to breath

4.

D) an outdoor movie screening

11) A public good in which exclusion is possible is called 1.

A) an exclusive good.

2.

B) a common good.

3.

C) an impure good.

4.

D) a club good.

12) Which of the following is NOT a club good? 1.

A) cable television

2.

B) a rock concert

3.

C) a country club

4.

D) a baseball bat

13) Markets tend to produce too little of an excludable public good because 1.

A) transaction costs are high.

2.

B) of the lack of rivalry.

3.

C) these goods are depletable.

4.

D) All of the above.

14) The total demand for a public good is found by 1.

A) horizontally summing all individual demands.

2.

B) vertically summing all individual demands.

3.

C) finding the demand from the median voter.

4.

D) dividing the marginal cost of the good by the number of voters.

15) The efficient quantity of a pure public good occurs when the marginal cost of producing that good equals the 1.

A) marginal benefit to the median voter.

2.

B) marginal benefit to each individual.

3.

C) sum of all individual marginal benefits.

4.

D) sum of all individual marginal benefits divided by the number of voters.

16) When majority rule voting is used to determine whether to purchase a public good, 1.

A) the efficient outcome is not assured.

2.

B) the median voter gets her way.

3.

C) the sum of the marginal benefits is ignored.

4.

D) All of the above.

17) When majority rule voting is used to determine whether to purchase a public good, 1.

A) the efficient outcome is assured.

2.

B) the median voter gets her way.

3.

C) the sum of the marginal benefits must equal marginal cost.

4.

D) the marginal benefit of the good to the median voter equals the good's marginal cost.

18) Which of the following goods is more likely to be excludable? 1.

A) a chocolate bar

2.

B) a concert at Times Square

3.

C) national defense

4.

D) ocean breeze

19) If the median voter voted against a project, we can infer that 1.

A) at least half of the voters also voted against it.

2.

B) at most a third of the voters voted against the project.

3.

C) the project was approved.

4.

D) at least half of the voters voted in favor of the project.

20) The median voter votes yes for a project if 1.

A) the value of the project for him is larger than the tax he needs to pay.

2.

B) the value of the project for him is smaller than the tax he needs to pay.

3.

C) the total benefits are positive.

4.

D) the total cost is zero.

For the following, please answer "True" or "False" and explain why.

21) Over-fishing of common fishing grounds happens because fishing grounds are a common property and social and private incentive are the same.

22) The efficient quantity of a public good occurs when the marginal cost of providing that good equals the sum of the marginal benefits to all individuals.

23) Explain the externality generated when a shepherd grazes sheep in a field that is common property that several other shepherds use.

24) Explain why state governments may charge a fee for beach access to address the commons problem.

.

25) Explain why the social demand curve for a public good is the vertical sum of the demand curves of each individual.

26) Suppose three neighbors must vote on the installation of a traffic light that costs $210. The cost of the light will shared by all three. Voter A values the light at $50; voter B values the light at $50; and voter C (who drives the most) values the light at $200. If the voting rule is that the majority wins, does the light get purchased? Is it efficient to purchase the light?

27) Suppose 100 citizens each derive marginal benefit from submarines according to the function MB = 10 - Q. If subs cost $100 each to produce, what is the efficient quantity of submarines?

28) Two neighboring farmers must each decide whether to contribute to a fence that separates their properties. The fence costs a total of $20. Both farmers currently have a profit of $30 each. With a fence to keep each farmer's animals from wandering onto the other's property, both farmers would experience a $15 rise in profits. Draw the payoff matrix and discuss the possible outcomes. Answer:

Microeconomics, 7e (Perloff) Chapter 19 Asymmetric Information

19.1 Adverse Selection

1) Adverse selection can occur when 1.

A) all persons involved in a transaction have full information.

2.

B) one person has information not available to others.

3.

C) post-agreement incentives result in workers shirking.

4.

D) nobody has any information about a particular product.

2) If you sell your DVD player on eBay, you will be better informed about the quality of the product than any potential buyer. This is called 1.

A) adverse selection.

2.

B) asymmetric information.

3.

C) moral hazard.

4.

D) opportunistic behavior.

3) A person who practices poisonous snake charming and does not reveal this to her health insurance company before purchasing insurance is an example of 1.

A) moral hazard.

2.

B) adverse selection.

3.

C) signaling.

4.

D) screening.

4) A person who starts practicing poisonous snake charming after signing a contract with a health insurance company is an example of 1.

A) moral hazard.

2.

B) adverse selection.

3.

C) signaling.

4.

D) screening.

5) In the automobile insurance market, adverse selection occurs when 1.

A) drivers with greater risks buy a policy with large deductibles.

2.

B) drivers with greater risks buy a policy with no deductibles.

3.

C) uninsured drivers drive recklessly.

4.

D) insured drivers drive recklessly.

6) Opportunism may occur when

1.

A) both parties have limited information.

2.

B) both parties have full information.

3.

C) one party has information the other does not.

4.

D) All of the above.

7) Adverse selection occurs when 1.

A) a person takes more risks that are not known to the life insurance company because he has life insurance.

2.

B) a person buys life insurance because he has a risky lifestyle that is not known to the life insurance company.

3. 4.

C) a person is a risk lover. D) pregnant women with health insurance make more doctor visits than uninsured pregnant women.

8) Adverse selection occurs when there is

1.

A) full information.

2.

B) unobserved behavior.

3.

C) an unobserved characteristic.

4.

D) a worker who shirks because his boss does not watch him.

9) If reckless drivers are more likely to buy automobile insurance than safe drivers are, 1.

A) a moral hazard has occurred.

2.

B) adverse selection has occurred.

3.

C) the market for insurance is efficient.

4.

D) then automobile insurance will be fairly priced.

10) What is one of the most important benefits of the Internet?

1.

A) The Internet has increased asymmetric information.

2.

B) The Internet has reduced asymmetric information.

3.

C) The Internet has increased moral hazard.

4.

D) The Internet has increased transaction costs.

11) If a bank offers mortgages that do not require the normal 20% down payment, the bank encourages 1.

A) people who know they might not pay off the mortgage.

2.

B) people who can't afford the down payment but can pay off the mortgage.

3.

C) people who know that they are going to pay off the mortgage.

4.

D) people who know they can't pay off the mortgage but who can afford the down payment.

12) If adverse selection exists in a market, 1.

A) it increases consumer surplus but reduces producer surplus.

2.

B) it reduces consumer and producer surplus.

3.

C) it reduces producer surplus but has no impact on consumer surplus.

4.

D) it increases both consumer and producer surplus.

13) The lemons problem is due to 1.

A) asymmetric information.

2.

B) moral hazard.

3.

C) hidden actions.

4.

D) symmetric information.

14) Which of the following is an example of a hidden characteristic? 1.

A) After purchasing life insurance, Billy joins the bomb squad.

2.

B) Only the owner knows about her car defects.

3.

C) Bobby let his employer know about his college degree.

4.

D) Sally provides her driving record to the car insurance company.

15) The market for used cars is shown in the above figure. Buyers cannot tell whether any given car is a lemon. Ten percent (10%) of all cars are lemons. Which of the following statements is true? 1.

A) All of the cars will be sold.

2.

B) No cars will be sold.

3.

C) Only lemons will be sold.

4.

D) Only good cars will be sold.

16) The market for used cars is shown in the above figure. Buyers cannot tell whether any given car is a lemon. Forty percent (40%) of all cars are lemons. Which of the following statements is true?

1.

A) All of the cars will be sold.

2.

B) No cars will be sold.

3.

C) Only lemons will be sold.

4.

D) Ten percent of the used cars sold will be lemons.

17) The market for used cars is shown in the above figure. Buyers cannot tell whether any given car is a lemon. Forty percent (40%) of all cars are lemons. Which of the following statements is true? 600.

A) All of the cars will be sold at $1,600.

601.

B) No cars will be sold.

602.

C) Only lemons will be sold at $1,600.

603.

D) Only lemons will be sold at $1,000.

18) The market for used cars is shown in the above figure. Buyers cannot tell whether any given car is a lemon. Ten percent (10%) of all cars are lemons. Which of the following statements is true? 900.

A) All of the cars sell for $1,900.

901.

B) Only lemons are sold for $1,900.

902.

C) Only lemons are sold for $1,000.

903.

D) Only good cars will be sold for $2,000.

19) The market for used cars is shown in the above figure. Buyers cannot tell whether any given car is a lemon. Ten percent (10%) of all cars are lemons. Which of the following statements is true? 900.

A) Only lemons are sold for $1,900.

901.

B) Only lemons are sold for $1,000.

902.

C) Buyers of lemons will pay too much for their cars.

903.

D) Buyers of good cars will pay too much for their cars.

20) A consumer is likely to avoid adverse selection and get a high-quality lunch at

1.

A) a snack bar at a traveling carnival.

2.

B) a vendor who parks her cart at a different location every noon.

3.

C) a restaurant in the center of a business district.

4.

D) a restaurant located next door to Disneyland.

21) Which of the following reduces the effects of asymmetric information? 1.

A) repeat purchases

2.

B) warranties

3.

C) building a reputation

4.

D) All of the above.

22) With asymmetric information firms might be reluctant to improve the quality of their products because

1.

A) it costs them more to produce the better quality product.

2.

B) they are not able to completely capture the benefits of the improvement.

3.

C) consumers do not value the better product.

4.

D) consumers are better informed about the product and value the new product less.

23) Asymmetric information will always cause 1.

A) efficiency problems.

2.

B) equity problems.

3.

C) Both A and B.

4.

D) None of the above.

24) Many people tend to buy multiple cars from the same manufacturer because 1.

A) doing so reduces the asymmetric information problem; consumers have better knowledge of quality.

2.

B) doing so reduces the asymmetric information problem; consumers have better knowledge of prices.

3.

C) doing so doesn't change the asymmetric information problem but consumers have more information.

4.

D) doing so increases the consumers brand loyalty.

25) eBay has a seller reputation system to provide 1.

A) consumers with a signal concerning seller quality.

2.

B) sellers a chance to signal other sellers concerning their quality.

3.

C) a reduction in monopoly power.

4.

D) improvements in investor relations.

26) What is one reason the eBay seller reputation system is important?

1. 2. 3. 4.

A) eBay transactions tend to be isolated, one-off transactions. B) eBay transactions tend to be repeat sales between the same seller and buyers. C) eBay transactions often involve stolen products. D) eBay transactions require a larger percentage than, say, Amazon transactions.

27) What could be a potential explanation for a firm selling virtually identical products under different brands? 1. 2. 3. 4.

A) Consumers are well informed about the quality of the products. B) Consumers believe that the products' quality is similar and thus firms are able to price discriminate. C) Consumers always favor private-label brands. D) Consumers believe that the products' quality differs and thus firms are able to price discriminate.

For the following, please answer "True" or "False" and explain why.

28) Adverse selection occurs when an agreement encourages undesirable behavior.

29) If sellers of good cars and sellers of lemons both offer a warranty on their cars, consumers will then be able to tell which cars are the lemons.

30) A downtown diner daily serving the same business people will be more likely to serve a tasty lunch than a snack bar at a tourist attraction.

31) Explain what may occur when a buyer and a seller have unequal amounts of limited information. Describe two different types of problems that may arise when asymmetric information exists. symmetric information may lead to opportunistic behavior where the informed person benefits at the expense of the person with less information. Adverse selection may occur where the informed person benefits form the less informed person not knowing about an unobserved characteristic of the informed person. Moral hazard may occur if the informed person takes advantage of the less informed person through an unobserved action.

32) The XYZ Co. is hiring salespersons. They will be paid a very attractive hourly rate that is independent of how much they sell. Describe an adverse selection that would take place. Describe a moral hazard that would take place.

33) The market for used cars is shown in the above figure. Buyers cannot tell whether any given car is a lemon. The percent of all cars that are lemons is θ. What value of θ is necessary for all cars to be sold?

34) The market for used cars is shown in the above figure. Ten percent (10%) of all cars are lemons. A mechanic is offering to inspect a car for sale and certify that a car is not a lemon. If car sellers are risk neutral, what is the highest price that a car seller would pay for such a service? Who would buy this service?

35) The market for used cars is shown in the above figure. Buyers cannot tell whether any given car is a lemon. For all cars offered for sale to be sold, the percent of all cars that are lemons is θ. What happens to θ if car buyers incur a $100 transaction cost when buying a used car? .

36) Explain how product liability laws can reduce adverse selection.

19.2 Reducing Adverse Selection

1) Used car buyers will believe that a car is of good quality when the seller signals the car's high quality by offering a warranty when 1.

A) a warranty on a lemon is costly to the seller.

2.

B) warranties are offered on all cars.

3.

C) warranties are only offered on lemons.

4.

D) a warranty on a good car is a false signal.

2) If the market interest rate is 5% and a bank advertises loans at 12%, the bank will receive 1.

A) no applications.

2.

B) applications from mostly low-risk borrowers.

3.

C) applications from mostly high-risk borrowers.

4.

D) a moral hazard.

3) If a life insurance company does not require a medical exam of its policyholders, it is most likely that the company

1.

A) charges above-average premiums.

2.

B) charges below-average premiums.

3.

C) charges no premiums.

4.

D) has only very healthy policyholders.

4) Assume Health Insurance is provided universally by the government. This would 1.

A) eliminate the problems of adverse selection.

2.

B) result in adverse selection.

3.

C) eliminate the problems of moral hazard.

4.

D) All of the above.

5) If a student achieves a high SAT score, this 1.

A) sends a signal to a college that the applicant will be a good college student.

2.

B) does not act as a screening device.

3.

C) is a moral hazard.

4.

D) provides a college with no information.

6) Life insurance companies often give applicants a physical examination to prevent 1.

A) the person from dying before obtaining the policy.

2.

B) signaling.

3.

C) adverse selection.

4.

D) profit maximization.

7) Some companies subject their applicants to extensive tests. Why? 1.

A) to reduce the informational asymmetry between the firm and the applicant

2.

B) to screen the applicant to avoid the problem of adverse selection

3.

C) to gather more information about the applicant

4.

D) All of the above.

8) The requirement that all drivers must carry auto insurance reduces 1.

A) moral hazard.

2.

B) the effectiveness of signaling.

3.

C) adverse selection.

4.

D) the chance of auto accidents.

9) If bad drivers can usually avoid being ticketed by the police, then insurance companies will 1.

A) use one's driving record as a signal.

2.

B) use one's driving record as a screening device.

3.

C) not be able to use one's driving record as a screening device.

4.

D) request driving records directly from the police and not from the individual applicant.

10) A physical examination is not a good screening device for life insurance companies if 1.

A) life-threatening diseases are usually undetected.

2.

B) doctors cannot be easily bribed to write a good report.

3.

C) medical history is a good predictor of life expectancy.

4.

D) one's current state of health is a good predictor of life expectancy.

11) Assume health insurance is provided universally by the government. This would 1.

A) force every taxpayer to bear the costs of adverse selection.

2.

B) force every taxpayer to bear the costs of moral hazard.

3.

C) force the government to deal with adverse selection problems.

4.

D) force foreign governments to deal with moral hazard problems.

12) If the government provides universal health insurance, what screening process will the government need? 1.

A) It won't need a screening test.

2.

B) It will only give health insurance to non-smokers.

3.

C) It won't use screening tests but it will use statistical discrimination.

4.

D) It won't use screening tests but it will use signals.

13) What role does a company like J.D. Power (which provides product satisfaction reviews) serve? 1.

A) It provides a screening test.

2.

B) It provides a signal of quality.

3.

C) It reduces moral hazard.

4.

D) It reduces costs of giving surveys.

14) Why would a firm rather a company like J.D. Power (which provides product satisfaction reviews) review its product than doing it within the firm? 1.

A) J.D. Power provides less credibility.

2.

B) J.D. Power provides greater credibility.

3.

C) J.D. Power is more honest than the average firm.

4.

D) J.D. Power has a government monopoly in providing product satisfaction reviews.

For the following, please answer "True" or "False" and explain why.

15) Signals can help prevent adverse selection as long as a false signal is costly to the person sending it. .

16) How can a warranty at the seller's expense signal that a product is of high quality?

17) Explain why some people who are applying for a job at a bank dress up, arrive early, and have their paperwork neatly completed for the job interview.

19.3 Price Discrimination Due to False Beliefs About Quality

1) Firms are able to price discriminate 1.

A) when all customers are uninformed about quality differences.

2.

B) when no customers are uninformed about quality differences.

3.

C) when some customers are uninformed about quality differences.

4.

D) when there is full information about quality available to all customers.

2) Selling the same product under different brand names allows a firm to price discriminate as long as 1.

A) customers know the products are identical.

2.

B) customers do not know the products are identical.

3.

C) the products really are not the same.

4.

D) the firm lets customers know that the products are identical.

3) If all consumers are uninformed about the quality of a product 1.

A) firms can increase product by selling the same product under a different name at a different price.

2.

B) firms will not be able to price discriminate.

3.

C) firms will price discriminate.

4.

D) firms will increase profits by charging different prices for the same product.

4) The number of brands of identical products will most likely increase as 1.

A) the number of informed consumers increases.

2.

B) the cost of producing many brands decreases.

3.

C) the number of uninformed consumers decreases.

4.

D) None of the above.

5) For much of its history, General Motors was characterized by having versions of the same car with different nameplates, e.g., Chevrolet vs. Cadillac. This business model has seemed less successful in recent years. What might explain this? 1.

A) Consumers have more information concerning costs and quality.

2.

B) Consumers have more information concerning costs but not quality.

3. 4.

C) Sellers have more information concerning costs but consumers have more information about quality. D) Consumers have less information concerning costs and quality.

For the following, please answer "True" or "False" and explain why.

6) A firm is more likely to adopt multiple brand names for the same product when the good is a non-durable.

7) The XYZ Co. sells shirts. Shirts with the company label on the tag are perceived to be of higher quality than shirts with the store's label. Yet, shirts are of identical quality regardless of label. The demand for perceived high-quality shirts is ph = 50 - qh. The demand for perceived low-quality shirts is pl = 10 + ph – ql. The firm can produce shirts at TC = qh + ql. How many shirts does the firm label as low quality and how many as high quality? What prices are charged?

8) Describe how a firm that sells only one brand of bleach may be able to increase its profit if it re-labels some of its bleach.

19.4 Market Power from Price Ignorance

1) Competitive firms are able to set price above marginal cost when 1.

A) the markup is less than the cost of going to another store.

2.

B) the markup is greater than the cost of going to another store.

3.

C) all consumers have full information.

4.

D) consumers know what other stores are charging.

2) When consumers have asymmetric information and when search costs and the number of firms are large, a single-price equilibrium in a competitive market 1.

A) is impossible.

2.

B) occurs when price equals average cost.

3.

C) occurs when price equals marginal cost plus the search cost.

4.

D) occurs when the price is the price a monopoly would set.

3) If there is zero search cost, then in the presence of asymmetric information, competitive firms will 1.

A) charge the monopoly price.

2.

B) charge the competitive price.

3.

C) charge zero price.

4.

D) shut down.

4) The Internet has made it possible to compare lots of prices without incurring a lot of cost. This 1.

A) has given firms added market power to price discriminate.

2.

B) has significantly reduced search cost and made markets more competitive.

3.

C) has enabled firms to charge higher prices to consumers with computers.

4.

D) has only affected technology markets.

5) The Internet has made it possible to compare lots of prices without incurring a lot of cost. If Internet access is unequally distributed throughout the population one would expect 1.

A) consumers with internet access to pay a higher price.

2.

B) consumers without internet access to pay a lower price.

3.

C) price discrimination against consumers without internet access.

4.

D) firms to charge the same price to all consumers.

6) As long as there is asymmetric information among consumers and positive search cost, if price is below the monopoly price and the same across all firms, then a competitive firm 1.

A) can always profit from raising its price.

2.

B) can always profit from lowering its price.

3.

C) can profit from raising its price but by no more than the search cost.

4.

D) can profit from lowering its price but by no more than the search cost.

7) With asymmetric information among consumers and positive search costs, a firm may 1.

A) raise its price above the monopoly price.

2.

B) price at the monopoly level.

3.

C) price at the full information competitive level.

4.

D) None of the above.

8) Empirical studies conclude that advertising 1.

A) raises prices in all markets.

2.

B) can reduce the prices of many goods.

3.

C) reduces the prices on all goods.

4.

D) has no impact on prices.

9) If consumers have limited information about price and search costs exist, then 1.

A) the result must be that all firms will charge the same price.

2.

B) the monopoly price must result.

3.

C) the full-information, competitive price is not an equilibrium.

4.

D) the difference in prices between firms will be greater than the search cost.

For the following, please answer "True" or "False" and explain why.

10) In the tourist-trap model, a consumer might pay more than marginal cost for a good sold in a competitive market if the cost of possibly finding the good cheaper is more than the markup over marginal cost.

11) In a competitive market with large search costs, many firms, and asymmetric information, why is the monopoly price the only possible single-price equilibrium?

12) Explain why high priced lawyers may support an industry ban on price advertising for lawyers. n advertising ban that prohibits lawyers from advertising prices allows lawyers to charge higher prices.

19.5 Problems Arising from Ignorance When Hiring

1) Firms under-invest in safety because 1.

A) firms are not concerned with safety.

2.

B) firms do not want their plants to be safe.

3.

C) firms are risk averse.

4.

D) firms do not enjoy all of the benefits from investments in safety.

2) Government mandated safety standard within firms 1.

A) will always decrease efficiency.

2.

B) can increase efficiency by avoiding a prisoner's dilemma outcome.

3.

C) are unnecessary because of asymmetric information.

4.

D) will create unfair competition among firms.

3) The above figure shows the payoff to two firms in an industry deciding to make an investment in worker safety. The dominant strategy for each firm 1.

A) is to do the opposite of the other firm.

2.

B) is to make the investment.

3.

C) is to not make the investment.

4.

D) does not exist.

4) The above figure shows the payoff to two firms in an industry deciding to make an investment in worker safety. The Nash equilibrium

1.

A) is for just one of the firms to make the investment.

2.

B) is for both firms to make the investment.

3.

C) is for neither firm to make the investment.

4.

D) does not exist.

5) The above figure shows the payoff to two firms in an industry deciding to make an investment in worker safety. Neither firm will make the investment because 1.

A) each can benefit from the other firm incurring the costs.

2.

B) there is no benefit to making the investment.

3.

C) each firm pays for the other firm's investment.

4.

D) society does not care about worker safety.

6) Workers do not know the safety records at individual firms; they only know industry averages. As a result,

1.

A) each firm tries to outdo each other in making safety improvements.

2.

B) each firm has the incentive to be the safest in its industry.

3.

C) the equilibrium level of safety is less than optimal.

4.

D) the optimal level of safety is achieved.

7) Investment in safety at the firm level poses a prisoners' dilemma because 1.

A) if each firm plays its dominant strategy, joint profits are maximized.

2.

B) if each firm plays its dominant strategy, joint profits are not maximized.

3.

C) neither firm has a dominant strategy.

4.

D) the Nash equilibrium is not achieved.

8) Some software firms require that applicants have passed certain standardized certification tests before being hired. This policy is necessary when 1.

A) cheap talk does not provide a credible signal.

2.

B) cheap talk does provide a credible signal.

3.

C) the interests of the firm and the applicant converge.

4.

D) the applicant is honest about her abilities.

9) A pooling equilibrium occurs when 1.

A) dissimilar workers are paid alike.

2.

B) firms can distinguish between workers of different qualities.

3.

C) workers of the same quality are paid different wages.

4.

D) all workers are overpaid equally.

10) If diplomas work efficiently in signaling productive capabilities to employers, the resulting equilibrium 1.

A) will be a separating equilibrium.

2.

B) will be a pooling equilibrium.

3.

C) will always be inefficient.

4.

D) will never be efficient.

11) Employers verify the facts of potential employees' resumes to avoid 1.

A) signaling.

2.

B) screening.

3.

C) cheap talk.

4.

D) moral hazard.

12) If getting accepted into college is very difficult because of high standards of intelligence and ability, but students learn absolutely nothing while in college, it is most likely that 1.

A) they will not be hired upon graduation.

2.

B) attendance sends a signal to employers regarding ability.

3.

C) nobody will want to go to college.

4.

D) a college degree is not a credible signal.

13) If low-quality workers are unable to obtain a college degree, then a separating equilibrium can occur if 1.

A) the cost of obtaining a degree is less than the wage premium paid to those who have obtained the degree.

2.

B) the cost of obtaining a degree is greater than the wage premium paid to those who have obtained the degree.

3.

C) the cost of obtaining a degree is zero.

4.

D) the wage premium paid to those who have obtained the degree is positive.

14) When relatively few workers have high ability, 1.

A) they will settle for the average wage.

2.

B) they will want to signal their ability.

3.

C) the premium for high ability is less than when most workers have high ability.

4.

D) they do not need to signal their ability.

15) If a college degree is used to signal high ability, but the cost of a college degree is relatively high, 1.

A) a separating equilibrium is achieved.

2.

B) a pooling equilibrium is achieved.

3.

C) even low-ability workers will attend college.

4.

D) the share of high-ability workers must be 1.

16) Screening and signaling in the labor market are inefficient 1.

A) unless college costs are relatively low.

2.

B) unless they result in a better job match.

3.

C) because the benefits are spread out over many firms.

4.

D) because they raise the wage paid to all workers.

17) Cheap talk works at placing workers in the right job as long as 1.

A) the interests of the worker and the firm coincide.

2.

B) all workers want the most demanding jobs regardless of their abilities.

3.

C) workers have different ability levels that are unknown to firms.

4.

D) firms are not profit maximizers.

18) Auto insurance rates are lower for young women relative to young men. An unusually reckless young woman driver benefits from this 1.

A) statistical discrimination.

2.

B) signal.

3.

C) screening.

4.

D) majority voting.

19) Very few players are drafted into the NFL when they play at Division II or Division III schools whereas more players are drafted in the MLB from these schools. What can explain this? 1.

A) Attending a Division II or Division III school sends a stronger signal of future and present quality in the NFL.

2.

B) Attending a Division II or Division III school sends a weaker signal of future and present quality in the MLB.

3.

C) A signal of future and present quality is more important in the MLB than in the NFL.

4.

D) Attending a particular school is like a screening test for life insurance.

20) If a person attends a public college and is not hired because the boss went to a private college, this might be an example of 1.

A) statistical discrimination.

2.

B) racial discrimination.

3.

C) screening.

4.

D) moral hazard.

21) A job advertised in an industry publication rather than a local newspaper is an attempt to 1.

A) use access to the industry publication as a screening mechanism.

2.

B) use access to the newspaper as a screening mechanism.

3.

C) use access to the industry publication as a means of statistical discrimination.

4.

D) use access to the newspaper as a means of statistical discrimination.

22) Firms are not allowed to ask certain questions during job interviews, including marital status or how many children you have. Why is this the case? 1.

A) People with children are more productive, thus it is unfair if the firm knows this information.

2.

B) Firms might use the answers to these questions as a means of statistical discrimination.

3.

C) People with children are less productive, thus the firm has no right to this information.

4.

D) Such questions violate the privacy clause of the U.S. Constitution.

23) If one job applicant truthfully reveals that they subscribe to an industry publication, the job applicant is 1.

A) using the trade publication as a screening tool.

2.

B) using the trade publication as a signaling tool.

3.

C) using the trade publication as a form of statistical discrimination.

4.

D) using the trade publication for the wrong reasons.

24) The minor league system in professional baseball can be thought of as a 1.

A) screening tool.

2.

B) signaling tool.

3.

C) form of statistical discrimination.

4.

D) being used for the wrong reasons.

25) A necessary condition for a separating equilibrium when workers can use education as a signal of their ability is 1.

A) the high-ability wage is larger than the sum of the degree cost plus the lowability wage.

2.

B) the cost of a degree is larger than the low-skill wage.

3.

C) the high-ability wage is smaller than the low-ability wage.

4.

D) the high-skill wage is equal to the cost of a degree.

26) In a labor-market pooling equilibrium with high-skill and low-skill workers and where a costly educational degree is used as a signaling device,

1.

A) the high-skill workers' wage is larger than the low-skill workers' wage.

2.

B) the low-skill workers' wage is zero.

3.

C) the high- and low-skill workers receive the same wage.

4.

D) the high-skill workers' wage is equal to the low-skill workers' wage plus the cost of obtaining a degree.

27) In a labor-market separating equilibrium with high-skill and low-skill workers and where a costly educational degree is used solely as a signal device, we can say that 1.

A) education is socially inefficient because it is costly and provides no useful skills to the worker.

2.

B) education is socially efficient.

3.

C) education is privately inefficient for high-skill workers.

4.

D) education is privately useful for low-skill workers only.

28) In a labor-market separating equilibrium with high-skill and low-skill workers and where a costly educational degree is used solely as a signal device, government can increase total social welfare by

1.

A) subsidizing school for low-skill workers.

2.

B) subsidizing school for all workers.

3.

C) eliminating schooling, i.e., banning wasteful signaling.

4.

D) subsidizing school for high-skill workers.

29) In a labor-market pooling equilibrium with high-skill and low-skill workers and where a costly educational degree is used as a signaling device, all else equal, an increase in the wage differential between high- and low-skill workers leads to 1.

A) an increase in the required minimum share of high-skill workers.

2.

B) a decrease in the required minimum share of high-skill workers.

3.

C) no change in the required minimum share of high-skill workers.

4.

D) None of the above answers are correct.

30) The cost, c, of a college education that serves only as a signal of a high-quality worker is $10,000. The wage of a known high-quality worker, wh, is $30,000. The wage for a known low-quality worker, wl, is $10,000. For what value of the share of the work force that is of high quality, s, is a pooling equilibrium possible? 1.

A) s > 0.5

2.

B) s = 0

3.

C) s = 0.25

4.

D) s < 0.45

31) The wage of a known high-quality worker, wh, is $10,000. The wage for a known low-quality worker, wl, is $4,000. The share of the work force that is of high quality, s, is 0.5. For what value of the cost, c, of a college education that serves only as a signal of a high-quality worker does a pooling equilibrium exist? 1.

A) c > $3,000

2.

B) c < $3,000

3.

C) c < $1,000

4.

D) c = $2,500

For the following, please answer "True" or "False" and explain why.

32) If an employer cannot distinguish the ability of workers a separating equilibrium will result. Answer: False. When an employer cannot distinguish the abilities of the workers, the employer will pay all workers an average wage. This is a pooling equilibrium.

33) Two firms, A and B, are faced with a decision on making investments in safety. They each currently earn profit of $500. A safety investment would cost $100 paid by the firm that makes the investment and would lower both firms' labor costs by $75 per firm. If both firms share the investment ($50 each) their labor costs are lowered by $100 per firm. Draw the payoff matrix for this game and determine the Nash equilibrium. Does it make sense for the firms in the industry to ask the government to force them to make the investment? Explain. Answer:

34) Joe wants to achieve the highest position possible with the XYZ Co. During the interview, he tells them he is capable of performing many difficult tasks. The company

feels there is a 10% chance he is lying. Given the payoff matrix in the above figure, what job level will the company offer to Joe? Why?

35) Suppose 10% of all workers are of high ability. If a firm knows a worker's ability, workers of low ability are paid $20,000 and workers of high ability are paid $30,000. A college degree can signal ability, and the cost of the degree is $11,000. Will there be a pooling equilibrium or a separating equilibrium?

36) The cost, c, of a college education that serves only as a signal of a high-quality worker is $20,000. The wage of a known high-quality worker, wh, is $75,000. The wage for a known low-quality worker, wl, is $50,000. For what value of the share of the work force that is of high quality, t, is a pooling equilibrium possible?

Microeconomics, 7e (Perloff) Chapter 20 Contracts and Moral Hazards

20.1 The Principal-Agent Problem

1) In which of the following contracts is the agent's payment unaffected by his performance?

1.

A) fixed-fee contract

2.

B) hire contract

3.

C) contingent contract

4.

D) sharing contract

2) The outcome of the state of nature effects the payoff to the agent under a 1.

A) fixed-fee contract.

2.

B) hire contract.

3.

C) contingent contract.

4.

D) All of the above.

3) Moral hazard occurs when contracts are written in such a way that 1.

A) the interests of agent and principal converge.

2.

B) the interests of agent and principal diverge.

3.

C) agents will wish to maximize the principal's utility.

4.

D) production and risk-bearing efficiency are achieved.

4) Efficiency in risk bearing implies that 1.

A) risk is completely eliminated.

2.

B) the least risk-averse party bears most of the risk.

3.

C) the most risk-averse party bears most of the risk.

4.

D) all of the risk is borne by just one of the parties regardless of the degree of risk aversion.

5) Production efficiency implies that 1.

A) joint profits are maximized.

2.

B) joint profits are minimized.

3.

C) joint profits are zero.

4.

D) joint profits can be increased.

6) Many professional sports athletes have incentive clauses in their contracts. These indicate that 1.

A) the team owner has asymmetric information.

2.

B) the athlete might engage in moral hazard, which the team owner wishes to avoid.

3.

C) the athlete might engage in adverse selection, which the team owner wishes to avoid.

4.

D) the athlete has stronger negotiators.

7) In professional golf, a tournament winner might win, say, $1 million, whereas second place wins only $600,000. Why might golf have a 40% reduction in prize money from first and second place? 1.

A) To ensure that there is enough money to pay those who finish near last.

2.

B) To motivate golfers to take risks they might not otherwise take.

3.

C) To motivate golfers to not take risks they might otherwise take.

4.

D) The Professional Golfers Association is a cartel.

8) In NASCAR, a race winner might win, say, $500,000, whereas second place wins $450,000. Why might NASCAR have a 10% reduction in prize money from first and second place? 1.

A) To ensure that there is enough money to pay those who finish near last.

2.

B) To motivate drivers to take risks they might not otherwise take.

3.

C) To motivate drivers to not take risks they might otherwise take.

4.

D) NASCAR is a cartel.

9) Season ticket holders often purchase their tickets before a season begins. There is often a discount associated with season tickets. Why?

1.

A) Buying season tickets entails risk, which consumers are willing to bear if they are compensated for doing so.

2.

B) Buying season tickets is a way of guaranteeing a supply of tickets for the secondary market.

3.

C) Selling season tickets entails greater transactions costs which team owners hope to avoid.

4.

D) Sports teams are local monopolies.

10) Season tickets entail a certain amount of risk due to unpredictable team quality, injuries, weather, and so forth. Who is most likely to purchase season tickets? 1.

A) Sally, who is a rabid fan and is not wealthy

2.

B) Jim, who is a rabid fan and is risk neutral

3.

C) Roger, who is a rabid fan but is risk averse

4.

D) Jenn, who is a fan and is very wealthy

11) What is one potential problem with nationalized health care?

1.

A) It might not be efficient in risk bearing.

2.

B) It is guaranteed to solve the principal-agent problem.

3.

C) It would be efficient in risk bearing.

4.

D) It might cost too much.

12) We can say that a contract is able to prevent moral hazard when 1.

A) it eliminates production inefficiencies due to moral hazard without shifting risk to risk-averse people.

2.

B) it eliminates production inefficiencies due to moral hazard without shifting risk to risk-loving people.

3. 4.

C) it shifts risk to risk-loving people. D) it eliminates production inefficiencies due to moral hazard and shifts risk to risk-averse people.

13) A key aspect of the principal-agent problem is that

1.

A) the principal cannot perfectly monitor the agent's actions.

2.

B) the principal can perfectly monitor the agent's actions.

3.

C) the output level does not depend on the agent's actions.

4.

D) the principal is always risk-averse.

For the following, please answer "True" or "False" and explain why.

14) Production inefficiency is most likely to occur when the principal has more information about work performance than the agent does.

15) The type of contract selected depends on the information available to the parties.

16) Describe the characteristics of an efficient contract between a principal and an agent.

17) Explain how more than one possible state of nature affects contract choices.

20.2 Using Contracts to Reduce Moral Hazard

1) Suppose an agent must pay the full marginal cost for an item but splits the marginal revenue with the principal. As a result, 1.

A) joint profit is maximized.

2.

B) joint profit is not maximized.

3.

C) the agent will not enter into such a contract.

4.

D) the agent wishes to sell as many items as he can.

2) In a store that sells souvenirs, suppose an agent receives a $1 commission for each unit sold, and the principal receives the residual profit. As a result, 1.

A) joint profit is maximized.

2.

B) the agent will sell until the principal's marginal cost equals $1.

3.

C) no agent would enter into such a contract.

4.

D) the agent wishes to sell as many units as he can.

3) Suppose the principal offers to share a percentage of the profit with the agent. Such a contract 1.

A) will yield the same income for the agent as a hire contract would.

2.

B) is incentive compatible.

3.

C) creates a production inefficiency.

4.

D) would not be acceptable to any agent.

4) If the principal has full information, production efficiency without supervision can occur with 1.

A) a fixed fee rental contract.

2.

B) a profit-sharing contract.

3.

C) an incentive-compatible contract.

4.

D) All of the above.

5) Suppose two owners of a store agree to split the profit equally regardless of the number of hours each spends working at the store. As a result, 1. 2.

A) production efficiency is achieved. B) each enjoys only half the marginal benefit of an additional hour working in the store.

3.

C) one will work all of the time while the other works zero hours.

4.

D) each will work as many hours as if he or she were the sole owner.

6) Sue offers to pay Al $50 for each painting of his that she sells in her gallery. Each painting sells for $75. The cost to Al of producing each painting is $55. Which of the following statements is true about this contract? 1.

A) This contract is efficient.

2.

B) This contract maximizes joint profit.

3.

C) Al will not participate in this contract.

4.

D) This is a fixed-fee contract.

7) In the presence of asymmetric information, the only contract that results in production efficiency and no moral hazard is the one in which 1.

A) the agent receives a fixed fee.

2.

B) the principal receives a fixed rent.

3.

C) profit is shared.

4.

D) revenue is shared.

8) In the presence of asymmetric information, a contingent contract 1.

A) achieves production efficiency.

2.

B) can lead to opportunistic behavior on the part of the agent.

3.

C) is impossible to write.

4.

D) will result in the principal earning all of the profit.

9) In the presence of asymmetric information, a hire contract 1.

A) achieves production efficiency.

2.

B) can lead to opportunistic behavior on the part of the agent.

3.

C) is impossible to write.

4.

D) will result in the principal earning all of the profit.

10) In the presence of asymmetric information, a piece-rate contract 1. 2.

A) achieves production efficiency. B) can lead to agents producing more output than would occur under a fixedrent-paid-to-the-principal contract.

3.

C) is impossible to write.

4.

D) will result in the principal earning all of the profit.

11) In a principal-agent problem, if the contract used leads to the maximum of the principal's and agent's combined value (profits, payoffs), we can say that this contract features 1.

A) inefficiency in production, since only the principal's profits should be maximized.

2.

B) inefficiency in production, since only the agent's payoffs should be maximized.

3.

C) efficiency in production.

4.

D) inefficiency in production, since the agent's payoffs should be maximized and the principal's profits should be minimized.

12) In a principal-agent problem, if the contract implies that the more risk-averse agent will bear less risk, we can say that this contract exhibits 1.

A) efficiency in risk-bearing.

2.

B) risk sharing is not optimal because the less risk-averse (or risk-neutral) agent should bear none of the risk.

3.

C) risk sharing is not optimal because all risk should be transferred to the most risk-averse agent.

4.

D) risk sharing is not optimal because risk-neutral agents should face no risk.

13) Suppose a plaintiff hires a lawyer to represent her in a court case. The lawyer will be paid a fixed fee. Under this contract, 1.

A) production efficiency is achieved.

2.

B) the client bears all of the risk.

3.

C) the lawyer has an incentive to lie about his hours worked.

4.

D) All of the above.

14) Suppose a plaintiff hires a lawyer to represent her in a court case. The lawyer will be paid by the hour. Under this contract, 1.

A) production efficiency is not achieved.

2.

B) the client bears all of the risk.

3.

C) the lawyer has an incentive to lie about his hours worked.

4.

D) All of the above.

15) In the presence of asymmetric information, 1.

A) all contracts are efficient.

2.

B) efficiency in risk bearing cannot be achieved.

3.

C) a trade-off exists between risk-bearing efficiency and production efficiency.

4.

D) no contracting will take place.

16) Suppose a plaintiff hires a lawyer to represent her in a court case. The lawyer will receive a share of the settlement if the plaintiff wins. Under this contract, 1.

A) production efficiency cannot be achieved.

2.

B) the client bears all of the risk.

3.

C) the lawyer bears all of the risk.

4.

D) the risk is shared.

17) Suppose a plaintiff hires a lawyer to represent her in a court case. She agrees to pay the lawyer a wage per hour. She knows precisely what the lawyer should do and how long each activity should take, and she can verify that the lawyer has correctly completed each activity. She can terminate the contract at any time. With this contract, 1.

A) the lawyer bears all the risk.

2.

B) the risk is shared by the lawyer and the plaintiff.

3.

C) production efficiency can be achieved.

4.

D) production efficiency is impossible.

18) Suppose a plaintiff hires a lawyer to represent her in a court case. Under which of the following contracts is production efficiency assured? 1.

A) The lawyer is paid by the hour.

2.

B) The lawyer receives a share of the settlement.

3.

C) The lawyer receives a fixed fee.

4.

D) The lawyer pays the client a fee for the right to the entire settlement.

19) Suppose a plaintiff hires a lawyer to represent her in a court case. To which of the following contracts would a highly risk-averse plaintiff agree? 1.

A) The lawyer is paid by the hour.

2.

B) The lawyer receives a share of the settlement.

3.

C) The lawyer receives a fixed fee.

4.

D) The lawyer pays the client a fee for the right to the entire settlement.

20) Under which one of the following conditions would a lawyer accept a case on a contingent basis? 1.

A) The lawyer is risk averse.

2.

B) The client is risk loving.

3. 4.

C) The lawyer has several cases on a contingent basis with payoffs that are not perfectly positively correlated. D) The lawyer is more risk averse than the client is.

21) Under which one of the following contracts does an agent have the least incentive to behave opportunistically? 1.

A) The agent pays a fixed fee to the principal for the right to all future payoffs.

2.

B) The agent works for the principal on an hourly basis.

3.

C) The agent receives a share of the profit.

4.

D) The agent works for the principal on a per unit basis.

22) Suppose a plaintiff hires a lawyer to represent her in a court case. Under which of the following contracts is efficiency in risk bearing assured? 1.

A) The lawyer is paid by the hour.

2.

B) The lawyer receives a share of the settlement.

3.

C) The lawyer receives a fixed fee.

4.

D) It is impossible to determine without the degree of risk aversion for each.

23) One reason that lawyers might prefer a contingent contract when representing a plaintiff in a tort case is that 1.

A) lawyers are risk neutral.

2.

B) diversification of many cases allows lawyers to reduce risk.

3.

C) lawyers are typically confident about winning every case.

4.

D) hourly rates for lawyers are usually very low.

24) A contingent contract can create production inefficiency; however, many principals accept this because 1.

A) inefficiency is inevitable.

2.

B) monitoring is costless.

3.

C) risk is reduced.

4.

D) profit will increase as a result.

25) If an agent is risk neutral and a principal is risk averse, which of the following contracts would be efficient in risk bearing? 1.

A) A fixed fee is paid to the agent.

2.

B) A fixed fee is paid to the principal.

3.

C) An hourly rate is paid to the agent.

4.

D) The agent enjoys a share of the profit.

26) Assume a lawyer signs a contingency fee contract with a plaintiff. The case ends up settling for $1 million, of which the plaintiff received $800,000. It is obvious this contract was efficient. 1.

A) True, the plaintiff received some money.

2.

B) False, it is not clear whether another lawyer would have settled for more.

3.

C) False, it is not clear whether the plaintiff's lawyer is very smart.

4.

D) True, the plaintiff and lawyer reached an agreement and both lived up to their end of the agreement.

27) Many art galleries keep 60% of the sale price of a painting. However, art galleries do not typically pay the artist while the painting hangs on the wall. This form of contingency contract may be efficient if 1.

A) the artist is less risk averse than the gallery.

2.

B) the gallery is less risk averse than the artist.

3.

C) the artist is unable to diversify across galleries or paintings.

4.

D) the gallery is unable to diversify across artists of paintings.

28) Author A accepts a $5,000 advance and a 10% royalty after 5,000 books are sold. Author B foregoes the advance and negotiates for a 15% royalty on all books sold. Author C decides to self publish his book and keep 50% of all sales revenue. In what order of risk aversion (from most to least) would you rank these authors? 1.

A) Author A, Author B, Author C

2.

B) Author A, Author C, Author B

3.

C) Author B, Author A, Author C

4.

D) Author C, Author B, Author A

29) Author A accepts a $5,000 advance and a 10% royalty after 5,000 books are sold. Author B foregoes the advance and negotiates for a 15% royalty on all books sold. Author C decides to self publish his book and keep 50% of all sales revenue. Which of these authors is most likely to have 10 books published? 1.

A) Author A

2.

B) Author B

3.

C) Author C

4.

D) They are all equally likely.

30) Author A accepts a $5,000 advance and a 10% royalty after 5,000 books are sold. Author B foregoes the advance and negotiates for a 15% royalty on all books sold. Author C decides to self publish his book and keep 50% of all sales revenue. Which of these authors expects to sell the fewest books? 1.

A) Author A

2.

B) Author B

3.

C) Author C

4.

D) They are all equally likely.

For the following, please answer "True" or "False" and explain why.

31) In the presence of asymmetric information, production efficiency is assured when the principal and agent share the profit.

32) With full information any contract will lead to production efficiency.

33) In the presence of asymmetric information with costless monitoring and enforcement, a hire contract results in production efficiency.

34) Sam is suing someone in court for $10,000. The probability that Sam will lose the case is 1/h where h is the number of hours that Sam's attorney works on the case. The lawyer charges $500 per hour if he is to be paid hourly, or he requests 20% of the settlement if he is to be paid on a contingency basis. Assuming both Sam and the attorney are risk-neutral wealth maximizers, is either contract efficient?

35) Rents for stores at shopping malls are usually tied to the profits of the store. Comment on how this arrangement affects the mall owner's income versus a fixed rent.

36) If information is asymmetric, explain why the hire contract is not efficient in production and a moral hazard exists, but the fixed fee to the principal contract is efficient and does not pose a moral hazard problem.

37) Sam hires an attorney to present a court case. If Sam wins the case, he will receive some money. This payoff is a function of the attorney's hours and which judge is assigned the case that day. Judge A is very understanding toward people in Sam's position, but judge B is very harsh toward people like Sam. Is it possible for Sam to get the attorney to deliver the optimal amount of effort and make the attorney bear all of the risk?

38) Sarah's demand for routine medical visits is q = 10 - 0.2p when she is healthy and q = 20 - 0.2p when she is sick. Medical visits cost $50 each if Sarah has no medical insurance. She is sick 20% of the time. Sarah is considering two different insurance plans. One offers free medical visits; the other plan costs less up front but requires that Sarah pay $5 per medical visit. Compare the two plans in terms of the trade-off between risk and moral hazard.

20.3 Monitoring to Reduce Moral Hazard

1) If a firm has established monitoring devices that have a 50% chance of detecting shirking, and an employee gains $5,000 from shirking, the employer can deter shirking by having employees post a bond equal to 500.

A) $2,500.

501.

B) $5,000.

502.

C) $10,000.

503.

D) $50,000.

2) Monitoring is often used by firms in an attempt to decrease 1.

A) shirking.

2.

B) piece rates.

3.

C) adverse selection.

4.

D) signaling.

3) As the probability of detecting shirking increases, the size of the bond necessary to deter shirking 1.

A) also increases.

2.

B) stays the same.

3.

C) decreases.

4.

D) increases at an exponential rate.

4) Which of the following workers is most likely to be asked to post a bond? 1.

A) construction contractor

2.

B) fast food worker

3.

C) sanitation worker

4.

D) book author

5) Which of the following would not be used by firms to deter shirking? 1.

A) requiring employees to post a bond

2.

B) offering a bonus after five years of service

3.

C) paying more than the market wage

4.

D) paying less than the market wage

6) The benefit to employers of deferred payments is that 1.

A) adverse selection is eliminated.

2.

B) employers cannot engage in any opportunistic behavior.

3.

C) these payments raise the cost of being fired, so more monitoring is needed.

4.

D) these payments raise the cost of being fired, so less monitoring is needed.

7) An efficiency wage premium serves the same function as a bond because, just as with a bond, the premium represents 1.

A) the amount the employee loses if caught shirking.

2.

B) the expected value of the amount the employee loses if he shirks.

3.

C) the cost of monitoring the employee.

4.

D) the gain to the employee if he shirks.

8) A profit-maximizing firm that uses an efficiency wage and monitors will increase the wage it pays its workers until 1.

A) the worker requires no monitoring.

2.

B) the worker receives the market wage and requires full-time monitoring.

3.

C) the cost of monitoring the worker equals the efficiency wage.

4.

D) the change in the workers' productivity from being monitored times the per time unit cost of monitoring equals one.

9) If an additional dollar spent on monitoring would reduce shirking by 10 minutes, then the firm will increase the worker's wage by $1 if this caused 1.

A) shirking to increase by less than 10 minutes.

2.

B) shirking to decrease by more than 10 minutes.

3.

C) shirking to decrease by less than 10 minutes.

4.

D) monitoring to become unnecessary.

10) If all firms pay an efficiency wage, then 1.

A) there is no cost to shirking because the shirking worker can receive his high wage at another firm after being caught and fired.

2.

B) the macroeconomy would enjoy a prolonged period of near-zero unemployment.

3.

C) there is a cost to shirking because the efficiency wage is less than it would have been if only a few firms paid it.

4.

D) there is a cost to shirking because the shirking worker will spend a greater time unemployed after being caught and fired.

11) If a professional sports athlete signs a new contract which defers compensation until years after she is retired, she is signaling 1.

A) that she does not plan to shirk in the future, regardless of whether she did so in the past.

2.

B) that she did shirk and she will do so in the future.

3.

C) that she did shirk but won't do so in the future.

4.

D) that she didn't shirk and she won't do so in the future.

12) In 2008, foreclosures reached a record high. Which of the following is NOT a possible reason for foreclosures? 1.

A) Many mortgages were initiated without a down payment.

2.

B) Many mortgages were initiated on secondary and investment homes.

3. 4.

C) Some mortgages were adjustable rate mortgages which might have dramatically increased monthly payments for some. D) Property values were increasing too fast.

13) An employee gains $500 from shirking. Thus, to deter shirking, the employer makes employees post a bond equal to $1,000, and installs monitoring devices to detect shirking. What is the probability that these devices can detect shirking? 1.

A) 30%

2.

B) 100%

3.

C) 50%

4.

D) 95%

14) A firm uses an efficiency wage scheme to deter workers from shirking. A risk-neutral worker will not shirk if 1.

A) the expected loss from being fired is larger than or equal to the gain from shirking.

2.

B) the expected loss from being fired is smaller than the gain from shirking.

3.

C) the gain from shirking is positive.

4.

D) the expected loss from being fired is zero.

For the following, please answer "True" or "False" and explain why.

15) When shirking at the workplace occurs, increased monitoring of workers is the only effective way to reduce this behavior.

16) Explain why a firm may hire managers to operate outlets near the firm's headquarters, but may sell franchise rights for the outlets located greater distances from the headquarters. (With a franchise, the firm sells a brand name and a method of doing business to someone who then owns and operates the outlet.)

17) Suppose the probability of an employee being caught shirking, q, is a function of the employer's monitoring, M, such that q = M/100. If workers must put up a $1,000 bond and the gain to each worker from shirking is $100, what is the employer's optimal level of monitoring that is just sufficient to discourage shirking?

18) Suppose an employer has monitoring devices established so that the probability of an employee being caught while shirking is 0.2. If the gain to the employee from shirking is $1,000, how large a bond will deter shirking?

19) Suppose employees pay a bond of $1,000 to an employer. The gain from shirking is $400. Monitoring devices have been installed so that there is a 50% chance of being caught if you are shirking. The company is considering the installation of additional monitoring devices to increase the chance of catching a shirker to 100%. They feel this is needed to deter all shirking. What is your recommendation to the company? Explain.

20.4 Checks on Principals

1) Under deferred compensation packages, 1.

A) a moral hazard occurs if a firms fires a good worker before the worker receives her deferred compensation.

2.

B) a moral hazard occurs if workers decide not to shirk so as to receive the deferred compensation.

3. 4.

C) moral hazards are avoided. D) workers' wages are below their marginal revenue product as they near retirement.

2) Workers can reduce the chance of an employer lying by 1.

A) obtaining more information about the firm's performance.

2.

B) having a representative on the board of directors.

3.

C) requiring that employers share the cost of an economic downtown.

4.

D) All of the above.

3) Wage reduction policies are less common than layoffs because 1.

A) workers never prefer wage reduction policies.

2.

B) workers always trust the firm to tell the truth.

3.

C) of asymmetric information.

4.

D) of adverse selection problems.

4) A deferred payment scheme is more likely to be accepted by a worker if 1.

A) the firm has a reputation of not firing senior employees to save pension costs.

2.

B) the worker is very young.

3.

C) the worker is very old.

4.

D) None of the above.

5) If a firm was owned by its employees, 1.

A) there is a higher probability that wage reductions would outweigh layoffs.

2.

B) those in charge would not act any differently than regular owners; there would still be layoffs.

3.

C) those not in charge would remain risk neutral.

4.

D) wage reductions would be lower than if the firm was run for profit.

6) Assume a firm is run as a zero-profit enterprise. Which of the following would be true? 1.

A) There is a higher probability that wage reductions would outweigh layoffs.

2.

B) Those in charge would not act any different than regular owners, there would still be layoffs.

3. 4.

C) Those not in charge would remain risk neutral. D) Wage reductions would be lower than they would be if the firm was run for profit.

7) Socially inefficient outcomes are possible when 1.

A) uninformed parties want to avoid opportunistic behavior by informed parties.

2.

B) informed parties engage in opportunistic behavior against uninformed parties.

3.

C) those in charge are risk neutral.

4.

D) workers do not own the firm.

For the following, please answer "True" or "False" and explain why.

8) Including an employee representative on the board of directors reduces the possibility of opportunistic employer behavior.

9) Explain why checks on principals might be necessary.

20.5 Contract Choice

1) One way to prevent workers from shirking is to

1.

A) hire only workers who are predisposed toward shirking.

2.

B) hire only workers who are predisposed toward not shirking.

3.

C) reduce monitoring to zero.

4.

D) pay workers a fixed fee.

2) Firms that seek to avoid hiring lazy workers that assert they are hardworking are trying to avoid 1.

A) adverse selection.

2.

B) moral hazard.

3.

C) screening.

4.

D) signaling.

3) A good salesperson can sell $1,000,000 worth of goods, while a poor one can sell only $100,000 worth of goods. Job applicants know if they are good or bad, but the firm

does not. A firm will offer job applicants a choice between a fixed salary and a 20% commission. Assuming risk-neutral salespersons and no opportunistic behavior, what level must the fixed salary be so that the firm can determine a prospective good salesperson from a poor one? 1.

A) between $0 and $20,000

2.

B) between $20,000 and $200,000

3.

C) greater than $200,000

4.

D) zero

4) If good salespeople are extremely risk averse, then a choice between a fixed-fee contract and a contingent contract 1.

A) avoids a moral hazard.

2.

B) will result in all job candidates choosing the contingent contract.

3.

C) will result in an efficient contract.

4.

D) may not be a good screening device.

5) What is one potential problem with offering a choice of contracts to two different employees? 1.

A) If Employee A is paid more than Employee B, Employee A might sue for discrimination.

2.

B) Employee A might be paid less than Employee B, proving statistical discrimination.

3.

C) The two employees might compare salaries without comparing riskpreferences, thereby running the risk of jealousy or claims of discrimination.

4.

D) The two employees might compare risk preferences without comparing salaries, thereby running the risk of jealousy or claims of discrimination.

6) Why would a firm knowingly hire lazy employees? 1.

A) Lazy employees have positive marginal revenue product.

2.

B) Lazy employees have successfully sued firms in the past.

3. 4.

C) The Americans with Disabilities Act mandates that laziness be considered a disability. D) Lazy employees make hard-working employees look good.

7) If a firm hires lazy employees, 1.

A) it must pay them differently or hard-working employees will engage in moral hazard.

2.

B) it must pay them more or hard-working employees will engage in moral hazard.

3.

C) it must fire them before their laziness spreads to hard-working employees.

4.

D) the lazy employees make hard-working employees look good.

8) A good salesperson can sell $500,000 worth of goods, while a poor one can sell only $100,000 worth of goods. Job applicants know if they are good or bad, but the firm does not. A firm will offer job applicants a choice between a fixed salary of $10,000 or a 10% commission. Assuming risk-neutral salespersons and no opportunistic behavior, can the firm determine a prospective good salesperson from a poor one? 1.

A) Yes, because a poor salesperson will always choose the fixed salary.

2.

B) Yes, because a good salesperson will always choose the fixed salary.

3.

C) No, because a poor salesperson is indifferent between the two contracts.

4.

D) No, because a good salesperson is indifferent between the two contracts.

9) To induce an agent to work hard, a principal may offer the agent a bonus, in other words, an extra payment if a performance target is hit. Suppose that the agent's performance is affected by factors beyond the agent's control, for example umbrellas are demanded more on a rainy day. Under what conditions may the bonus not induce the agent to work harder? 1.

A) The agent is very risk averse.

2.

B) The agent is very risk loving.

3.

C) The agent is risk neutral.

4.

D) The principal is risk neutral.

10) A good salesperson can sell $100,000 worth of goods, while a poor one can sell only $10,000 worth of goods. Job applicants know if they are good or bad, but the firm does not. A firm will offer job applicants a choice between a fixed salary of $2,000 or a commission on the sale. Assume risk-neutral salespersons and no opportunistic behavior. Given that the firm wants to distinguish a prospective good salesperson from a poor one, what should be the commission on sales? 1.

A) Commission should be larger than 50%.

2.

B) Commission should be larger than 40%.

3.

C) Commission should be between 2% and 20%.

4.

D) Commission should be smaller than 2%.

11) A good salesperson can sell $200,000 worth of goods, while a poor one can sell only a smaller amount worth of goods. Job applicants know if they are good or bad, but the firm does not. A firm will offer job applicants a choice between a fixed salary of $20,000 or a 20% commission. Assume risk-neutral salespersons and no opportunistic behavior. Given that the firm wants to distinguish a prospective good salesperson from a poor one, what should be the sales amount of a poor salesperson? 1.

A) more than $150,000

2.

B) less than $100,000

3.

C) more than $100,000

4.

D) $100,000

For the following, please answer "True" or "False" and explain why.

12) A trade-off typically exists between incurring a moral hazard and making an adverse selection.

13) A good salesperson can sell $1,000,000 worth of goods, while a poor one can sell only $100,000 worth of goods. Job applicants know if they are good or bad, but the firm does not. A firm will offer job applicants a choice between a fixed salary and 20% commission. Assuming risk-neutral salespersons and no opportunistic behavior, what level must the fixed salary be so that the firm can distinguish a prospective good salesperson from a poor one, and thereby avoid hiring a poor one?

14) A good salesperson can sell $1,000,000 worth of goods, while a poor one can sell only $100,000 worth of goods. Job applicants know if they are good or bad, but the firm does not. A firm will offer job applicants a choice between a fixed salary of $25,000 or 20% commission. Assuming risk-neutral salespersons and the possibility of opportunistic behavior, will this choice of contracts allow the firm to distinguish between good salespersons and bad ones before the hiring decision is made?

FOR THE STUDENTS WHO NEED GRADE ‘A’ IN THEIR STUDIES Hi, hope you are having a great day…

We are a group of 24 writers having profound expertise in Business and Computer Science subjects. We can help you score A grade in your Accounting, Marketing, Finance, Economics, Management, Mathematics, Statistics, Information System, System Modeling, C++, Java Programming, Network Administration, Enterprise Administration, Database, Web Design, Networking, Internetworking, Data warehouse etc… We can also provide help with Psychology, Nursing, Health, History, English Literature, Political Science, Ethics, Humanity etc classes.

We can help with essays, term papers, research papers, dissertation, Ilabs, mymatlab, Wileplus, quizzes, exams, discussion questions etc.

You can expect: We understand each student has different requirement and we tend to treat each student according to his/her satisfaction.

We will provide original assignments, plagiarism free and to custom requirement. We will always meet deadlines. Our support will be 24/27, even in holidays. Our pricing will be fair. We will do free revisions if you want to make changes in provided work.

Email us for more information, query and quote.

[email protected]

View more...

Comments

Copyright ©2017 KUPDF Inc.
SUPPORT KUPDF